Ndeb Released Question Bank - 2017

April 4, 2018 | Author: Basant Shams | Category: Medical Specialties, Mouth, Medicine, Clinical Medicine, Diseases And Disorders


Comments



Description

A patient experiences pain and some gingival The proposed mechanism by which a calciumswelling in the anterior segment of the hydroxide preparation initiates secondary mandible. The mandibular lateral incisor has a dentin formation in direct pulp cappings is by shallow restoration, is tender to percussion and gives a positive response to the electric pulp A. releasing calcium ions. tester. There is some mobility. The most likely B. stimulating differentiated ameloblasts to diagnosis is lay down dentin. C. stimulating fibroblasts to elaborate nuclei A. acute apical abscess. of the first order. B. acute serous pulpitis. D. stimulating undifferentiated cells of the C. lateral periodontal abscess. tissue to differentiate into odontoblasts. D. acute suppurative pulpitis. E. chronic ulcerative pulpitis. When resecting the apical aspect of a tooth during endodontic apical surgery, it is The placement of a retentive pin in the appropriate to proximal regions of posterior teeth would most likely result in periodontal ligament perforation A. perform a 45-degree bevel. in the B. remove as little of the root as possible. C. avoid curettage of the periradicular lesion. A. mesial of a mandibular first premolar. D. place an amalgam retrofill. B. distal of a mandibular first premolar. C. distal of a mandibular first molar. D. mesial of a mandibular first molar. For composite resin preparations, cavosurface enamel margins are bevelled because The air-water spray used as a coolant in high 1. a bevelled margin produces a more speed cutting of a cavity will favorable surface for etching. 2. a bevelled margin improves the edge 1. decrease pulp damage. strength of the composite resin. 2. reduce frictional heat. 3. after etching, the bonding agent reduces 3. keep the operating site clean. microleakage. 4. reduce clogging of cutting instruments. 4. the procedure eliminates the need to polish the restoration. A. (1) (2) (3) B. (1) and (3) A. (1) (2) (3) C. (2) and (4) B. (1) and (3) D. (4) only C. (2) and (4) E. All of the above. D. (4) only E. All of the above. Note: Some of the items in the Released Test Item Bank may have been discontinued due to outdated science or errors. In addition, the format of some items is not currently used. The NDEB periodically reviews the bank to improve its quality, content, and translation. ©The National Dental Examining Board of Canada 2017 When using a high-speed handpiece for cavity Fractured incisal angles in the permanent teeth preparation, the heat generated is directly of adolescent patients are best restored using related to the A. glass ionomer. 1. duration of cutting. B. gold castings. 2. size, speed and sharpness of the bur. C. full coverage restorations. 3. use of air and water spray. D. acid etch composite resin techniques. 4. effectiveness of the suction. A. (1) (2) (3) B. (1) and (3) What are the purposes of using occlusal C. (2) and (4) splints? D. (4) only E. All of the above. 1. To change the pattern and degree of tactile afferent neural impulses. 2. To immobilize teeth. 3. To prevent teeth from disturbing occlusal Which of the following instruments can be used sensory input. for placing gingival bevels on cast gold inlay 4. To produce a permanent change in the preparations? occlusion. 1. Margin trimmers. A. (1) (2) (3) 2. Enamel hatchets. B. (1) and (3) 3. Carbide finishing burs. C. (2) and (4) 4. Small diamond discs. D. (4) only E. All of the above. A. (1) (2) (3) B. (1) and (3) C. (2) and (4) D. (4) only Histologically, a pulp polyp consists of all of E. All of the above. the following EXCEPT for A. a mass of collagenous fibres. B. epithelial rests of Malassez. Which of the following is the most probable C. proliferating capillaries. postoperative complication of intracoronal D. fibroblasts. bleaching for a tooth that has an inadequate internal seal? A. Fracture. B. Discolouration. C. Internal resorption. D. External root resorption. Note: Some of the items in the Released Test Item Bank may have been discontinued due to outdated science or errors. In addition, the format of some items is not currently used. The NDEB periodically reviews the bank to improve its quality, content, and translation. ©The National Dental Examining Board of Canada 2017 Treatment of primary herpetic The primordial cyst probably results from gingivostomatitis should include A. cystic degeneration of the stellate 1. palliative treatment. reticulum early in the formation of the 2. steroid therapy. tooth. 3. control of secondary infection. B. epithelial remnants in the periodontal 4. application of dilute hydrogen peroxide. ligament. C. an extension of pulpal inflammation after A. (1) (2) (3) death of the pulp. B. (1) and (3) D. failure of formation of the enamel matrix. C. (2) and (4) E. transformation of the dental lamina. D. (4) only E. All of the above. Histological sections of a lesion removed from the apex of a carious tooth show immature Tooth mobility may be due to fibrous tissue and chronic inflammatory cells. The most likely diagnosis is a/an 1. excessive occlusal force. 2. decreased osseous support. A. acute periapical abscess. 3. periodontal abscess. B. odontogenic fibroma. 4. gingival inflammation. C. radicular cyst. D. periapical granuloma. A. (1) (2) (3) E. central fibroma. B. (1) and (3) C. (2) and (4) D. (4) only E. All of the above. Radiographs of Garre's osteomyelitis show A. formation of sequestra. B. a worm-eaten pattern of bone destruction. Lichen planus occurs most frequently on the C. thickening of the cortex. D. sinus tracts. A. buccal mucosa. B. tongue. C. floor of the mouth. D. gingiva. Increasing the kilovoltage setting on the dental x-ray machine results in A. more gamma radiation. Enlargement of the gingiva, described as B. greater collimation. idiopathic fibromatosis, is best described as C. more penetration. D. greater secondary radiation at the level of A. degeneration. the skin. B. inflammation. C. hyperplasia. D. neoplasia. Note: Some of the items in the Released Test Item Bank may have been discontinued due to outdated science or errors. In addition, the format of some items is not currently used. The NDEB periodically reviews the bank to improve its quality, content, and translation. ©The National Dental Examining Board of Canada 2017 Oral lesions that fail to heal may be associated Which of the following can cause a contact to stomatitis? 1. tuberculosis. A. Dentifrice. 2. syphilis. B. Lipstick. 3. neoplasia. C. Acrylic. 4. diabetes. D. Antibiotics. E. All of the above. A. (1) (2) (3) B. (1) and (3) C. (2) and (4) D. (4) only Fordyce's granules are E. All of the above. A. ectopic sebaceous glands. B. ectopic sweat glands. C. small calcified nodules. In dental radiology, patient protection from D. aberrant mucous glands. radiation is most important for A. patients receiving antibiotics. B. patients receiving corticosteroids. A large encapsulated fluid-filled tumour is C. individuals over fifty-years of age. removed from the hard palate. The most D. pregnant women. appropriate method of determining the nature E. young adults. of this lesion is to A. examine the fluid under a microscope. B. culture the fluid and examine for bacterial A salivary calculus is a growth. C. submit the tissue for histological A. sialolith. examination B. rhinolith. D. submit the tissue for exfoliative C. phlebolith. cytological study. D. thrombolith. E. aspirate the fluid for electrophoresis study. An abnormal decrease in the flow of saliva is Acute maxillary sinusitis is associated with A. ptyalism. B. sialometaplasia. 1. pain in the posterior maxillary teeth. C. xerostomia. 2. nasal discharge. D. pyroglossia. 3. tenderness of posterior maxillary teeth to percussion. 4. increase of pain when bending over. A. (1) (2) (3) B. (1) and (3) C. (2) and (4) D. (4) only E. All of the above. Note: Some of the items in the Released Test Item Bank may have been discontinued due to outdated science or errors. In addition, the format of some items is not currently used. The NDEB periodically reviews the bank to improve its quality, content, and translation. ©The National Dental Examining Board of Canada 2017 The collimator of an x-ray tube Chronic disseminated Langerhans cell disease A. produces a more homogeneous x-ray A. produces a solitary eosinophilic lesion. beam. B. produces bony defects as focal areas of B. prevents secondary radiation. bony rarefaction. C. focuses the x-ray beam. C. occurs only in adult life. D. restricts the diameter of the x-ray beam. D. is a malignant lesion. The quantity of radiation output in a dental X- Which of the following is/are associated with ray apparatus is a function of dentin dysplasia type I? 1. time. 1. Obliteration of pulp chambers. 2. kVp. 2. Normal appearance of clinical crowns. 3. ma. 3. Small underdeveloped roots. 4. filtration. 4. Periapical radiolucent areas. A. (1) (2) (3) A. (1) (2) (3) B. (1) and (3) B. (1) and (3) C. (2) and (4) C. (2) and (4) D. (4) only D. (4) only E. All of the above. E. All of the above. A zinc phosphate cement base Radiographically, a benign bone neoplasm is differentiated from a malignant one because in A. has the same radiopacity as amalgam. the benign lesion B. is less radiopaque than amalgam. C. has the same radiopacity as gold. 1. the margins are irregular and fade into the D. cannot be seen on a radiograph. surrounding bone. 2. the cortex remains intact. 3. there can be perforation of the periosteum. 4. the margins are defined and demarcated. Papillary hyperplasia under a denture is usually due to (an) A. (1) (2) (3) B. (1) and (3) A. moniliasis. C. (2) and (4) B. ill fitting denture. D. (4) only C. allergy to denture cleanser. E. All of the above. D. avitaminosis. Note: Some of the items in the Released Test Item Bank may have been discontinued due to outdated science or errors. In addition, the format of some items is not currently used. The NDEB periodically reviews the bank to improve its quality, content, and translation. ©The National Dental Examining Board of Canada 2017 On an occlusal radiograph, it is possible to A patient with hyperthyroidism may exhibit misdiagnose the midline palatal suture as a A. weight gain. A. fracture. B. delayed eruption of teeth. B. palatal cyst. C. exophthalmos. C. granuloma. D. gingival inflammation. D. abscess. In fibrous dysplasia Median anterior maxillary cysts occur in the A. foci of cartilage are a common A. nasal bone. histological finding. B. incisive canal and in the palatine process. B. an inflammatory infiltrate is C. zygomatic process. characteristically present. D. hamular process. C. there are characteristic changes in the blood chemistry. D. a ground-glass appearance is present on radiographs. Squamous cell carcinomas of the lip occur most frequently on the A. commissures. The most frequent location of a dentigerous B. lower lip near the midline. cyst is the C. inner surface of upper lip. D. inner surface of lower lip. A. third molar area. E. upper lip near the midline. B. symphysis of the mandible. C. midline of the hard palate. D. apical area of a devitalized tooth. E. premolar area. The clinical appearance and texture of an early carcinoma of the floor of the mouth can be A. red and soft. The prolonged use of antibacterial lozenges or B. white and rough. mouthwashes contributes to the development of C. ulcerated and indurated. D. All of the above. A. oral candidiasis. B. geographic tongue. C. cancrum oris. D. Koplik's spots. Oral foci of infection are of greatest clinical E. aphthous ulcers. significance in the presence of A. polycythemia vera. B. iritis and uveitis. Ludwig's angina may cause C. eczema and urticaria. D. rheumatoid arthritis. A. respiratory obstruction. E. subacute bacterial endocarditis. B. cavernous sinus thrombosis. C. suppurative encephalitis. D. subdural empyema. Note: Some of the items in the Released Test Item Bank may have been discontinued due to outdated science or errors. In addition, the format of some items is not currently used. The NDEB periodically reviews the bank to improve its quality, content, and translation. ©The National Dental Examining Board of Canada 2017 Tissue from a multilocular radiolucent area of White lesions of the oral mucosa may be the posterior mandible microscopically shows produced by follicular areas lined with cylindrical cells resembling the enamel organ. The most likely 1. thickening of the epithelium. diagnosis is a/an 2. increase of the keratinized layers. 3. coagulation by heat or chemicals. A. neurofibroma. 4. mycotic infection. B. ameloblastoma. C. central fibroma. A. (1) (2) (3) D. periodontal cyst. B. (1) and (3) E. dentigerous cyst. C. (2) and (4) D. (4) only E. All of the above. Intraoral soft tissue examination will NOT assist in the diagnosis of Pyogenic granuloma is most frequently found A. lichen planus. on the B. sinusitis. C. erythema multiforme. A. tongue. D. anemia. B. gingiva. E. vitamin deficiencies. C. buccal mucosa. D. tonsillar pillars. E. lips. Which of the following conditions are associated with AIDS? Which of the following is NOT associated with 1. Acute marginal periodontitis. osteogenesis imperfecta? 2. Hairy leukoplakia. 3. Candidiasis. A. Brown teeth. 4. Geographic tongue. B. Brittle bones. C. Thin enamel. A. (1) and (2) D. Blue sclerae. B. (1) (2) (3) C. (1) and (4) D. All of the above. Inflammation involving the bone marrow of the jaw caused by infection from a tooth or the periodontium is called Which of the following lesions has a tendency to bleed easily? A. osteoma. B. periostitis. A. Pyogenic granuloma. C. osteomyelitis. B. Osteoma. D. osteosclerosis. C. Fibroma. E. pericementitis. D. Papilloma. E. Lipoma. Note: Some of the items in the Released Test Item Bank may have been discontinued due to outdated science or errors. In addition, the format of some items is not currently used. The NDEB periodically reviews the bank to improve its quality, content, and translation. ©The National Dental Examining Board of Canada 2017 Which of the following anatomic spaces is Chronically inflamed submandibular lymph most likely to be involved as a result of an nodes are apical infection of a mandibular third molar? A. soft. A. Sublingual. B. not palpable. B. Submandibular. C. firm. C. Submental. D. fixed. D. Submasseteric. Oral leukoplakia has the most favourable Laboratory examination of the blood of a prognosis when it is patient with an acute bacterial infection would show A. present in a non-smoker. B. accompanied by pain. A. lymphocytosis. C. infected with Candida albicans. B. leukocytosis. D. speckled in appearance. C. monocytosis. E. on the hard palate. D. leukopenia. E. eosinophilia. Median anterior maxillary cysts are found in Oral lichen planus has lesions which A. the zygomatic process of the maxilla. B. the incisive canal. A. bleed readily. C. the uvula. B. occur in the debilitated. D. the hamular process. C. exhibit a positive Nikolsky's sign. D. histopathologically show lymphocytic infiltration. Soft, white, elevated plaques of the oral mucosa are characteristic of Swelling related to increased tissue fluid is A. angioma. called B. candidiasis. C. actinomycosis. A. thrombosis. D. herpes simplex. B. edema. E. submucous fibrosis. C. hematoma. D. embolism. E. surgical emphysema. The redness of an inflammatory lesion of oral mucosa is due to A. increased number of capillaries. B. increased size of capillaries. C. decreased thickness of epithelium. D. decreased connective tissue elements. E. All of the above. Note: Some of the items in the Released Test Item Bank may have been discontinued due to outdated science or errors. In addition, the format of some items is not currently used. The NDEB periodically reviews the bank to improve its quality, content, and translation. ©The National Dental Examining Board of Canada 2017 Which of the following most appropriately Which of the following cements can chemically describes a traumatic neuroma? bond to enamel? A. A slow-growing, painless neoplasm. 1. Zinc phosphate cement. B. A slow-growing, hypersensitive nodule. 2. Polycarboxylate cement. C. A tumour at a recent extraction site. 3. Reinforced zinc oxide eugenol cement. D. A tumour of the tongue. 4. Glass ionomer cement. A. (1) (2) (3) B. (1) and (3) An occluded submandibular duct can be C. (2) and (4) diagnosed by D. (4) only E. All of the above. A. history. B. palpation. C. sialography. D. occlusal radiographs. Early signs and symptoms of localized alveolar E. All of the above. osteitis (dry socket) include 1. bleeding. 2. bad odour. The typical history of a mucocele is 3. pus formation. 4. pain. A. a slowly growing tumour mass. B. a pain immediately before eating. A. (1) (2) (3) C. a trauma, swelling, rupture, disappearance B. (1) and (3) and recurrence. C. (2) and (4) D. an ulcerated area on buccal mucosa. D. (4) only E. frequent bleeding. E. All of the above. Median palatine cysts are classified as Immediately after the extraction of a tooth, postoperative instructions should include A. developmental. B. residual. 1. analgesics. C. idiopathic. 2. regular diet. D. odontogenic. 3. application of cold. 4. frequent rinsing of the socket. A. (1) (2) (3) Lidocaine (Xylocaine) is an example of a local B. (1) and (3) anesthetic which is chemically classified as an C. (2) and (4) D. (4) only A. amide. E. All of the above. B. ester. C. aldehyde. D. ethamine. E. aminide. Note: Some of the items in the Released Test Item Bank may have been discontinued due to outdated science or errors. In addition, the format of some items is not currently used. The NDEB periodically reviews the bank to improve its quality, content, and translation. ©The National Dental Examining Board of Canada 2017 D. vestibular disturbances. (2) and (4) is D. B. B. Incision at fracture site. Extraction of a tooth is CONTRAINDICATED E. thiopental. 1. (2) and (4) 1. gastrointestinal symptoms (when A. 5%. during administration of D. 3. C. removal of a torus mandibularis. 4. (1) and (3) C. Cardiac arrhythmias are most commonly seen C. 80%. a physical examination. 4. Immobilization of fracture. ©The National Dental Examining Board of Canada 2017 . the format of some items is not currently used. D. is allergic to nitrous oxide. All of the following are side effect of D. photosensitivity. molar tooth. halothane. 3. had a myocardial infarct six months ago. and translation. the B. A. 10%. an oral examination. in the dental office for a patient who A. is nervous. B. Rigid fixation of fragments. (1) and (3) MINIMALLY acceptable percentage of oxygen C. E. B. 2. The NDEB periodically reviews the bank to improve its quality. Preoperative evaluation of a healthy patient requiring elective oral surgery in hospital A. content. administration is an indication that the patient D. has not eaten for some time. 50%. D.Which of the following is a principle of closed Loss of sensibilit over the distribution of the fracture management? inferior dental nerve is a possible complication from A. administered orally). A. C. All of the above. (4) only 2. (4) only E. removal of an impacted mandibular third C. has received the nitrous oxide too quickly. Debridement of fracture site. A. E. an uncomplicated removal of a mandibular second molar. Patient nausea during nitrous oxide C. (1) (2) (3) In a safe general anesthetic mixture. is hypothyroid. appropriate laboratory tests. 20%. nitrous oxide. is 4 months pregnant. ethyl ether. All of the above. In addition. prolonged tetracycline therapy EXCEPT A. an acute osteomyelitis of the mandible. B. superinfection. C. Note: Some of the items in the Released Test Item Bank may have been discontinued due to outdated science or errors. has a Factor~VIII deficiency. (1) (2) (3) should include B. B. discoloration of newly forming teeth. a complete history. D. C. Tous les énoncés ci-dessus. inhibits growth of intestinal bacteria. Opioids predispose to more bleeding. and translation. It is the drug of choice as an antidote in Protracted use of tetracycline may produce atropine poisoning. (4) only C. (1) (2) (3) B. E. D. ©The National Dental Examining Board of Canada 2017 . A. (1) and (3) A. A. It is the neurotransmitter at both B. Desflurane. pressure. pain. stimulate medullary centers. 1. When used alone. combines chemically with vitamin K. C. Which of the following can be mistaken on a radiograph for a chronic alveolar abscess? When comparing opioids with NSAIDS which of the following is correct? 1. lidocaine. C. Nitrous oxide. the format of some items is not currently used. NSAIDS cause more constipation. decrease excitability of the motor cortex. D. In addition. B. 3. 4. produce a state of unconsciousness. produce delirium. aminophylline. Note: Some of the items in the Released Test Item Bank may have been discontinued due to outdated science or errors. (1) et (3) C. Sevoflurane. C. touch. (2) and (4) B. content. sympathetic and parasympathetic ganglia. digitalis. The NDEB periodically reviews the bank to improve its quality. Opioids impair gastric motility to a greater 3. Posterior palatine foramen. procainamide. (4) seulement E. It is rapidly hydrolysed in the body by D. extent. (2) et (4) D. (1)(2)(3) D. symptoms of vitamin K deficiency because tetracycline A. which of the following agents will not produce satisfactory anesthesia? All of the following drugs are useful in the treatment of cardiac arrhythmias EXCEPT A. B. D. B. D. reduce perception of painful stimuli. Periapical cemento-osseous dysplasia. General anesthetics can do all of the following EXCEPT A. cholinesterase. NSAIDS cause more drowsiness. E. Isoflurane. C. is detoxified by the liver. C. 4. All of the above. D. A. B. interferes with the conversion of prothrombin to thrombin. 2. 2. It can produce both muscarinic and nicotinic actions. B. Mental foramen. proprioception.The first sensation lost after administration of a Which of the following statements is/are true local anesthetic is regarding acetylcholine? A. Nasopalatine foramen. A. D. Medical history. All of the above. 3. acrylic template to cover socket opening The mode of action of the sulfonamides upon and saline rinses. C. A. 4. D. B. (1) (2) (3) B. antrostomy for retrieval of root. (2) and (4) Which of the following is the most important D. content. 4. branchospasm. (1) and (3) protein. A. C. cyanosis. competing for nutrients in the tissue environment of the microorganisms. Immediate toxic reactions to local anesthetic administration are most commonly due to Early anoxia is characterized by A. agent. the format of some items is not currently used. (4) only factor in the preoperative evaluation of a E. C. Therapy should consist of 1. interfering with the synthesis of folic acid. The NDEB periodically reviews the bank to improve its quality. has anticonvulsant properties. B. A. inhibiting the biosynthesis of 3. amnesia. 2. Laboratory data. An infected root is accidentally displaced into C. tooth in line of fracture. C. (1) and (3) C. the maxillary sinus. Note: Some of the items in the Released Test Item Bank may have been discontinued due to outdated science or errors. A. reveals perforation of the sinus lining. Electrocardiogram. In addition. B. direction of blow. susceptible bacteria is by 2. p-aminobenzoic acid. All of the above. hypersensitivity to the vasoconstrictor. Examination of the socket D. D. proprioceptor nerve action. antibiotic coverage and observation. E. ©The National Dental Examining Board of Canada 2017 . excessive blood level of the anesthetic D. and translation. (2) and (4) D. (4) only. B. has topical anesthetic properties. Pulse and blood pressure. deterioration of the anesthetic agent. TMJ mobility. patient? A. hypersensitivity to the anesthetic agent. interfering with the synthesis of cell wall B. is a local anesthetic agent. (1) (2) (3) C. closure of oro-antral communication and antibiotic coverage.Displacement of mandibular fractures is Lidocaine dependent upon 1. is an antiarrhythmic agent. bradycardia. B. 3. 3. ampicillin. laceration of the periodontal membrane. 2. 1. 4. ankylosis. Use of nitrous oxide analgesia produces 4. distal drift of the adjacent premolar. B. D. increase in overbite. (1) and (3) If a patient is allergic to penicillin. 4. Tooth 1. ©The National Dental Examining Board of Canada 2017 . encroaching upon the space for tooth 1. sulcus depth. B. (1) (2) (3) A. All of the above. C. D. A. accelerated healing. contour. a transient vasoconstriction. C. loss of pulp vitality. C. the intrusion of a permanent D. (1) and (3) C. (4) only infection would be E. streptomycin. and translation. tinnitus as a result of A. positive chemotaxis. without encroaching upon the space for tooth 1. All of the above. (4) only D. A. 2.2 will D. mesial drift of the adjacent molar. central nervous system reaction. the most C. medullary stimulation.1. (2) and (4) An 8 year old has lost tooth 1. cephalexin. declomycin. root resorption. In a 7 year old.In achieving hemostasis. The NDEB periodically reviews the bank to improve its quality. (2) and (4) C. myocardial depression. external cold The normal gingiva of the child patient is application produces diagnosed on the basis of A. A. histamine release. Loss of a permanent maxillary first molar may Cardiovascular collapse caused by a high result in circulating dose of a local anesthetic is due to 1. the format of some items is not currently used. B. 3. clindamycin. D. 2. A.2. 1. (2) and (4) appropriate antibiotic for an odontogenic D. B. E. content. A. (1) and (3) B. increased vascular permeability. (1) (2) (3) B. vagal stimulation. (1) (2) (3) B.1. C. overeruption of the opposing tooth. tight fitting gingival collar. (4) only most likely erupt E. C. dysphoria. B. increased pressure in the middle ear. stippling. Note: Some of the items in the Released Test Item Bank may have been discontinued due to outdated science or errors. E. In addition. cochlea effect.1. All of the above. peripheral action on the eardrum. distally to the space for tooth 1. central incisor can cause E. (1) and (3) C. All of the above. Poor oral hygiene during orthodontic treatment Which of the following processes is NOT may result in active in causing tooth eruption? 1. 6 to 12 years. could constitute a problem requiring consultation with an orthodontist. 1 to 5 years. prevent the eruption of the permanent B. Pressure from periapical tissue. B. D. Note: Some of the items in the Released Test Item Bank may have been discontinued due to outdated science or errors. E. (2) and (4) C. (2) and (4) Following the premature loss of the deciduous D. loss of stippling. maintain arch length. (4) only molars. accurately determined using the A. has a keratinized surface. 13 to 16 years. (1) (2) (3) B. bleeding. and translation. displays varying degrees of stippling. A. In an 11 year old with an otherwise acceptable 3. occlusion. 3. the Angle classification is most E. C. gingival desquamation. A. could be surgically exposed to speed its D. All of the above. teeth. facial profile. The NDEB periodically reviews the bank to improve its quality. Growth of the dentin. C. 4. D. is attached by collagen fibres to the cementum. an impacted maxillary canine 4. ©The National Dental Examining Board of Canada 2017 . E. Any age. B. In addition. 2. D. (1) and (3) removed to allow the canine to erupt. A. could be extracted. could be retained and the first premolar B. A. content. prevent extrusion of opposing teeth. the format of some items is not currently used. permanent molars. A. 1. is bound firmly by the periosteum to the alveolar bone. In a healthy periodontium. D. 2. C. Growth of the enamel. permanent canines. attached gingiva C. Growth of the root. D. All of the above.A space maintainer in the posterior segment Primary herpetic lesions of the oral cavity are will most likely to occur during A. (4) only eruption. permanent incisors. (1) (2) (3) B. B. retard eruption of the permanent teeth. edema. C. Deep overbite with minimal overjet. (2) and (4) A. 4. proliferative and degenerative changes of regions are broader than in the anterior the junctional epithelium. C. In addition. is excreted rapidly by the kidney. 3. oral hygiene.Which one of the following statements is Histopathologic alterations associated with the FALSE? pathogenesis of periodontal disease include A. All of the above. periodontal disease. a secondary occlusal trauma. is deposited in bone. All of the above. E. A. E. 3. (1) and (3) following findings is/are potentially damaging? C. cells. junctional epithelium. a primary occlusal trauma. (4) only 1. (2) and (4) D. (1) and (3) C. Extruded teeth. passes the placental barrier. A. 3. collagen destruction subjacent to the D. neutrophils. (4) only B. (4) only E. (1) (2) (3) The Silness-Löe Index measures B. A. inflammatory exudate that can involve location. 4. the format of some items is not currently used. 4. Note: Some of the items in the Released Test Item Bank may have been discontinued due to outdated science or errors. Marginal ridge discrepancies. Healthy gingiva does not bleed. (1) and (3) C. lymphocytes and plasma B. which of the B. Healthy gingiva is bright red in color. B. 2. ©The National Dental Examining Board of Canada 2017 . is deposited in teeth. Mobility of teeth WITHOUT loss of bone support suggests The fluoride ion A. C. C. 1. an atrophic condition of the periodontium. All of the above. (2) and (4) D. attachment level. Wide occlusal tables resulting from excessive wear. The NDEB periodically reviews the bank to improve its quality. region. 2. D. and translation. content. The interdental papillae in the posterior 2. (1) (2) (3) When analyzing occlusion. deepening of the gingival sulcus. (1) (2) (3) B. probing depth. D. Stippling in healthy gingiva varies with 1. (1) and (3) C. If a periodontal probe is inserted 4 or 5mm to the base of a pocket on the mesialbuccal of a tooth and then pushed facially causing Gingival hyperplasia may be blanching. content. connective tissue vascularity. B. consist of bone. transeptal. horizontal. A protective bite plate is indicated in all of the In the development of gingivitis. All of the above. (1) and (3) C. 2. C.Which of the following factors may affect Supragingival calculus is most often found on probing depth measurements? the 1. alveolar bone loss. None of the above. 4. (2) and (4) A characteristic of a periodontal pocket is D. D. buccal of mandibular anterior teeth. B. In addition. C. Probe type. there is an inadequate zone of attached C. C. free gingival. familial. the degree of keratinization. gingival hyperplasia. this indicates that A. A. manage temporomandibular joint pain D. All of the above. gingival edema. ©The National Dental Examining Board of Canada 2017 . prevent migration of teeth. Note: Some of the items in the Released Test Item Bank may have been discontinued due to outdated science or errors. palatal of maxillary molars. oblique. reduce tooth mobility. prevent excessive tooth wear. The NDEB periodically reviews the bank to improve its quality. All of the above. 4. (4) only E. B. the fibre following EXCEPT to groups first lost are A. A. B. gingival hyperplasia exists. the lateral wall of the pocket does not E. B. C. (4) only E. and translation. A. 2. A. Angulation of probing. Periodontal health. (2) and (4) D. (1) (2) (3) B. amount of melanin pigmentation. Probing force. idiopathic. A. E. 3. B. gingiva. dysfunction syndrome. subgingival deposits. D. A. C. the format of some items is not currently used. lingual of mandibular molars. D. The color of gingiva is influenced by 1. drug induced. control a bruxism habit. 3. (1) (2) (3) B. lingual of mandibular anterior teeth. C. A. interproximal areas. and B. 4. palatal surface of maxillary anterior teeth. A. locate tooth surfaces that can act as guiding planes. The NDEB periodically reviews the bank to improve its quality. the amount of gingival shrinkage depends upon A. between l0 and 20 years of age. direct access for thorough debridement. Following root planing. (1) (2) (3) C. orthodontics. 2. All of the above. buccal and lingual surfaces of molars. In the design of a removable partial denture. A. All of the above. All of the above. (4) only E. whether the pocket orifice is broad or narrow. further oxidation. the degree of suppuration present. (1) and (3) range of the solder. the degree of gingival edema present. decrease the setting time. (4) only melting range. B. D. B. High humidity in a room where zinc oxide and B. E. (2) and (4) D. determine the path of insertion. (1) (2) (3) E. All of the above. dissolve surface oxides and prevent 3. 3. pocket reduction. selective grinding. between 20 and 30 years of age. B. Destructive occlusal forces can be reduced by The main purpose of flux in soldering is to 1. the thickness of the gingiva. (2) and (4) D. (1) (2) (3) E. ©The National Dental Examining Board of Canada 2017 . increased opportunity for new attachment. A. prevent recrystallization and grain growth. 4. (2) and (4) D. (1) and (3) C. bifurcations and trifurcations. eugenol impression paste is being mixed will C. not affect the setting. prevent oxidation and lower the melting B. A. locate the height of contour. and translation. B. content. D. 3. frequently occurs 2. the format of some items is not currently used. All of the above. C. before age 10. A. (4) only E. 4. tooth extraction. In addition. D. Note: Some of the items in the Released Test Item Bank may have been discontinued due to outdated science or errors. A. dissolve surface oxides and lower the D. E. B.Infraosseous defects may occur at the The benefits of open flap debridement alone include 1. C. D. A. 2. increase the setting time. at any age. locate retention areas. C. (1) and (3) C. B. restorative treatment. after age 30. prevent any setting. the objectives of surveying the diagnostic cast are to Primary herpetic gingivostomatitis most 1. (4) only The major disadvantage of zinc phosphate used E. magnesium oxide. to cement crowns is the A. sulfate for 60 minutes. avoid burning the soft tissues. (1) (2) (3) B. C. "tempered" in warm water before placement in D. lack of edge strength. (1) and (3) B. restore tooth function. reduce contraction error. 2% potassium sulphate for 2 to 5 minutes will A. 1. D. A. Note: Some of the items in the Released Test Item Bank may have been discontinued due to outdated science or errors. initiate a chemical reaction. teeth on the working side make contact in lateral excursions. assess facial contour and lip support. 2. (2) and (4) C. 4. ©The National Dental Examining Board of Canada 2017 . D. Storage in air. zinc-oxide. reversible hydrocolloid impression material? B. 4.While the teeth are set in wax. accelerate the set of the stone. retard the set of the stone. evaluate esthetics. minimize the distortion of the hydrocolloid material during the storage time. B. zinc phosphate. All of the above. A. minimize distortion. C. reduce thermal conductivity. lateral excurison. "Cuspid guided occlusion" occurs when the A. The NDEB periodically reviews the bank to improve its quality. silica. A. the format of some items is not currently used. E. D. B. Storage in 100% humidity for 30 minutes. be biologically acceptable. calcium hydroxide. pulp irritation. A fixed bridge pontic should C. the mouth in order to A. Storage in 2% solution of potassium excursions on the working side. 3. (4) only stone. content. development of heat during setting. inhibit the formation of bubbles in the D. dentures are The principal ingredient of a zinc phosphate tried in to cement powder is 1. canine and lateral incisors make contact in B. E. B. posterior teeth make no contact in lateral D. 2. reduce galvanic reactions between The immersion of a hydrocolloid impression in abutments and other restorations. Heated impression modeling compound is C. Slow removal from undercuts. low crushing strength. B. In addition. and translation. 3. (1) and (3) C. (1) (2) (3) A. (2) and (4) D. C. verify the vertical dimension of occlusion. C. teeth on the nonworking side make Which of the following prevents distortion of a contact in lateral excursions. verify the maxillomandibular records. D. All of the above. horizontal condylar inclination. C. primary retention. All of the above. Wrought gold alloy clasps are superior to cast polycarboxylate cement has gold clasps of the same cross-sectional area because they A. A. D. 3. (1) (2) (3) of the abutment tooth most often altered to B. content. A. ©The National Dental Examining Board of Canada 2017 . C. concentration of the fillers and E. 2. are coarser grained. A. B. C. the format of some items is not currently used. superior biologic compatibility. have a higher proportional limit. distal. E. relationship of the maxilla to the hinge axis. vertical dimension of occlusion. B. the refractory cast. lingual. D. 1. longer working time. (2) and (4) D. lateral force transmission. intercondylar distance. (1) (2) (3) Reversible hydrocolloids transform from sol to B. B. buccal. (1) and (3) provide clasp reciprocity is C. preparation of occlusal rest areas. In removable partial denture design. a surveyed diagnostic cast prior to 1. C. E. In designing a removable partial denture. 3. mylohyoid. temperature. Compared with zinc-phosphate cement. All of the above. B. percent composition by weight of water. the surface A. genioglossus. 2. plasticizers. (2) and (4) D. mesial. B. The NDEB periodically reviews the bank to improve its quality. B. the location of clasps is determined on A. (4) only A. 4. increased compressive strength. In addition. concentration of potassium sulfate. a master cast after tooth preparations are A facebow is used to record the completed. occlusal. C. D. are more accurately adapted to the tooth. 4. and translation. indirect retention. vertical stability. C.The muscle of the floor of the mouth in the Rests on terminal abutment teeth for a cast molar region which requires special attention in metal removable partial denture provides the final mandibular denture impression is the A. (1) and (3) gel and gel to sol as a function of the C. D. hyoglossus. exhibit greater flexibility D. lower film thickness. geniohyoid. (4) only. Note: Some of the items in the Released Test Item Bank may have been discontinued due to outdated science or errors. by the rest. are circular in cross-section. All of the above. removable partial denture frameworks can be made A. D. All of the above. D. 3. determined from radiographs. inhibit polymerization. C. surface. the enamel is thicker on the lingual D. base metal alloy 4. in the anterior segment of the arch. external to it. flasking and processing procedures. an important factor in abutment tooth selection. 2. and translation. more ductile. (2) and (4) 2. it placed on the lingual surface of a canine rather will than on the incisal surface because A. C. 4. complete dentures on the original stone casts are rearticulated in order to correct occlusal disharmony produced by A. lighter. determined during surveying of the I-bar clasp arms and circumferential clasp arms diagnostic cast. the comparison of the length of root A. Note: Some of the items in the Released Test Item Bank may have been discontinued due to outdated science or errors.Unsupported. the format of some items is not currently used. (4) only C. A removable partial denture rest should be If hydroquinone is added to the monomer. inaccurate jaw relation records. visibility and access are better. activate polymerization. C. B. with greater casting accuracy. (4) only approach the tooth undercut area from a E. (1) (2) (3) 1. errors in registering of centric relation record. (1) (2) (3) B. E. None of the above. A. After processing. retained in bone to the amount of tooth B. (1) and (3) A. 3. (1) and (3) gingival to the height of contour. D. The NDEB periodically reviews the bank to improve its quality. B. less leverage is exerted against the tooth B. near the tuberosities. C. C. C. more corrosion resistant. originate from the framework and D. gingival direction. provide retention by the resistance of metal to deformation rather than frictional resistance of parallel walls. hyperplastic tissue in an The crown-root ratio is edentulous maxilla is most often found 1. terminate in retentive undercuts lying B. the cingulum of the canine provides a natural recess. initiate polymerization. content. In comparison to gold alloys. (2) and (4) B. ©The National Dental Examining Board of Canada 2017 . In addition. in the vault. both A. Setting not affected by a well controlled A. ©The National Dental Examining Board of Canada 2017 . Dental amalgam A. phosphorus pentoxide. (2) and (4) silicones. Bonding to enamel. Polysulfide impression materials 2. 4. C. B. 1. lead peroxide. has a satisfactory resistance to compressive stress. after polymerization. 3. content. placement of chemical-impregnated cords D. carbon disulfide. E. (1) and (3) condensation and addition polymerization C. B. Release of fluoride. the format of some items is not currently used. hydrogen peroxide. demonstrate. (1) (2) (3) C. C. sodium peroxide. Which of the following are characteristics of restorative glass ionomer cements? 1. electrosurgical means. C. In addition.Dental polysulfide rubber impression materials Effective tissue displacement with elastic are polymerized with the following initiator: impression materials can be accomplished by A. The NDEB periodically reviews the bank to improve its quality. is inversely proportional to temperature and humidity. into the gingival sulcus. (1) and (3) rubber impression materials C. (1) (2) (3) The accuracy of the polysulfide and silicone B. Irritating to pulpal tissues. injection of the material into the gingival B. 2. (2) and (4) D. 3. have a better detail reproduction than both B. All of the above. linking chains. All of the above. is subject to solubility and disintegration in the oral fluids. and translation. have a better dimensional stability than addition cured silicones. condensation silicone. sulcus. B. Note: Some of the items in the Released Test Item Bank may have been discontinued due to outdated science or errors. (4) only D. adheres chemically to the walls of the cavity. cross. a firm tray material. compares favorably with reversible hydrocolloids. 4. A. have a lower "tear strength" than moist environment. is better than that of polyethers. D. E. E. A. (4) only A. A. A. ©The National Dental Examining Board of Canada 2017 . content. All of the above. retard the final set. release of internal stress. The retention of an indirect. 3. (4) only E. strength. the format of some items is not currently used. extra-coronal 2. No change in compressive strength but lower tensile strength. All of the above. demonstrate less creep or flow. B. glass ionomer cements B. C. B. (4) only placed on abutment teeth? E. D. as possible in order to decrease distortion caused by A. A. (2) and (4) 4. corrosion. (1) (2) (3) 2. B. (1) (2) (3) B. Resorption of bone. protect the cement from moisture. After initial setting. continued expansion of the wax. Decrease in width of periodontal containing amalgam is contaminated with ligament. C. (1) and (3) 3. and translation. drying-out of the wax. should have a coating agent applied in order to C. E. A. saliva during condensing? 1. (2) and (4) D. restoration can be improved by 3. C. produces less stress on the abutment tooth 2. create a smooth finish. D. D. has a lower yield strength. demonstrate less marginal breakdown in C. Which of the following would occur if a zinc D. Increased surface pitting. 1. Increase thickness of cementum. (1) and (3) Which of the following is NOT a direct C. All of the above. due to a high modulus of elasticity. hasten the final set. (1) and (3) C. is economical to fabricate. generally have lower compressive D. decreasing the taper of the preparation wall. B. Increased expansion.Dental amalgams that are made from alloys The retentive arm of a combination clasp containing 6% copper. Reduced flow or creep. protect the cement from ultraviolet light. (1) (2) (3) B. The NDEB periodically reviews the bank to improve its quality. In addition. (2) and (4) physiological response to additional forces D. can be used to engage deeper undercuts clinical service. 4. (4) only Inlay wax patterns should be invested as soon E. 4. during removal and insertion. A. an antirotation key. adding grooves. Increased density in cribiform plate. compared to those made (wrought wire retentive arm and cast lingual from alloys containing 13% copper arm) is better than a cast arm because it 1. are more resistant to tarnish and A. reduced flow. lengthening the clinical crown. Note: Some of the items in the Released Test Item Bank may have been discontinued due to outdated science or errors. D. A. Eosinophilic granulomas A. The most appropriate diagnosis is A. hemangioma. B. numerous unerupted supernumerary teeth. cause the patient to sweat. Place a pilot hole at the dentino-enamel B. C. can induce 8th cranial nerve damage. (2) and (4) C. D. Determine the subgingival anatomic 4. (1) and (3) junction. (1) (2) (3) 4. 4. Epinephrine is one drug used in the Osteogenesis imperfecta is manifested by management of an acute anaphylactic reaction (Type I allergic reaction) because it A. punched-out radiolucencies in the jaws. (2) and (4) D. enhances the activity of some preparation? neuromuscular blocking agents. (1) (2) (3) A. The NDEB periodically reviews the bank to improve its quality. the format of some items is not currently used. C. (4) only D. 1. All of the above. pleomorphic adenoma. 2. In addition. E. produce an indirect sympathomimetic examination reveals a stratified squamous action. E. is used to treat tuberculosis. 3. and translation. granular cell tumour (myoblastoma). C. protruding lesion depression. are a nonlipid reticulo-endotheliosis. stimulates heart muscle and increases heart D. ©The National Dental Examining Board of Canada 2017 . (2) and (4) D. (4) only E. (1) (2) (3) A. osteoporosis and anemia. All of the above. (1) and (3) C. 3. All of the above. B. multiple fractures and blue sclera. are caused by a virus. Microscopic 2. (1) and (3) B. vascular spaces. nitrous oxide may 1. occur in infants. rate. increases systolic blood pressure. C. epithelium covering loose. (1) and (2) B. (4) only D. produce signs of inherent myocardial A patient has a smooth. relaxes bronchial muscles. E. 2. 3. contours. red. B. 2. (4) only A. Remove caries and unsupported enamel. 4. Note: Some of the items in the Released Test Item Bank may have been discontinued due to outdated science or errors. papilloma. (2) and (4) C.Streptomycin Which of the following should be done prior to pin hole placement in an extensive amalgam 1. When used for conscious sedation. fibrous connective 3. 1. produce numbness of the extremities. at the tip of the tongue. Examine the radiograph. B. is a broad-spectrum antibiotic. All of the above. fibroma. (1) (2) (3) E. content. tissue with many thin-walled. produces vasoconstriction in many vascular beds. are found at the apices of nonvital teeth. 3. Central incisors. B. melting the wax onto the die. (1) (2) (3) B. burning sensation in the premolar region. (1) (2) (3) satisfactory intraoral radiograph can be B. C. (1) and (3) produced with skin exposures as low as C. (2) and (4) D. D.5R. fibres of the buccinator muscle. 1. If a complete mandibular denture causes a 2. (4) only A.000R. this is 3. D. (4) only D. (2) and (4) C. complains of a burning sensation in the palate. mental branch of the inferior alveolar E. B. A. Note: Some of the items in the Released Test Item Bank may have been discontinued due to outdated science or errors. B. C. C. When making an indirect wax pattern on a stone die. E. First premolars. a coping and telescopic crown are used on C. All of the above. post dam. A. A. D. underlying bone. The NDEB periodically reviews the bank to improve its quality. content. frenum. incisive foramen. The most likely area requiring relief is at the C. the use of a separating medium. due to the denture exerting pressure in/on the 4. First molars. the abutment. Which of the following maxillary teeth are more likely to present with a palatal abscess? 1.10mR. a non-rigid connector is placed in the E. birth. and translation.A tilted molar can be used as a fixed partial Permanent first molars begin to calcify at denture abutment if A. a A. ©The National Dental Examining Board of Canada 2017 . fixed partial denture. Using current radiographic techniques. buccal frenum area.1. 12 to 15 months. improved adaptation will result from A patient with new complete dentures A. soaking the die in cold water. lingual branch of the mandibular division B. 150 . 4. 1 . All of the above. 100 . its long axis is within 25° of the long axis of the other abutments. In addition. it undergoes orthodontic uprighting first. E. Lateral incisors. B. nerve. 1 to 4 months in utero. 7 to 11 months.600mR. 1 . D. (1) and (3) of the trigeminal nerve. 2. D. 3 to 6 months. the format of some items is not currently used. burnishing the wax to the die. foveae palatinae. Apical. a functional heart murmur. cortical bone. B. Cephalosporins. distal and mesial interdental gingiva. Mesial. B. A. occlusal interference. sweating. Urticaria. In addition. D. (4) only Inefficient progress in permanent molar E. ©The National Dental Examining Board of Canada 2017 . (1) (2) (3) B. Note: Some of the items in the Released Test Item Bank may have been discontinued due to outdated science or errors. adverse effect of antibiotic therapy? D. the pathway of patients with inflammation is A. Which of the following is the most severe C. Which of the following should NOT be D. Angioneurotic edema. through the periodontal ligament. chronic periodontitis. prescribed to a pregnant patient? A. A. Hypoglycemia in a diabetic patient can be characterized by The interdental gingival col is the area between 1. mental confusion B. C. D. 3. A. 4. C. palpitations. A. D. through the epithelial attachment. Erythromycin. B. Rotational. an organic heart murmur. interseptal bone. Which of the following basic forcep D. B. the format of some items is not currently used. B. mitral valve prolapse. process. Anaphylactic reaction. A. B. The NDEB periodically reviews the bank to improve its quality. All of the above. C. bundle bone.Prophylactic antibiotic therapy is indicated for In horizontal alveolar bone loss. (2) and (4) D. facial and lingual attached gingiva. perivascularly. Cloxacillin. cancellous bone. facial and lingual interdental gingiva. Tetracycline. and translation. uprighting is most likely due to A. content. movements is NOT used for extracting teeth? E. the 2. Infrabony defects occur most frequently in C. Flatulence. into the marrow spaces of the crestal bone. nausea. an osseointegrated dental implant. Diarrhea. (1) and (3) C. A. through the cortical bone of the alveolar D. lack of anchorage. C. C. B. Lingual (palatal). C. heavy force application. osteogenic. D. 3. develops from gingivitis. A. Patients with occlusal parafunctional habits may have A. goes through stages of tissue destruction B. (1) (2) (3) B. 3. results in occlusal traumatism. 4. used to control bleeding. mandibular prognathism. visual examination. (1) and (3) A. A. C. soreness in masticatory muscles. and quiescence. occlusal wear facets. Note: Some of the items in the Released Test Item Bank may have been discontinued due to outdated science or errors. (2) and (4) Sutures can be D. C. The condyle of the mandible is unique because A. dysplasia of the anterior cranial base. resorbable or non-resorbable. used to stretch soft tissues into position. The NDEB periodically reviews the bank to improve its quality. All of the above. (2) and (4) B. testing for mobility of teeth. both interstitial and appositional bone formation are present. D. radiographic examination. D. All of the above. (1) and (3) C. it develops from Meckel's cartilage. B. E. D. C. ©The National Dental Examining Board of Canada 2017 . Hydroxyapatite used in dental surgery is 2. resistant to fracture. 1. is associated with continuous destruction of the alveolar bone. probing. In addition. tooth mobility. and translation. the format of some items is not currently used. C. 4. used to hold soft tissues into position. (4) only E. B. this would likely indicate B. D. content. 2. C. Periodontitis 1. protrusive maxillary incisors. maxillary prognathism. used to fill osseous defects. A. All of the above. (4) only C. derived from tooth enamel. A. both primary and secondary cartilage growth centres are present. (1) (2) (3) B.Detection of periodontal pockets is done by If the norm for the cephalometric angle SNA is 82 degrees and a patient's SNA is 90 degrees. corrected with a porcelain veneer? D. ©The National Dental Examining Board of Canada 2017 . Cross bite on tooth 1. 4. radiographic changes are present only in periodontitis. The NDEB periodically reviews the bank to improve its quality. A. Soft tissue hyperplasia. B. Alveolar ridge resorption. Ferric sulphate (Astringedent®). C. content. Aluminum chloride (Hemodent®). (1) and (3) periodontitis is C. Diastema between 1. and translation. (4) only 1. (2) and (4) D. Penicillin. Tetracycline. E. C. (1) and (3) C. (1) and (3) ill-fitting complete dentures.A dental laboratory has returned a removable Which of the following is the most effective partial denture framework. E. Clindamycin. All of the above. All of the above. casting error. A. tensile strength of amalgam. A. For amalgam restorations. All of the above. gingivitis. Enamel hypoplasia. 3. 1. compressive strength of enamel. Angular cheilitis. D. D. distortion in the final impression. a 90 cavosurface A. (4) only A. 3. insufficient retention. Peg lateral incisor. (1) (2) (3) The following conditions can be the result of B. C. Erythomycin. B. D. changes in gingival colour are present Which of the following should NOT be only in gingivitis. possible cause(s) of the problem is/are B. 3. Epinephrine (Racord®). a stable fit could not be achieved. In addition.1. radiographic changes are present only in E.3. improper pour of the master cast.1 and 2. 2. (2) and (4) 1. C. The A. 2. C. Carcinoma. 4. (2) and (4) D. Aluminum sulphate (Pascord®). E. B. compressive strength of amalgam. (4) only 2. Nystatin. The framework fit pharmacologic treatment for angular cheilitis? the master cast well but when tried in the mouth. Note: Some of the items in the Released Test Item Bank may have been discontinued due to outdated science or errors. 4. Which of the following hemostatic agents is most likely to create a systemic reaction? A. condensing of amalgam. (1) (2) (3) The difference between gingivitis and B. the format of some items is not currently used. (1) (2) (3) angle accommodates the B. D. changes in gingival colour are present only in periodontitis. Severe anaphylaxis. Maxillary third molar. 1. dental extraction. D. Rubber dam. 3. C. A severe angina attack. In addition. Confusion. All of the above. content. D. The NDEB periodically reviews the bank to improve its quality. management of an anxious patient. which of the following teeth is most Nitrous oxide likely to spread infection to the submandibular space? A. stabilization. it progresses more quickly. resistance to horizontal force. A mild asthmatic attack. and translation. 4. ©The National Dental Examining Board of Canada 2017 . In an acute apical abscess (acute periapical abscess). High velocity suction. D. D. dental plaque is not involved. Staff education. (1) and (3) D. Severe hypotension. B. is a substitute for behavioural B. Epinephrine should be administered immediately in which of the following emergencies? A. Mandibular first molar. Xerostomia. D. All of the above. (4) only E. may supplement behavioural management of an anxious patient. E. it is usually associated with abrasion. B. A. C. 4. Mandibular lateral incisor. is a substitute for local anesthesia for a D. 3. B. would have no analgesic effect at a dose of 60%. B. Note: Some of the items in the Released Test Item Bank may have been discontinued due to outdated science or errors. the age of onset is younger. direct retention. C. Surgical mask. (1) and (3) A. C. 2. 2. indirect retention. Increased urination. (2) and (4) B. A. the format of some items is not currently used. Mandibular second bicuspid. Mandibular third molar. (1) (2) (3) B. (4) only C. A. C.A reciprocal clasp arm on a removable partial Which of the following procedures should be denture will provide used to reduce the risk of exposure to mercury vapour? 1. The initiation of cemental caries differs from enamel caries because Which of the following is NOT a common side effect from a tricyclic antidepressant? A. Orthostatic hypotension. (2) and (4) E. (1) (2) (3) C. C. 4. be rigid. Immediate loading of the implant. chances of successful osseointegration of a dental implant? A. (1) (2) (3) immediate application of heat for at least B. (2) and (4) 2. (2) and (4) D. 1. Management of this hematoma includes A. 2. Using a heavy body material in the Which of the following can increase the impression tray. A good initial stability of the implant. E. the format of some items is not currently used. poor injection technique. C. be coated with an appropriate adhesive. A moist retraction cord. C. A. have occlusal stops. Electrosurgery. Local anesthetic injected into dental abscesses Which of the following is/are (an) acceptable is rarely effective because mean(s) to obtain the gingival retraction that is needed when using an elastomeric impression A. B. This nerve block is not commonly 4. (1) (2) (3) B. edema dilutes the local anesthetic. The formation of this hematoma indicates 2. content.Following administration of a posterior If a polyvinyl siloxane material is used to make superior alveolar nerve block. All of the above. 4. (4) only E. the impression tray must correct? 1. A copper band that is removed when the impression is made. ©The National Dental Examining Board of Canada 2017 . Note: Some of the items in the Released Test Item Bank may have been discontinued due to outdated science or errors. In addition. (1) and (3) the first 6 hours. associated with hematoma formation. An atraumatic surgical approach. cover the hard palate. All of the above. and translation. D. (4) only 3. (1) and (3) C. 3. The patient may experience trismus the D. a hematoma the final impression for a maxillary cast occurs. A. (1) (2) (3) B. prosthodontics? B. there is excessive vasoconstriction. The availability of dense cancellous bone. E. 3. (1) and (3) 1. bacteria can metabolize the local material for making a final impression in fixed anesthetic. D. (2) and (4) D. Which of the following statements is restoration. All of the above. the tissue is too acidic. (4) only next day. C. The NDEB periodically reviews the bank to improve its quality. (2) and (4) B. ©The National Dental Examining Board of Canada 2017 . Acetylsalicylic acid. (2) and (4) Overhangs on restorations predispose D. 3. antipyretic and anti-inflammatory effects? A. Diazepam. (1) (2) (3) B. (2) and (4) A. Which of the following has analgesic. Advanced ridge resorption. Bradykinin. D. target-like. chicken wire. enhanced food retraction. The NDEB periodically reviews the bank to improve its quality. 3. (2) and (4) D. C. Amphetamines The vibrating line of the palate is 1. 2. (4) only B.Which of the following could cause phonetic The direct immunoflourescence pattern seen in problems for patients with removable dentures? pemphigus vulgaris has been described as 1. soap-bubble. E. Posterior teeth placed in a buccal A. (1) (2) (3) D. cotton wool. position. B. decrease fatigue. C. (4) only C. 3. E. an area which marks the movement of the 3. All of the above. D. Anterior teeth that are too long. content. easily located on a cast. D. and translation. Note: Some of the items in the Released Test Item Bank may have been discontinued due to outdated science or errors. enhanced plaque retention. 4. a useful landmark in complete denture A. All of the above. corrugated. (1) and (3) C. increased caries susceptibility. soft palate. 2. (1) (2) (3) fabrication. (1) and (3) C. always on the hard palate. the format of some items is not currently used. (4) only E. 1. A. (4) only E. C. (1) (2) (3) B. 2. Excessive bulk in the palatal area. restricted plaque removal. (1) and (3) A. 4. increase mental alertness. 4. All of the above. 2. 4. B. 1. Acetaminophen. are useful in controlling arrhythmias. (1) and (3) E. In addition. increase salivation. All of the above. ©The National Dental Examining Board of Canada 2017 . A. (4) only E. (2) and (4) A. (1) and (3) E. content. maxilla. Note: Some of the items in the Released Test Item Bank may have been discontinued due to outdated science or errors. placing a gingival bevel. (4) only B. the primary molars. (1) (2) (3) D. All of the above. worsen with forward growth of the 4. develop into an Angle Class I 1. (1) (2) (3 malocclusion with late mandibular B. tissue adaptation. (2) and (4) D. drowsiness. malocclusion with normal exfoliation of 2. A.An Angle Class II dental malocclusion in the Possible side effects of therapeutic doses of mixed dentition will most likely codeine are A. 1. B. (1) (2) (3) 4. partial denture is/are to improve 2. space for the veneer material. soft palate. hard palate. 3. margin at least 1mm supragingivally. E. not change as the maxilla and mandible grow. occlusion. All of the above. C. (1) (2) (3) D. (1) and (3) C. develop into a skeletal malocclusion with D. All of the above. fit of the framework. 3. B. (4) only growth of the maxilla and mandible. 2. All of the above. respiratory depression. (1) and (3) growth. (1) and (3) C. C. lengthening the axial walls. (2) and (4) D. maxillary complete denture is the 2. constipation. B. The NDEB periodically reviews the bank to improve its quality. A. C. and translation. definite finish line. the format of some items is not currently used. function. 3. (2) and (4) D. 4. In addition. adding an occlusal dovetail. The primary retention of a Class II gold inlay is achieved by The primary purpose(s) of relining a distal extension base of a removable cast framework 1. rough surface. B. E. nausea. (4) only E. develop into an Angle Class I A. 3. The tooth preparation for a porcelain veneer must have a The primary stress bearing area of the 1. alveolar ridge. zygoma. increasing the parallelism of walls. 4. C. nitrogen. has difficulty must have a breathing and has a bluish tinge to his complexion. content. and translation. space for the veneer material. A. margin at least 1mm supragingivally. ameloblasts migrate apically down the root. ©The National Dental Examining Board of Canada 2017 . intentional devitalization followed by a vesicles. (1) and (3) C. bonded amalgam core build-up. air. C. who is a heavy smoker. emphysema. The tooth preparation for a porcelain veneer has developed a barrel chest. post and core restoration. C. A. the format of some items is not currently used. D. (1) (2) (3) B. D. An incisional biopsy. 2. a breakdown product.0mm remaining coronal tooth Enamel pearls form when structure. (2) and (4) D. apply fluoride and improve oral hygiene. Note: Some of the items in the Released Test Item Bank may have been discontinued due to outdated science or errors. Nicotinic stomatitis. Aphthous stomatitis. place a direct composite restoration. D. definite gingival finish line. D.4 reveals The small bubble normally seen in a local caries penetrating one third into the mesial anesthetic cartridge is enamel. place an amalgam restoration. cardiac insufficiency. The most likely diagnosis is 1. C. A. place a porcelain inlay. acute upper respiratory infection. candidiasis? B. 3. C. A vital canine is to be used as the anterior abutment of a four unit fixed partial denture and it has 2. In addition. Herpetic gingivostomatitis. cells of the epithelial root sheath do not B. B. A. B. (4) only E. up. The most appropriate management of tooth 1. C. C. A cytological smear. cells of the dental follicle fail to develop. A complete blood count.4 is to A. Denture stomatitis. D. B. a pin retained composite resin core build- C. a pin retained amalgam core build-up. epithelial rests transform into ameloblast D. primary cancer of the lung. oxygen. D. All of the above. B. For which of the following is nystatin oral suspension an appropriate treatment? Which of the following should be performed to confirm a diagnosis of pseudomembranous A. 4.A bite-wing radiograph of tooth 1. coarse diamond finish. The most acceptable foundation restoration would be A. An excisional biopsy. migrate away from the dentin. B. A. A 50 year old patient. The NDEB periodically reviews the bank to improve its quality. B. 3. 2. For an acid-etched Class III composite resin. content. that has not been exposed to radiation.The most appropriate antibiotic for a patient Dental implants are CONTRAINDICATED in with an odontogenic infection and a history of patients who penicillin allergy is 1. capture soft tissue in a supporting form. tobramycin. (1) and (3) C. (4) only E. eliminate the need for internal retention. have unrepaired cleft palates. (2) and (4) In comparison to visible light. travel faster. methicillin. D. pad. (1) (2) (3) B. developing solution is to chemically 2. Bleeding time. (1) and (3) C. (1) and (3) C. X-rays D. C. remove silver halide from the emulsion E. B. All of the above. 4. The NDEB periodically reviews the bank to improve its quality. capture the retromylohyoid area. the patient’s is done primarily to coagulation mechanism should be evaluated using which test? 1. ©The National Dental Examining Board of Canada 2017 . 2. Partial thromboplastin time. (1) (2) (3) the emulsion. A. have higher energy. D. remove silver halide from the emulsion C. Von Willebrand’s Factor. All of the above. simultaneously with impression making. A. have a longer wave length. are taking anticoagulants. can be focused. (4) only D. C. B. prevent displacement of the retromolar B. 4. A. (2) and (4) that has been exposed to radiation. A. An altered cast impression technique for free- A patient is currently on warfarin. 3. reduce silver halide to metallic silver in A. allow jaw relation records to be made D. (4) only E. are over age 80. Note: Some of the items in the Released Test Item Bank may have been discontinued due to outdated science or errors. clindamycin. A. the emulsion. aid in finishing. oxidize silver halide to metallic silver in 4. the format of some items is not currently used. (1) (2) (3) B. improve convenience form. and translation. C. (2) and (4) D. D. B. the cavosurface margin of the cavity can be bevelled to The purpose of phenidone in radiographic 1. increase the surface area for etching. All of the above. In addition. Prothrombin time or INR. 3.4. Before a end extension mandibular partial denture cases planned extraction of tooth 3. tetracycline. A. have uncontrolled diabetes mellitus. B. conductivity. D. candidiasis. precedes alveolar bone loss. D. (1) (2) (3) B. cortical bone. have a large surface area in contact with the ridge. A. B. A. The NDEB periodically reviews the bank to improve its quality. genetic. Position of the flap after suturing. functional. herpetic gingivostomatitis. C. expansion coefficient. 2. B. ©The National Dental Examining Board of Canada 2017 . the loss of C. axial wall should be 1. follows alveolar bone loss. D. herpangina. facial and lingual proximal cavosurface D. 4. C. (1) and (3) C. (2) and (4) clinical attachment D. modulus. Level of plaque control. be contoured by scraping the master cast. In determining the ideal proximal outline form for a Class II amalgam cavity preparation in a molar the Which of the following is the best predictor for successful periodontal flap surgery? 1. environmental. C. 2. All of the above. (1) (2) (3) A pontic should B. pemphigus. gingival cavosurface margin must clear A. The primary stimulus for growth of the the tissue that has the highest intensity is mandible is A. B. (4) only B. 4. blood. proximal walls diverge occlusally. B. 3. C. A. epigenetic. content.5mm deep. 3. 1. Type of initial incision. skeletal muscle. margins must just clear contact with the adjacent tooth. E. A. (1) and (3) During periodontal disease activity. D. Extent of flap reflection. Note: Some of the items in the Released Test Item Bank may have been discontinued due to outdated science or errors. fat. exert no pressure on the ridge. C. (4) only E.On T1-weighted magnetic resonance images. the format of some items is not currently used. contact nonkeratinized tissue. C. (2) and (4) A. is concomitant with alveolar bone loss. and translation. All of the above. D. Which thermal property is most important in selecting a restorative material to protect the pulp from excessive temperature changes? A child on antibiotic therapy would be more likely to develop A. In addition. contact with the adjacent tooth. diffusivity. Mandibular second premolar. the same as the proportional limit. A patient wearing complete dentures complains A. allergy to denture base material. A. wettability of the dental stone. periodontal pocket depth. resist deformation. The most likely cause is A. decreased tissue tolerance. hydrocolloid. All of the above. (2) and (4) D. A. 1. B. D. and translation. the same as the stress at fracture. adequate length for retention and B. C. B. 4. an inaccurate centric relation record. space for thickness of porcelain. This is often an indication of C. infrabony pocket topography. higher incidence of two canals? C. surface tension of an irreversible B. C. decreased by work hardening. neoplastic invasion of the inferior mandibular nerve. space for thickness of metal that will D. Distobuccal root of the second molar. a single path of insertion. higher than the proportional limit. have two roots and two canals? E. In addition. The NDEB periodically reviews the bank to improve its quality. C. C. impingement of denture on the mental Which of the following teeth is most likely to nerve. A dental radiograph will accurately indicate A. 2. Maxillary second premolar. the extent of furcation involvements. D. B. the format of some items is not currently used. The preparation of an anterior tooth for a The yield strength of an orthodontic wire is metal-ceramic crown should provide A. Mandibular first premolar. localized pressure. Note: Some of the items in the Released Test Item Bank may have been discontinued due to outdated science or errors. (1) and (3) bilaterally. D. B. Mesiobuccal root of the second molar. defective occlusal contacts. C. (4) only A. the height of the bone on the facial Which permanent maxillary molar root has a surfaces of the teeth. surface tension of a silicone impression material. D. B. Maxillary first premolar.Voids on a dental cast could be caused by the One week after receiving a complete denture a high patient returns with an isolated sore spot. ©The National Dental Examining Board of Canada 2017 . 3. E. (1) (2) (3) of tingling and numbness in the lower lip B. content. C. A. None of the above. Distobuccal root of the first molar. wettability of the impression material. impingement of denture on the mandibular nerve. D. incorrect vertical dimension. resistance form. E. Mesiobuccal root of the first molar. D. removal of food debris from the canal. Metaproterenol. D. Following radiation therapy to the mandible. immerse in ethyl alcohol. C. C. the format of some items is not currently used. C. Greater palatine. soft tissue necrosis. pellets. fracture. Which statement is FALSE regarding the use of Tetracycline therapy instituted either in the a barbed broach? It is used for second trimester or post partum to the infant is responsible for all the following EXCEPT A. removal of gutta-percha during the enamel. Epinephrine. C. C. minor changes in the hydroxyapatite of D. development of malignancy. wipe with an alcohol soaked gauze. autoclave for a full cycle. A. removal of paper points and cotton permanent teeth. In addition. result in A. B. discolouration of deciduous teeth and C. Which of the following applies to gutta-percha? A. Which of the following drugs should NOT be percha cones prior to obturation is to administered to a patient in order to alleviate symptoms of an acute asthmatic attack? A. content.25% sodium hypochlorite solution. nonsurgical retreatment. The NDEB periodically reviews the bank to improve its quality. important in determining the appropriate dose D. actinomycosis. A. predisposing the infant to candidal infections. B. Gender. Weight. D.The most appropriate way to disinfect gutta. Age. B. extraction of mandibular teeth is most likely to D. B. Medical history. and translation. It is a good thermal conductor. osteomyelitis. Isoproterenol. permanent maxillary first molar? E. of drug for a patient? A. Note: Some of the items in the Released Test Item Bank may have been discontinued due to outdated science or errors. removal of vital or nonvital pulp tissue. D. A. Which of the following nerves does NOT C. It can be softened by heat. Nasopalatine. It can be inserted easily into fine canals. Which one of the following factors is LEAST C. ©The National Dental Examining Board of Canada 2017 . Posterior superior alveolar. B. D. Hydrocortisone. immerse in a 5. B. B. B. require anesthesia for the extraction of a D. forming a complex with the developing tooth material. Middle superior alveolar. It is resorbable. Which of the following should NOT be administered to a patient with chest pain consistent with a myocardial infarction? Abnormal development of the first pharyngeal arch may produce defects in the A. E. divide. in the greatest discrepancy between the B. A. C. C. A. produce matrix. Which of the following movements will result A. A. D. Retrusive. hydrocortisone. C. Inversely proportional to wavelength. survive. electron emission occurs at trauma EXCEPT the A. tooth sensitivity. Morphine. Acetylsalicylic acid. B. Which of the following does NOT describe the A small hinge articulator was used for the energy of x-ray photons exiting the x-ray unit? fabrication of a cast gold onlay for tooth 4. atropine. focusing cup. palate and the hyoid bone. Protrusive. D. Note: Some of the items in the Released Test Item Bank may have been discontinued due to outdated science or errors. Laterotrusive. Oxygen. Mono-energetic. D. Normally distributed. expression. and translation. D. tooth mobility. Epinephrine. radiographic evidence of increased C. The NDEB periodically reviews the bank to improve its quality. Mediotrusive. A. maxilla and the muscles of facial E. B. odontoblasts fail to C. mandible and the external nose. differentiate. periodontal ligament space. B. ©The National Dental Examining Board of Canada 2017 . B. zygomatic bones and the external ears. Accessory root canals develop because root B. D. B. target. loss of pulp vitality. D. nitroglycerin. B. Nitroglycerin. diphenhydramine. filament. The first drug used for the management of anaphylaxis is A. In addition. the format of some items is not currently used. D.All of the following are signs of occlusal In an x-ray tube. articulator and the patient? C. epinephrine. D. C. C. Proportional to frequency. anode.6. content. type 2 diabetic. C. E. noted. Clinical examination D. diabetes mellitus. hyperlipidemia. and translation. right facial paralysis is A. congenital methemoglobinemia. Note: Some of the items in the Released Test Item Bank may have been discontinued due to outdated science or errors. D. oxygen saturation. gout. E. of the following EXCEPT D. A. the format of some items is not currently used. Which of the following local anesthetics is classified as an ester? Following the administration of a right inferior alveolar nerve block. D. content. B. Lidocaine. application of pit and fissure sealants. conservative Class I amalgams. Local anesthetic containing epinephrine is CONTRAINDICATED for a patient with A. In addition. present. preventive resin restorations. narrow palatal vault. pterygopalatine fossa. shaped arch. prophylactic odontotomy. ©The National Dental Examining Board of Canada 2017 . blood pressure. undercut palatal torus is C. B. D. inadvertent injection into the C. parotid gland. pupil diameter. E. Procaine. D. C. severe asthma. Articaine. Mepivacaine. E. heart rate. Acetylsalicylic acid should be avoided in each C. very few teeth remain in a flat or U- E. This condition was caused by B.During general anesthesia. C. B. there is a high. pseudocholinesterase deficiency. B. all of the following The full palatal major connector is indicated should be monitored EXCEPT where A. The most appropriate management is A. E. Addison’s disease. The only defects are deeply stained grooves in C. A. reveals no evidence of caries in the grooves. the posterior teeth. palatal tissue is soft and compressible. respiratory rate. sublingual gland. facial canal. B. submandibular region. no treatment. A 45 year old patient has 32 unrestored teeth. D. Bupivacaine. A. gastric ulcer. The NDEB periodically reviews the bank to improve its quality. a well-defined. sulfite sensitivity. B. (1) (2) (3) B. B. stimulate cyclo-oxygenase activity.For a cast gold restoration. increases the thickness of gold. The child has no pain. 4. C. C. Geographic tongue. narrow fissures. buccal or lingual. B. A. B. 6 months in utero. Codeine. Titanium. polymerization shrinkage. D. 2. In addition. content. and translation.0ppm. promote wound healing. 1. increases retention. What is the most likely diagnosis? to initiate a hypersensitivity reaction? A. Ibuprofen. (2) and (4) ketoralac. Acetaminophen. A 4 year old child presents for an emergency D. E. Which of the following is a CONTRAINDICATION for placement of a Polyvinylsiloxane impression materials have fissure sealant on a permanent molar? high A. examination. (1) and (3) Non-steroidal anti-inflammatory drugs. buccal. B. dimensional stability. Note: Some of the items in the Released Test Item Bank may have been discontinued due to outdated science or errors. A. the format of some items is not currently used. The NDEB periodically reviews the bank to improve its quality. improves marginal adaptation. Which of the following drugs controls and reduces inflammation? A. a gingival bevel is The mesial furcation of maxillary permanent used instead of a shoulder because a bevel first molars is best probed from the 1. Deep. tooth crowns except third molars is D. A. D. C. Silver amalgam. linear expansion. 3. by-product formation. B. such as C. C. D. lingual. Candidiasis. B. A. 5 years. Inadequate moisture control. B. Meperidine HCl. ingestion will NOT cause discolouration of any C. The youngest age at which tetracycline B. 15 years. protects the enamel. 10 years. Primary herpetic gingivostomatitis. Community water supply fluoridated at D. C. The mother is concerned about white patches on the child’s tongue. Gold. ©The National Dental Examining Board of Canada 2017 . are beneficial in the treatment of D. C. Hairy tongue. C. A. reduce prostaglandin E2 synthesis. Tooth erupted more than one year. (4) only periodontal disease because they E. A. Nickel-chromium. Chrome-cobalt. eats and drinks normally and has a history of repeated use of amoxicillin for otitis Which of the following materials is most likely media. All of the above. Uncontrolled diabetes. in patients with a negative skin test to B. The NDEB periodically reviews the bank to improve its quality. B. disease? B. patients presenting with generalized C. D. fungal. The etiologic agent for necrotizing ulcerative gingivitis (NUG) is A. D. giant cell lesions. A 60 year old patient in chronic renal failure B. remain coronal to the mucogingival junction. Prolonged NSAID therapy. hyperpituitarism. aggressive periodontitis.Incomplete tooth fracture Acute anaphylactic reactions to penicillin are LEAST likely to occur A. D. avoid attached gingiva. B. B. ©The National Dental Examining Board of Canada 2017 . is associated with medium to large-sized C. Bleeding time. D. be partial thickness. A. most commonly involves the supporting penicillin. Hematocrit. D. C. Systemic corticosteroid therapy. gingivitis. Platelet count. viral. In addition. Metronidazole has been prescribed for a patient taking warfarin daily for the last 2 years. glaucoma. can readily be diagnosed using transillumination. E. an allergic reaction to the drug. hypoparathyroidism. C. This patient should be evaluated for A. A. Note: Some of the items in the Released Test Item Bank may have been discontinued due to outdated science or errors. have a wide base. and translation. when the drug is administered parenterally. chronic periodontitis. D. bacterial. INR. B. within minutes after drug administration. hyperparathyroidism. the format of some items is not currently used. elicits dull. in patients who have already experienced restorations. C. gingival hyperplasia. C. prolonged pain on chewing. Systemic antibiotics may be indicated for B. C. cusps. Which Which of the following is the most likely cause of the following must be closely monitored? of osteoporosis. has bilateral radiolucent mandibular lesions. D. A surgical flap for the extraction of a tooth should be designed to A. hypertension and peptic ulcers in a 65 year old with Crohn’s A. Chronic renal failure. Histological analysis reveals that these are D. protozoan. content. Malabsorption syndrome. hyperthyroidism. C. A. A patient presents with pain from tooth 4. less often in elderly patients. 3. 1. Note: Some of the items in the Released Test Item Bank may have been discontinued due to outdated science or errors.7 crown. Clinical and radiographic A. 3. the format of some items is not currently used. anteriorly. B. 2. During the administration of local anesthesia. B. more often with the removal of maxillary examinations reveal tooth 4.7. Decrease in glycogenolysis. long buccal nerve block. more often with the removal of maxillary A. remove entire bridge and assess restorability of abutments.7 An oroantral communication occurs which is an abutment for a 4 unit bridge from 4. C. Decrease in diastolic pressure. ©The National Dental Examining Board of Canada 2017 . remove 4. C.0mm. content. E. Increase in muscular contractility.7 has extensive second premolars. A. B. anterior superior alveolar block. prescribe an antibiotic and an analgesic first molars.4 to 4. The daily maximum dose of ibuprofen for an adult is Facial paralysis following an inferior alveolar nerve block is a result of injecting the solution A. 4. perform endodontic therapy through the 4.200 mg. positive aspiration of blood will occur most often in a/an Which of the following occurs when epinephrine is administered intravenously? A. D. In addition. distal caries and apical rarefying osteitis.600 mg. section the bridge at 4. codeine. inferiorly. C.0mm.0mm.5mm. D. mental or incisive block. B. 2. posteriorly. D. A. B. too far B. inferior alveolar block. D.7 crown and assess 4. A. The NDEB periodically reviews the bank to improve its quality. and translation. C. ibuprofen. C.000 mg. C. B. D. D.400 mg. posterior superior alveolar block. tramadol. 1. less often when the maxillary sinus has most appropriate initial management is to pneumatized into the alveolus.4. acetaminophen. and reappoint the patient. E. superiorly.5mm.A periodontal screening and recording (PSR) The antithrombotic effects of aspirin may be score of 3 for a sextant indicates that probing compromised by depth does NOT exceed A. D. C. 5. Increase in arrhythmias.7. C. D. 6. The B. A. sloughing. Endodontic treatment. Condensing silicone. C. D. D. E.200 mg. B. Necrotizing ulcerative gingivitis. D. D. C. stroke. primate spaces. amoxicillin. During dental treatment. For acute dental pain. B. B. C. D. and translation. meperidine. Polyether. myocardial infarction. pneumonia. C. The most likely diagnosis is A. C. B. 3. lorazepam. 4. Splinting of teeth.600 mg. alcohol.Naloxone reverses respiratory depression In which of the following situations can topical caused by corticosteroids be used? A.000 mg. Periodic observation. A. Sublingual nitroglycerin. 2. pulmonary embolus. A. D. Subcutaneous epinephrine. C. B. A. the daily maximum cumulative dose of acetylsalicylic acid is When performing a frenectomy. Polyvinyl siloxane. secondary bleeding. the format of some items is not currently used. D. A. Note: Some of the items in the Released Test Item Bank may have been discontinued due to outdated science or errors. leeway spaces. phenobarbital. ©The National Dental Examining Board of Canada 2017 . In addition. Polysulfide. content. becomes pale and sweaty. Which of the following is the most appropriate management for a tooth displaying crazing of Which of the following impression materials is the enamel? the LEAST flexible when set? A. spaces between primary incisors are called A. D. nystatin. B. Angular cheilitis. C. C. D. B. irritation. Erosive lichen planus. Herpes labialis. Stainless steel band. physiological spaces. 3. C. B. a minimal amount of anesthetic solution is used to prevent A. All the following medications can be used for the treatment of Candida albicans EXCEPT In a normally developing occlusion. The NDEB periodically reviews the bank to improve its quality. a 45 year old male patient complains of a tight constriction of his What should be the immediate management of chest. C. Inhaled salbutamol. B.400 mg. Oral ibuprofen. Candidiasis. D. feels nauseous an acute anginal episode? and attempts to vomit. fluconazole. chlorhexidine. freeway spaces. distortion of the tissues. B. B. B. pulp tissue left in the root canal. C. reamer. precursors of sarcoma. over-instrumentation. Percussion of a tooth is used to evaluate all of the following EXCEPT Following the removal of a vital pulp. occlusal trauma. the format of some items is not currently used. E. lateral perforation. asymptomatic necrotic teeth with incompletely formed apices. the arch form can be more accurately C. B. E. Hedstrom file. D. The patient A. benign neoplasms. the recommended time during endodontic procedures is a interval between bitewing radiographic examination for the detection of dental caries is A. asymptomatic necrotic teeth with D. 3. Note: Some of the items in the Released Test Item Bank may have been discontinued due to outdated science or errors. D. content. D. overlooked. 4. asymptomatic vital teeth with completely formed apices. B. periapical status. returns with apical periodontitis. tuberculomas. and translation. Granular cell myoblastoma. ©The National Dental Examining Board of Canada 2017 . A. common cause is C. pulpal status. asymptomatic vital teeth with B. infection. ankylosis. 24 months. barbed broach. (1) and (3) C.The most efficient cutting instrument used For an adult patient. Ranula. A. C. dependent upon caries risk. 12 months. incompletely formed apices. The most B. malignant neoplasms. C. C. C. non-neoplastic granulomatous lesions. D. A. k-type file. (4) only E. D. A. Mucoepidermoid carcinoma. 6 months. The oral examination of the edentulous patient should include digital palpation because 1. All of the above. spicules under the mucosa may be A. B. occur in the floor of the mouth? 2. completely formed apices. Lymphangioma. undercut areas may be hard to visualize. (1) (2) (3) B. evaluated D. the root canal is medicated and sealed. Pleomorphic adenoma. the thickness of the mucosa can better be Which of the following lesions is most likely to evaluated. The NDEB periodically reviews the bank to improve its quality. In addition. Pulpotomy is the treatment of choice in carious pulp exposures of Oral peripheral giant cell lesions of the periodontium are most likely A. (2) and (4) D. convert the latent image to black metallic silver. Fissural. 4. (2) and (4) B. B. radiation received by patient. D. (4) only C. increase sharpness. The NDEB periodically reviews the bank to improve its quality. The purpose of using intensifying screens in extra-oral radiography is to Which of the following cysts is most likely to undergo transformation into an ameloblastoma? A. the patient. Osteochondroma. Ewing's tumor. radiographic contrast. and translation. decrease scatter radiation. Osteosarcoma. Radicular. B. Note: Some of the items in the Released Test Item Bank may have been discontinued due to outdated science or errors. Reduced deposition of metallic silver on the emulsion. 1. 4. (1) (2) (3) B. D. (1) (2) (3) C. 4. All of the above. Dentigerous. A. (4) only C. Increased exposure time. 3. Fibrous dysplasia. 3. The function of the fixer solution is to Which of the following bone lesions of the 1. D. C. In addition. (1) and (3) A. remove unexposed silver halide. intensity of central beam. harden the emulsion. (2) and (4) E. the format of some items is not currently used. (4) only E. 3. (4) only E. D. Increased processing time. ©The National Dental Examining Board of Canada 2017 . A. improve resolution. All of the above. mandible is/are malignant? 2. (2) and (4) B. 1. (1) and (3) A. (1) (2) (3) C. (1) and (3) D. secondary radiation. Reduced attenuation of the X-ray beam in 2. (1) and (3) D. continue the action of the developer. Traumatic. 4. (1) (2) (3) B. All of the above. All of the above. 2. C. (2) and (4) E. decrease exposure time. content. 2. 3.Proper collimation of the useful beam for the What could cause an area on an analog film size and target-film distance will reduce radiograph to be darker? the 1. A. cemento-osseous dysplasia). (4) only E. palate. 4. (4) only E. content. Heat test. All of the above. tongue. buccal mucosa. increased penumbra of the image. 3. (4) only D. The NDEB periodically reviews the bank to improve its quality. the format of some items is not currently used. Compared to the bisecting angle technique. the nasopalatine foramen may be mistaken for 1. E. infiltrates bone. contains fluid. results of the biopsy. C. All of the above. patient’s symptoms. and translation. discrete. radiographic appearance. enlarges rapidly. ©The National Dental Examining Board of Canada 2017 . A 2cm. reduced distortion of the image. a naso-alveolar cyst. A radicular cyst A periradicular granuloma can be differentiated from a periradicular cyst by the A. the paralleling technique will result in a Which of the following tests can be used to determine the pulp status of a tooth with an all- 1. 4. (1) and (3) C. A. traumatic injury. gingiva. C. D. D. (1) (2) (3) A. periapical osseous dysplasia (periapical 3. (1) and (3) C. is associated with a vital tooth. 4. periapical granuloma. 2. refer patient to family physician. white lesion of the buccal mucosa has not resolved after elimination of all local irritants. Note: Some of the items in the Released Test Item Bank may have been discontinued due to outdated science or errors. C. The most appropriate management is to A syphilitic gumma is most commonly found on the A. a simple bone cyst/traumatic bone cyst. (1) and (3) B. response to percussion testing. E. (1) (2) (3) D. a radicular cyst. B. D. Cold test with ice stick. increased magnification of the image. cauterize it.Root resorption may be associated with Radiographically. B. B. excessive orthodontic forces. A. reduced dose to the thyroid gland. ceramic crown? 2. B. (2) and (4) C. 1. apply toluidine blue staining. A. (1) (2) (3) B. re-examine at 6 month intervals. (2) and (4) D. C. 2. E. lip. C. 3. Cold test with ethyl chloride spray. (2) and (4) D. Electric pulp test. B. In addition. B. an incisive canal cyst. All of the above. A. D. A. perform an incisional biopsy. C. elevated blood D. C. In addition. B. CONTRAINDICATED for pharmaco- sedation? A. pressure. 3. the last part of the CNS (Central Nervous System) to be depressed is A. D. Lidocaine. B. Convulsions and loss of consciousness. A. Benzocaine. healing is painful but not delayed. difficulty with extraction. A. C. weak but regular pulse. 30 minutes. Minor tranquilizers. spinal cord. Tachycardia. socket. palpitations. Which local socket". content. Antihistamines. 2. Barbiturates. Tetracaine. sweating. and translation. routine use of antibiotics is advised as a C. cerebellum. midbrain. Sudden pallor. no fibrin is formed in the extraction A. prophylactic measure. B. (1) (2) (3) B. Note: Some of the items in the Released Test Item Bank may have been discontinued due to outdated science or errors. D. reduction of arch length. (4) only which of the following drugs is E. (1) and (3) C. 1 to 2 minutes. Opioids. B. D. the format of some items is not currently used. cardiac arrhythmia. B. delayed eruption of the underlying premolar. headache. oblongata. Procaine. occasional loss of consciousness. An ankylosed deciduous molar can cause The psychomotor recovery time from nitrous oxide sedation is 1. a focal osteomyelitis exists in which the clot has disintegrated. ©The National Dental Examining Board of Canada 2017 .Healing of extraction wounds is sometimes A patient has a proven allergy to para-amino complicated by a condition known as a "dry benzoic acid derivatives. D. Which of the following is most likely related to the administration of excessive vasoconstrictor? In general anaesthesia. immediate. reduction in alveolar bone. 10 minutes. 4. In this condition anesthetic solution can be used safely? A. The NDEB periodically reviews the bank to improve its quality. E. All of the above. A. medulla. D. D. C. the B. (2) and (4) When a patient has a history of porphyrism. C. Flushing of the skin and tremors. (1) and (3) C. It is only determinate of skeletal age. All of the above. 3. A. A 3 year old with a 4mm overjet. ©The National Dental Examining Board of Canada 2017 . A. (2) and (4) An overjet of 8mm is most often associated D. Class I cuspid relationship. (4) only Which of the following patients should be E. and translation. It provides a precise measure of skeletal 2. primary tooth is lost prematurely. maxillary canines and first molars. Class II cuspid relationship. A. C. Note: Some of the items in the Released Test Item Bank may have been discontinued due to outdated science or errors. An 8 year old with a previous thumb C. mandibular canines and first molars. B. 1. 4. All of the above. (1) and (3) E.The eruption of a permanent central incisor The developing permanent tooth may be delayed by 1. D. (2) and (4) D. alveolar bone. (1) and (3) C. effect will be on the growth of the 2. the primary habits. 2. dense fibrous tissue. C. Skeletal age may be estimated by comparing the image to a standard. B. The NDEB periodically reviews the bank to improve its quality. Class III cuspid relationship. A. the format of some items is not currently used. may show deviated eruption times if the 3. All of the above. palate. (4) only E. maxillary canines and lateral incisors. referred to orthodontically close a maxillary midline diastema? 1. (1) (2) (3) D. (4) only B. An 8 year old with no abnormal oral If a child's teeth do NOT form. 2. All of the above. usually erupts earlier in girls. A 14 year old with no abnormal oral habits. (1) (2) (3) 4. In addition. early loss of a deciduous incisor. 4. A. a retained deciduous incisor. (2) and (4) D. has a more protrusive path of eruption in the anterior region. C. development. D. (1) and (3) C. diagnosis. mandibular canines and lateral incisors. mandible. 3. B. (4) only with E. (1) (2) (3) B. habit. (1) (2) (3) B. 3. (2) and (4) A. It is of minimal value in orthodontic 4. in the anterior region. a supernumerary tooth. maxilla. Which of the following is correct with respect to the hand-wrist radiograph? The primate spaces are located between the A. B. content. lies apically and lingually to primary teeth 1. a greater range of activation. the strategic removal of primary teeth. possible. A crossbite of a maxillary central incisor is Inadequate space for the eruption of the most appropriately treated maxillary second premolar is most frequently caused by the premature loss of the deciduous A. B. central incisor. The NDEB periodically reviews the bank to improve its quality. tooth shape. cartilaginous. 3. 3. patient age. following eruption of the maxillary permanent canines. C. C. 4. correcting mouth breathing as early as 1. All of the above. content. malocclusion may be reduced by 1. D. (1) (2) (3) B. (2) and (4) E. C. division 1 malocclusion has the best prognosis when the Bone laid down by the periosteum is 1. stage of dental maturation. C. All of the above. permanent lateral incisors. Loops and helices in orthodontic arches result 2. A. (4) only The severity of an Angle’s Class II E. 3. 4. during the eruption of the maxillary permanent central incisors. Correction of an Angle Class II. first molar. second molar. appositional. (1) and (3) D. (1) and (3) A. tooth has erupted. (4) only E. (2) and (4) B. skeletal bases are harmonious. 2. maxillary incisors are tipping labially. cancellous. (1) (2) (3) C. B. endochondral. preventing thumbsucking and lip biting in habits. (4) only C.The most significant factor in the predictable "Dental age" is defined as the correction of an anterior crossbite is the A. 2. and translation. (1) (2) (3) B. easier insertion. maintaining the integrity of the primary dentition. canine. (1) and (3) C. A. mesio-distal spacing. B. B. overbite. a decreased level of force application. A. In addition. (2) and (4) D. dentition is mildly crowded or spaces. the format of some items is not currently used. D. permanent central incisors. A. number of years elapsed since a given C. A. All of the above. following eruption of the maxillary D. ©The National Dental Examining Board of Canada 2017 . lower face height is long. eruption time of a given tooth. Note: Some of the items in the Released Test Item Bank may have been discontinued due to outdated science or errors. improved tissue response. following eruption of the maxillary B. 4. D. D. A. D. D. thoroughness of condensation. content.Thumbsucking does NOT usually affect the The most important diagnostic element in permanent dentition if the habit is assessing the periodontal status of a patient is the A. glass ionomer cements. (1) (2) (3) In minimizing the firing shrinkage of porcelain. B. 4. D. mobility of the teeth. 1. zinc phosphate cements. results of vitality testing. space. All three of the materials expand slightly upon cooling from mouth temperature (37°C) to room temperature (20°C). A. B. The NDEB periodically reviews the bank to improve its quality. extra teeth. (4) only A. deposited in the nails. unusual dental arch development. In addition. C. congenitally missing teeth. C. B. C. (1) and (3) the principal factor is the C. E. B. space. deposited in teeth. polyether and I malocclusion is addition cured silicone impression materials. widening of the periodontal ligament A. and translation. ©The National Dental Examining Board of Canada 2017 . C. deposited in bone. The most common etiology of an Angle’s Class In comparing polysulfide. Bis-GMA resin cements. the format of some items is not currently used. D. narrowing of the periodontal ligament C. A. calcium hydroxide cements. Lead oxide is used as an activator in 2. ratio of flux to feldspar. cemental tears. Note: Some of the items in the Released Test Item Bank may have been discontinued due to outdated science or errors. during curing. C. fusion temperature. silicones. After one week. All of the above. addition cured silicones Fluorides taken systemically are will undergo more distortion than polysulfides. depth of periodontal pockets. horizontal bone loss. B. than B. associated with sleeping. D. Trauma from occlusion may be diagnosed radiographically by the presence of Zinc-oxide-eugenol cements are less soluble A. excreted in the urine. uniformity of particle size. (2) and (4) D. B. 3. discontinued before eight years of age.arch size discrepancy. D. All three of the materials contract slightly C. which of the following statements is true? A. B. tooth size . is of low intensity. discontinued before four years of age. base impressions should be poured C. Speech defects associated with a maxillary C. A. and clinical crown are equal in length.A removable partial denture is preferable to a The purpose of a temporary restoration in an fixed bridge when the anterior tooth is to A. A. residual ridges are severely resorbed. angular and box shaped with parallel vertical walls. positioning anterior teeth incorrectly. D. prevent tooth movement. endothermic. is twice the length of the clinical crown. 1. within 8 hours. polysulfide B. flat and with an obtuse angle to the C. wearing a maxillary complete denture is most C. A. likely to be associated with A. rounded and spoon shaped. 4. B. In a fixed bridge. maintain aesthetics. posterior palatal extension too far An occlusal rest preparation should be posteriorly. 1. after 24 hours. exothermic. providing excessive bulk of denture base. prevent gingival inflammation and D. the format of some items is not currently used. (2) and (4) proximal surface of the tooth. D. C. an allergy to the acrylic resin. A. a knife-edge bony ridge will D. (4) only E. heavy smoking. abutment teeth have large undercuts. (1) and (3) C. (2) and (4) denture. D. occlusion with posterior natural teeth. In addition. A. an ill-fitting denture and poor oral partial denture can be caused by hygiene. C. (1) (2) (3) B. (1) and (3) In treatment planning for a removable partial C. within 1 hour. The NDEB periodically reviews the bank to improve its quality. make impression-making difficult. C. is half the length of the clinical crown. and translation. replacing the teeth too soon after extraction. All of the above. the most favorable ratio for an abutment tooth is when the root A. 2. abutment teeth are rotated. (1) (2) (3) B. (4) only E. The polymerization of methyl methacrylate is A. 2. To ensure the greatest accuracy. ©The National Dental Examining Board of Canada 2017 . cause difficulty in tooth selection. B. B. B. content. B. Papillary hyperplasia on the palate of a patient B. recession. abutment teeth are tipped. necessitate relief to the partial denture. within 10 minutes. 4. Note: Some of the items in the Released Test Item Bank may have been discontinued due to outdated science or errors. All of the above. hydrophilic. D. protect dentin and pulp. E. 3. 3. edentulous areas are large. hydrolytic. potential for systemic reaction. E. the major temporary restoration? connector should begin 3-6mm from the free gingival margin in order to 1. Cementing a full gold crown with zinc phosphate includes 1. All of the above. 8% racemic epinephrine may be hazardous for 3. some patients because of its 4. local caustic action on the gingival tissue. A. (2) and (4) D. detected at the delivery appointment. protect the pulp. cleaning excess cement off while setting. The interocclusal distance (freeway space) A. (1) and (3) C. and centric relation. reduce bond strength. In addition. reduce microleakage.The major connector of a removable partial Prior to cementing an onlay in a vital tooth denture should be designed to using a resin cement. C. is the difference between occlusal vertical dimension and hinge axis registration. a fixed B. (2) and (4) D. having excess cement covering the margins. D. the application of cavity varnish will A. prevent rotation of the major connector in A. D. To decrease abutment tooth sensitivity. is the distance between centric occlusion B. Contacts with adjacent teeth that prevent connectors. Need for significant occlusal adjustment. polycarboxylate cement. application of continuous loading. improve seal. B. The use of a retraction cord impregnated with 2. allow sufficient length for the minor 3. ©The National Dental Examining Board of Canada 2017 . C. act as a stress-breaker. D. Hypersensitivity of the abutments that decreases after permanent luting. The NDEB periodically reviews the bank to improve its quality. the format of some items is not currently used. acrylic resin cement. tissues. A. and translation. B. (4) only E. B. connect rigidly the component parts of the partial denture. Exposed gingival margins in an area. can be caused by an inadequate In a removable partial denture. prevent inflammation of the gingival 4. Note: Some of the items in the Released Test Item Bank may have been discontinued due to outdated science or errors. A. reduce prevent postoperative sensitivity. D. C. C. (1) and (3) C. All of the above. 2. (1) (2) (3) C. B. local astringent action. A. C. applying a thick coat of cement internally. B. content. dissipate vertical forces. glass ionomer cement. is usually l0mm in the premolar region. is the difference between occlusal vertical bridge may be temporarily seated using dimension and rest vertical dimension. (4) only E. Which of the following problems of a permanent fixed bridge. complete seating of the bridge. improve phonetics. B. A. zinc oxide eugenol cement. (1) (2) (3) an antero-posterior direction. A. avoiding covering the pad with the direction. (2) and (4) D. D. B. 4. (1) (2) (3) B. temperature of the cathode filament. B. B. which insufficient space between the maxillary tuberosity and the retromolar pad will require 1. occur if the second or third molars are 1. Electrosurgery. occur if a supracrestal fiberotomy is performed. after 24 hours. and translation. B. (1) and (3) C. 3. maxillary base. D. 4. 2. ©The National Dental Examining Board of Canada 2017 . (1) and (3) C. originates above the height of contour. Note: Some of the items in the Released Test Item Bank may have been discontinued due to outdated science or errors. (2) and (4) D. The NDEB periodically reviews the bank to improve its quality. (4) only E. the format of some items is not currently used. occur if retainers are worn until the 3. D. mandibular growth is complete. removed. not covering the tuberosity with the tooth. traverses a portion of the suprabulge of the B. All of the above. A circumferential clasp arm on a tooth is one For a patient with complete dentures. relapse of the mandibular incisors CANNOT A. C. (1) (2) (3) tuberosity. surgically reducing the retromolar pad. Displacement cords. content. surgically reducing the maxillary A. quantity of X-rays produced. energy of the X-rays. contrast of the image. be predicted from characteristics of the Kilovoltage controls the original malocclusion. Following orthodontic alignment. extends more than 180° around the tooth. All of the above. Aluminum chloride impregnated cord. after 30 minutes. approaches the undercut from an occlusal A. In addition. Aluminum sulfate saturated cord. C. mandibular base. (4) only E. immediately. 2.Which of the following materials/techniques is If an impression were taken with a polysulfide CONTRAINDICATED when using polyvinyl impression material of teeth exhibiting severe siloxane impression materials? external undercuts a stone model should be poured A. A. C. C. C. all cusps have penetrated the wax record A. (1) and (3) D. (4) only protrusive position.Cleft palate may result in a higher incidence of Which of the following increases the sharpness of a radiograph? 1. A. B. there is no perforation of the wax record. 70%. The NDEB periodically reviews the bank to improve its quality. intestinal parasites. In inhalation analgesia. the wax record to insure that A. Propylthiouracil is a drug used in the treatment of A. the patient has not closed in a lateral abutment. 4. A. When uprighting a molar to be used as a bridge B. C. B. (4) only E. B. sodium retention. the format of some items is not currently used. vertical changes due to tipping of the D. altered crown morphology. leukemia. denture construction will 1. C. content. D. local periodontium. A. 3. 80%. the dentist should examine E. orthognathic surgery. 2. impair aesthetics. what is the safe 3. consideration must be given to the position. C. and are in contact with the opposing teeth. (2) and (4) D. 60%. traumatize the underlying supporting maximal nitrous oxide concentration that can tissues. the incisal edges of the anterior teeth have made contact. In addition. All of the above. C. use of fixed appliances for optimum control. hyperthyroidism. ©The National Dental Examining Board of Canada 2017 . D. Long object-to-film distance. induce generalized soreness over the ridges. A. All of the above. residual space for a pontic. B. and translation. supernumerary teeth. Small focal spot. Note: Some of the items in the Released Test Item Bank may have been discontinued due to outdated science or errors. 50%. B. cause the dentures to click. All of the above. (1) (2) (3) B. Short focal spot-to-film distance. be delivered? 4. (1) and (3) C. molar. (1) (2) (3) C. Reduction of free way space in complete E. congenitally missing teeth. (2) and (4) In taking an interocclusal wax record in a D. 2. 3. (4) only E. B. All of the above. C. Prilocaine. Longer trituration of alloy and mercury will Which of the following drugs is used in the result in an amalgam restoration which will treatment of Candida albicans infections? have A. D. The NDEB periodically reviews the bank to improve its quality. D. Lidocaine. C. All central and lateral incisors. antiplatelet agent. Tetracycline. (1) and (3) incisors and first molars. wider preparation margins. (1) and (3) A. and translation. D. molars. E. the tray has adequate space for the impression material. (2) and (4) D. Procaine. C. anti-inflammatory agent. utilizing grooves or boxes. In addition. C. 4. Sodium salicylate is administered to a patient with rheumatoid arthritis because it is a/an A. a custom tray A. All of the above. Note: Some of the items in the Released Test Item Bank may have been discontinued due to outdated science or errors. content. Only mandibular central and lateral B. greater expansion during setting. increasing axial height. Which of the following anesthetic agents are hydrolyzed by plasma cholinesterase? In complete denture construction. (2) and (4) B. B. (4) only C. reducing the taper. there is relief for muscle attachments. Mepivacaine. 2. first molars and first premolars. A. 4.The retention form of a full crown preparation Which permanent teeth will commonly be can be improved by present in an 8 year old child? 1. Nystatin. A. reduced strength. Penicillin. 1. Chlorhexidine. D. sedative. (1) (2) (3) D. (1) (2) (3) B. 2. increased flow. C. ©The National Dental Examining Board of Canada 2017 . All central and lateral incisors. B. C. the flanges are not overextended. antiseptic. the format of some items is not currently used. A. should be fabricated to ensure that B. All central and lateral incisors and first 3. the distal extension is adequate. 4. (1) (2) (3) B. The most appropriate management is 3. In addition. A. E. C. B. Cleft palate. (1) and (3) D. pulpotomy and seal temporarily. Which Angle’s malocclusion is most 2. a deviated closure to maximal 4. periodontal damage. intercuspidation. bilateral. Cleidocranial dysostosis. 4. large. pulpectomy and seal temporarily. Angle’s "subdivision" refers to an abnormal molar relationship that is The ANB angle in severe Class II A. within normal limits. endodontic treatment. A. faster movement. He is brought to your office within one hour of the injury. Cephalometric standards 1. (2) and (4) D. division 1. traumatic. (4) only E. (1) (2) (3) C. are racially biased. C. Class III. root resorption. functional. All of the above. small. (1) (2) (3) C. C. B. C. (4) only E. pulp cap and restore. 3. Class II. (4) only E. B. division 2. 2. Excessive force in orthodontic tooth movement The defining future of a skeletal crossbite is is positively correlated with A. C. B. coincident midlines. A periapical radiograph 1. an interference free closure to maximal 2. A. are the basis for diagnosis. B. (2) and (4) D. transitional. commonly associated with mouth breathing? 3. A. A. (1) and (3) D. 1. (1) and (3) C. All of the above. D. A. B. reveals that the apex is incompletely formed. D.A 9 year old boy sustains a fracture of the Which of the following congenital problems crown of his central incisor with minimal pulp often results in a malocclusion? exposure. Class I. intercuspidation. Class II. may vary with patient development. content. slower movement. ©The National Dental Examining Board of Canada 2017 . Ectodermal dysplasia. and translation. The NDEB periodically reviews the bank to improve its quality. All of the above. include a range of application. malocclusions is most often B. a large mandible. Pierre Robin syndrome. Note: Some of the items in the Released Test Item Bank may have been discontinued due to outdated science or errors. (2) and (4) D. unilateral. the format of some items is not currently used. the space between the tray and the teeth is E. and translation. (4) only 2. 2. E. Thyroid collar and lead apron. ignored because pressures of the lip will cause the problem to recur. C. (1) and (3) will be improved if C. unsatisfactory temporary crown A. practitioner preference. uncontrolled tipping of the permanent teeth. ignored because pressures of the tongue will correct it as it erupts. High kilovoltage. The NDEB periodically reviews the bank to improve its quality. (1) (2) (3) The accuracy of alginate impression materials B. occlusal prematurities. All of the above. 3. the format of some items is not currently used. C. (1) and (3) C. 2. Rectangular collimation. High speed films. E. 4. All of the above. 1. tooth sensitivity.The appliance of choice to correct an anterior Which of the following methods decrease crossbite is determined by the radiation exposure to patients? A. lingually directed path should be D. ©The National Dental Examining Board of Canada 2017 . 1-2mm. the space between the tray and the teeth allows 4-5mm of alginate. 3. (1) (2) (3) restorations may result in B. (4) only 1. D. cooperation of the patient. deepening of the overbite. 4. In addition. B. (4) only A. For teeth prepared as abutments for fixed bridges. 3. (2) and (4) D. A. (1) (2) (3) B. allowed to erupt into cross-bite and then corrected. amount of overbite. 4. (2) and (4) 1. E. lingual tipping of the mandibular incisors. gingival recession. age of the patient. corrected before it reaches the occlusal plane. C. A. content. Note: Some of the items in the Released Test Item Bank may have been discontinued due to outdated science or errors. B. All of the above. D. the impression is soaked in water for 1 hour. A. B. (2) and (4) D. allowed to erupt until all incisors can be banded. tooth migration. (1) and (3) Serial extraction may result in C. D. regional extraction spacing. the impression is removed slowly from A maxillary central incisor that is erupting in a the undercuts around the teeth. 4. Acetaminophen.r. C. maxillary premolars. B. E. The most likely etiology is A. (1) and (3) p. (1) and (3) A. the format of some items is not currently used. C. (2) and (4) 3. mandibular premolars. (2) and (4) thumbsucking has an Angle Class II D. D. patient with a acid for 15-20 seconds confirmed allergy to codeine.r. decalcifies the intertubular and peritubular 1. increases dentinal permeability.6 hours B. (1) (2) (3) followed by 400mg every 4 . ©The National Dental Examining Board of Canada 2017 . SNB = 80. In addition. D. dental and neuromuscular. Epinephrine. 4. Hydromorphone. 2mg every 4 . 2.n. Metabisulfite. Condensation silicone. removes the smear layer. Polyvinylsiloxane. E. The NDEB periodically reviews the bank to improve its quality. A. neuromuscular. Infectious mononucleosis. 4. Irreversible hydrocolloid. skeletal. and translation. the following? D. 2. Lidocaine. All of the above. (4) only malocclusion with a SNA = 82 and a E.Which of the following constituents of a local The incidence of tooth loss due to periodontal anesthetic cartridge is most likely to be disease is highest for allergenic? A. 1.n. mandibular incisors. Ketorolac. (2) and (4) B. Shingles. D.6 hours p. (1) (2) (3) B. D. Polysulfide rubber. Which of the following impression materials has the best dimensional stability? A. All of the above. Acid etching of dentin with 10-15% phosphoric For a 20 year old. Ibuprofen. A. C. following the removal of an impacted third 2. (4) only 10mg every 4 . Chickenpox. 10mg every 4 .6 hours p. E. dental. mandibular molars. All of the above. Note: Some of the items in the Released Test Item Bank may have been discontinued due to outdated science or errors. D. 80kg. content. (1) (2) (3) B.r. Hydrochloric acid. (4) only C. B. 800mg 1 hour pre-operatively A.r. opens the dentinal tubules. Oral hairy leukoplakia. molar? 3.n. Epstein-Barr virus is associated with which of C.6 hours dentin. B. C. (1) and (3) A 12 year old male with a history of C. 3. which of the following is/are appropriate for pain control 1. skeletal and neuromuscular.n. p. 650mg with oxycodone. maxillary molars. E. (1) and (3) The custom tray used in making a final C. Which of the following is NOT a feature of D. Diabetic acidosis. the format of some items is not currently used. extend to the bottom of the vestibule. Premature wear of occlusal surfaces. be stored in water until ready for use. Note: Some of the items in the Released Test Item Bank may have been discontinued due to outdated science or errors. In addition. B. bruxism? A. A. B. D.0mm primary molar with carious pulp exposure and cusp fracture is a/an What effect(s) could this Bolton relationship have on an Angle Class I malocclusion? A. Hypoglycemic shock. 2 years. C. C. Acute adrenal insufficiency.5mm The most appropriate treatment for a vital  maxillary “6” excess of 3. The NDEB periodically reviews the bank to improve its quality. A. extraction and placement of a space maintainer. D. Angina. ©The National Dental Examining Board of Canada 2017 . Anaphylaxis. Deeper overbite. Maxillary crowding. C. Incomplete anesthesia. 6 years. 4 years. content. pulp capping and composite resin 2. All of the above. E. and translation. D.Which of the following could be a What is the earliest age that a congenitally complication when performing a dental missing mandibular second bicuspid can be extraction on an insulin-dependent diabetic confirmed? patient? A. A. B. dentist administer epinephrine? C. Vasovagal syncope. pulpotomy and stainless steel crown. D. (1) (2) (3) B. pulp capping and amalgam restoration. C. A Bolton relationship has determined a  maxillary “12” excess of 3. Increased overjet. Erosion. D. restoration. A. C. 4. B. Acute adrenocortical insufficiency. have a horizontal handle. Maxillary spacing. B. Increased mobility of teeth. Radiographic evidence of the widening of the periodontal ligament. 1. For which of the following reasons would a B. 8 years. 3. (4) only E. Increased bleeding. create adequate space for the impression material. (2) and (4) complete denture impression must D. Irreversible hydrocolloid. alginate impressions B. between 21 and 30 years of age. Corrected conventional tomography. and translation. Primary herpetic gingivostomatitis most frequently occurs A. Addition silicone. before age 10. speed of the electrons. anterior nasal spine. can be poured twice with little effect on Increasing the kVp results in accuracy of the resulting cast. Peutz-Jeghers. 3. penumbra. Arthrography. C. C. D. (1) and (3) C. After setting. content. D. B.Kilovoltage controls the To ensure a clinically acceptable setting time. tartaric acid. B. after age 31. the format of some items is not currently used. D. contrast. Polyether. salicylic acid. A. A. have higher tear strength than polyvinylsiloxane impressions. (4) only E. Note: Some of the items in the Released Test Item Bank may have been discontinued due to outdated science or errors. All of the above. structures on periapical radiographs EXCEPT B. maleic acid. nasopalatine/incisive canal. ©The National Dental Examining Board of Canada 2017 . Gorlin-Goltz. In addition. itaconic acid. 4. D. B. D. increased long scale image contrast. B. nasal septum. Magnetic resonance imaging. phosphoric acid. A. C. B. TMJ disc? C. C. Computed tomography. Sjögren’s. C. Gardner’s. B. NOT recommended for making a final impression for fabrication of dies for a porcelain fused to metal crown? All of the following appear as midline A. absorb water. Which syndrome presents with multiple cysts of the jaws? A. A. A. What is the best imaging modality to assess the B. (1) (2) (3) E. D. polyalkenoic cements contain 1. The NDEB periodically reviews the bank to improve its quality. decreased long scale image contrast. penetrating power of radiation. 2. (1) and (4) D. between l1 and 20 years of age. Which of the following impression materials is D. C. increased short scale image contrast. C. decreased short scale image contrast. Condensation silicone. A. zygomatic process of the maxilla. D. remain dimensionally stable for 12 hours. ©The National Dental Examining Board of Canada 2017 . The NDEB periodically reviews the bank to improve its quality. increased diameter of the dentinal tubules. B. lower film thickness. lower solubility in oral fluids. reverse maxillary curve of Wilson. D. Gingivoaxial. ionized by the acidic pH. Axiopulpal. higher compressive strength. rapidly redistributed by the increased B. A. toxicity of vasoconstrictor. D. blood flow. Attachment loss. sodium. Increased tooth mobility. hydrogen peroxide and ethyl chloride. B. the format of some items is not currently used. C. D. Bleeding upon probing. rapidly degraded by released enzymes. D. chloride. C. which of the Local anesthetics are less effective in inflamed following indicates that gingivitis has tissue because they are progressed to periodontitis? A. calcium. content. Mesioaxial. C. A. possibility of intravascular administration. D. B. C. and translation. Local anesthetics block nerve conduction by interfering with ionic movement of A. Class III skeletal malocclusion. elimination of the collagen fibres. E. sodium hypochlorite and EDTA. D. elimination of the smear layer. calcium hydroxide and phenol. C. with maxillary anterior spacing is a/an glass ionomer cement has a/an A. Class II. potassium. In addition. B. chlorhexidine and chloroform. B. B. D.At a six month recall visit. toxicity of local anesthetic. C. D. demineralization of the superficial dentin. D. Aspiration prior to a local anesthetic injection Which line angle is NOT present in a Class V reduces the amalgam cavity preparation? A. Occlusoaxial. Change in gingival colour. The occlusal parameter most likely associated When compared to zinc phosphate cement. accentuated maxillary curve of Wilson. Distoaxial. possibility of paresthesia. C. C. diluted by the edematous fluid. A 10-15 second application of 37% phosphoric The smear layer created by root canal acid on prepared dentin will result in all of the instrumentation can be removed by following EXCEPT A. B. B. Note: Some of the items in the Released Test Item Bank may have been discontinued due to outdated science or errors. division 2 dental malocclusion. ability to release fluoride. A. C. A. long buccal block. antisialagogue properties? A. patient’s radiation dose is E. A. A rubber dam drain is always placed and sutured to assist drainage. the drug of choice for D. anterior superior alveolar block. A. fluctuant swelling. the presence of minimal attached gingiva. C. C. delay breakfast until after the extractions. During the administration of local anesthesia. an intravascular injection will occur most often D. Ibuprofen. bleeding on probing. C. Immediate cessation of pressure and pain. the management of angina is A. In addition to oxygen. Acetylsalicylic acid. incisive block. D. E. D. posterior superior alveolar block. ©The National Dental Examining Board of Canada 2017 . content. B. B. D. and translation. fast emulsion film. for the extraction of two teeth under local anesthesia? A. atropine. Delay breakfast and insulin until after the Which of the following drugs has the strongest dental appointment. The most effective way of minimizing a D. a lead apron and thyroid collar. B. C. C. appointment. prescription radiography. diphenhydramine. Codeine.Which of the following preoperative Which of the following is true about incision instructions are most appropriate for a well. In addition. B. C. Penicillin. Atropine. A. inferior alveolar block. epinephrine. The NDEB periodically reviews the bank to improve its quality. bone loss. Eat breakfast before the appointment and B. acetylsalicylic acid. The procedure is only indicated with a delay insulin injection until after the localized. A. B. intensifying screens.m. nitroglycerin. Note: Some of the items in the Released Test Item Bank may have been discontinued due to outdated science or errors. Eat breakfast before the appointment and take insulin according to the regime prescribed by the physician. Take insulin before the appointment and always possible. progressive attachment loss. A sign of gingivitis is D. Profound anesthesia of the surgical site is B. C. and drainage of an acute apical abscess controlled insulin-dependent diabetic patient (acute periradicular abscess)? who is scheduled at 9:00 a. in a/an E. the format of some items is not currently used. C. (2) and (4) impression then pouring the impression. has recently become ulcerated is D. The mesial furcation of maxillary first molars is D. inferior alveolar nerve block. Note: Some of the items in the Released Test Item Bank may have been discontinued due to outdated science or errors. excisional biopsy. B. Topical anesthesia. after the extraction without the denture in E. Ibuprofen. after the extraction with the denture in the mouth and pouring the impression leaving the denture in the impression. A. Which of the following is NOT a property of D. Pronounced midpalatal raphe. E. The NDEB periodically reviews the bank to improve its quality. present on the buccal mucosa for 6 months and C. best probed from the E. before the extraction with the denture in B. Which of the following cause sedation? 1. white lesion. B. (1) (2) (3) A. observation. Meperidine. the mouth. The impression 4. after the extraction with the denture in the lidocaine? mouth. Poor denture base adaptation. removing the denture from the C. Gow-Gates block. B. C. A. Muscle relaxation. C. Triazolam. buccal or lingual. buccal. removing the denture from the impression then pouring the impression. Penicillin V. Insufficient posterior palatal seal. C. D. and translation. B. that has been B. C. D. All of the above. Which drug is most adversely affected by ingestion of antacids? The appropriate management for an avascular A. 5 x 3mm in size. Cephalexin. content.The local anesthetic technique requiring the What is the most likely cause of a maxillary needle to contact the neck of the condyle is the denture dislodging when the patient opens wide or makes extreme lateral excursions? A. to add a tooth to the denture should be made A. A. incisional biopsy. Codeine. Coronoid process interference. D. In addition. D. (4) only B. Labial frenum impingement. aspiration biopsy. Vazirani-Akinosi block. posterior superior alveolar nerve block. the format of some items is not currently used. Erythromycin. C. 2. (1) and (3) the mouth. Antiarrhythmic action. lingual. A mandibular central incisor is to be extracted 3. B. cytologic examination. A. Tetracycline. and added to a partial denture. Local anesthesia. ©The National Dental Examining Board of Canada 2017 . E. Tetracycline. ©The National Dental Examining Board of Canada 2017 . celecoxib inhibits both COX-1 and COX-2. Penicillin. Magnesium trisilicate. lower renal toxicity. celecoxib inhibits mostly COX-2. COX-2. Which of the following is NOT an effect of Which of the following drugs can interfere with acetaminophen? the effectiveness of oral contraceptives? A. Codeine. Anticholinergics. A. increased platelet aggregation inhibition. B. The framework fits the master cast well but when tried in the mouth a stable fit cannot be achieved. C. D. celecoxib inhibits mostly COX-2. Amoxicillin. A. B. C. D. Anticonvulsants. D. A. and translation. B. The most likely cause is Which class of drugs does NOT have analgesic effects? A. Acetaminophen. Penicillin V. increased safety for asthmatic patients. D. B. Ibuprofen inhibits mostly COX-1. B. Ibuprofen inhibits both COX-1 and C. In addition. D. Antipyretic effect. long-term use LEAST appropriate to be prescribed to a of celecoxib is associated with pregnant patient? A. control of secondary infection. Analgesia. Erythromycin. distortion in the final impression. C. increased cardiac protection. application of dilute hydrogen peroxide. topical steroids. D. the format of some items is not currently used. B. Inhibition of CNS cyclooxygenase. D. C. partial denture framework. shrinkage of the alloy during casting. When comparing ibuprofen and celecoxib used in therapeutic doses. B. improper major connector design.Which of the following antibiotics is the Compared to acetylsalycilic acid. Antidepressants. reduced gastrointestinal ulcerations. improper casting temperature. C. The NDEB periodically reviews the bank to improve its quality. Salicylates. C. None of the above. Anti-inflammatory effect. content. C. D. B. Ibuprofen inhibits mostly COX-1. E. Note: Some of the items in the Released Test Item Bank may have been discontinued due to outdated science or errors. A. Both ibuprofen and celecoxib are potent A dental laboratory has fabricated a removable inhibitors of COX-1 and COX-2. which of the following Treatment of primary herpectic statements is true? gingivostomatitis should include A. cauterization. Xenogenic. B. increase its chemical reactivity. Note: Some of the items in the Released Test Item Bank may have been discontinued due to outdated science or errors. periodontally involved abutment teeth. V. 2400 mg. ©The National Dental Examining Board of Canada 2017 . B. xerostomia. E. D. D. C. Alloplastic. localized.g. spraying a surfactant on the impression. D. A. D. B. A. increase its fusion temperature. The procedure is only indicated with a D. 4000 mg. diarrhea. M. evidence of bruxism. Increasing the amount of network modifiers (e. decrease its thermal expansion. achieving drainage. C. D. hydrocolloid. deep vertical anterior overlap. E.Which of the following statements about A zirconia-based ceramic fixed partial denture incision and drainage of an acute apical abscess can be used for a patient with (acute periradicular abscess) is FALSE? A. D. high surface tension of an irreversible C. C. convulsions. upon heating. C. J. C. fluctuant swelling. B. tremors. F. 3600 mg. The incision should be sutured after B. B. low surface tension of a silicone A. Na2O) in porcelain will A drug with anticholinergic side effects may cause A. CaO. Profound anesthesia of the surgical site is not always possible. cantilever pontic. using a hydrophilized addition silicone. D. likely to have a successful result? B. Which sound is associated with the most common speech defect in complete denture patients? Which type of bone grafting material is most A. A. content. B. the format of some items is not currently used. decrease its potential for devitrification C. The NDEB periodically reviews the bank to improve its quality. Allogenic. S. In addition. and translation. impression material. C. Voids in a gypsum cast are most likely the For acute dental pain. the daily maximum result of cumulative dose of acetaminophen is A. long clinical crowns. Autogenous. 3200 mg. Relief of the pressure and pain is immediate after treatment. (4) only D. (1) (2) (3) A. The NDEB periodically reviews the bank to improve its quality. the format of some items is not currently used. following intravenous antibiotics is most 3. A patient had a coronary arterial stent placed 1 year ago following a myocardial infarction and has been asymptomatic since. E. perform only emergency dental treatment D. Sulfonamide. E. crowding. ©The National Dental Examining Board of Canada 2017 . avoid using epinephrine impregnated A. D. overjet. All of the above. D. not recline the patient more than 45 degrees. A. prescribe antibiotics to prevent infective endocarditis. The most The most appropriate type of bone grafting appropriate management is to material for use in the maxillofacial region is A. mild generalized chronic periodontitis. actinomycosis is suspected. retraction cord. B. (1) and (3) B. Note: Some of the items in the Released Test Item Bank may have been discontinued due to outdated science or errors. condition? A. seizure. appropriate for the management of the 4. purulent increased draining fistula in the right submandibular area. D. probing. 55 year old patient has alveolar bone The most common medical emergency in the loss on 40% of teeth. overbite. content. xenogenic. B. B. E. Erythromycin. The most likely diagnosis is B. C. C. and translation. A healthy. syncopal episode. Cephalosporin. moderate localized chronic periodontitis. In addition. myocardial infarction. autogenous. for 1 year. B. allergic reaction. C. 3-4mm of clinical dental office is a/an attachment loss and probing pocket depths ranging between 5-6mm with bleeding upon A. Penicillin. C. allogenic. Aminoglycoside. mild localized chronic periodontitis.Overeruption of the posterior teeth results in A patient presents with a chronic. use local anesthetics without epinephrine. Culture results are inconclusive but 1. cerebrovascular accident. alloplastic. moderate generalized chronic periodontitis. E. C. D. (2) and (4) C. rotation. Which of the 2. generator during the exposure. reinforce the root. C. prescribe a different antibiotic. A. diabetes mellitus. stand behind a barrier during exposure. C.1? A. A. C. and translation. content. use chlorhexidine solution as a rinse prior to therapy. retain the core build up. ©The National Dental Examining Board of Canada 2017 . A. Doxepin. Ammonium thiosulfite. reinforce the remaining coronal tooth D. Extrusion. C. Verapamil. D. management for periodontal therapy for this C. stand 2 metres away from the x-ray B. B. D. B. C. B. A. mitral valve prolapse with regurgitation. Phenytoin. functional heart murmur. wear a radiation monitoring device (film badge) during exposure. which traumatic dental injury personnel who expose radiographs is to have is most likely to result in pulp necrosis on them tooth 1. Intrusion. patient is to D. Subluxation. B. prosthetic heart valve. The NDEB periodically reviews the bank to improve its quality. wear a lead apron during exposure. Sodium sulfite. When compared to quaternary ammonium oral rinses. structure. concentration. E. substantivity. potency. The most appropriate B. Silver. toxicity.Antiobiotic prophylaxis prior to tooth A post is used in an endodontically treated extraction is required for (a) tooth to A. obturate the canal. C. Which of the following drugs will NOT cause gingival hyperplasia? Which substance found in radiograph processing solutions is of most concern A. 0. In addition. D. D. the format of some items is not currently used. maintain penicillin at the present level. D. increase the dosage of penicillin. organic heart murmur. Note: Some of the items in the Released Test Item Bank may have been discontinued due to outdated science or errors. The most effective method for protecting dental In a 17 year old. unrelated condition. Luxation. Cyclosporine.12% chlorhexidine oral rinses have a higher antimicrobial activity due to increased A patient with a history of infective endocarditis is currently taking penicillin for an A. environmentally? B. C. Acetic acid. B. D. local destruction of normal oral flora. order a complete blood count. E. thickening of the basement membrane. B. obtain bacterial cultures. Which of the following symptoms are B. perform an incisional biopsy. Ketorolac 10 mg every 4 hours. 4 hours. vomiting. two-wall osseous defects. A. Note: Some of the items in the Released Test Item Bank may have been discontinued due to outdated science or errors. and cardiovascular collapse. C. and translation. The most appropriate initial severe asthma and nasal polyps following an management of this patient is to emergency pulpectomy? A. Cyclosporine. C. E. and bronchial constriction. D. hematuria. Ibuprofen. nausea. Deafness. the format of some items is not currently used. obtain a cytologic smear. consistent with an anaphylatic reaction to C. Urticaria. A pale 8 year old patient has generalized which of the following is/are appropriate for gingival enlargement and spontaneous pain management for an adult with a history of bleeding. The prognosis of guided tissue regeneration C. Which of the following medications increases a patient’s risk for intraoral candidiasis? A. obtain a fasting blood glucose level. increased number of epithelial cells. B. D. B. Acetaminophen 1000 mg every 6 hours. horizontal bone loss. local immunosupression. A. A. one-wall osseous defects. Pentobarbital. content. cross-reacting antigens in the tongue. dizziness. E. ©The National Dental Examining Board of Canada 2017 . In addition.Assuming the daily maximum is not exceeded. Acetylsalicylic acid 650 mg every C. prolonged local vasoconstriction. diarrhea. Oliguria. bronchial constriction A. D. C. increased production of proteoglycans. Crystalluria. every 6 to 8 hours. C. B. D. Pilocarpine. D. diarrhea. acute anemia and bronchial constriction. a systemic antibacterial action. three-wall osseous defects. bronchial (GTR) is best for the treatment of constriction. Hydantoin treatment causes gingival Asthmatic patients using corticosteroid inhalers hyperplasia as a result of a/an may develop candidiasis on the dorsal surface of the tongue because of A. Naproxen 250 mg. B. The NDEB periodically reviews the bank to improve its quality. Warfarin. penicillin? D. and cardiovascular collapse. B. 1. C. the format of some items is not currently used. E. D. E. D.6. hydrogen peroxide. are polarized in opposite directions. the most effective A.4. ©The National Dental Examining Board of Canada 2017 . Note: Some of the items in the Released Test Item Bank may have been discontinued due to outdated science or errors. The usual adult dosage of codeine administered orally is A. palpation. penicillin V. B. 250-500mg. D. D. irrigating solution for dissolving organic debris B. C. C. (2) and (4) D. sodium hypochlorite. Pain to cold that ceases after removal of B.7. the stimulus. an allergic reaction to the drug. Draining sinus tract that traces to the apex of the tooth. content. 2-5mg. Which element found in radiograph processing solutions is of most concern environmentally? In endodontic therapy. (1) and (3) to develop a furcation involvement? C. The NDEB periodically reviews the bank to improve its quality. 1. C. erythromycin. 30-60mg. B. within minutes after drug administration. calcium hydroxide. ameloblasts and An anaphylactic reaction to penicillin is most odontoblasts likely to occur A. 4. C. In addition. None of the above. Silver. ethylenediaminetetraacetic acid (EDTA). and translation. 2. A. in patients who have already experienced C. 1.5. (4) only A. A. 3. D. D.In a developing crown. begin to produce matrix simultaneously. nonmineralized matrix. 500-1000mg. 1. initially produce an organic parenterally. when the drug is administered B. origin is B. A. metronidazole. (1) (2) (3) Which of the following teeth is LEAST likely B. Lead. is C. Copper. All of the above. Mercury. have rapid cell division. 1. tetracycline. Which of the following is consistent with reversible pulpitis? A. Painful response to percussion and D. when the drug is administered orally. B. Discontinuous lamina dura and a The antibiotic of choice for infections of pulpal periapical radiolucency. Metronidazole. Which of the following drugs is the most appropriate treatment? A. immediately prior to the dental procedure. nerve. ©The National Dental Examining Board of Canada 2017 . The tissue which is most sensitive to radiation B. elevating the legs. D. water:powder ratio. B. D. ensuring the airway is open. lymphoid. lupus erythematosus. muscle. Acetylsalicylic acid 650 mg every 4 hours. D. C. pemphigus vulgaris. A 43 year old patient with a history of severe A positive Nikolsky's sign is a diagnostic asthma and nasal polyps has an emergency feature of pulpotomy. He has signs and C. and translation. four hours prior to the dental procedure. E. erythema multiforme. Phenethicillin. completed a 7-day course of ampicillin for a B. administering oxygen. C. C. Acetaminophen 650 mg every 4 hours. Acyclovir. content. respiratory infection. chronic marginal gingivitis. mixing time. composition. Nafcillin. lichen planus. D. The NDEB periodically reviews the bank to improve its quality. Nystatin. E. Clindamycin. one day prior to the dental procedure. C. E. B. Assuming daily recommended doses are followed. administering epinephrine. B. The best method to control the setting time of Which of the following penicillins is most an irreversible hydrocolloid without affecting effective against Gram-negative organisms? its physical properties is to alter the A. A. Naproxen 250 mg every 6-8 hours. oral amoxicillin should anesthetic administration does NOT include be taken A. A. two days prior to the dental procedure. Methicillin. contains elastic fibres.When given prophylactically to prevent The management of syncope following local infective endocarditis. is keratinized. symptoms consistent with oral candidiasis. A. is C. one hour prior to the dental procedure. C. placing in a supine position. the format of some items is not currently used. In addition. C. is closely bound to underlying muscle. B. dental pulp. Ampicillin. Penicillin V. D. Ampicillin. the most appropriate drug management? B. Ketorolac 10 mg every 4 hours. E. A. E. D. water temperature. D. Note: Some of the items in the Released Test Item Bank may have been discontinued due to outdated science or errors. has no basal cell layer. Healthy attached gingiva A 70 year old insulin-dependent patient has just A. B. which of the following is A. B. D. D. C. C. C. prn. indicator of periodontal stability? A. D. B. A. Codeine. followed by 400 mg. Gingival inflammation. between 25 and 50%. anterior crowding. the format of some items is not currently used.Which of the following should NOT be In severe gingival recession. gingivalis. Absence of bleeding on probing. A. the A. Which of the following should NOT be prescribed for a patient receiving warfarin? A gingival pocket (pseudopocket) is A. Ketorolac. if the marginal prescribed for a patient receiving warfarin? tissue extends to the mucogingival junction. Acetaminophen. B. Codeine. Hydromorphone 2 mg. root caries of the adjacent tooth. P. deepened probing depth. is generally not progressive in nature. P. collagenolytic activity in chronic periodontitis? D. Oxycodone. ©The National Dental Examining Board of Canada 2017 . Macrophages. Acetaminophen. 1 hour preoperatively. B. Neutrophils. B. Plaque index. B. A. A. jaw fracture from contact sports. intermedia. Ibuprofen 800 mg. In addition. Acetaminophen 650 mg. characterized by B. What is the primary source of elevated C. D. and translation. every 4-6 hours C. Periodontitis D. likelihood of complete root coverage after B. C. Oxycodone. Note: Some of the items in the Released Test Item Bank may have been discontinued due to outdated science or errors. B. affects individuals with various susceptibility at different rates. A. content. Which of the following is appropriate to provide postoperative pain control for an 80kg. every 4-6 hours prn. C. B. Acetylsalicylic acid. every 4-6 hours prn. greater than 75%. Patient compliance with maintenance visit. and there is loss of interdental tissue. with oxycodone Which of the following is the most predictable 10 mg. loss of alveolar bone. loss of attachment. gingival grafting is C. affects different parts of dentition in the same individual similarly. less than 20%. C. The NDEB periodically reviews the bank to improve its quality. C. The removal of a partially impacted mandibular 20 year old patient with an allergy to codeine third molar is appropriate for the prevention of following the removal of 4 erupted third molars? A. (1) (2) (3) B. C. D. Prednisone. Note: Some of the items in the Released Test Item Bank may have been discontinued due to outdated science or errors. (1) and (3) Which of the following is most likely to have C. Ibuprofen. In a patient with renal failure. C. Cortisone. Ibuprofen. A. Penicillin V. Dexamethasone. Penicillin V. 3. Codeine. B. Amitriptyline. 2. In addition. Which of the following is the most potent D. Clonazepam. Calculus attaches to tooth surfaces by all of the The Controlled Drugs and Substances Act following EXCEPT controls the distribution and use of which of the following? A. Triamcinolone. Metronidazole. All of the above. mechanical locking to tooth/root A. A. Diazepam. ©The National Dental Examining Board of Canada 2017 . C. Amitriptyline. Phenelzine. Clindamycin. Oxycodone. hemidesmosomes. which of the Which of the following is associated with drug- following does/do need a dose interval induced bruxism? adjustment? A. D. A. D. irregularities. D. E. Root sensitivity. B. 4. content. the format of some items is not currently used. A. corticosteroid? E. 1. Acetaminophen. C. B. Bone level. Ibuprofen. (2) and (4) its effect reduced in a patient with genetic D. Amoxicillin. The NDEB periodically reviews the bank to improve its quality.In addition to plaque index and probing depths. Amoxicillin. Acetaminophen. Clindamycin. and translation. B. organic pellicle. B. B. C. Erythromycin. B. Bleeding on probing. C. Fluoxetine. D. (4) only polymorphism of CYP2D6? E. close adaption to cementum. The Controlled Drugs and Substances Act which of the following must be re-examined controls the distribution and use of which of the following the completion of initial periodontal following? therapy? A. C. Hydrocortisone. D. Which drug is indicated for the management of B. Arrested carious lesions in enamel are softer than intact enamel. B. Decreasing the retentive undercut. ©The National Dental Examining Board of Canada 2017 . dimension of occlusion. Note: Some of the items in the Released Test Item Bank may have been discontinued due to outdated science or errors. to complete the prescribed medication. to avoid consuming alcohol. the most likely to predispose a patient to seizures? physiologic rest position A. C. Sodium cromoglycate. C. provides a quide to establish the vertical B. Decreasing the taper. In addition. B. an acute asthmatic attack C. Increasing the length. content. C. D. high testosterone levels. Salbutamol. the format of some items is not currently used. Ibuprofen. D. Acetaminophen. Increasing the diameter. provides a guide for the selection of cusp E. E. Codeine. The NDEB periodically reviews the bank to improve its quality. C. determines the shape of the compensating curve. appropriate in the management of a patient experiencing an acute asthmatic attack? A. is useful to the determination of condylar Which of the following medications is most inclination. B. Fluticasone. D. A. B. Patients who have been prescribed an acetaminophen/codeine combination must be advised Osteoporosis is linked to A. Zileuton. and translation. A. B. inclination. Caries progression through enamel can A. B. that it can cause hypertension. Triamcinolone. C. high progesterones levels. enamel. Which of the following statements is true? Which of the following will increase the A. Meperidine. low estrogen levels. A. D. D. C.Administration of which of the following is In complete denture contruction. determines the level of the occlusal plane. Fluticasone. take as long as 6 to 8 years. Salbutamol. low androgen levels. D. D. A major percentage of radiographically flexibility of a removable partial denture clasp detected proximal radiolucensies in enamel arm? are cavitated. Ketorolac. Budesonide. that it can cause diarrhea. Caries progression can only be arrested in C. due to its ability to A. agent for conscious sedation. 3. but she was not concerned submandibular gland. A. C. C. 2 B. non-selectively reduce the oral microbial flora. D. general anesthetic agent. All of the D. ©The National Dental Examining Board of Canada 2017 . 2000 Which of the following are signs of aggressive The primary use of nitrous oxide and oxygen is periodontitis? as a(n) 1. A. since she has had a similar condition all of her E. C. A.8 C. Specific periodontal microbial pathogens. D. (1) (2) (3) B. C. loss of taste from the anterior 2/3 on the left side of the tongue. B. Prednisolone. the format of some items is not currently used. The NDEB periodically reviews the bank to improve its quality. allergic reaction to cinnamon. decreased salivary output from the left at least 3 years. 20 C. Chlorhexidine is an effective antiplaque agent D. on the buccal mucosa. The mother tells the sublingual gland. (2) and (4) the lingual nerve is damaged.One millilitre of a 2 solution of lidocaine Which of the following drugs does NOT cause hydrochloride contains how many milligrams gingival enlargement? of the drug? A. leukoplakia. (1) and (3) During the extraction of an impacted tooth 3. All of the above. dentist that she has noticed this appearance for D. Nifedipine. and translation. white sponge nevus. A. dorsum of the tongue C. neutralize the bacterial end-products such as lactic acid. numbness of the floor of the mouth on the life. frictional keratosis. Onset before the age of 35. Cyclosporine. Rapid attachment loss. deviation of the tongue to the left on A 10 year old has diffuse bilateral white areas protrusion. (4) only following can occur EXCEPT a E. damage the microbial cell membrane. B. Note: Some of the items in the Released Test Item Bank may have been discontinued due to outdated science or errors. bind to the positively charged pellicle. B. The most likely diagnosis is left side. 4. In addition. substitute agent for local anesthesia. Ulcerations of the gingiva. A. decreased salivary output from the left and the floor of the mouth. Phenytoin. B. content. B. 2. 200 D. precludes the use of local anesthesia. the maxillary and mandibular cuspids. Multiple myeloma.A patient on broad spectrum antibiotics for Which of the following antibiotics may be 4 weeks has widespread. only the fossae of teeth. sore. B. wrinkle her forehead or smile on the left side. a chemical bond to clean dentin? C. contralateral subarachnoid hemorrhage. C. leukoplakia. pemphigoid. cusps of mandibular teeth. the format of some items is not currently used. A. The most likely diagnosis is The selective grinding rule for removing premature contacts on the working side of A. B. requires the services of an anesthesiologist. D. Pemphigus vulgaris. C. Hygienic. Horner's syndrome. D. Neomycin. The use of conscious sedation for a restorative E. ©The National Dental Examining Board of Canada 2017 . D. C. candidiasis. red and white cross-allergenic with penicillin? oral mucosal lesions. E. D. Saddle. B. A patient is not able to close her left eye. Erythromycin. Zinc-oxide and eugenol. Note: Some of the items in the Released Test Item Bank may have been discontinued due to outdated science or errors. Calcium hydroxide. is contraindicated in children. acute mastoiditis. procedure in an office environment A. lingual and buccal cusps of maxillary teeth. In addition. C. E. C. The most likely diagnosis is A. may be used in conjunction with B. E. only the mandibular teeth. Ovoid. Which of the following dental materials shows B. narcotics. Glass ionomer. C. Conical. A. the buccal cusps of maxillary and lingual E. facial nerve paralysis. and translation. Cephalexin. B. content. A. fracture of the base of the skull. Erythema multiforme. erythema multiforme. D. Zinc-phosphate. Monocytic leukemia. Tetracycline. D. B. Clindamycin. Modified ridge lap. A differential diagnosis for gingival hyperplasia should include which of the Which of the following pontic designs will following conditions? make it most difficult to maintain optimal oral hygiene? A. The NDEB periodically reviews the bank to improve its quality. erosive lichen planus. complete dentures is to grind B. A. C. D. D. E. requires effective local anesthesia. (1) (2) (3) B. Undereruption. (4) only recording (PSR) for a patient. Note: Some of the items in the Released Test Item Bank may have been discontinued due to outdated science or errors. eccentric movements by multiple C. 4. B. heavily restored. strong. moderate. A. a tooth in the sextant has supraerupted. lateroprotrusive registrations. D. C. A. B. D. definitive. A. decreased eruption of the anterior teeth. B. Which maxillary central incisor characteristic The etiology of an open bite malocclusion of is the most limiting in the construction of a dental origin is most often the result of Class II functional appliance? 1. Periodontal pockets CANNOT be reduced by When an orthodontic force is applied to a maxillary right canine. used in adult treatment. increased eruption of the anterior teeth. guided tissue regeneration. mobile. increased eruption of the posterior teeth. D. B. decreased eruption of the posterior teeth. Overeruption. ©The National Dental Examining Board of Canada 2017 . The NDEB periodically reviews the bank to improve its quality. periodontal ligament is altered within B. labially prominent. 3. and translation. 2. the format of some items is not currently used. All of the above. Retroclination. nonvital. D. one hinge axis movement. weak. open flap curettage. a mucogingival problem is present. C. blood flow to the A. the code E. days. C. D. dentition. hours. delayed until the eruption of the permanent C. A.Isolated gingival recession is most frequently Orthodontic growth modification should be seen on teeth that are A. B. B. D. D. (1) and (3) C. C. (2) and (4) When performing a periodontal screening and D. B. A. asterisk (*) is used for a sextant when A. Proclination. large centric relation-maximum intercuspation slides. seconds. C. A modified hinge non-adjustable articulator is limited in its accuracy to reproduce dynamic jaw movements because it can only reproduce The correlation between malocclusion and temporomandibular dysfunction is A. content. a tooth in the sextant needs to be extracted. occlusal adjustment. started as early as possible. C. minutes. all the teeth in the sextant are missing. scaling and root planing. individualized to maximize effect. In addition. junctional epithelium. (1) and (3) D. 2. content. C. 7 other permanent teeth. 2. The NDEB periodically reviews the bank to improve its quality. antiplatelet action. possible from the distal extension base(s). Locating direct and indirect retainers as far as possible from the distal extension base(s). transverse fibres of the periodontal ligament. D. B. B.The etiology of a deep bite malocclusion of Which of the following is/are locally delivered dental origin is most often the result of antimicrobial(s) agent(s) used to treat infected periodontal pockets? 1. extend to the B. B. (1) (2) (3) C. All of the above. decreased eruption of the posterior teeth. 1. Doxycycline. (2) and (4) E. Ibuprofen. close as possible to the distal extension D. increased eruption of the anterior teeth. (1) and (3) A. A. In addition. A daily dose of 81 mg of acetylsalicylic acid is C. B. (1) (2) (3) B. base(s). D. anti-inflammatory function. 3. E. Chlorhexidine. attached gingiva. 5 other permanent teeth. from the distal extension base(s) and the D. 1 other permanent tooth. the probe tip will most likely A. ©The National Dental Examining Board of Canada 2017 . 3. Locating direct retainers as far as possible C. Which of the following analgesics can exacerbate peptic ulcers? When probing a healthy peridontium using light forces. 4. indirect retainers as close as possible to the distal extension base(s). 4. Generalized aggressive periodontitis is characterized by attachment loss affecting the Which of the following will increase resistance first molars and incisors and at least to dislodging forces on a removable partial denture? A. All of the above. increased eruption of the posterior teeth. Note: Some of the items in the Released Test Item Bank may have been discontinued due to outdated science or errors. analgesic properties. D. (2) and (4) B. A. the format of some items is not currently used. (4) only C. Acetaminophen. 3 other permanent teeth. Locating direct and indirect retainers as C. Codeine. A. antipyretic effect. (4) only. C. Clindamycin. and translation. decreased eruption of the anterior teeth. Metronidazole. Tramadol. Locating direct retainers as close as used for its possible to the distal extension base(s) and the indirect retainers as far as A. A fixed partial denture with a single pontic is deflected a certain amount. battered child syndrome. Anterior labial. D. can be left in place when What is the Miller’s class for gingival recession that extends beyond the mucogingival junction A. Angulation of the impaction. In addition. close to the tooth roots. C. C. content. four times as much. creep. Posterior buccal. III. Abnormal mobility. low modulus. The NDEB periodically reviews the bank to improve its quality. B. C. D. there is no plan to place a dental implant with no interproximal attachment loss? in the site. eight times as much. D. Shape of the crown. B. von Willebrand desease. deep in the bone. coherent manner. twice as much. B. Anterior palate. the root is small. I. The mucogingival junction with no Following extraction of teeth. D. a span of two A 6 year old has circumscribed suppurative similar pontics will deflect lesions that look like cigarette burns on the left ear lobe and the right knuckles. osteogenesis imperfecta. B. B. C. high ductility. Note: Some of the items in the Released Test Item Bank may have been discontinued due to outdated science or errors. stress relaxation. and translation. Drug-induced gingival overgrowth will most Which of the following is most likely to affect likely occur on the interdental papillae of the difficulty of extraction of an impacted which area? mandibular third molar? A. D. C. II. the root fragment is mobile and no more than 7-8mm in length. root fragments interproximal attachment loss is a Miller Class. Loss of thermal sensitivity. Posterior palate. D. cooperative and respond to the dentist in a B. IV. A. A. Münchhausen syndrome. B. Size of the pulp chamber. A. C. the format of some items is not currently used. B. the maxillary sinus is pneumatized and C. E. D.Which of the following is NOT a sign of pulpal The need to frequently replace intact necrosis in immature teeth? orthodontic elastics is a direct consequence of A. D. The most likely diagnosis is C. Coronal discolouration. impetigo. Root width. ©The National Dental Examining Board of Canada 2017 . The parents are A. not infected and located A. Periradicular radiolucency. the same amount. calcium salt. C. have higher energy. Asthmatic reaction to acetylsalicylic acid. A gastric ulcer. mistaken as a pathologic finding on periapical radiographs of the premolar segments of the maxilla? Which of the following is a A. Concurrent use of penicillin V. Nasolacrimal canal. What is the most likely diagnosis? A. Nasolacrimal canal. decrease fluidity. Which of the following may be mistaken as a pathologic finding on periapical radiographs of the premolar segments of the maxillary dental arch? A. A. drugs is the most appropriate management? A. excruciating pain in the right A. Stainless-steel. Which of the following structures may be D. duration and can be elicited by lightly touching C. Cobalt chromium. C. B. C. Succinylcholine.An infant has asymptomatic small whitish A common filler added to resin to produce outgrowths at the junction of the soft palate and dental composites is hard palate. B. B. In addition. An allergy to acetaminophen. C. minimize oxidation. Which of the following orthodontic alloy wires C. The pain is of short B. Carbamazepine. D. Nickel-titanium. Tylenol 3®? C. Nasopalatine foramen. B. zinc oxide. quartz. B. In comparison to visible light. ß-titanium. B. Penicillin. Nasopalatine foramen. A. enhance melting. ©The National Dental Examining Board of Canada 2017 . have a longer wave length. Fordyce granules. X-rays D. Bony septa in the maxillary sinus. Epstein pearls. increase stiffness. infraorbital region. Flux is added to the casting metal during melting to A healthy 40 year old female patient complains of periodic acute. Note: Some of the items in the Released Test Item Bank may have been discontinued due to outdated science or errors. the skin of the area. has the greatest effectiveness of activation? D. Acetazolamide. C. travel faster. Pneumatisation of the alveolar process. D. content. CONTRAINDICATION to the use of B. D. can be focused. D. Dental lamina cysts. Which of the following D. and translation. A. Bohn nodules. Mental foramen. B. Mental foramen. the format of some items is not currently used. C. The NDEB periodically reviews the bank to improve its quality. A. E. In addition. Odontoma. B. Ameloblastic fibroma. A. and translation. E. Vasovagal syncope. a D. hyperbaric oxygen treatment prior to tooth extraction. B. unexplainable genetic factors. room. prior to the radiation therapy. Five years ago. hyperbaric oxygen treatment prior to and D. a 35 year old patient received D. content. Myocardial infarction. The most appropriate If removal of teeth is indicated in a patient who management is is to receive radiation therapy for a carcinoma of the tongue. the teeth should be extracted A.000 epinephrine. well-defined in the anterior maxilla. prolonged retention of a primary incisor. modulated radiotherapy of 3000 cGy to the mandible for treatment of Hodgkin’s disease. The NDEB periodically reviews the bank to improve its quality. An anaphylactoid reaction. An anterior crossbite of a permanent maxillary Which of the following conditions produces a incisor in a mixed dentition is most often radiolucent image? associated with A. Note: Some of the items in the Released Test Item Bank may have been discontinued due to outdated science or errors. C. chemotherapy followed by adjuvant intensity. ill-defined in the anterior maxilla. most appropriate management is to B. A. lingually situated supernumerary teeth. take additional antihypertensive Which is the most likely cause of this reaction? medication. extract the tooth using a maximum of four cartridges. of 2% lidocaine with 1:100. extraction. extract the tooth using a maximum of two Following an inferior alveolar block injection cartridges. B.7 with local anesthesia is planned. An allergic reaction. D. Sialolithiasis. premature eruption of a maxillary incisor. to prescribe antibiotics following tooth B.A patient with a history of hypertension The most frequent radiographic appearance of presents for an emergency dental extraction. C. ©The National Dental Examining Board of Canada 2017 . ill-defined in the posterior mandible. metastatic carcinoma in the jaws is a His blood pressure is 158/100 mmHg. Intravascular injection. A. D. well-defined in the posterior mandible. Osteosclerosis. B. The extraction of tooth number 3. extract the tooth and advise the patient to patient experiences a transient tachycardia. six months post-radiation therapy. C. a functional shift.000 epinephrine for local anesthesia. The radiolucency dentist intends to use lidocaine 2% with 1:100. direct the patient to a hospital emergency D. B. The A. A. the format of some items is not currently used. C. C. immediately post-radiation therapy. following tooth extraction. C. during the radiation therapy. C. levels of plaque fluid in deep periodontal E. D. D. D. B. mandibular second premolar to the second molar. cratering. destruction. C. B. maxillary canine to the first molar. ©The National Dental Examining Board of Canada 2017 . C. D. periodontitis? B. mandibular central incisor to the first premolar. content. 500 psig. altered texture. B. Amount of microbial deposits inconsistent with disease severity. Diseased sites infected with Aggregatibacter (Actinobacillus) actinomycetemcomitans. A. delayed tooth eruption. C. dentures cover his taste buds. maxillary first premolar to the first molar. and translation. bleeding. D. Rapid attachment loss and bone D. codeine. A. B. 1500 psig. steroid hormones act as growth factors. C. edema. free nerve endings are now covered by the B. In addition. In Turner’s syndrome. A. A.Which of the following unilateral fixed partial When an oxygen tank is half full. C. the format of some items is not currently used. Note: Some of the items in the Released Test Item Bank may have been discontinued due to outdated science or errors. a wide maxilla. B. micrognathia. The NDEB periodically reviews the bank to improve its quality. 1000 psig. dentures. pockets. gingival blood vessels with increased B. C. C. tramadol. The most reliable diagnostic sign of gingival Prevotella intermedia increases significantly in inflammation is pregnancy gingivitis because of increased A. the patient is most likely to exhibit Which of the following characteristics is NOT seen in all patients with aggressive A. A 50 year old edentulous patient is complaining that spicy foods are no longer enjoyable since he received his new complete upper and lower dentures. ibuprofen. oligodontia. 2000 psig. acetaminophen. C. what is the dentures is most likely to have insufficient pressure gauge reading? periodontal support? A fixed partial denture extending from the A. colonization of shallow pockets. inflammation. Familial aggregation of diseased individuals. dentures are obstructing air flow to the D. olfactory epithelium. retractability. The most likely The antithrombotic effects of acetylsalicylic reason for this occurrence is that the acid may be compromised by A. canines. the format of some items is not currently used. C. edentulous patient if A. the patient is wearing dentures when the transposed are the image is produced. minimal surgical debridement after delaying 3 months. administered bisphosphonates for the past 3 years. 2. C. (1) and (3) C. B. thermal test. (2) and (4) D. minimal surgical debridement A. B. periapical. D. 3. diagnosis of periradicular periodontitis associated gingivitis? 1. In addition. aggressive surgical debridement C. mandibular canine and mandibular first image. (1) (2) (3) B. periodontal. D. premolars. probing. ©The National Dental Examining Board of Canada 2017 . D. pulpal. A. The NDEB periodically reviews the bank to improve its quality. content. a digital panoramic machine is used. panoramic radiographes of a completely C. tooth results in a positive response. Prostaglandins. Proinflammatory cytokines. premolar. Note: Some of the items in the Released Test Item Bank may have been discontinued due to outdated science or errors. 4. Matrix metalloproteinases. When percussion on the occlusal surface of a E. B. periapical radiograph. A. the most likely etiology of inflammation is A. immediately. The permanent teeth most commonly B. they are not close enough to the focal trough of the machine. Leukotoxin. E. C. D. molars. mandibular incisor and mandibular canine. She now has an area of exposed necrotic bone with sharp edges in an edentulous maxilla.Which of the following has a direct effect on Which of the following tests is/are useful in the periodontal tissues in diabetes mellitus. Lipopolysaccharides. maxillary canine and maxillary first A 50 year old female had intravenously premolar. incisors. immediately. aggressive surgical debridement after delaying 3 months. and translation. (4) only. maxillary upper lateral incisor and maxillary canine. electric pulp test. D. A. C. Residual root tips will not be visible on B. the patient moves during production of the B. All of the above. The most appropriate The permanent teeth most frequently ankylosed management is to perform are the A. D. C. 4. 50. B. B. content. and translation. chemotherapy and intensity-modulated A. management for the extraction of tooth 3. 0º. – 5º. The NDEB periodically reviews the bank to improve its quality. 0. supernumerary? His medical management involved surgery. following tooth extraction. A. What is the most appropriate film size for making an occlusal radiograph in a 3 year old Which of the following statements is true patient? regarding the radiographic appearance of furcation involvements? A. B. C. 12 months. 2. + 5º.A 55 year old patient had a squamous cell What percentage of natal teeth are carcinoma of the oral cavity 5 years previously. D. B. The percentage of documented child abuse D. hyperbaric oxygen treatment prior to tooth year old patient with high caries risk is extraction. radiograph. dentition is A. 9 months. Note: Some of the items in the Released Test Item Bank may have been discontinued due to outdated science or errors. 50-60. will still result in an accurate cast when poured D. B.7 under local anesthesia is A. 6 months. Furcation involvements cannot be seen on make a bite-wing radiograph in the primary radiographs. + 10º. using local anesthetic without epinephrine. cases estimated to involve orofacial injuries is A. ©The National Dental Examining Board of Canada 2017 . Which of the following impression materials C. A. transfusion with packed red blood cells. They are best evaluated with periapical radiographs. In addition. Polyether. The most appropriate C. radiotherapy (IMRT) of 65 Gy which included B. C. 90. Polysulfide. A definitive diagnosis can be made from a C. D. B. Bone loss is greater than it appears on a radiograph. C. 1. 10. 30-40. – 10º. Addition silicone. the format of some items is not currently used. two weeks after making the impression? E. C. 3 months. The most appropriate recall interval for an 8 B. the mandible. Condensation silicone. hyperbaric oxygen treatment prior to and A. The most appropriate vertical angulation to D. 10-20. Penicillin V. C. naloxone. All of the above. Multiple drug ingestion is less likely to 4.Which of the following drugs inhibits salivary The CNS depressant effects of midazolam are flow? reversed by the administration of A. B. Diazepam. D. D. Pilocarpine. Development of a dentigerous/follicular C. Propranolol. C. D. Which of the following statements regarding geriatric patients is correct? Mucosal thickening in the maxillary sinus A. D. Nystatin. Probantheline. flumazenil. Loratadine. Amitriptyline. D. A. Development of a benign neoplasm adjacent to its crown. content. Osteonecrosis of the adjacent bone. B. A. ranitidine. C. Which of the following drugs is CONTRAINDICATED in patients with Long Which of the following conditions is NOT a QT syndrome? sequela of a tooth completely impacted in bone? A. may be caused by a periapical infection. (2) and (4) their oral health. Glycopyrrolate. Note: Some of the items in the Released Test Item Bank may have been discontinued due to outdated science or errors. Acyclovir. the format of some items is not currently used. ©The National Dental Examining Board of Canada 2017 . D. may be associated with nasal discharge. Drugs that primarily depend on pulmonary 1. B. (1) and (3) to multiple side effects that may impact on C. (1) (2) (3) C. B. C. diphenhydramine. Azithromycin. A. (4) only E. B. C. Clotrimazole. Which of the following drugs inhibits salivary Which of the following drugs can be flow? administered to manage pain following the acute onset of a migraine? A. and translation. is often usually an incidental radiographic prescribed in smaller doses or at greater finding. may fill the entire air space of the sinus. B. The NDEB periodically reviews the bank to improve its quality. Nifedipine. Nystatin. In addition. time intervals. D. cyst around its crown. excretion for elimination should be 2. A. Multiple drug use predisposes the elderly B. External resorption of the tooth. result in toxicity due to inhibition of biotransformation. 3. Alendronate. B. Sumatriptan. D. extraction of planned when the patient is an adult.Which of the following lesions may appear The most appropriate time to initiate surgical radiolucent on a radiograph? treatment of a prognathic mandible immediately follows the A. let the permanent canine erupt into a mesial position and reposition it into an Angle’s Class I as soon as possible. late mixed dentition. tongue trap appliance. A. B. The vibrating line of the palate The most appropriate management for atypical swallowing is A. A. delineates the movement of the soft palate. A 10 year old patient is missing a permanent maxillary left lateral incisor. myofunctional treatment. C. encourage the permanent left canine to D. Note: Some of the items in the Released Test Item Bank may have been discontinued due to outdated science or errors. cessation of growth. A dental implant is In orthodontic treatment. B. A. A. E. producing physiologic actions opposite to D. Dense bone island/idiopathic osteosclerosis. D. B. C. competitive blocking of histamine at the cellular receptor site. C. improve stability. Periapical osseous dysplasia. increase in vertical dimension. There are no other orthodontic problems. decrease in the curve of Spee. occipital traction. that of histamine. early mixed dentition. correct a midline deviation. In addition. let the permanent canine erupt into a mesial position and reposition it into an Angle’s Class I before placing the implant. C. C. let the permanent canine erupt into a mesial position and place the implant distal to the canine. denture fabrication. C. primary dentition. increase in frequency of eruption B. is always on the hard palate. acting on the central nervous system. C. Osteopetrosis. and translation. the format of some items is not currently used. content. is a well-defined line across the palate. B. C. The NDEB periodically reviews the bank to improve its quality. D. D. Hawley appliance. Sialolith. extract the primary maxillary left canine as B. D. Premature loss of primary molars may result in Epinephrine antagonizes the effects of histamine by A. B. ©The National Dental Examining Board of Canada 2017 . is not a useful landmark in complete D. The most premolars will help to appropriate management is to A. B. erupt into an Angle’s Class I relationship. open the vertical dimension. decrease in the overbite and overjet. soon as possible. irregularities. labialize the incisors. preventing the release of histamine. lymphadenopathy and bad breath. What by the width of the is the most probable occlusal relationship at age 20? A. A. Hand-foot-mouth disease. The patient’s temperature is 38. Angle Class I with anterior overlap. the mesiodistal width of permanent an end-to-end first molar relationship with maxillary canines and premolars is determined normal skeletal and incisor relationships. Root resorption of primary teeth can be B. Erythema multiforme. Primary herpetic gingivostomatitis. gingiva. What is the most probable diagnosis? Root resorption of primary teeth can be delayed A. A. D. incisors. C. D. D. second molars. Angle Class I without anterior overlap. exam reveals bilateral painful regional B. In addition. C. canines. Chlorhexidine. solution for controlling pulpal bleeding after a pulp exposure prior to pulp capping? A. Sodium hypochlorite. ©The National Dental Examining Board of Canada 2017 . primary canines and mandibular molars. D.5ºC. seen on the hard palate. The mandibular intercanine width increases until full eruption of the permanent A 3 year old patient complains of pain of the mouth and headaches for 24 hours. Saline. occlusal crossbite. third molars. the tongue and the lips. and translation.In a Tanaka and Johnson mixed dentition A patient in the late mixed dentition stage has analysis. B. primary canines and maxillary molars. the presence of premature occlusal contact. Angle Class II with anterior overlap. the absence of a permanent replacement. B. the absence of a permanent replacement. Angle Class II without anterior overlap. Herpetiform aphtaus ulcers. C. A. permanent mandibular incisors. the presence of premature occlusal contact. The NDEB periodically reviews the bank to improve its quality. inflammation of the pulp. C. permanent maxillary incisors. the premature loss of adjacent primary teeth. D. Note: Some of the items in the Released Test Item Bank may have been discontinued due to outdated science or errors. accelerated by C. the format of some items is not currently used. B. C. Vesicles are C. Ferric sulfate. the soft palate. content. the premature loss of the adjacent primary teeth. B. D. the D. inflammation of the pulp. by B. Which of the following is the most appropriate E. The clinical A. (4) only E. 3. (2) and (4) C. bondability to tooth structure. All of the above. Total etching and dentin bonding. Reinforcing the fracture line with 4. 3. 1. the format of some items is not currently used.If a tooth has an inadequate ferrule. (4) only E. All of the above. Having a single larger fragment to temporization. (1) (2) (3) E. 2. (4) only D. (2) and (4) C. When preparing a post space in a figure-8 Posts in endodontically treated teeth should shaped premolar. retention from a post in the root. Note: Some of the items in the Released Test Item Bank may have been discontinued due to outdated science or errors. A. (1) (2) (3) A. All of the above. (1) and (3) C. Extend core material 2mm apically in the nonposted section of the figure-8. ©The National Dental Examining Board of Canada 2017 . 4. (1) and (3) C. provide better retention. All of the above. good radiopacity. 4. Use of a groove in the fragment. A. All of the above. content. In addition. require less dentin removal. (2) and (4) D. Elective endodontic treatment gaining composite. (1) and (3) B. Surgical crown lengthening. 1. Remove tooth structure to make an oval B. high strength. exert a wedge effect on the remaining dentin. (4) only E. (1) and (3) B. 2. and translation. Orthodontic eruption. Achieve post fit between the mesial and 3. 2. C. are more consistent with root anatomy. When compared to parallel-sided posts. E. Sub-gingival preparation and prolonged 3. A. (1) (2) (3) 4. 2. a low elastic limit to flex with the tooth. The NDEB periodically reviews the bank to improve its quality. tapered posts 1. which of Which of the following will improve the the following is/are effective strategies to prognosis of a rebonded fractured tooth increase tooth structure available for crown fragment? preparation? 1. 4. (1) (2) (3) D. 2. (1) and (3) shaped canal. which of the following is/are have appropriate? 1. (4) only A. B. (2) and (4) D. distal walls of the straightest canal. (2) and (4) D. (1) (2) (3) B. rebond. 3. Use one post in the straightest canal. Use of non-end cutting rotary instruments. The most effective appliance to correct mid. A. Bruxism. 3. In addition. (2) and 4) When using fixed orthodontic brackets. All of the above. Tipping. and translation. C. E. Which of the following is NOT a risk factor for primary molar ankylosis? A. of the root. D. The NDEB periodically reviews the bank to improve its quality. (4) only 4. Removing all caries in the pulp cap area. (1) (2) (3) B. content. 3. C. of pulp vitality after indirect pulp capping? A. 2. Missing permanent successor. D. Use of calcium hydroxide. All of the above. Which of the following improves the prognosis 4. rotate. (4) only will commonly E. 1. E. inter-arch elastics. translate. teeth D. reverse-pull headgear. (1) and (3) C. Intrusion. (1) and (3) C. tip. Having a well sealed restoration. B. (4) only orthodontic treatment? E. D. Development of a craniofacial syndrome. (1) and (3) B. D. C. (1) and (3) 2. B. Trauma. Discomfort. A. chin cup. Torqueing. Reducing other traumas to the pulp. Minimal canal enlargement. All of the above. 1. B. B. All of the above. A. 4. (1) (2) (3) 1. Note: Some of the items in the Released Test Item Bank may have been discontinued due to outdated science or errors.Which of the following is/are part of optimal Temporary anchorage devices (TADS) have post preparation procedure? the greatest effect on which orthodontic movement? 1. face retrusion in the early mixed dentition is (a) A. 4. Genetics. ©The National Dental Examining Board of Canada 2017 . (2) and (4) Which of the following is/are a risk of D. (1) (2) (3) B. Diameter of the post that is half the width C. (2) and (4) 3. Root resorption. (1) (2) (3) A. (4) only D. intrude. Passive post fit. Decalcification. 2. the format of some items is not currently used. Extrusion. bionator. 3. 2. (2) and (4) C. ? C. Nystagmus. Composition of the resin cement. A. Decreased consciousness. B. control the dental pain following the adjustment of orthodontic appliances in a child? Which of the following is NOT an early A. Decreased heart rate.35. (Gorlin Syndrome). 0. B. In addition. D. Pulpectomy is the most appropriate treatment in a primary tooth with A. A. and translation. D. What is the most appropriate medication to E. Slurred speech. Familial colorectal polyposis (Gardner’s Syndrome). E.25. 0.20. B. D. Type of acid etch. overdose? C. A. Acetone breath. D.15. ©The National Dental Examining Board of Canada 2017 . What is the dose of epinephrine in an B. C. 0. Acetaminophen. Apert Syndrome. Which of the following variables has the greatest effect on bond strength? A. Moist skin. The NDEB periodically reviews the bank to improve its quality. D. D. 0. Bronchospasm. Abdominal pain. Note: Some of the items in the Released Test Item Bank may have been discontinued due to outdated science or errors. advanced internal root resorption.30. C. Patient oral hygiene. C. radicular pulp necrosis. Moisture control. 0. Increased respiratory rate. of hyperglycemia? C. EpiPen®Jr. Size of the bracket base. Aspirin. Ibuprofen. B. Anaphylaxis. clinical manifestation of local anesthetic B. advanced external root resorption. Hypotension. content. A.What is the most probable syndrome affecting For which of the following medical a 9 year old patient with a history of 3 emergencies is atropine most commonly used? keratocystic odontogenic tumours (odontogenic keratocyst)? A. Nevoid basal cell carcinoma syndrome C. reversible pulpitis. D. Which of the following is NOT a manifestation B. Naproxen. D. B. the format of some items is not currently used. Crouzon Syndrome. Bradycardia. C. C. osteosarcoma. warfarin. reduction of fever. B. C. C. bleeding from the gastrointestinal tract. shortening of bleeding time. (2) and (4) A patient with end-stage renal disease has D. B. obtain a serum parathormone (PTH) level. 4. 1. 0. 0. D. Obtain a cytologic smear from the B. codeine. D. The most appropriate management for this patient is to The following are possible effects of acetylsalicylic acid EXCEPT A. associated with vital mandibular teeth. (1) and (3) C. B. A histopathologic analysis reveals the presence of multinucleated giant cells.25 mg. B. osteoma. sarcoma. 2.35 mg. B.40 mg. A. The effect of the combination of two drugs is less than the sum of their individual effects. content. The effect of the combination of two drugs is equal to the sum of their A. One drug’s action blocks the effect of the A benign neoplasm of bone is called a/an second drug’s action. 0.20 mg. In addition. E. extract the involved teeth. individual effects. A. D. D. Order a complete blood count. E. Which of the following is most appropriate to establish the diagnosis? A. All of the above. Obtain bacterial cultures. A. commence antibiotic therapy. (4) only several multilocular radiolucent lesions E. E. The NDEB periodically reviews the bank to improve its quality.What is the dose of epinephrine in an Epi Increased bleeding is associated with a Pen®? prolonged administration of A. D. The effect of the combination of two gingiva. perform a mandibular resection.30 mg. 0. ©The National Dental Examining Board of Canada 2017 . 0. acetylsalicylic acid. drugs is greater than the sum of their C. suppression of inflammatory response. individual effects. (1) (2) (3) B. C. the format of some items is not currently used. D. acetaminophen. torus. initiate endodontic treatments. A pale 8 year old patient presents with Which of the following best describes drug generalized gingival enlargement with synergism? spontaneous bleeding. C. 3. and translation. Excisional biopsy. Note: Some of the items in the Released Test Item Bank may have been discontinued due to outdated science or errors. fibrous dysplasia. 1. increase in gingival crevicular fluid. a functional shift. Basal and alveolar bone respond 2. the periodontal pocket. deep overbite. diastemas. is firmly attached to the tooth. B. the alveolar bone. 3. identically to pressure. bone. is demarcated from the attached gingiva by the mucogingival junction. B. C. Osteoid is a highly mineralized bundle 4. 2. (2) and (4) D. (1) and (3) C. 3. dilation of small blood vessels. enlargement of the marginal gingiva. A. ©The National Dental Examining Board of Canada 2017 . 4. (4) only E. increased vascular permeability. the connective tissues of the gingiva. A. the causative from a patient's circulatory system through the microorganisms are found in A. D. A. the periodontal ligament. kidneys.The majority of nitrous oxide is eliminated In chronic periodontitis. (1) and (3) C. increased overjet. (1) and (3) C. B. Basal and alveolar bone are histologically identical. Which of the following is/are correct? A suprabony pocket is associated with A. The effects of plaque on vascularity of the D. liver enzymes. D. spontaneous bleeding. (2) and (4) Marginal gingiva D. All of the above. C. 1. content. All of the above. an associated habit. (1) (2) (3) B. subgingival calculus. lungs. gingival connective tissue result in 1. B. Note: Some of the items in the Released Test Item Bank may have been discontinued due to outdated science or errors. A. C. C. a normal swallowing reflex. A. with 4. A. is demarcated from the attached gingiva by the free gingival groove. (4) only E. An anterior open bite is commonly associated 3. All of the above. (4) only bilateral maxillary constriction is E. the format of some items is not currently used. The NDEB periodically reviews the bank to improve its quality. a horizontal growth pattern. (1) (2) (3) B. unilateral crossbite. (1) (2) (3) B. E. 2. C. GI tract. In addition. horizontal loss of alveolar bone. (2) and (4) The most common presenting feature of D. plasma. proliferation of small blood vessels. and translation. B. Bruxism may be associated with Dislodgement of a maxillary complete denture may be caused by 1. premature contacts in the centric relation. 2. balancing prematurities. 1. under extension. 3. stress. 2. improper occlusion. 4. anxiety. 3. overextension. 4. thickness of distobuccal flange. A. (1) (2) (3) B. (1) and (3) A. (1) (2) (3) C. (2) and (4) B. (1) and (3) D. (4) only C. (2) and (4) E. All of the above. D. (4) only E. All of the above. Success of an endosseous dental implant is dependent upon On a semi-adjustable articulator, the incisal guide table represents 1. biocompatibility of the material. 2. design. A. a reference point for the establishment of 3. a period of non-function. occlusal vertical dimension. 4. immediate loading. B. the anterior equivalent of the condylar guidance. A. (1) (2) (3) C. a mechanical equivalent of the horizontal B. (1) and (3) and vertical overlap of the anterior teeth. C. (2) and (4) D. the mechanical equivalent of the Curve of D. (4) only Wilson. E. All of the above. Which pontic type is best for a knife edge The function of the compensating curve is residual ridge where esthetics is not a major concern? A. to help provide a balanced occlusion in complete dentures when the mandible is A. Sanitary. protruded. B. Conical. B. to aid in establishing an incisal guide C. Ridge lap. plane. D. Modified ridge lap. C. the same as the function of the curve of Spee. In an edentulous patient, the coronoid process A. limits the distal extent of the mandibular denture. B. affects the position and arrangement of posterior teeth. C. aids in determining the location of the posterior palatal seal. D. limits the thickness of the maxillary buccal denture flange. Note: Some of the items in the Released Test Item Bank may have been discontinued due to outdated science or errors. In addition, the format of some items is not currently used. The NDEB periodically reviews the bank to improve its quality, content, and translation. ©The National Dental Examining Board of Canada 2017 Which of the following removable partial Which of the following could cause clicking dentures has the LEAST amount of rotation sounds during speech in denture wearers? around the fulcrum? A. Excessive vertical dimension. A. Kennedy Class I. B. Nonbalanced occlusion. B. Kennedy Class II. C. Excessive buccal flange thickness. C. Kennedy Class III. D. Reduced vertical overlap (overbite). Which of the following are the most A characteristic of a group function occlusion appropriate for use as overdenture abutments? is A. Central incisors. A. the teeth on the nonworking side contact B. Second premolars. in lateral excursion. C. Canines. B. the teeth on the working side contact in D. First premolars. lateral excursion. C. the canine and lateral incisors contact in lateral excursion. D. the posterior teeth on both working and Which of the following clinical findings is nonworking sides contact in lateral NOT associated with combination (Kelly’s) excursion. syndrome? A. Reduction in morphologic face height. B. Advanced anterior maxillary ridge A laboratory remount of processed dentures is resorption. done in order to correct occlusal disharmony C. Down growth of the maxillary produced by errors primarily in the tuberosities. D. Advanced alveolar bone resorption under A. mounting of the casts on the articulator. the posterior partial denture base areas. B. registration of jaw relation records. C. processing of acrylic dentures. D. registration of condylar guidance. A cast partial denture replacing teeth 3.5 - 3.8 and 4.5 - 4.8 was permanently relined with acrylic resin. At the delivery appointment, Radiographic examination reveals early when the rests of the framework are fully evidence of internal resorption. The most seated on the abutment teeth, the denture base appropriate management is does not contact the supporting tissues. The most likely cause of the problem is A. apical surgical intervention. B. immediate pulp extirpation. A. the denture reline resin shrunk during C. immediate pulpotomy. polymerization. D. observation and re-evaluation in 3-6 B. excess pressure was placed on the rests months. during the impression procedure. E. calcium hydroxide pulpotomy. C. excess pressure was placed on the denture base area during the impression procedure. D. the patient’s tissues have remodelled. Note: Some of the items in the Released Test Item Bank may have been discontinued due to outdated science or errors. In addition, the format of some items is not currently used. The NDEB periodically reviews the bank to improve its quality, content, and translation. ©The National Dental Examining Board of Canada 2017 The function(s) of the reciprocal clasp arm Conversion of a flush terminal plane to a is/are to mesial step/Class I terminal plane in the absence of orthodontics is primarily the result 1. act as an indirect retainer. of 2. stabilize the abutment teeth. 3. act as a direct retainer for the distal base. A. loss of the mandibular primate space. 4. counteract any force transmitted by the B. greater maxillary than mandibular retentive arm. forward growth. C. differences in leeway between the A. (1) (2) (3) maxillary and mandibular arches. B. (1) and (3) D. distal movement of the maxillary first C. (2) and (4) permanent molars. D. (4) only E. All of the above. When a removable partial denture is fabricated to occlude with natural teeth, the occlusal form The inferior border of the lingual bar of a of the artificial teeth is influenced by the removable partial denture should A. endodontic condition of the abutment A. displace the lingual frenum. teeth. B. be in contact with the cingula of the B. occlusal form of the remaining teeth. incisor teeth. C. need to produce a fully balanced C. be superior to the gingival border. occlusion. D. be as inferior as the movement of the frenum of the tongue will permit. Which of the following should be evaluated for surgical removal before new complete dentures In a free-end distal extension partial denture, are fabricated? the most effective means of limiting applied loads to abutment teeth is by 1. Mandibular tori. 2. Epulis fissuratum. A. splinting abutments to adjacent teeth. 3. Papillary hyperplasia. B. using the abutment teeth without 4. Sharp, prominent mylohyoid ridges. splinting. C. using monoplane denture teeth. A. (1) (2) (3) D. maintaining a stable base-tissue B. (1) and (3) relationship. C. (2) and (4) D. (4) only E. All of the above. Note: Some of the items in the Released Test Item Bank may have been discontinued due to outdated science or errors. In addition, the format of some items is not currently used. The NDEB periodically reviews the bank to improve its quality, content, and translation. ©The National Dental Examining Board of Canada 2017 When a partial denture is to be fabricated The mesial furcation of the permanent where occlusal adjustment of the natural teeth maxillary first molar is best assessed from is required, this should be performed which aspect of the tooth? A. prior to taking the final impressions. A. Mesiobuccal. B. prior to registering centric relation but B. Mesiopalatal. after final impression taking. C. Midmesial. C. after the framework has been constructed. Which of the following may affect probing Diazepam depth measurements of a periodontal pocket? 1. is a benzodiazepine. 1. Probing force. 2. is contraindicated in a patient with acute 2. Diameter of the probe tip. narrow angle glaucoma. 3. Angulation of the probe. 3. is anxiolytic. 4. Subgingival calculus. 4. produces amnesia. A. (1) (2) (3) A. (1) (2) (3) B. (1) and (3) B. (1) and (3) C. (2) and (4) C. (2) and (4) D. (4) only D. (4) only E. All of the above. E. All of the above. Gingivitis is characterized by The most common form of periodontal disease is A. bleeding on probing. B. loss of attachment. A. gingivitis. C. alveolar bone loss. B. chronic periodontitis. D. apical migration of the junctional C. gingival hyperplasia. epithelium. D. aggressive periodontitis. A patient with periodontal disease may A crown margin can be extended subgingivally complain of when required 1. loose teeth. 1. for esthetics. 2. bleeding gingiva. 2. to increase retention. 3. pain. 3. to reach sound tooth structure. 4. gingival recession. 4. for caries prevention. A. (1) (2) (3) A. (1) (2) (3) B. (1) and (3) B. (1) and (3) C. (2) and (4) C. (2) and (4) D. (4) only D. (4) only E. All of the above. E. All of the above. Note: Some of the items in the Released Test Item Bank may have been discontinued due to outdated science or errors. In addition, the format of some items is not currently used. The NDEB periodically reviews the bank to improve its quality, content, and translation. ©The National Dental Examining Board of Canada 2017 A major clinical problem of penicillin therapy Properties of glutaraldehyde include is its 1. rapid formation of cross linkages which A. high toxicity. limit penetration of pulp tissue. B. allergenicity. 2. minimal effect on pulp tissues. C. rapid development of tolerance. 3. excellent disinfection against oral flora. D. narrow spectrum. 4. minimal effectiveness against viruses and spores. A. (1) (2) (3) Which of the following anesthetic agents is/are B. (1) and (3) metabolized by plasma cholinesterase? C. (2) and (4) D. (4) only 1. Prilocaine. E. All of the above. 2. Lidocaine. 3. Mepivacaine. 4. Procaine. Bioavailability of orally administered drugs A. (1) (2) (3) may be influenced by B. (1) and (3) C. (2) and (4) 1. formulation of the drug. D. (4) only 2. gastrointestinal perfusion. E. All of the above. 3. pH of the gastrointestinal tract. 4. presence of other substances in the gastrointestinal tract. Which of the following drugs has/have sedative A. (1) (2) (3) properties? B. (1) and (2) C. (2) and (4) 1. Benzodiazepines. D. (4) only 2. Barbiturates. E. All of the above. 3. Meperidine. 4. Phenytoin. A. (1) (2) (3) Propoxyphene has the potential for drug abuse B. (1) and (3) because C. (2) and (4) D. (4) only A. it is a potent analgesic with a euphoriant E. All of the above. activity equal to morphine. B. in combination with alcohol its effects are greatly intensified. C. it cannot be detected in the bloodstream. Note: Some of the items in the Released Test Item Bank may have been discontinued due to outdated science or errors. In addition, the format of some items is not currently used. The NDEB periodically reviews the bank to improve its quality, content, and translation. ©The National Dental Examining Board of Canada 2017 Nitrous oxide Diagnostic casts for a fixed bridge allow the dentist to 1. provides good muscle relaxation. 2. is non-explosive and non-flammable. 1. visualize the direction of the occlusal 3. is a potent anesthetic. forces. 4. provides rapid induction and recovery. 2. assess occlusion more accurately. 3. plan the pontic design. A. (1) (2) (3) 4. assess the esthetics using a diagnostic wax- B. (1) and (3) up. C. (2) and (4) D. (4) only A. (1) (2) (3) E. All of the above. B. (1) and (3) C. (2) and (4) D. (4) only E. All of the above. Which of the following will produce hemostasis when applied topically? 1. Oxidized cellulose. The surgical risk for a patient with organic 2. Absorbable gelatin sponge. heart disease depends upon 3. Microfibrillar collagen. 4. Topical bovine thrombin. A. cardiac reserve. B. blood pressure. A. (1) (2) (3) C. respiration. B. (1) and (3) D. pulse rate. C. (2) and (4) E. cardiac output. D. (4) only E. All of the above. A sedative drug should Advantages of resin bonded bridges are A. cause excitement. B. eliminate all sensation. 1. tooth structure conservation. C. produce unconsciousness. 2. short chair-side time. D. produce a mild state of central nervous 3. lower cost for patient. system depression. 4. improved esthetics compared to traditional bridges. A. (1) (2) (3) Adequate position of a needle in the lumen of a B. (1) and (3) blood vessel for a venipuncture is confirmed by C. (2) and (4) the D. (4) only E. All of the above. A. presence of blood upon aspiration. B. rapidity of filling of the syringe upon aspiration. C. pain associated with vessel puncture. Note: Some of the items in the Released Test Item Bank may have been discontinued due to outdated science or errors. In addition, the format of some items is not currently used. The NDEB periodically reviews the bank to improve its quality, content, and translation. ©The National Dental Examining Board of Canada 2017 Which of the following drugs will influence The terminal plane relationship of primary your periodontal treatment planning? second molars determines the 1. Insulin. A. arch length between permanent second 2. Cortisone. molars. 3. Nitroglycerin. B. future anteroposterior positions of 4. Warfarin. permanent first molars. C. vertical dimensions of the mandible upon A. (1) (2) (3) eruption of permanent first molars. B. (1) and (3) D. amount of leeway space that is available C. (2) and (4) for permanent premolars and canines. D. (4) only E. All of the above. What is the most appropriate appliance to correct an Angle Class I malocclusion with a Which of the following antibiotics is the most labially tipped maxillary central incisor and appropriate for a patient with an open fracture spacing in a 15 year old patient? of the mandible? A. Growth modification appliance. A. Clarithromycin. B. Hawley with an active labial bow. B. Tetracycline. C. Molar distalizing appliance. C. Amoxicillin. D. Bilateral expansion appliance. D. Erythromycin. Premature loss of mandibular primary cuspids Which of the following nerves are anesthetized in Angle Class I and Class II cases results in by an infraorbital nerve block? increased 1. Palpebral. 1. overjet. 2. Lateral nasal. 2. arch width. 3. Labial. 3. overbite. 4. Zygomatico-facial. 4. leeway space. A. (1) (2) (3) A. (1) (2) (3) B. (1) and (3) B. (1) and (3) C. (2) and (4) C. (2) and (4) D. (4) only D. (4) only E. All of the above. E. All of the above. Leeway space is most efficiently maintained by a/an A. lingual arch. B. tongue crib. C. anterior bite plate. D. functional appliance. Note: Some of the items in the Released Test Item Bank may have been discontinued due to outdated science or errors. In addition, the format of some items is not currently used. The NDEB periodically reviews the bank to improve its quality, content, and translation. ©The National Dental Examining Board of Canada 2017 What is the most appropriate management of A 10 year old patient with a lateral functional an ectopically erupting premolar with the shift due to a maxillary bilateral posterior primary predecessor firmly in place? crossbite A. Natural exfoliation of the primary tooth. 1. should not be treated until growth is B. Luxation of the primary tooth. complete. C. Extraction of the primary tooth and 2. should be diagnosed in maximum monitor. intercuspation. D. Extraction of the primary tooth and space 3. requires rapid palatal expansion. maintenance. 4. has a centric occlusion-centric relation (CO-CR) shift caused by occlusal interferences. A 12 year old patient has the following A. (1) (2) (3) cephalometric values: B. (1) and (3) SNA = 87° (N = 82°) C. (2) and (4) SNB = 80° (N = 80°) D. (4) only. Mandibular Plane Angle = 32° (N = 32°) E. All of the above. FMA = 26° (N = 26°) The patient is a skeletal Angle Class II with a Which of the following is/are side effects of A. normally protrusive maxilla and a rapid maxillary expansion? retrognathic mandible. B. protrusive maxilla and an orthognathic 1. increase in the vertical dimension. mandible. 2. mesial movement of the maxilla. C. protrusive maxilla and a retrognathic 3. increase in the maxillary inter-molar mandible. width. D. retrusive maxilla and an orthognathic 4. increase in the mandibular inter-molar mandible. width. A. (1) (2) (3) B. (1) and (3) The occlusal parameter most useful to C. (2) and (4) differentiate between an overbite of dental or D. (4) only skeletal origin is the E. All of the above. A. mandibular curve of Spee. B. mandibular curve of Wilson. C. maxillary curve of Wilson. Early orthodontic treatment of a retrognathic D. maxillary incisors morphology. mandible should occur E. maxillary lip length. A. before age 5. B. between the ages of 6 and 8. C. just prior to the pubertal growth spurt. D. after skeletal maturity. Note: Some of the items in the Released Test Item Bank may have been discontinued due to outdated science or errors. In addition, the format of some items is not currently used. The NDEB periodically reviews the bank to improve its quality, content, and translation. ©The National Dental Examining Board of Canada 2017 Increased overjet, moderate facial convexity Which of the following drugs has/have and a deep labio-mental sulcus is most antisialagogue properties? characteristic of which facial type? 1. Acetaminophen. A. Prognathic. 2. Atropine. B. Orthognathic. 3. Acetylsalicylic acid. C. Retrognathic. 4. Scopolamine. A. (1) (2) (3) B. (1) and (3) The most common order of serial extraction in C. (2) and (4) the maxillary arch is D. (4) only E. All of the above. A. primary canines, first premolars, primary first molars. B. primary canines, primary first molars, first premolars. A pontic exerting too much pressure against the C. first premolars, primary canines, primary ridge may cause first molars. D. primary first molars, primary canines, first 1. fracture of the solder joints. premolars 2. hypertrophy of the soft tissue. 3. crazing of the gingival portion of the porcelain. 4. resorption of the alveolar bone. A previously well-controlled periodontal patient now demonstrates the presence of A. (1) (2) (3) bleeding in 60% of sites and significantly B. (1) and (3) increased probing depths. The patient has most C. (2) and (4) likely developed D. (4) only E. All of the above. A. diabetes mellitus. B. hyperchromatosis. C. osteoporosis. D. hypercalcemia. Which patient is LEAST likely to be predisposed to liver toxicity following a dose of 1,000 mg of acetaminophen? Cephalosporins A. An adult with liver cirrhosis. B. A chronic alcoholic. 1. may be cross-allergenic with penicillin. C. A diabetic. 2. have a narrower spectrum than penicillin. D. A 15kg, 4 year old child. 3. have a mechanism of action similar to that of penicillin. 4. may cause cholestatic hepatitis. A. (1) (2) (3) B. (1) and (3) C. (2) and (4) D. (4) only E. All of the above. Note: Some of the items in the Released Test Item Bank may have been discontinued due to outdated science or errors. In addition, the format of some items is not currently used. The NDEB periodically reviews the bank to improve its quality, content, and translation. ©The National Dental Examining Board of Canada 2017 ©The National Dental Examining Board of Canada 2017 . B. common cause of recurrence of pockets is B. Magnesium. D. Altered lymphocyte activity. C. 3. of incorrect length. (1) and (3) E. D. Impaired polymorphonuclear phagocytosis. 4. masseter muscle. Calcium. content. tendon of the temporal muscle. A. In addition. synaptic inhibition in the dorsal column. D. traumatic occlusion. A. failure to splint. The NDEB periodically reviews the bank to improve its quality. external oblique ridge. 2. periapical. Chloride. E. B. (1) (2) (3) adrenal cortex. C. which of the following ions? C. A. the most A. C. C. release of adrenal steroids from the A. D. Sodium. Local anesthetics interfere with the transport of B. C. C. poor oral hygiene. B. inhibition of prostaglandin synthesis. B. systemic disease.The shape of the distobuccal flange of a A significant mechanism by which mandibular denture is determined by the acetylsalicylic acid produces its analgesic and anti-inflammatory effect is 1. The amount of radiation is best reduced by A. panoramic. buccinator muscle. decreasing the target-object distance. Generalized subgingival calculus. decreasing the object-film distance. occlusal. D. lowering kVp and increasing milliamperage. bite-wing. the format of some items is not currently used. Bacterial plaque. A. Potassium. (4) only E. C. local anesthetic effect on pain fibres. The most appropriate radiograph to assess an interproximal vertical bony defect of the alveolar bone is the What is the primary etiologic factor for generalized aggressive periodontitis? A. with incorrect horizontal angulation. D. and translation. B. of incorrect shape. D. using collimation. B. antagonism of histamine. Note: Some of the items in the Released Test Item Bank may have been discontinued due to outdated science or errors. All of the above. with incorrect vertical angulation. Overlapped interproximal contacts in a bite- wing radiograph are caused by an X-ray tube cone Following periodontal flap surgery. (2) and (4) D. Nausea. 2. provide for a rigid restoration. (2) and (4) well-circumscribed radiolucency at the apex of D. radiation dose. 4. 4. B. A. (1) (2) (3) B. place the porcelain metal joint away from C. Panoramic. sclerosing osteitis. E. 2. 2. by the porcelain. D. (4) only a vital mandibular incisor? E. the soft tissues. Diarrhea. C. B. effect(s) of codeine? 1. D. periapical cemental dysplasia. hyperthyroidism. 3. (1) and (3) What is the most likely diagnosis of a small C. hyperparathyroidism.Appropriate collimation of the X-ray beam for Multiple radiolucent lesions of the jaws are the film size reduces features of 1. A. basal cell nevus syndrome. control thermal conductivity. (4) only E. Note: Some of the items in the Released Test Item Bank may have been discontinued due to outdated science or errors. All of the above. All of the above. 4. the format of some items is not currently used. 3. Cone beam computed tomography. a periapical granuloma. The NDEB periodically reviews the bank to improve its quality. All of the above. 4. (1) and (3) C. E. secondary radiation. image definition. 2. and translation. Which radiographic examination provides the Pontic design for a metal-ceramic bridge most diagnostic information for the should replacement of a permanent mandibular first molar with an osseo-integrated implant? 1. A. (1) and (3) C. multiple myeloma. (1) (2) (3) B. (1) (2) (3) B. All of the above. Which of the following is/are possible side D. 3. allow for complete coverage of the metal A. Drowsiness. 1. (2) and (4) C. (1) and (3) B. (2) and (4) D. (2) and (4) D. (4) only. In addition. Respiratory depression. (1) (2) (3) A. ©The National Dental Examining Board of Canada 2017 . radiographic contrast. Occlusal. a radicular cyst. content. (4) only. Periapical. 3. A. number of bony walls. content. The NDEB periodically reviews the bank to improve its quality. weight of plaque obtained from facial D. candida albicans infection. (2) and (4) C. severe temporomandibular dysfunction. 1. (1) and (3) C. a true unilateral crossbite. 3. number of bony walls. apical to the junctional epithelium. (2) and (4) A shift from bilateral crossbite with coincident D. A. A. defect can be used to confirm the C. B. 2. and translation. (4) only midlines to a right-sided posterior crossbite E. depth in the alveolus. (1) (2) (3) B. root width. coronal to the junctional epithelium. B. periodontal ligament space. not erupt. occlusal interference and functional shift. Radiographs of a periodontally related osseous B. quantity of plaque at the gingival margin. 2. loss of crestal cortication. In addition. C. the format of some items is not currently used. with non-coincident midlines most likely indicates A. erupt out of position. 20g force. the tooth may impacted mandibular third molar? A. A. at the level of the junctional epithelium. C. ill-fitting dentures. 4. angulation. defect show the C. colony forming units of Gram-negative B. All of the above. tooth. measurement of the defect. cause resorption of the root of another 1. (1) and (3) bacteria. location of the epithelial attachment. inadequate denture hygiene. The Plaque Index of Silness and Loe measures 4. xerostomia. All of the above. D. the tip of the periodontal probe is most likely located A. C. measurement of the defect. Note: Some of the items in the Released Test Item Bank may have been discontinued due to outdated science or errors. Radiographs of a periodontally related osseous B. 3. All of the above. E. A. B. Denture stomatitis can be associated with D. (1) (2) (3) B. ©The National Dental Examining Board of Canada 2017 . presence of a furcation involvement.Which of the following should be considered If there is insufficient arch space for a when assessing the difficulty of removal of an permanent tooth to erupt. location of the epithelial attachment. When probing a healthy gingival sulcus with a D. A. C. (4) only surfaces. E. All of the above. The periodontal probe should be inserted into B. Lateral jaw. the format of some items is not currently used. B. aminide. parallel to the long axis of the tooth. B. ester. B. The selection of a vasoconstrictor for a local Local anesthetics anesthetic depends upon the A. B. radiographs will appear D. (1) and (3) C. C. A. (1) (2) (3) than the nervous tissue. group function occlusion. (4) only E. selectively interfere with the propagation 4. D. potentials in nerve fibres. 3. black. with a firm lateral motion. D. duration of the procedure. gray. brown. non-passive retentive arms. interfere with the propagation of action 2. apical to the level of the alveolar crest. ethamine. When developer solution is old and oxydized. B. content. In an infrabony periodontal pocket. and translation. C. need for hemostasis. C. The NDEB periodically reviews the bank to improve its quality. the gingival sulcus C. Occlusal. Periapical. Which of the following radiographs is best to D. do not have an effect on any other tissue A. Bitewing. aldehyde. with a firm pushing motion. A. parallel to the root surface. ©The National Dental Examining Board of Canada 2017 . Panoramic. type of procedure. at the level of the alveolar crest. (2) and (4) D. coronal to the level of the alveolar crest. B. In addition. C. blue.Movement of a mandibular distal extension Procaine is a local anaesthetic which is (Class 1) partial denture away from the denture chemically classified as an bearing tissues when the patient opens is primarily caused by A. C. overextended borders. 1. amide. B. the tip of the periodontal probe is located A. C. E. Note: Some of the items in the Released Test Item Bank may have been discontinued due to outdated science or errors. of action potentials in nociceptive fibres. D. medical status of the patient. A. diagnose caries and early alveolar bone loss? A. do not readily pass the blood-brain barrier. content. 4. (1) (2) (3) A. All of the above. are over age 80. C. arrested. (2) and (4) D. A. Porcelain/ceramic thickness for fracture resistance. (1) (2) (3) 3. E. 3. accelerated. 4. and translation. All of the above. The NDEB periodically reviews the bank to improve its quality. (2) and (4) the path of insertion. is greater than the sum of their individual effects. gamma radiation. the crown/root ratio. secondary radiation. collimation. a drug produces active metabolites. A. delayed. All of the above. a drug accumulates in the body at a faster B. (1) and (3) 4.Tooth reduction for anterior tooth preparation Increasing the kilovoltage results in greater for metal-ceramic and all ceramic crowns is dictated by which of the following factor(s)? 1. 4. 3. 2. (4) only. (1) and (3) B. (4) only D. penetration. are taking anticoagulants. Parallelism of axial walls for facilitating C. Crown length for adequate retention. tissue/bone reaction. unaltered. a drug is concentrated in the kidneys. the format of some items is not currently used. A. the concentration of the drug increases with time. 2. (1) (2) (3) B. the quality of cooperation. have uncontrolled diabetes mellitus. resistance. E. A. E. the eruption of the permanent successor could Dental implants are CONTRAINDICATED in be patients who 1. Clearance for occlusal function. Following very early loss of a primary tooth. All of the above. 2. ©The National Dental Examining Board of Canada 2017 . (4) only Orthodontic tooth movement is generally more E. (4) only E. (4) only D. In drug synergism 2. D. (2) and (4) C. efficient with adolescents than in adults because of the difference in 1. (2) and (4) D. 1. 3. All of the above. (1) (2) (3) B. 1. 3. (1) (2) (3) B. A. B. (1) and (3) C. 2. (1) and (3) rate than it is destroyed or removed. E. have unrepaired cleft palates. (2) and (4) C. Note: Some of the items in the Released Test Item Bank may have been discontinued due to outdated science or errors. In addition. D. the effect of the combination of two drugs 4. the rate of growth. (1) and (3) C. (4) only 1. loss of attachment. C. (2) and (4) A. Ionized and water solubility. lip habits. (4) only 1. D. within the apical half of the root. and translation. (1) and (3) 1. All of the above. (1) and (3) The signs of chronic periodontitis include C. inflammation. periodontitis. A. C. C. premature wear of occlusal surfaces. All of the above. 2. Corticosteroids may be used for the management of A. (1) (2) (3) B. 4. 1. arthritis. 2. periodontal pockets. within the cervical one third of the root. When a simple tipping force is applied to the crown of a single-rooted tooth.Which permanent tooth usually erupts first? Migration of teeth may be associated with A. Maxillary canine. bruxism. E. Mandibular central incisor. All of the above. (4) only B. allergy. the periodontal ligament. 4. In addition. TMJ discomfort. Note: Some of the items in the Released Test Item Bank may have been discontinued due to outdated science or errors. (2) and (4) 2. C. asthma. Maxillary central incisor. at the apex. content. D. D. Which properties increase the tendency of a A. (4) only 3. the format of some items is not currently used. radiographic evidence of the widening of E. 3. Ionized and low lipid solubility. increased mobility of teeth. bone resorption. All of the above. Addison's disease. (1) (2) (3) drug to cross cell membranes? B. (1) and (3) A patient with bruxism is likely to demonstrate C. tongue habits. B. A. 4. Non-ionized and high lipid solubility. Mandibular first molar. (2) and (4) D. 3. B. (2) and (4) D. (1) (2) (3) B. The NDEB periodically reviews the bank to improve its quality. ©The National Dental Examining Board of Canada 2017 . 4. (1) and (3) C. E. Non-ionized and low lipid solubility. the centre of rotation is located A. 2. at the cervical line. D. (1) (2) (3) B. E. D. 3. 3. All of the above. observe it until it exfoliates. pain. when The facial profile most often associated with an is it appropriate to leave root tips in the socket? Angle’s Class III malocclusion is 1. Root tip size is less than 2 mm. During the extraction of a primary tooth. 1. hyalinization of periodontal ligament. E. 3. late mixed dentition. D. (1) (2) (3) the same metal in the cast condition in that B. (1) and (3) B. D. All of the above. content. if heated sufficiently. C. restorative procedures. 3. 4. In addition. above 7. B. A. (1) and (3) C. A. increased. (4) only D. 4. (1) and (3) C. the format of some items is not currently used. D. observe it until it becomes symptomatic. the yield strength and hardness are E. (4) only E. (1) and (3) C. the grains are deformed and elongated. (4) only 2. All of the above. (2) and (4) 1. the resistance to corrosion is decreased. Note: Some of the items in the Released Test Item Bank may have been discontinued due to outdated science or errors. Signs and symptoms of occlusal traumatism are Prolonged heavy orthodontic forces may result in 1. late primary dentition. periodontal ligament space. (2) and (4) A.4.4. early permanent dentition. extract it. loss of pulp vitality. straight. (2) and (4) C. E. at 7. C.The optimal time for orthodontic treatment Ionization of local anesthetics is facilitated by a involving growth modification is during the tissue pH A. inefficient tooth movement. There is no infection in the area. (2) and (4) D. 3. A metal in the wrought condition differs from A. (1) (2) (3) A. A 4 year old child has a chronically infected non-restorable primary first molar. ©The National Dental Examining Board of Canada 2017 . extract it and place a space maintainer. 4. B. recrystallization can occur. All of the above. Tips will not interfere with development or eruption of permanent teeth. 2. (4) only B. (1) (2) (3) B. devitalization of the teeth. radiographic evidence of increased 2. The most A. B. convex. Tips will not interfere with ensuing C. 4. pain. and translation.4. below 7. A. concave. (1) (2) (3) appropriate management is to B. 2. early mixed dentition. C. tooth mobility. The NDEB periodically reviews the bank to improve its quality. D. (4) only of a hypoglycemic reaction. 2. The NDEB periodically reviews the bank to improve its quality. All of the above. The rate limiting step of tooth movement is 3. (1) and (3) C. original neoplasm may invade the area of surgery. In addition. E. force application. 2. a low-carbohydrate snack is available in 3. 3. Supracrestal. B. 14 years. occlusal interference. 4. D. sclerosed blood vessels compromise A. B. bone deposition. the patient has eaten his regular morning B.An 8 year old child has an 8 mm diastema An insulin-dependent diabetic patient has between teeth 1. 2 years. A. normal development. Diagonal. an extra dose of insulin is available in case D. A. This could indicate rushed in late for an early morning 1 hour long the presence of appointment. the patient has taken his regular dose of insulin. case of an insulin reaction. 2. 6 months in utero. B. the format of some items is not currently used. healing. 4. Which of the following should be verified? That 1. (4) only the fact that the E.1. A. C. Oblique. 1. bone resorption. C. (1) and (3) breakfast. an enlarged frenum. (1) and (3) C. a cyst. Note: Some of the items in the Released Test Item Bank may have been discontinued due to outdated science or errors. 7 years. A. (2) and (4) D. All of the above. patients are immunocompromized. (1) (2) (3) B. content. (1) (2) (3) D. B. and translation. All of the above. one or more mesiodens. (4) only E. (2) and (4) Surgery of irradiated bone is complicated by D. A. ©The National Dental Examining Board of Canada 2017 . (1) (2) (3) C. C. (2) and (4) D.1 and 2. bone becomes brittle. Which of the periodontal ligament fibres are Tetracycline will cause dentin discolouration most commonly associated with orthodontic when ingested at the age of relapse? 1. Horizontal. 4. provide 3-dimensional control of tooth movement. Note: Some of the items in the Released Test Item Bank may have been discontinued due to outdated science or errors. the format of some items is not currently used. (1) and (3) effects when uprighting a mesially tipped C. 4. enhance mandibular forward growth. Four months into pregnancy. 4. D. (1) (2) (3) B. Fixed orthodontic appliances are generally more efficient than removable appliances because conventional fixed appliances Maxillary midline diastema can be caused by A. 2. B. content. A. provide intermittent force application. (2) and (4) D. promote improved oral hygiene. (2) and (4) D. 1. safely extract a tooth in a dental office setting? extraction complication of dry socket? A. The NDEB periodically reviews the bank to improve its quality. C. Intrusion of the buccal segment. Buccal tipping of the buccal segment. a tongue thrust habit. (1) and (3) C. (4) only E. a thumb-sucking habit. Extrusion of the second mandibular molar. D. (4) only appliances? E. restricts maxillary forward growth. enhances anterior tooth movement. B. All of the above. All of the above. 1. (2) and (4) mandibular second molar with fixed D. (2) and (4) D. (1) and (3) 4. 4. A. 2. 2. Suffering from Factor VIII deficiency. In addition. 1. 3.Which of the following clinical approaches In which of the following patients can you would reduce the tendency to the post. Two weeks after a myocardial infarct. B. All of the above. are individualized to each patient. Use of systemic antibioprophylaxis. 1. C. a mesiodens. (4) only E. and translation. C.2% chlorhexidine gluconate. Pre-extraction control of pericoronitis. (1) (2) (3) 3. All of the above. Presenting with bilateral submandibular 3. (1) (2) (3) What are the anticipated orthodontic side B. Post-extraction socket dressing with B. 2. Lingual tipping of the second molar. ©The National Dental Examining Board of Canada 2017 . topical chlorhexidine. (4) only A. congenitally missing lateral incisors. A. (1) (2) (3) E. (1) and (3) C. 0. Occipital and/or cervical extra-oral anchorage 3. restricts posterior tooth movement. Use of a pre-operative mouthrinse of space abcess. 3. perform an incisional biopsy. (4) only the amount of E. A. (2) and (4) Periodontitis is differentiated from gingivitis by D. (1) and (3) C. E. refer patient to family physician. All of the above. Loss of attachment. (4) only A. place a large strip of iodoform gauze.Which of the following is/are clinical signs of The epithelial attachment does NOT migrate gingivitis? apically in 1. 4. cyanosis of gingival tissue. All of the above. (1) and (3) management would be to C. (1) and (3) C. white lesion of the buccal mucosa has not resolved after elimination of all A. B. subgingival calculus. In addition. E. 3. ©The National Dental Examining Board of Canada 2017 . content. the sulcular epithelium C. (1) (2) (3) treatment would be to B. B. plaque accumulation. 4. (2) and (4) A. C. Gingivitis is associated with D. 3. Loss of stippling. A. C. cauterize it. aggressive periodontitis. The NDEB periodically reviews the bank to improve its quality. the format of some items is not currently used. (1) (2) (3) B. The most appropriate B. discrete. D. (1) (2) (3) local irritants. the presence of minimal attached gingiva. 4. (2) and (4) D. Note: Some of the items in the Released Test Item Bank may have been discontinued due to outdated science or errors. prescribe an antibiotic and recall after 24hrs. B. is permeable to bacterial enzymes and toxins. B. apply toluidine blue staining. the bony floor of the sinus is removed with the tooth. bone loss. second molar. A 2cm. 2. 2. apical migration of the epithelial attachment. Your immediate A. 2. E. D. gingival inflammation. gingival hyperplasia. C. 1. E. In gingivitis. and translation. necrotizing ulcerative periodontitis. chronic periodontitis. D. irrigate the sinus repeatedly and place a dressing in the tooth socket. 1. may be ulcerated. (4) only filling the tooth socket. A. re-examine at 6 month intervals. is a barrier to bacterial invasion. undergoes both degenerative and During the extraction of an isolated maxillary proliferative changes. D. All of the above. Gingival hyperplasia. approximate the soft tissue as accurately as possible without irrigating. Bleeding on probing. bleeding on probing. discomfort of the patient. (4) only E. 2. (1) and (3) periodontal condition includes C. hyperemic and edematous gingival tissue. using a larger collimator. increasing the focal spot-to-film distance. C.10ma.65ma. All of the above. C. Reduce the exposure time. The sharpness of a radiographic image can be improved by A. (4) only A. 85kVp . using a larger focal spot. necrotizing ulcerative gingivitis. Use a lower speed receptor. Radiation dose to the patient is reduced by B. 65kVp . B. A. 4. the B. 4. B. analgesic properties. D.Which of the following Excessively dark radiographs are the result of milliamperage/kilovoltage combinations results in an X-ray beam with the most penetration? 1. anti-inflammatory function. 2. excessive milliamperage. using a high speed film. 2. D. 1. (1) (2) (3) E. (1) (2) (3) recommended when making radiographs on an B. 3. 1. Increase the kVp. exposure time. (2) and (4) D. The NDEB periodically reviews the bank to improve its quality. 4. kilovoltage. (1) (2) (3) root planing alone will occur if the patient's B. Note: Some of the items in the Released Test Item Bank may have been discontinued due to outdated science or errors.15ma. D. increasing the object-to-film distance. A daily dose of 81 mg of acetylsalicylic acid is used for its The depth of penetration of X-rays depends on A.5ma. using low kVp. C. ©The National Dental Examining Board of Canada 2017 . 10kVp . (2) and (4) D. the format of some items is not currently used. Increase the mA. D. (4) only A. Probing depth reduction following scaling and A. Which of the following modifications is A. B. 75kVp . and translation. density of the subject. underdevelopment. All of the above. C. In addition. content. antiplatelet action. using an aluminum filter. chronic periodontal pockets. All of the above. gingival hyperplasia due to phenytoin therapy.40ma. A. (1) and (3) 8 year old child compared to an adult? C. backward placement of the film. 3. C. B. D. milliamperage. E. overexposure. 3. (1) and (3) C. E. antipyretic effect. increasing the target-film distance. (2) and (4) D. 90kVp . Which of the following is/are vital signs? A. D. B. Pupil size and reactivity. used to reduce the frequency of seizures in persons with grand mal seizure disorder? A. In addition. Phenobarbital. Pulse and respiratory rate. Amobarbital. content. Height and weight. of the tooth. B. Allergenicity. 3. the format of some items is not currently used. red blood cell count. C. locate the height of contour. (2) and (4) D. Development of tolerance. B. High toxicity. 4. Body temperature. E. white blood cell count. D. A. D. Complete blood count. C. Pulse and respiratory rate. (4) only E. A. 4. C. Pupil size. (4) only Which of the following barbiturates can be E. A. Which of the following is/are vital signs? Which of the following is associated with the use of penicillin? 1. tripoding a B. (1) (2) (3) B. Secobarbital. (2) and (4) D. enhance resistance form of the abutment. ©The National Dental Examining Board of Canada 2017 . the most fixed partial denture abutment will valuable laboratory test(s) is/are the 1. locate unfavorable tissue undercuts. All the above. For a removable partial denture. (1) and (3) C. (1) and (3) C. 2. (1) (2) (3) A. 3. cast is used to C. improve embrasure form. Pentobarbital. Thiopental. 2. return the cast to the surveyor. All of the above. 2. Blood pressure. E. The NDEB periodically reviews the bank to improve its quality. All of the above. and translation. (2) and (4) C. reduce the possibility of pulpal exposure. 1. determine the path of insertion. 4. direct occlusal forces along the long axis 3. A. (4) only D.Minor tooth movement to correct an inclined For an acute bacterial infection. hemoglobin level. Renal failure. Note: Some of the items in the Released Test Item Bank may have been discontinued due to outdated science or errors. (1) and (3) B. D. (1) (2) (3) B. culture and sensitivity test. 90o axiogingival angle. reduce the diameter of the primary beam. 1.6 to a more normal E. The three unit fixed 4. Gold-silver. 4. Nickel-chromium. less irritating. (4) only C. about the same as the average amount received from cosmic radiation. (1) and (3) B. B. reduce the exposure time. All of the above. The average annual dose of ionizing radiation that a patient receives from dental radiographs A. B. In addition. more hygienic. (1) (2) (3) A. Gold-platinum-palladium. to the existing occlusion. lower the kilovoltage.8mm into the ray beam. 2. increased vertical angulation of the x-ray A 60 year old patient requests the replacement beam. 2. (2) and (4) A. A. (2) and (4) with a fixed partial denture. after restoring tooth 1. ago. B. greater than the average amount received D. E. a better thermal conductor. Gold-palladium.6 and replacing it C.A preparation for a metal-ceramic crown with a The main purpose of X-ray collimation is to porcelain butt joint margin should have a A. Note: Some of the items in the Released Test Item Bank may have been discontinued due to outdated science or errors. of tooth 4. about the same as the average amount received from medical sources. C. a metal base is C. space of the missing tooth. All of the above.6 has extruded 1. (1) and (3) For a removable partial denture. (4) only from natural sources. Tooth 1. C. Overlapping contacts on a bitewing radiograph D. content. 90o cavosurface margin. (2) and (4) preferred to an acrylic base because metal is D. after devitalizing and preparing tooth 1. The NDEB periodically reviews the bank to improve its quality. after extracting tooth 1. (1) (2) (3) B. 1. All of the above. incorrect horizontal angulation of the x. 2. (1) (2) (3) is B. low when compared to the average total of a resin-bonded fixed bridge? amount of radiation received. 1. (4) only E. 4. stronger. which was extracted many years 3. B. should be fabricated A. What is the alloy of choice for the framework D. D. result from the 1. and translation. the format of some items is not currently used. subgingival finish line. 3. plane of occlusion.6 for a cast crown.6. 3. ©The National Dental Examining Board of Canada 2017 . decreased vertical angulation of the x-ray bridge replacing the mandibular first molar beam. (1) and (3) C. malalignment of teeth. D. C. D. A.2mm shoulder. filter out low energy X-rays. Epinephrine in a local anesthetic solution will E. 2. assist hemostasis at the site of injection. increased vascularity of the jaws. fluoro aluminosilica powder and polyacrylic acid. D. decrease absorption of the anesthetic. A. D. B. reinforced zinc oxide and eugenol cement. reduced vascularity of the jaws. conjugation with serum proteins. C. ©The National Dental Examining Board of Canada 2017 . Which of the following is a sign of a severe Glass ionomer cements contain toxic reaction to ketoconazole? A. build up of tolerance. fluoro aluminosilica powder and C. A. Acute overdose of acetaminophen is most likely to lead to A 78 year old patient presents with several carious lesions on the root surfaces of the A. The NDEB periodically reviews the bank to improve its quality. microfilled composite resin. Jaundice. C. D. Xerostomia. The most likely cause of postoperative sensitivity with Class II composite resin A. All of the above. (1) (2) (3) restorations is B. content. overheating during the finishing process. In addition. maxillary posterior teeth. (4) only B. toxicity of the restorative material. the format of some items is not currently used. C. and translation. A. assist in post-operative healing. E. (1) and (3) C. increased brittleness of the jaws. rapid renal excretion. 3. E. depression of the respiratory centre. anaphylactic shock. D. orthophosphoric acid. D. hepatic toxicity. material of choice is a C. B.An end result of ionizing radiation used to treat The short action of thiopental is due to oral malignancies is A. glass ionomer cement. D. Note: Some of the items in the Released Test Item Bank may have been discontinued due to outdated science or errors. acid etching of dentin. B. 1. Hypertension. The restorative B. (2) and (4) A. C. rapid redistribution. 4. microleakage at the interface. zinc oxide and distilled water. Salivary gland swelling. D. C. hybrid composite resin. D. deformity of the jaws. silver amalgam. rapid metabolism. prolong the action of the anesthetic agent. B. B. severe gastric bleeding. zinc oxide and polyacrylic acid. placement. control polymerization shrinkage in composite resins. be taken only for surgical purposes. B. Type IV gold alloy. C. A. not be taken until end of second trimester. myocardial infarction. purposes. more taper. temporary management of which of the following? bridges because of superior A. B. be taken by panoramic radiography only. D. hardness. ©The National Dental Examining Board of Canada 2017 . A radiograph may performing periodontal surgery for patients with A. a shorter retentive arm. C. C. D. enhance the bond between a porcelain B. brittleness of the composite. C. the format of some items is not currently used. cobalt-nickel partial denture clasp will require C. hepatotoxicity. coronary artery disease. Asthma. content. B. a thicker cross section. nephrotoxicity. facilitate the soldering of gold castings. D. An overdose of acetaminophen can result in A. B. reduce the surface tension when investing D. excessive polymerization shrinkage. means that chromium- B. In addition. Gastric ulcer. increase gastric motility. D. A patient who is four months pregnant requires Antibiotic coverage should be provided when an extraction. B. compared to a A. A. Adrenal corticosteroids The higher modulus of elasticity of a chromium-cobalt-nickel alloy. veneer and the resin cement. neurotoxicity. increase heart rate. and translation. cardiotoxicity. fracture toughness. C. wear resistance. prosthetic heart valve. presence of voids within the material. dental implants. D. D. dimensional stability. B. be taken for diagnostic or treatment A. C. The NDEB periodically reviews the bank to improve its quality. B.Methyl methacrylate resins will perform better Acetylsalicylic acid is indicated for the than composite resins for long span. Myocardial infarction. C. inadequate moisture control during a wax pattern. A. A silane coupling agent is used to The most frequent cause for composite resin restoration failure is A. Note: Some of the items in the Released Test Item Bank may have been discontinued due to outdated science or errors. reduce inflammation. D. Hemophilia. C. D. a shallower undercut. cause vasodilation. Bonding to enamel. A. lateral jaw radiographs. 4. a unilateral distal extension base. (1) and (3) A. A. The NDEB periodically reviews the bank to improve its quality. 1. All of the above. 2. D. C. D. (2) and (4) B. the format of some items is not currently used. 2. In addition. A. complete tooth support. Setting is affected by moisture. Note: Some of the items in the Released Test Item Bank may have been discontinued due to outdated science or errors. increasing the rate of addition of the D. (2) and (4) E. C. B. evidence of periorbital edema. Just prior to the eruption of the first 3. the liquid. As soon as diagnosed. 2. B. decreasing the particle size. Just prior to the eruption of the maxillary B. a post and core retained crown. fistula related to a periapical lesion. Fractures of the maxilla can best be diagnosed by Which of the following are characteristics of restorative glass ionomer cements? 1. D. (1) and (3) canines. 4. reduce the tin-mercury phase. (1) (2) (3) D. clinical examination. C.The setting of a zinc-phosphate cement can Periapical surgery is CONTRAINDICATED best be retarded by for a tooth that has a A. slightly more. reduce tarnish resistance. B. All of the above. (4) only E. (2) and (4) D. C. the same. The coefficient of thermal expansion of the metal relative to the porcelain for constructing The main reason for adding copper to a dental a metal-ceramic crown should be amalgam alloy is to A. (1) (2) (3) B. (1) (2) (3) C. Irritating to pulpal tissues. D. anteroposterior radiograph of the skull. (1) and (3) D. make amalgamation easier. vertical root fracture. large periapical lesion. powder to the liquid. B. increase expansion. 3. C. 4. Indirect retention is required for a removable What is the most appropriate time to extract a partial denture which has mesiodens? 1. cooling the glass mixing slab. an anterior extension base. (4) only C. and translation. Release of fluoride. Just prior to the eruption of the maxillary central incisors. ©The National Dental Examining Board of Canada 2017 . C. permanent molars. significantly less. content. slightly less. bilateral distal extension bases. increasing the concentration of water in B. All of the above. A. A. (4) only E. 3. D. (1) (2) (3) A. Which of the following is B. microstructure. Lidocaine. E. D. pallor. and translation. (2) and (4) D. All of the above. horizontal fracture of the maxilla. Pain. 2. Take a radiograph. Prilocaine. (1) and (3) C. (1) (2) (3) B. Stiffness of wire is a function of 1. Procaine. the format of some items is not currently used. diameter of the wire segment. E. Mepivacaine. Diplopia. (2) and (4) D. fracture involving the orbital floor. (1) and (3) C. (1) and (3) B. An opposing free end saddle removable partial. (4) only E. 2. CONTRAINDICATED for this patient? C. 2. ©The National Dental Examining Board of Canada 2017 . length of the wire segment. pyramidal fracture of the maxilla. All of the above. C. 4. 1. The NDEB periodically reviews the bank to improve its quality. 3. Prescription of penicillin V. All of the above. A. A. 4. Note: Some of the items in the Released Test Item Bank may have been discontinued due to outdated science or errors. infection. Swelling of the orbit. (2) and (4) C. Previous orthodontic treatment. 3. A physical sign of impending syncope is 3. Administration of 2% xylocaine with 1:100. In addition. B. C. Heavily restored abutment. cranio-facial dysjunction. A. (4) only D. D. Which of the following is a CONTRAINDICATION for the use of a resin bonded fixed partial denture (acid etched bridge or “Maryland Bridge”)? A.000 epinephrine. content. B. Malocclusion. elevation of blood pressure. Acetylsalicylic acid for pain control. alloy composition. fracture of the zygomatic arch. B. B. A. (1) (2) (3) C. tachycardia. (4) only E. A Le Fort I fracture is a A 25 year old female in her first trimester of pregnancy presents with an acute dental A.Which of the following may be associated with Which of the following anesthetic agents is/are a fracture of the mandible? metabolized by plasma cholinesterase? 1. Angle Class II malocclusion. 4. are depressants of the cerebral cortex. occlusal trauma is present. In addition. (4) only E. Anaphylactic reaction to penicillin is most C. in patients with a negative skin test to mucogingival junction indicates that penicillin allergy. content. A. (2) and (4) D. when the drug is administered parentally. initiates gingivitis. (2) and (4) D. the base of the pocket extends beyond the 4. and translation. Trauma from occlusion A. there is an increase in crevicular fluid. All of the above. there is no attached gingiva. gingival hyperplasia is present. initiates periodontitis. A. ©The National Dental Examining Board of Canada 2017 . has open gingival embrasures. may have convulsant effects. may cause physical dependency. 3. the format of some items is not currently used. affects the blood supply to gingivae. The width of the attached gingiva can be C. (1) (2) (3) B. 2. gingival surface is convex in all B. (4) only E. B. B. (1) and (3) C. gingival surface is concave and adapts 2. C. 3. gingival inflammation in the area has been eliminated. B. All of the above. 1. in patients who have already experienced A midfacial probing depth measurement where an allergic reaction to the drug. A. 1. short and fat. (1) and (3) directions. accurately measured if D. the probing depth is 3mm or less. gingival recession is present. B. (1) (2) (3) 4. The NDEB periodically reviews the bank to improve its quality. B. C. (1) and (3) C. (1) (2) (3) E. tall and thin. a late maturer. likely to occur D.A pontic replacing a mandibular first molar Tranquilizers and hypnotics should be designed so that it/its 1. An endomorph is typically A. scaling has not been done. 4. within minutes after drug administration. conceals the porcelain to metal junction on its gingival surface. are depressants of the respiratory centre. 2. (4) only A. an early maturer. (2) and (4) D. Note: Some of the items in the Released Test Item Bank may have been discontinued due to outdated science or errors. D. closely to the ridge. 3. A. C. All of the above. a dental malocclusion. Class III. intensity of pain. Note: Some of the items in the Released Test Item Bank may have been discontinued due to outdated science or errors. primary and the permanent incisors. dental calculus. Hydromorphone. B. D. a skeletal malocclusion. a faulty restoration. The NDEB periodically reviews the bank to improve its quality. primary molars and the premolars. C. D. space. the format of some items is not currently used. maxillary and mandibular tooth material. response to a vitality test. temporomandibular disorder. Class II. Meperidine. D. E. perform a space analysis. D. C. content. mandibular second premolar. regain space in the arch. A. Morphine. Which of the following drugs is a natural alkaloid obtained from opium? Thumbsucking after the age of six most often results in A. maxillary and the mandibular primate D. nature of swelling. C. mandibular canine. occlusal trauma. dentition is to B. D. B. Methadone. extract the deciduous canines. maxillary lateral incisor. C. strip all proximal contacts. B. maxillary canine. An acute lateral periodontal abscess associated with a lateral periodontal cyst can be Leeway space is the difference in size between differentiated from an acute abscess of pulpal the origin by the A. dental plaque. type of exudate. B. A. E. C. D.The primary etiologic factor for gingivitis is The most appropriate management for a patient with mild anterior crowding in an early mixed A. ascorbic acid deficiency. incipient. Class I. B. mouth breathing. C. ©The National Dental Examining Board of Canada 2017 . B. C. The most commonly impacted permanent tooth A furcation involvement in which the probe is the extends completely through the furcation is classified as A. B. E. and translation. C. In addition. Codeine. A. D. A. degree of tooth mobility. needle gauge decreases. C. inflammatory and immune reactions. lipid solubility of the un-ionized form. In addition. Extraoral maxillary division block. content. C. D. B. the format of some items is not currently used. D. All of the above. digitalis. needle length increases. A. A. ©The National Dental Examining Board of Canada 2017 . Acute apical abscess with cellulitis. Posterior superior alveolar block. B. C. Previously treated tooth with acute B. C. D. Pulp reactions to caries include all of the Facial nerve paresthesia is most likely to occur following. Antibiotics are indicated for management of The penetration of a local anesthetic into nerve which of the following conditions? tissue is a function of the A. Hyperthyroidism. amide linkage between the aromatic nucleus and the alkyl chain. C. quinidine. E. decrease in dentin permeability. evaporation of the intratubular fluid. Chronic apical abscess. pentobarbital sodium. B. Posterior palatine block. and translation. only organic matter. B. D. Inferior alveolar block. B. Irreversible pulpitis. Acromegaly. length of the central alkyl chain. B. C. ester linkage between the aromatic C. B. EXCEPT from which of the following injections? A. depth of injection increases. periapical periodontitis. Sodium hypochlorite in endodontic therapy dissolves A. only inorganic matter. Hyperparathyroidism. Hypogonadism. both organic and inorganic matter Needle deflection increases as A. Note: Some of the items in the Released Test Item Bank may have been discontinued due to outdated science or errors. The NDEB periodically reviews the bank to improve its quality. A.The most effective drug for relief of angina Which of the following diseases predisposes a pectoris is patient to exaggerated cardiovascular effects of epinephrine? A. nucleus and the alkyl chain. D. C. nitroglycerin. D. codeine. tertiary dentin formation. D. All of the following are possible effects of acetylsalicylic acid EXCEPT For a patient with cardiovascular disease. A.The use of aspirating syringes for the Between the ages of 5 and 18 years. All of the above. C. C. Note: Some of the items in the Released Test Item Bank may have been discontinued due to outdated science or errors. ester. anesthesia B. amide. shortening of bleeding time. affects blood pressure less than general anesthesia. Methylparaben. (4) only E. is responsible for bacteremia. affects blood pressure more than general D. the undercut and non-undercut areas. cerebral cortex. (2) and (4) D. suppression of inflammatory response. aldehyde. C. the position of the survey line on the cast. corpus callosum. The NDEB periodically reviews the bank to improve its quality. changing the tilt of the cast on the surveyor alters the Which of the following is NOT a component of A. Sodium metabisulphite. hypothalamus. D. decreases 0-1mm. C. B. (1) and (3) D. 3. bleeding from the gastrointestinal tract. C. C. and translation. administration of local anesthetics is mandibular arch length recommended because A. B. increased. anesthesia. D. is in an intravascular location. In addition. A. the effectiveness of local anesthesia is B. local A. acid. decreases 3-4mm. ©The National Dental Examining Board of Canada 2017 . 1. 1:100.000 epinephrine? B. their use reduces the frequency of accidental intravenous injection. C. aspiration of blood is proof that the needle D. A. Sodium chloride. When designing a removable partial denture. reduction of fever. path of insertion of the planned a dental cartridge containing 2% lidocaine with removable partial denture. spinal cord. (1) (2) (3) C. A. increases 0-1mm. their use removes the hazard of rapid injection and provides a distinct saving of time. content. Water. B. Lidocaine is an 4. the direction of forces applied to the B. The central action of caffeine is principally on the A. B. increases 3-4mm. the format of some items is not currently used. 2. partial denture. D. to the coronal aspect of the epithelial The highest incidence of congenitally missing attachment. D. likely diagnosis is a/an C. Propranolol. with C. C. C. Condylion to Gnathion. has a high positive predictive value (PPV) D. the tip of the periodontal probe extends A. The NDEB periodically reviews the bank to improve its quality. In addition. distal wedge periodontal surgery. Porion to Gnathion. C. B. Which of the following statements is true about diazepam? It Loss of sensation over the distribution of the A. content. Radiographic A. lateral incisors is most likely seen in a patient B. narrow 8mm periodontal pocket on the buccal surface of the tooth. D. The most B. Note: Some of the items in the Released Test Item Bank may have been discontinued due to outdated science or errors. B. a forceps removal of a mandibular second molar. removal of an impacted mandibular third Canada. One-wall bony defect. B. molar tooth. Condylion to Menton. When probing the sulcus of healthy gingiva. congenital heart disease. is indicative of past inflammation. D. does not produce a dependence syndrome. A. Captopril. of B. for further attachment loss. endodontic lesion. There is papillae? an isolated. Three-wall bony defect. apical to the epithelial attachment. A. is indicative of current inflammation. causes amnesia. Which of the following drugs may cause Tooth 4. removal of a torus mandibularis. In which of the following defects is bone D.5 was treated endodontically 2 years generalized enlargement of interdental ago and is now sensitive to percussion. to the alveolar bone crest. Two-wall bony defect. C. Down syndrome. hyperthyroidism. findings are within normal limits. B. B. ©The National Dental Examining Board of Canada 2017 . and translation. C. chronic periodontal lesion. Nifedipine.Frankfort plane extends from Presence of bleeding on probing A. regeneration most likely to occur? A. unilateral cleft lip and palate. C. B. the format of some items is not currently used. vertical root fracture. A. C. is available without prescription in A. Porion to Orbitale. improves performance of fine motor inferior dental nerve is a possible complication skills. Digitalis. an osteoma appears A. a dentigerous cyst. incisors above their apices. esthetics. E. axial contours of the tooth. (1) (2) (3) space. radicular cyst. surrounded by a radiopaque line. Note: Some of the items in the Released Test Item Bank may have been discontinued due to outdated science or errors. foramen is located between the roots of the maxillary A. the nasopalatine 4. 1. Which of the following pontic designs is most likely to cause soft tissue irritation? In the mandibular third molar region. All of the above. D. an eruption cyst. (1) and (3) A. The radiolucent area suggests D. occlusal function. B. radiolucent. nasolabial cyst. D. Polished porcelain. 4. C. proximal contacts. (2) and (4) A. 2. B. (1) (2) (3) B. Polished acrylic. radiolucent. septum. (1) (2) (3) D.Radiographically. (4) only B. (1) and (3) E. The NDEB periodically reviews the bank to improve its quality. widening of the periodontal ligament 1. In addition. circumscribed radiolucent area 3cm in diameter B. A. radiopaque. All of the above. branchial cyst. and translation. On a periapical radiograph. either radiopaque or radiolucent. vertical destruction of the interdental 3. (2) and (4) B. B. contains the crown of the developing third C. 2. a A. central and lateral incisors above their apices. (1) and (3) C. C. C. C. central incisors below their apices. 3. nasopalatine cyst. All of the above. space. (2) and (4) D. content. 4. decreased definition of the lamina dura. 3. central and lateral incisors below their D. (4) only E. the nasopalatine foramen Traumatic occlusion may cause may be misdiagnosed as a 1. 2. a simple bone cyst/traumatic bone cyst. the format of some items is not currently used. molar. narrowing of the periodontal ligament A. ©The National Dental Examining Board of Canada 2017 . Glazed porcelain. Radiographically. A provisional crown must restore the D. Polished gold. Stafne's idiopathic bone cavity. C. (4) only apices. (1) (2) (3) A. (1) (2) (3) B. a dentigerous cyst. punched out radiolucent lesions. Note: Some of the items in the Released Test Item Bank may have been discontinued due to outdated science or errors. the format of some items is not currently used. (2) and (4) D. content. Which of the following is CONTRAINDICATED for this patient? 1. euphoria.Therapeutic doses of morphine administered A discontinuous lamina dura may be a intramuscularly may produce consequence of 1. eburnated bone. (4) only D. 3. 2. The mental foramen. (2) and (4) C. parathyroid hyperplasia. 1. B. dysphoria. The NDEB periodically reviews the bank to improve its quality. (1) and (3) B. metastatic carcinoma. (4) only E. drowsiness. a nasopalatine cyst. A 25 year old female in her third trimester of Which of the following entities can present as pregnancy presents with an acute dental periapical radiolucencies? infection. prescription of ibuprofen. C. pulpitis. C. Extraction using 2% lidocaine with 1:100. (2) and (4) D. A periapical granuloma. All of the above. (2) and (4) C. 4. 3. (1) (2) (3) A. ©The National Dental Examining Board of Canada 2017 . and translation. ground glass appearance. All of the above. A radicular cyst. All of the above. 2. 3. E. D. (4) only E. the foramen of the incisive canal. regular radiolucent line around the roots. B. Prescription of clindamycin. 4. (1) (2) (3) D. The lamina dura is a/an A. regular radiopaque line around the roots. A. In addition. 4. 4. constipation. (1) and (3) C.000 epinephrine. All of the above. (1) and (3) C. A. 3. 1. A. D. multiple radiopaque lesions. The radiographic images most suggestive of multiple myeloma is A. Prescription of penicillin V. B. irregular radiolucent line around the roots. A periapical radiolucency associated with a B. A metastatic carcinoma. (1) and (3) B. (4) only E. 2. a periapical granuloma. generalized hypercementosis. 2. vital maxillary central incisor can represent C. odontoma. A. C. (4) only C. Central giant cell granuloma. (2) and (4) B. and translation. All of the above. a chronic periapical abscess. Demineralization of teeth. Note: Some of the items in the Released Test Item Bank may have been discontinued due to outdated science or errors.Which of the following landmarks may be A radiograph displaying a “cotton-wool”' identified on periapical radiographs of the appearance and generalized hypercementosis is mandible? suggestive of 1. 2. Hypercementosis. Mylohyoid ridge. finding in hyperparathyroidism? A. 3. C. (1) and (3) C. 4. B. A. Central giant cell granuloma. (1) (2) (3) B. 4. Paget’s disease. Osteopetrosis. a periapical cyst. 3. A well circumscribed 3mm radiolucency in the C. fibrous dysplasia. Keratocystic odontogenic tumor. Multiple myeloma. D. 3. apical region of the mandibular second D. premolar may be E. D. E. E. calcifying epithelial odontogenic tumour. ©The National Dental Examining Board of Canada 2017 . D. adenomatoid odontogenic tumour. C. 2. All of the above. Letterer-Siwe disease. Mental foramen. Multiple keratocystic odontogenic tumors. (4) only What is the most significant radiographic E. In addition. Rampant caries. Odontogenic myxoma. The NDEB periodically reviews the bank to improve its quality. Coronoid process. E. (1) and (3) C. (4) only E. Which lesion/s may appear radiographically as Which condition produces a radiopaque image? multilocular radiolucencies? A. External oblique ridge. Generalized loss of lamina dura. 1. (1) (2) (3) B. (1) (2) (3) B. B. Ameloblastoma. content. ameloblastic fibro-odontoma. B. A. osteogenesis imperfecta. (1) and (3) A. (2) and (4) D. Osteomalacia. D. 2. a periapical granuloma. cleidocranial dysplasia. the mental foramen. osteopetrosis. (2) and (4) D. 1. All of the above. the format of some items is not currently used. A mixed lesion with a radiolucent rim and 4. corticated border causing impaction of a permanent tooth is most likely a/an A. most likely cause of this bleeding is a/an D. A. to consider that prostaglandins B. detecting A. B. extraction of tooth 4. completion of a mandible cartilage model. The x-rays which are most apt to be absorbed by the skin and cause an injury are An alcoholic patient with bleeding tendencies may require injection of which vitamin prior to A. a normal apex. In addition. The start of a mandibular bone formation coincides with When prescribing nonsteroidal anti- inflammatory drugs (NSAIDs). advanced alveolar atrophy. D. D. Bite-wing radiographs are most valuable for D. an apical radiolucency.7. B. B. rays of long wave-length.A patient complains of acute pain 24 hours A mandibular fracture during normal after the insertion of a restoration in a tooth mastication is most likely to occur in a patient with no pre-existing periapical pathology. D. osteoporosis. B. The NDEB periodically reviews the bank to improve its quality. content. C. The C. dry socket. an impacted tooth at the inferior border. cervical caries. aluminum filtered rays. undiagnosed coagulopathy. osteosclerosis. The radiograph will show A. central rays. a large intraosseous lesion. The with tooth is vital and tender to percussion. D. infection of the socket. condensing osteitis. impair blood coagulation. C. D. severe bleeding from the extraction site. protect the gastric mucosa. induce vasoconstriction. first trimester. disturbance of the hemostatic plug. prevent edema. C. Vitamin B12. Vitamin C. Vitamin K. proximal surface caries. hyperemia of the pulp. tooth germ development. deep penetrating rays. Vitamin D. The highest risk of radiation-induced congenital malformation occurs during the Twenty-four hours following the simple A. D. B. it is important A. C. C. C. C. B. D. and translation. A. C. third trimester. occlusal carious lesions. the format of some items is not currently used. A. A. ©The National Dental Examining Board of Canada 2017 . second trimester. bone formation in the maxilla. a patient presents with B. Note: Some of the items in the Released Test Item Bank may have been discontinued due to outdated science or errors. an extraction? B. neonatal period. completion of fusion of the facial processes. 3-4 seconds. “Ground glass” is the classical description of Direct pulp capping of permanent teeth in the radiographic appearance found in children under the age of 12 years is most likely to be successful for A. Paget’s disease of bone. content. 9-10 seconds. A. In a dental office. A. fibrous dysplasia. C. D. pulp exposures 3-5mm in size. Hyperventilation. fever related hypoplasia. panoramic. periapical and occlusal. C. A. B. The NDEB periodically reviews the bank to improve its quality. teeth with open apices.The most appropriate radiograph(s) to Extensive caries located only on the primary determine the location of an impacted maxillary incisors of a 2 year old is generally maxillary cuspid is/are indicative of A. ankyloglossia. if a patient is not breathing but Localized gingival recession of a permanent has a pulse. A. D. localized aggressive periodontitis. During CPR. B. teeth that are symptomatic. D. acute osteomyelitis. B. B. occlusal. demand breast or bottle feeding. C. Myocardial infarction. vitamin C deficiency. how often should a breath be mandibular incisor in an 8 year old can be given? Every caused by A. C. B. retained natural colour of the tooth. D. Note: Some of the items in the Released Test Item Bank may have been discontinued due to outdated science or errors. Bronchospasm. formation of pulp stones. D. periapical. early periapical osseous dysplasia B. 7-8 seconds. C. traumatic occlusion. Anaphylaxis. pulp regeneration. In addition. B. the format of some items is not currently used. important clinical criterion of success is B. teeth that are hyperemic. 5-6 seconds. (periapical cemento-osseous dysplasia). amelogenesis imperfecta. necrotizing ulcerative gingivitis. the most A. E. and translation. C. D. ©The National Dental Examining Board of Canada 2017 . D. C. C. completion of root formation. what is the most common cause of respiratory distress? After partial pulpotomy of a permanent central incisor in an 8 year old child. fluorosis of the primary dentition. D. D. observe it until it becomes symptomatic. To minimize fractures of amalgam restorations B. prescribe an antibiotic. perform a pulpectomy. content. A. primary first molar is to C. B. In the absence of its permanent successor. The most important factor in stainless steel crown retention in a primary tooth is the A. discolouration. Mesiodens. D. D. The NDEB periodically reviews the bank to improve its quality. D. Note: Some of the items in the Released Test Item Bank may have been discontinued due to outdated science or errors. C. is more susceptible to dental caries. extract it and place a space maintainer. C. A. rounded axiopulpal line angles. a large labial frenum. Dens invaginatus. B. A. In addition. cavity preparations should resorption. D. clinically exhibit C. temperature sensitivity. ©The National Dental Examining Board of Canada 2017 . C. remove carie and place a temporary restoration. a E. should be treated endodontically to prevent root resorption. D. and translation. D. adenomatoid odontogenetic tumour. Gemination. should be extracted. the format of some items is not currently used. primary first molar of a 7 year old A. percussion sensitivity. Ankylosed primary second molars may B. EXCEPT A. In a 4 year old the most appropriate treatment B. B. A. permanent incisors could be associated with the follwings. a mesiodens. buccolingual displacement. preservation of the coronal bulge. maintenance of parallelism. B. non-restorable maxillary lateral incisors. Space shortage. prescribe an analgesics. C. observe it until it exfoliates. infra-occlusal position. throbbing toothache the previous night is to A. concave pulpal floors.Which of the following conditions is the most The most appropriate management for a child likely cause of a maxillary midline diastema? with a primary tooth that caused a severe. have C. a congenital absence of permanent for a chronically infected. perform a pulpotomy. extract it only. C. may remain for years with no significant in primary teeth. lateral walls parallel to the external surface A diastema between two maxillary central of the tooth. B. bevelled cavosurface margins. D. insertion of retentive pins. E. placement of accessory grooves. direct pulp capping. D. extract the contra-lateral molar. decreased overjet. C. C. B. perform space analysis. D. Weekly 0. pulpotomy. zinc phosphate. Twice daily brushing with fluoride pulpotomy on a primary tooth? toothpaste. climate. occlusal restorations on all primary first C. Which of the following is an indication for a D. A. regularly assess the arch development. Signs and symptoms of reversible pulpitis. B. Involvement of interradicular bone with a maintainer is fistula. Following the extraction of a primary first molar in a 4 year old child. polycarboxylate. E. A. the most fistula. C. C. Daily sodium bicarbonate rinse. content. B. D. and translation. The NDEB periodically reviews the bank to improve its quality. extract the opposing molar. A 6 year old child has a non-vital primary mandibular second molar which has a draining sinus tract from the bifurcation area.2% NaF rinse. total daily fluoride intake. with closed A. the most appropriate management is to A. increased intercanine space. B. Radiographic evidence of internal root resorption. A. Fluoride varnish at 1 month intervals. C. D. insert a space maintainer. B. the format of some items is not currently used. ©The National Dental Examining Board of Canada 2017 . glass ionomer. extraction. B.The extraction of a primary maxillary central Which of the following is the most important incisor at the age of 6 years will cause preventive therapy for an 8 year old living in a non-fluoridated community. Note: Some of the items in the Released Test Item Bank may have been discontinued due to outdated science or errors. contacts in the posterior quadrants and disto- B. molars? D. Involvement of interradicular bone with no In a child with a high caries risk. daily water consumption. loss of intercanine space. A. appropriate cement for a band and loop space C. D. water fluoride concentration. observation. In addition. no change in intercanine space. The most appropriate management is The most important factor in determining the dosage of systemic fluoride supplementation is A. zinc oxide eugenol. extract the mesiodens and allow passive enamel only. C. Proceed with treatment without rubber periapical involvement. This finding indicates that D. is nonvital and endodontic therapy is indicated. B. electric pulp tester. and translation. The most appropriate management is to However. pulpotomy. The tooth cannot extracted. Wait 15 minutes until more profound anesthesia is achieved. extract the mesiodens and orthodontically the tooth extrude 2. old child.5ml of 2% incisor with a necrotic pulp and a wide open lidocaine with 1:100.1 unerupted D. A. regeneration. B. prescribe an antibiotic. wait for eruption following a facial injury resulting in a fracture and then extract it. of a maxillary central incisor that involves the C. appropriate management is to A. B. Anesthetize the long buccal nerve with the occurs on a permanent first molar of a 7 year remaining lidocaine. root canal therapy. content. The most appropriate immediate D. The tooth is vital and has no D. the format of some items is not currently used. extraction. ©The National Dental Examining Board of Canada 2017 . The NDEB periodically reviews the bank to improve its quality. should be observed and tested again at a management is to later date. during placement of a rubber dam perform a/an clamp on the first permanent molar.1 for another year. maintain space with a removable appliance. The most E. B.1. In addition. management? C. uncover the mesiodens. Which of the following is the most appropriate B. pulpectomy. be found. apex. is nonvital and should be extracted. A 1. A 7 year old has just lost a permanent maxillary C. test adjacent teeth for vitality.5mm diameter carious pulp exposure C. A. order a soft tissue x-ray. reveal a palatally located mesiodens.1. pulpotomy.1 C. The tooth tests negative to an eruption of 2. A.000 epinephrine. has a root fracture and should be central incisor due to trauma.A child has received a successful inferior An 8 year old patient has a permanent central alveolar nerve block using 1. Note: Some of the items in the Released Test Item Bank may have been discontinued due to outdated science or errors. the child complains that the “tooth ring” is hurting. A 9 year old presents for treatment immediately B. D. D. monitor the eruption of 2. The most appropriate dam. apexification. pulp capping. A. partial pulpotomy The radiographs of a 9 year old with tooth 1. Anesthetize the lingual nerve with the remaining lidocaine. initial treatment is to perform a/an A. completely erupted and tooth 2. A. B. C. An overjet of 8mm is usually associated with D. (1) (2) (3) B. maxillary and lateral nasal processes. gonial angle. D. C. 2. the format of some items is not currently used. Koplik’s spots. diagnosis of a congenitally missing mandibular C. 3 years. 5 years. The NDEB periodically reviews the bank to improve its quality. the main growth site is in the stabilize the root’s surface. D. lateromedial extensions. Pterygopalatine. dental follicle cells migrate into the epithelial diaphragm. Frontomaxillary. C. (2) and (4) D. A. Resorbs along the posterior rami. content. of the following characteristics? D. B. Has completed 100% of its growth by age 13 in females. Note: Some of the items in the Released Test Item Bank may have been discontinued due to outdated science or errors. 1. (4) only. lateral and medial palatine processes. 3. C. posterior border of the ramus. Grows more vertically than horizontally. A.Which of the following conditions is an The principal growth sites of the maxilla in a example of a mucous retention phenomenon? downward and forward direction include which of the following sutures? A. Ranula. Palatine. Cleft lip is caused by the failure of which of the following processes to fuse? A. second premolar? A. condylar cartilage. All of the above. epithelial diaphragm forms too many E. Zygomaticomaxillary. inferior and lateral aspects of the body of The upper lip is the result of fusion between the the mandible. maxillary and mandibular processes. 7 years. B. ©The National Dental Examining Board of Canada 2017 . What is the earliest age to confirm the B. Has latent post-pubertal growth potential. B. A. The normal growing mandible exhibits which C. Nicotine stomatitis. A. medial and lateral nasal processes. C. B. Maxillary and lateral nasal. and translation. Median palatine. E. D. cementoblasts fail to develop on and In the mandible. Angle Class III cuspid relationship. D. D. B. Residual cyst. periodontal ligament forms too soon. Angle Class I molar relationship. B. E. (1) and (3) Developing roots will split when the C. 4. maxillary and medial nasal processes. A. Maxillary. Nasopalatine cyst. C. Angle Class II cuspid relationship. Angle Class I cuspid relationship. 1 year. In addition. 2 severe interproximal attachment loss affecting has the best long term prognosis if 1. A. E. with 50% overbite. Normal articulation of primary canines is D. growth spurt. generalized aggressive periodontitis. mesial inclined plane of the maxillary B. involves the 4 permanent first premolars. B. the arches. B. In addition. plane of the mandibular canine. horizontal bone loss.2 is the first molars and incisors. but Correction of a lingual crossbite of tooth 1. The most likely diagnosis is A. is commenced with bilateral expansion of B. permanent molars. The distal inclined plane of the maxillary canine articulates with the mesial inclined canine articulates with the mesial inclined plane of the mandibular canine. Which of the following would differentiate Orthopedic correction of a mild skeletal Angle clinically between an acute apical abscess Class III malocclusion should be started (acute periradicular abscess) and an acute periodontal abscess on a single rooted tooth? A. D. generalized chronic periodontitis. Note: Some of the items in the Released Test Item Bank may have been discontinued due to outdated science or errors. localized chronic periodontitis. adolescent will most frequently present with B. ideally inclined. C. B. lingually inclined. end-to-end. content. the permanent canines are end-to-end. plane of the mandibular canine. Pain upon percussion. necrotizing ulcerative gingivitis. Tooth mobility. shortly after eruption of the upper first C. B. C. The mesial inclined plane of the maxillary canine articulates with the distal inclined canine articulates with the distal inclined plane of the mandibular canine. D. is best suited to Class I dental and skeletal D.Which of the following represents the normal In an Angle Class I occlusion the relationship of the primary canines? A. A. C. distal inclined plane of the maxillary A. with 50% overbite. just prior to the pre-pubertal growth spurt. Pain upon palpation. D. Serial extraction During orthodontic treatment. B. localized aggressive periodontitis. ideally inclined. with 5% overbite. C. the primary canines are end-to-end. requires leeway space maintenance. with 5% overbite. C. A 16 year old healthy patient has good oral hygiene with minimal plaque and calculus. ©The National Dental Examining Board of Canada 2017 . immediately following the pre-pubertal A. the format of some items is not currently used. C. D. D. and translation. is the treatment for Class II skeletal malocclusions with severe space shortage. C. malocclusions with minimal space shortage. a healthy A. Pulp vitality testing. The NDEB periodically reviews the bank to improve its quality. vertical bone loss. shortly after eruption of the upper second permanent molars. lingually inclined. gingivitis. C. content. actinomycetemcomitans. ©The National Dental Examining Board of Canada 2017 . and translation. B. B. D. Smoking tobacco. Gram-positive aerobes. The predominant micro-organisms associated D. Diagnosis of periodontitis is clinically D. (1) and (3) The periodontal condition showing localized C. C. A. develops from gingivitis. A. Porphyromonas gingivalis. and quiescence. is mineralized subgingival plaque. (1) (2) (3) B. Prevotella intermedia. Tooth migration. C. periodontitis as a manifestation of a with periodontitis are systemic disease. aggressive localized periodontitis. Gram-negative anaerobes. Note: Some of the items in the Released Test Item Bank may have been discontinued due to outdated science or errors. bleeding from the base of the periodontal pocket. B. A. Poorly controlled diabetes. 4. goes through stages of tissue destruction B. (2) and (4) advanced vertical bone loss involving the first D. is associated with bone loss. A. Gram-negative aerobes. B. a periodontal pocket. loss of periodontal attachment. is the cause of periodontitis. drinks. Gram-positive anaerobes. The NDEB periodically reviews the bank to improve its quality. derives its dark colour from food and B. the format of some items is not currently used. pain upon probing. D. C. (4) only molars and the incisors is diagnosed as E. Fremitus. Poor oral hygiene. All of the above. chronic localized periodontitis. necrotizing ulcerative periodontitis. C. A. Widened periodontal ligament. accumulates predominantly on mandibular incisors and maxillary molars. D. 3. loss of periodontal attachment. In addition. Gingival recession. C. Aggregatobacter (Actinobacillus) C. D. tooth mobility. is caused by occlusal trauma. Coronary heart disease. demonstrated by A. Fusobacterium nucleatum. B. D. 2.Which of the following is NOT a sign of Which of the following is NOT a risk factor for occlusal trauma? periodontitis? A. B. Subgingival calculus The principal microorganism in localized aggressive periodontitis is A. A. C. Clinical diagnosis of periodontitis requires the Periodontitis presence of 1. bleeding upon probing. Severe chronic periodontitis is defined by B. C. B. Primary herpetic gingivostomatitis. Pregnancy. destruction of vessels of the periodontal ligament. lack of stippling. lack of stippling. Gingival trauma. Dental plaque. A.Which of the following is LEAST likely to The most common complaint of a patient with influence the development of plaque induced gingivitis is gingivitis? A. Diabetes. painful gums. Cyanotic. Gluten intolerance. Red. swollen. Which of the following is an etiologic factor in D. The NDEB periodically reviews the bank to improve its quality. Note: Some of the items in the Released Test Item Bank may have been discontinued due to outdated science or errors. Class II furcation involvement in D. C. Necrotizing ulcerative gingivitis. the format of some items is not currently used. In addition. Red. C. B. POOREST prognosis? A. increased stippling. D. A. A. and 6mm pockets. content. bad breath. Which of the following conditions has the D. and translation. presence of purulent exudate. periodontal pockets greater than 5mm. D. lack of stippling. C. E. swollen. D. Gingival tissue with 1mm pocket labial to a mandibular incisor. attachment loss greater than 5mm. swollen. Pemphigus. C. B. changes in the texture of the gingiva. C. Acute stress. presence of a Class I furcation maxillary molars. D. Traumatic occlusion. bleeding on brushing. Recurrent aphthous stomatitis. Pink. Lack of attached gingiva. Gingival tissue with edematous red interdental papillae. A. induced gingivitis is due to C. Which of the following is contagious? Which of the following changes in colour. induced gingivitis? C. Firm gingival tissue with generalized 5 B. ©The National Dental Examining Board of Canada 2017 . a vitamin C deficiency. contour and texture are indicative of plaque B. B. Calculus. Gingival bleeding associated with plaque B. the development of necrotizing ulcerative gingivitis? A. excessive pocket depth. A. D. involvement. microulceration of sulcus epithelium. cleft formation. 4. D. (1) and (3) surface. in a 25 year old patient. (1) and (3) C. C. Severe refractory chronic periodontitis. 4. mouth. A cemento-enamel projection. D. probing depth. (1) (2) (3) periodontitis in a 30 year old patient. rigidness of the splint applied. Localized severe aggressive periodontitis 2. oral hygiene status. Localized severe aggressive periodontitis in a 16 year old patient. thoroughness of the curettage of the root B. (1) (2) (3) B. Generalized severe aggressive A. B. C.Which type of periodontitis is generally treated Maintenance care for a patient treated for WITHOUT systemic antibiotics? periodontal disease includes periodic assessment of A. (4) only C. All of the above. C. 1. sensitivity to thermal A. teeth with short roots. (4) only E. B. In addition. gingival inflammation. Resective osseous surgery is best suited for periodontal sites with A. fibrotic. Severe generalized chronic periodontitis. content. effectiveness of the irrigation of the socket. length of time the tooth has been out of the 2. C. tooth mobility. early to moderate bone loss. desensitizing the roots with an appropriate medicament. (1) and (3) A. adjusting the occlusion. edematous. All of the above. and translation. 1. 3. D. The NDEB periodically reviews the bank to improve its quality. 2. D. Narrow root separation. change is reduced by B. E. (2) and (4) D. severe intrabony defects. 3. Wide root separation. Note: Some of the items in the Released Test Item Bank may have been discontinued due to outdated science or errors. 3. A. B. below the mucogingival junction. Which of the following conditions has the WORST prognosis for a furcation involved tooth? After periodontal surgery. Root planing is used in the treatment of pockets The prognosis for a replanted developed which are permanent tooth is most influenced by the 1. C. the format of some items is not currently used. All of the above. (4) only E. keeping the roots free of bacterial plaque. A. severe attachment loss. D. 4. infrabony. replaning the roots. E. (2) and (4) B. A bifurcation ridge. use of an antibiotic. (1) (2) (3) E. ©The National Dental Examining Board of Canada 2017 . (2) and (4) D. (2) and (4) D. 4. There is no indication that this tooth requires periodontal treatment. ©The National Dental Examining Board of Canada 2017 . access to perform osseous recontouring. All of the above. D. Coronally positioned flap. D. and junctional epithelium. calculus. infrabony pockets. fibrotic gingival enlargements. B. D. no sign of inflammation or loss of attachment? A. In periodontal flap surgery. calculus and contaminated cementum. None of the above. E. reduction in pocket depth is due to The objective of root planing during periodontal therapy is to remove A. E. derives from subgingival plaque. D. does not have a definite predelection for a 2. epithelial attachment. A surgical flap approach to periodontal pocket B. sever the attachment of the oblique fibres of the periodontal ligament. remove the sulcular lining of the pocket. Following root planing. suprabony pockets. Root planing. the format of some items is not currently used. specific site or sites. plaque. C. retention of gingiva. D. content. aid in healing. excise the keratinized gingiva. In addition. elimination permits C. connective tissue attachment. D. pseudopockets. Note: Some of the items in the Released Test Item Bank may have been discontinued due to outdated science or errors. All of the above. A. healing by primary intention. (4) only E. plaque and calculus exclusively. Localized gingivectomy. the initial incision is made to A. C. A. C. osseous contours. healing by primary intention. shrinkage of the gingival tissue. contaminated cementum C. 3. B.Gingivectomy is indicated for Subgingival calculus 1. plaque. and translation. Autogenous connective tissue graft. All of the above. A. The NDEB periodically reviews the bank to improve its quality. plaque. Which of the following periodontal procedures is indicated on a maxillary canine that will receive a full crown with subgingival margins The gingivectomy approach to pocket when the abutment has 1mm of attached elimination results in gingiva. B. B. retention of all or most of the attached C. (1) (2) (3) D. adequate access to correct irregular A. B. A. gingiva. B. All of the above. calculus and crevicular epithelium. all cementum associated with periodontitis. B. (1) and (3) C. may take longer to form compared to supragingival calculus. C. correct gingival architecture. facilitate plaque control. remove granulation tissue. In addition. D. provide access for periodontal debridement. laterally positioned flap. the format of some items is not currently used. C. D. necrotizing ulcerative gingivitis. A. C. Most bone loss following the placement of dental implants occurs at Regular use of oral irrigators will A. prevent plaque formation. The primary objective of periodontal flap surgery in the treatment of periodontitis is to The most reliable measurement of the effectiveness of root planing at re-evaluation 4- A. remove debris. remove plaque. C. A. increased sulcular fluid flow. free autogenous gingival graft. C. encourage wound healing. subepithelial connective tissue graft. molars. absence of bleeding upon probing. B. enhance regeneration. D. 2 – 4 years. D. Mesial of maxillary incisors. content. D. A. create new attachment. B. Lingual of mandibular first premolars. double papilla pedicle graft. E. Distal of the palatal roots of maxillary B. root smoothness. absence of plaque. Note: Some of the items in the Released Test Item Bank may have been discontinued due to outdated science or errors. A. disuse atrophy. D. D. C. Mesial of maxillary first premolars. C. remove calculus.Which of the following root surfaces have A patient presents with 5mm of gingival concavities that make root planing difficult? recession on the labial of tooth 1. generalized chronic periodontitis. plaque-induced gingivitis. B. The most predictable surgical procedure to achieve root A. add bone support. and translation.3. B. B. coverage on this tooth is a B. The NDEB periodically reviews the bank to improve its quality. 0 – 1 year. C. 6 weeks later is B. prevent bacteremia. ©The National Dental Examining Board of Canada 2017 . C. A daily chlorhexidine rinse following periodontal flap surgery is primarily used to Pathologic migration of teeth is a clinical feature of A. 10 – 12 years. D. 5 – 7 years. The NDEB periodically reviews the bank to improve its quality. content. (1) and (3) C. D. necrotizing ulcerative gingivitis. (4) only E. Which A. tendency to recur. A. E. of the following medications is most B. D. Aggressive periodontitis has all of the A 58 year old woman complains of electric following features EXCEPT shock-like pain on the left side of the chin and lip when eating or taking a hot shower. and translation. All of the above. Exfoliative cytology is indicated in the 3. Amoxicillin. In addition. mixed radiopaque/radiolucent C. A. E. lichen planus. All of the above. herpes simplex. Calcifying epithelial odontogenic tumour. C. Odontogenic adenomatoid tumour. B. osteoporosis of the alveolar process. examine the fluid under a microscope. A. Cementoblastoma. submit the tissue for histological analysis. inflammatory origin. Note: Some of the items in the Released Test Item Bank may have been discontinued due to outdated science or errors. D. culture the fluid. Periapical osseous dysplasia (periapical cemento-osseous dysplasia). Nystatin is the most appropriate drug to treat A. decrease in tooth mobility. Ibuprofen. appropriate to confirm the diagnosis? C. D. aphthous ulceration. order blood tests. B. the format of some items is not currently used. B. Which of the following is/are associated with C. perform a cytologic smear. 1.Disuse atrophy of the periodontium causes Keratocystic odontogenic tumours (odontogenic keratocysts) have a/an A. C. B. specific periodontal microbial pathogens. periradicular abscess. C. ©The National Dental Examining Board of Canada 2017 . A. benign mucous membrane pemphigoid. E. The most appropriate method to diagnose a cystic tumour is to A. B. (1) (2) (3) B. Carbamazepine. diagnosis of 4. radiographic evidence of bone loss. erythema multiforme. Acetaminophen and codeine. candidiasis. changes in the arrangement of fibre bundles. ulcerations of the gingiva. C. B. appearance. aphthous stomatitis. (2) and (4) D. rapid attachment loss. D. solid consistency. narrowing of the periodontal ligament. an impacted tooth? D. D. 2. 3. D. thrombocytopenic purpura. (4) only B. B. D. salicylate poisoning. can appear cystic radiographically. dentoalveolar abscess. xerostomia? 1. E. gingiva. The most common site of intraoral squamous cell carcinoma is the Leukemic gingivitis has a similar clinical appearance to A. palate. (4) only Which of the following is/are associated with E. bronchiectasis. A. Atropine administration. In addition. 4. Mikulicz's disease. 1. D. D. fracture. rhinitis. squamous cell carcinoma. Oral lesions can be found in all of the 3. buccal mucosa. epulis granulomatosum. Acute anxiety state. A. Note: Some of the items in the Released Test Item Bank may have been discontinued due to outdated science or errors. infectious mononucleosis. A. (1) and (3) C. (1) (2) (3) B. content. B. Geographic tongue. (4) only E. ©The National Dental Examining Board of Canada 2017 . Candidiasis.Osteomyelitis of the mandible may follow The most likely diagnosis of a proliferative lesion found at a denture periphery is a/an 1. B. ascorbic acid deficiency gingivitis. C. are metastatic lesions. (1) (2) (3) Hemangiomas of the jaws B. (1) and (3) C. Sjögren syndrome. 2. the format of some items is not currently used. D. 2. All of the above. C. floor of the mouth. (1) (2) (3) B. 3. D. (2) and (4) A. A. leukemia. An odour of acetone on the breath can be found in patients with Which of the following conditions is/are associated with AIDS? A. A. All of the above. Vincent’s angina. necrotizing ulcerative periodontitis. C. (1) and 3) C. All of the above. C. diabetes mellitus. giant cell granuloma. Hairy leukoplakia. pernicious anemia. following conditions EXCEPT for 4. D. Acute marginal periodontitis. B. radiotherapy. (2) and (4) D. A. 2. C. infectious mononucleosis. are malignant. The NDEB periodically reviews the bank to improve its quality. C. obstructive jaundice. (2) and (4) D. B. never occur in bone. epulis fissuratum. and translation. 4. Traumatic bone cyst. All of the above. white plaques. radiotherapy. 4. C. content. candidiasis. which of the following medications is the most appropriate? A pleomorphic adenoma is characterized by A. (2) and (4) D. (4) only In the treatment of necrotizing ulcerative E. C. D. C. herpes zoster. A. D. enucleation. B. resection. Erosion. B. E. metastases. An analgesic. Keratocystic odontogenic tumour (odontogenic keratocyst). B. Note: Some of the items in the Released Test Item Bank may have been discontinued due to outdated science or errors. 3. obtain a complete blood count. A. Attrition. pain. B. D. Hypoplasia. the format of some items is not currently used. D. D. pemphigus vulgaris. A. D. hairy tongue. The most appropriate management of a painless Which of the following is NOT a true cyst? ulcer of 2 months duration affecting the lateral border of the tongue is to 1. Radicular cyst. ©The National Dental Examining Board of Canada 2017 . The most appropriate treatment of an ameloblastoma is The characteristic oral lesion(s) of pemphigus is/are A. candidiasis. C. B. The most likely diagnosis is A. psoriasis. obtain a cytologic smear. A. 2. A systemic antibiotic. gingivitis with associated lymphadenopathy. perform an incisional biopsy. An anti-inflammatory. In addition. Abrasion. C. B. prescribe systemic antibiotics. (1) and (3) C. recurrance. C. chemotherapy. Lateral periodontal cyst. E. C. B. A topical antibiotic. (1) (2) (3) B. Fordyce granules. ulceration.An 80 year old man develops multiple painful Which of the following conditions may result skin vesicles along the distribution of the right from horizontally brushing the teeth? infraorbital nerve. vesicles and bullae. re-evaluate in 6 months. D. A. The NDEB periodically reviews the bank to improve its quality. and translation. C. Mesial release incision. edentulism. Inverted. be placed on the crown of the tooth. (1) (2) (3) B. the most appropriate D. cystic ameloblastoma. ranula. All of the above. osteomyelitis. socket. allow for a 180° rotation. A. B. A. C. inject the joint with hydrocortisone. Mesio-angular. D. hemorrhage. A. and translation. B. B. The patient is severely hyperthyroid and Management of a “dry socket” should include not under treatment. A. C. immobilize for ten days. Note: Some of the items in the Released Test Item Bank may have been discontinued due to outdated science or errors. time. saline irrigation of socket. A patient dislocates his mandible for the first C. supportive therapy. Narrow based. D. management is to A. B. 3. Able to be repositioned over bone. The tooth is seated in a pneumatized B. C. A.Management of a patient with an acute Which impacted mandibular third molar is periradicular abscess should include usually easiest to remove? 1. forceps D. hygroma. Disto-angular. Marsupialization is the most appropriate C. After reduction. D. be pulled coronally during extraction. placement of topical antibiotics in the D. age. inject the joint with a sclerosing solution. B. drainage. CONTRAINDICATION for a single tooth extraction in a general dental office? A. (1) and (3) C. Which of the following is a basic design C. The NDEB periodically reviews the bank to improve its quality. Limited bone exposure. The tooth has internal root resorption. the format of some items is not currently used. ©The National Dental Examining Board of Canada 2017 . (2) and (4) With respect to extraction of teeth. Horizontal. muscle attachments. D. Which of the following is a D. C. The patient is a well-controlled diabetic. surgical technique in the management of D. be applied parallel to the long axis of the tooth. 4. In addition. a prescription for systemic antibiotics. have the patient exercise the mandible to principle for a mucoperiosteal flap? avoid trismus. elimination of the cause. vigorous curettage of the socket. B. 2. Displacement of fractures is influenced by A. A. content. external hot compresses. B. (4) only beaks should? E. maxillary sinus. D. pen V. the format of some items is not currently used. continued growth is likely to result in a supernumerary tooth. apex pointing to the dentino-enamel D. B. E. dentin without pulpal involvement. A. In addition. hyperemia. the cementum only. Note: Some of the items in the Released Test Item Bank may have been discontinued due to outdated science or errors. the enamel and in the dentin up to 1mm. Periodontal disease. diet high in citrus fruit. C. E. (2) and (4) C. metronidazole. occlusal parafunction. All of the above. 4. cephalosporin. development of root caries? 1. content. C. D. ©The National Dental Examining Board of Canada 2017 . junction. The NDEB periodically reviews the bank to improve its quality. acute pulpitis. apex pointing to the enamel surface. C. Xerostomia. The most likely cause is A. Dentigerous cysts should be completely 3. (1) and (3) recurrence potential. extract the tooth in the office using preoperative sedation and local anesthesia without a vasoconstrictor. B. D. xerostomia. extract the tooth in the office using local Which of the following factors influence(s) the anesthesia with a vasoconstrictor. C. ampicillin. A. C. D. B. The most appropriate surfaces of all maxillary anterior teeth. B. A. A. and translation. D. with local anesthesia. D. (4) only become osteoblastic. bulimia. the epithelial lining of the cyst has a high B. admit the patient to hospital for extraction A. A vertical cross-section of a smooth surface carious lesion in enamel appears as a triangle An incipient carious lesion is described as with the being in A. erythromycin. base facing toward the pulp. the enamel only.The most appropriate antibiotic for a periapical A patient reports pain on mastication since the dental abscess is placement of a metal-ceramic crown 2 weeks earlier. base at the dentino-enamel junction. A healthy 66 year old patient who had a A 19 year old female with an otherwise healthy myocardial infarct eight years previously dentition presents with erosion of the lingual requires an extraction. supraocclusion. B. The anatomy of the cemento-enamel enucleated because junction. 2. dentin hypersensitivity. the epithelium of the cyst can degenerate and form neoplastic cells. admit the patient to hospital for extraction C. This is management is to most likely caused by A. C. (1) (2) (3) B. the connective tissue of the cyst wall can D. with general anesthesia. B. A diet high in refined carbohydrates. perform a pulpectomy. directions C. place amalgam restorations over the next few months. requires the replacement of a large MOD C. The tooth is hypersensitive to heat. 2. B. D. D. restore with amalgam. swab the exposure with chlorhexidine. (1) (2) (3) A 16 year old patient has multiple extensive B. a cracked tooth. the mandible may be observed to exhibit a shift from centric relation to centric A. In addition. All of the above. excavate caries and place temporary If a patient in her first trimester of pregnancy restorations within the next few weeks. restore all teeth with composite resin over appropriate treatment is to the next few months. delay any treatment until the hygiene amalgam restoration with extensive recurrent improves. the most D. hydrodynamic theory. ©The National Dental Examining Board of Canada 2017 . C. lateral. B. forward. B. caries and thermal sensitivity. The C. 4. Note: Some of the items in the Released Test Item Bank may have been discontinued due to outdated science or errors. C. and translation. upward. most appropriate management is to D. A. occlusion in all but one of the following B. perform a pulpotomy. A. (4) only preventive regime and to E. dentinogenesis process.Sensitivity related to a noncarious cervical When performing a functional analysis of lesion is most likely explained by the occlusion. (2) and (4) management is to place the patient on a D. The most likely diagnosis is give the most favourable prognosis for successful direct pulp capping? A. D. the format of some items is not currently used. thermal conductivity of dentin. A. neurogate mechanism. A. inorganic component of dentin. The most appropriate C. traumatic occlusion. Pain is relieved by warmth and direct Which of the following clinical findings will pressure. restore with reinforced zinc oxide During cavity preparation with a rubber dam. The exposure is small. place a pulp capping material. small mechanical pulp exposure occurs. A patient complains of tooth pain which is sharp and stabbing when chewing sweet or cold foods. D. delay treatment until after the baby is born. B. 1. B. backward. E. a eugenol. a carious lesion with pulpal inflammation. 3. No hemorrhage at the exposure site. C. (1) and (3) carious lesions. a carious lesion with pulp degeneration. A. restore with a composite resin. content. The exposure site is uncontaminated. The NDEB periodically reviews the bank to improve its quality. application of a resin based pit and fissure D. Requires endodontics or extraction. application of a glass ionomer pit and fissure sealants. inflammatory. Intravascular injection. incision and drainage. Expansion. What is the most likely diagnosis A. content. the format of some items is not currently used. D. and translation. mobility of the tooth. old patient shows a 4mm diameter. painless. E. apicoectomy. Allergic reaction. A. preventive resin restoration. pain on percussion.A 45 year old patient has 32 unrestored teeth. Biopsy is not necessary. fluoride varnish application. The most common clinical finding in the diagnosis of an acute apical abscess is The most significant factor contributing to the A. C. D. lymph node enlargement. type of post utilized. sealants. Syncope. C. neoplastic. Note: Some of the items in the Released Test Item Bank may have been discontinued due to outdated science or errors. B. D. Clinical examination reveals no evidence of caries in the grooves. observation. A. endodontic treatment. B. composite resin. C. radiolucent. endodontically treated tooth is the C. A periapical granuloma is all of the following The only defects are deeply stained grooves in EXCEPT the posterior teeth. amalgam. A. D. A. 23 year old patient experiences a warm sensation. light headedness and then loses consciousness Which of the following is characteristic of approximately 30 seconds following the periapical osseous dysplasia (periapical injection of 1. Pain. conservative Class I amalgams. D. A. nausea. C. The NDEB periodically reviews the bank to improve its quality. remaining coronal tooth structure. to open the tooth for drainage. In addition. to the enamel of the central pit is a/an C. The most appropriate management is B.000 epinephrine for an inferior alveolar nerve block. prophylactic odontotomy. B. C. C. no treatment. long-term success of the restoration of an B.1. D. The tooth has a normal response to vitality tests. ©The National Dental Examining Board of Canada 2017 . defined radiolucency at the apex of tooth 4. presence of a cellulitis. for his situation? B. A healthy. discoloration of the crown. Radiographic examination of a healthy 38 year D. well- E. presence of extracoronal coverage. B. type of core material used. E. permanent molar with occlusal caries confined B. diaphoresis. Overdose of local anesthetic. The most appropriate management is The most appropriate treatment for a A.8ml of 2% lidocaine with cemento-osseous dysplasia)? 1:100. C. An analgesic. 3. D. All of the above. discolouration of deciduous teeth. rhinophyma. 4. D. B. Note: Some of the items in the Released Test Item Bank may have been discontinued due to outdated science or errors. ©The National Dental Examining Board of Canada 2017 . Shortness of breath. content. (1) and (3) B. the format of some items is not currently used. A very apprehensive patient experiencing pain Tetracyclines have all of the following may be prescribed a barbiturate. A. diabetic. (1) (2) (3) A. antibiotic. The NDEB periodically reviews the bank to improve its quality. chloral properties EXCEPT hydrate or an antihistamine to control the anxiety. In which of the following would you A. (2) and (4) C. treatment? C. D. C. A topical antibiotic. You would suspect rapidly develops a facial rash. they form stable complex with the B. discolouration of permanent teeth. Edema of the lips. patient with chronic renal disease. B. antidepressant. A systemic antibiotic. Flurbiprofen is an A. In the treatment of necrotizing ulcerative gingivitis with associated lymphadenopathy. these drugs? A/An B. which of the following medications is the most Tetracycline therapy instituted either in the appropriate? second trimester or post partum in the infant is responsible for A. B. absorption is impaired when taken with expect an exaggerated response to the use of milk. Hyperventilation. emphysema.Shortly after the administration of an inferior A patient has a history of shortness of breath alveolar nerve block. (4) only D. discolouration of the deciduous or permanent teeth. developing tooth matrix. the enamel. minor changes in the hydroxyapatite of C. D. In addition. they predispose patients to candidial infection. D. An anti-inflammatory. Pallor and perspiration. cardiac insufficiency. and translation. C. 1. C. A. anti-inflammatory. they may be substituted for amoxicillin in patients that require coverage to prevent subacute bacterial endocarditis. E. 2. a healthy adult patient and ankle edema. Which of the following signs and symptoms should be A. arthritic patient. watched for before initiating the planned dental B. asthma. muscular relaxant. C. D. patient may include C. A. clindamycin. D. B. the antibiotic for a patient with mitral valve most appropriate antibiotic to treat an infection prolapse with regurgitation undergoing a of endodontic origin is surgical dental procedure? A. Inhaled nitrous oxide. Codeine. Acetylsalicylic acid. A. C. Nystatin is the most appropriate drug to treat B. Postoperative suture removal. necrotizing ulcerative gingivitis. Inferior alveolar nerve block. D. C. ©The National Dental Examining Board of Canada 2017 . Which of the following require prophylactic antibiotics prior to dental procedures causing a bacteremia? Appropriate management for the relief of symptoms of primary herpetic A. C. Subcutaneous epinephrine. A. B. C. Which of the following medications is CONTRAINDICATED in the management of a patient who is taking warfarin? A. Intramuscular penicillin. and translation. dexamethasone elixir. prosthetic heart valve? B. Making an alginate impression. content. D.Which is the most appropriate prophylactic In a patient who is allergic to penicillin. arrhythmia. amoxicillin. D. B. None of the above. Clindamycin. B. B. D. angina. aphthous stomatitis. Amoxicillin. acyclovir capsules. Oral ibuprofen. C. azithromycin. C. candidiasis. hypertension. The NDEB periodically reviews the bank to improve its quality. In addition. D. triamcinolone acetonide in Orabase®. Erythromycin. C. Cardiac stents one year after placement. D. Coronary artery bypass grafts. Oral nitrate is used to treat the symptoms of A. periodontal abscess. tetracycline. Implanted cardiac pacemakers. Note: Some of the items in the Released Test Item Bank may have been discontinued due to outdated science or errors. E. tachycardia. gingivostomatitis in an immunocompromised B. Clindamycin. Vancomycin. Which of the following drugs will have the most rapid onset of action? Which of the following procedures requires antibiotic prophylaxis for a patient with a A. Prosthetic cardiac valves. A. penicillin V. Acetaminophen. the format of some items is not currently used. D. B. Restoration of occlusal caries. steroid therapy. symptomatic irreversible pulpitis. (1) and (3) gingivitis includes C. 3. (1) (2) (3) Initial treatment of necrotizing ulcerative B. E. (4) only 1.Generalized malaise and elevated body The most appropriate treatment of necrotizing temperature may be associated with ulcerative gingivitis in a patient with lymphadenopathy is A. D. enhance wound healing. E. oral hygiene instruction. application of dilute hydrogen peroxide. 2. Treatment of primary herpetic gingivostomatitis should include Retention of an inlay is improved by 1. B. (4) only D. (1) and (3) C. periodontal debridement. B. Note: Some of the items in the Released Test Item Bank may have been discontinued due to outdated science or errors. placing a gingival bevel. (1) and (3) B. oral hygiene instruction. (4) only B. 2. 4. protect the wound from injury. antibiotic therapy. and translation. relieve the occlusion. (1) (2) (3) A. (2) and (4) C. (4) only E. increasing the parallelism of walls. gingivoplasty. A. D. C. content. The immediate treatment of a periodontal A. All of the above. (2) and (4) D. acute apical abscess. 1. A. establish drainage. (2) and (4) A. (1) (2) (3) B. 2. debridement. (1) (2) (3) abscess is to B. 3. D. C. prescribe an antibiotic. C. All of the above. prescribe an analgesic. The purpose of a periodontal dressing is to A. occlusal adjustment. addition of an occlusal dovetail. control of secondary infection. The NDEB periodically reviews the bank to improve its quality. D. (2) and (4) D. E. 4. palliative treatment. the format of some items is not currently used. (1) and (3) C. 1. asymptomatic apical periodontitis. lengthening the axial walls. 4. ©The National Dental Examining Board of Canada 2017 . 3. All of the above. All of the above. 2. All of the above. asymptomatic irreversible pulpitis. occlusal adjustment. 4. increase patient comfort. 3. In addition. C. B. length of extraoral time. C. When removal of carious dentin results in an exposure of nonvital pulp. post. C. postpone treatment and recheck status periodically. The mechanical objectives when preparing the root canal system for obturation with gutta- percha should include The prognosis for an avulsed tooth is mainly affected by A. test the pulp vitality and perform root cement. and translation. canal therapy on teeth with no response. the labial surface of the root is traumatized 3 years ago in an accident. the most appropriate management is to The anterior palatine foramen is most likely to be radiographically misdiagnosed as a A. trephination. nasolabial cyst. A. A. D. D. cotton moistened with eugenol. Elective endodontic treatment may be safely and successfully undertaken for all of the The most appropriate emergency management following EXCEPT of a mature permanent tooth with acute irreversible pulpitis is A. D. E. B. removal of irregularities. D. The NDEB periodically reviews the bank to improve its quality. and curette the periapical area. development of a continuously tapering cone in the root canal. C. necrotic cementum removal. A. hemophiliacs. storage solution. C. cement the post. occlude the cavity with a light packing of B. C. B. extract the teeth and place a bonded totally within the canal and cement the bridge. perform endodontic treatment. cyst of the incisive papilla. perforated. D. rarefying osteitis.During post preparation on a maxillary central A patient's 4 mandibular incisors were incisor. In addition. B. cement the post using a resin-based A. Note: Some of the items in the Released Test Item Bank may have been discontinued due to outdated science or errors. then raise a flap and seal B. incision and drainage. ©The National Dental Examining Board of Canada 2017 . re-prepare the canal so the post is now C. patients with a history of rheumatic fever. The most appropriate management Radiographs now show apical radiolucencies is to associated with all 4 teeth. The most appropriate management is to A. D. pregnant patients during first trimester. the format of some items is not currently used. pulp extirpation. extract the tooth. pulpectomy. pregnant patients during second trimester. B. content. C. B. apical surgery. maintenance of an intact foramen. D. place a temporary restoration and observe. perform root canal therapy on all 4 teeth the defect surgically. pulpotomy. all of the above. cap the exposed pulp horn with calcium hydroxide. A. Codeine. D. the format of some items is not currently used. recurrent aphthous ulcers. denture stomatitis. In addition. resect the apical section of the root management is to containing the separated instrument. disinfect gutta-percha points? D. splint the teeth.5 to 1mm short of the radiographic C. C. D. Acetylsalicylic acid. C. perform an apicoectomy and place a loosening of 3 maxillary incisors. A. B. apex. The most appropriate C. the most appropriate termination point of apical root canal preparation is A. B. Ketorolac. and translation. Which of the following should NOT be prescribed for a patient receiving warfarin? During endodontic treatment a file separates. splint the teeth and treat endodontically immediately. Penicillin. at recall examination. Dry heat sterilization. Which of the following is the most appropriate for pain management following an emergency pulpectomy for an adult with a history of severe asthma and nasal polyps? Which one of the following is the most appropriate initial treatment for internal A. A patient has suffered a blow resulting in the B. treat Which of the following may be used to endodontically. Pulp capping. B. B. Chemical solutions. C. D. Acetaminophen. Autoclave. A. changes at the apex are evident. D. 0. primary herpetic gingivostomatitis. Flame sterilization. Metronidazole. Note: Some of the items in the Released Test Item Bank may have been discontinued due to outdated science or errors. Vitality tests retrograde filling. C. give negative readings. Acetaminophen. perform pulpectomies on the teeth. content. B. A. fill the root canals and replant. check vitality in one month and if negative. extract the tooth. the most appropriate management is to A. D. 3mm short of the radiographic apex. D. The NDEB periodically reviews the bank to improve its quality. C. slightly through the apical foramen. No radiographic C. Pulpotomy. Pulpectomy. complete the root canal filling and monitor A. remove the teeth. informing the patient. Naproxen. necrotizing ulcerative gingivitis (NUG). The fragment is 3mm long and is lodged tightly B. ©The National Dental Examining Board of Canada 2017 . in the apical third of the canal. Apicoectomy. resorption? B. In addition to D.When performing endodontic treatment on a Metronidazole can be used to treat vital tooth. to the point where the patient feels sensation. ©The National Dental Examining Board of Canada 2017 . laboratory fabricated final restoration. incisors. The most appropriate retraction of the incisors. (1) (2) (3) B. reduce the occlusal out of occlusion and restore with a MOD amalgam. bond to dentin. D. cusp cap the buccal and lingual cusps and Metallic salts are included in root canal sealers restore with a MOD amalgam. restore with a bonded MOD composite resin. The NDEB periodically reviews the bank to improve its quality.Radiographs of the mandibular incisor teeth of Maxillary incisor protrusion can be treated by a 45 year old healthy patient reveal periapical radiolucencies.6 but cannot afford a with intracanal calcium hydroxide. the format of some items is not currently used. (2) and (4) D. Interim C. In addition. C. restore with a MOD amalgam. crown-root ratio of the abutments. reduction and genioplasty. set more rapidly. D. (1) and (3) C. A. position of the abutments in the arch.6 with the best followed by apical curettage. prognosis is to D. restoration has just had endodontic therapy B. 3. with a history of previous trauma and internal resorption is A. 4. observation over 6 months for further A patient with a pre-existing MOD amalgam resorption. B. 2. stronger. alveoloplasty B. C. A. perform a biopsy of the radiolucent segment. amount of periodontal support of the abutments. The choice and number of abutments for a fixed partial denture is influenced by the 1. The teeth are vital and 1. A. All of the above. content. (4) only E. complete instrumentation and medication completed on tooth 4. 3. place a drain in the affected area. premolar extraction with orthodontic asymptomatic. 4. C. All of the above. perform endodontic therapy on the four and prosthodontic replacement. extraction of the incisors. to make the sealers D. extraction and replacement with a fixed or removable prosthesis. (1) (2) (3) B. management is to 2. and translation. observe periodically. premolar extraction with surgical repositioning of the anterior dentoalveolar A. A. immediate instrumentation and obturation restorative management of 4. (4) only The most appropriate management for a tooth E. (1) and (3) C. B. Note: Some of the items in the Released Test Item Bank may have been discontinued due to outdated science or errors. (2) and (4) D. radiopaque. length of the span. lesion. observed with a therapeutic dose of E. apexogenesis and conventional nonsurgical root canal treatment. it will minimally affect the crown to root C. C. ©The National Dental Examining Board of Canada 2017 . B. an Which of the following requires antibiotic apicoectomy of approximately 3mm is prophylaxis for a patient with a prosthetic heart recommended because valve? A. a greater resection may expose a post in the canal. Anti-inflammatory action. The NDEB periodically reviews the bank to improve its quality. eliminated. Gabapentin. postextraction bleeding. C. A. Procaine. Morphine. Valproic acid. A patient who is jaundiced because of liver D. Dermatitis. Which of the following is used in the emergency management of a patient suffering status epilepticus? Treatment options for an immature tooth with a A. Nausea. Gastrointestinal irritation. content. Increased bleeding. acetaminophen? A. Note: Some of the items in the Released Test Item Bank may have been discontinued due to outdated science or errors. Prilocaine.During surgical root canal treatment. pulmonary embolism. and translation. B. Bupivacaine. C. Which local anesthetic is subject to inactivation C. it will allow better access for retrofill placement. In addition. B. D. Constipation. Antipyretic action. the format of some items is not currently used. Mepivacaine. Oliguria. D. B. necrotic pulp include B. disease has an increased risk of A. anaphylactic shock. D. Restoration of occlusal caries. C. apexogenesis and revascularization. C. Inferior alveolar nerve block. D. A. ratio. cardiac arrest. B. apexification and conventional nonsurgical root canal treatment. Which of the following is a sign of an allergic D. C. Hyperventilation. Lidocaine. apexification and revascularization. Endodontic instrumentation beyond apex. by plasma esterases? D. A. Which one of the following effects is typically D. Diazepam. B. B. reaction to penicillin? E. Making an alginate impression. most accessory and lateral canals will be A. pulpal abscess. early apical abscess formation. Acute trauma. Sclerosing osteitis/condensing osteitis in the periapical region is indicative of a/an Pain upon vertical percussion on the incisal A. the format of some items is not currently used. C. The NDEB periodically reviews the bank to improve its quality. C. E. The surrounding soft tissues are within normal limits. C. and translation. An infected pulp may cause a/an Severe throbbing tooth pain which increases A. In addition. failure to isolate and dry the tooth. B. periapical radiograph reveals a 5mm radiolucency associated with the distal root apex. B. the presence of multiple canals in various B. possible presence of C. symptomatic apical periodontitis (acute 10 years ago. Aging. D. chronic inflammation of the pulp.6 had endodontic treatment completed D. Tooth 3. simple bone cyst/traumatic bone cyst. chronic apical abscess (chronic the tooth. Note: Some of the items in the Released Test Item Bank may have been discontinued due to outdated science or errors. Symptomatic irreversible pulpitis.6 is a/an results can be explained by all of the following EXCEPT A. B.Which of the following is LEAST likely to Which of the following can be characterized by cause pain? a narrowing of pulp chambers and root canals? A. A. tooth mobility. It is asymptomatic but a periradicular periodontitis). D. Taurodontism. periapical osseous dysplasia (periapical periradicular periodontitis). Chronic apical abscess (chronic C. A. acute inflammation of the pulp. periradicular periodontitis). content. dentigerous cyst. abscess). acute apical abscess (acute periradicular stages of pulp pathosis. A. symptomatic irreversible pulpitis. abscess). symptomatic apical periodontitis (acute C. periradicular abscess). pulp polyp. asymptomatic apical periodontitis (chronic C. C. edge of an anterior tooth may indicate the B. D. The most likely diagnosis for Erratic and inconsistent electric pulp test tooth 3. B. asymptomatic apical periodontitis (chronic periradicular periodontitis). D. chronic apical abscess (chronic periradicular abscess). B. poor contact between the electrode and D. symptomatic apical periodontitis (acute periradicular periodontitis). cemento-osseous dysplasia). acute apical abscess (acute periradicular D. Reversible pulpitis. periradicular abscess). B. periradicular cyst. D. Symptomatic apical periodontitis (acute A. periradicular periodontitis). reversible pulpitis. Amelogenesis imperfecta. keratocystic odontogenic tumour when the patient lies down is a symptom of (odontogenic keratocyst). ©The National Dental Examining Board of Canada 2017 . from idiopathic osteosclerosis in that sclerosing D. erodes rapidly through bone. the cortical plate is more than 4mm thick. occurs only in young adults. pulp chamber and the root canal space are obliterated and the periodontal ligament space appears normal. perform endodontic treatment and fabricate a porcelain veneer. is asymptomatic. A periradicular granuloma Which of the following statements is true A. Cavernous sinus thrombosis. sealed. C. Note: Some of the items in the Released Test Item Bank may have been discontinued due to outdated science or errors. What is the most likely EXCEPT diagnosis? A. Fracture of these teeth are usually due to loss of coronal tooth structure. Spontaneous drainage of pus. regarding endodontically treated teeth? B. occurs at the apex of a vital tooth. painful. B. when a lateral root perforation needs to be C. A. when the apical foramen cannot be sealed A. ©The National Dental Examining Board of Canada 2017 . What is the most serious complication of an acute apical abscess (acute periradicular Apicoectomy is CONTRAINDICATED when abscess) in the maxilla? A. expansile. B. A patient complains of the discolouration of an C. upper central incisor. radiopaque. Radiographically. bony support. D. Pulp necrosis. In addition. Asymptomatic irreversible pulpitis. there is a granuloma at the apex of the C. associated with the apex of the tooth. The most appropriate management would be to A. Pain persists following surgery is indicated in all of the following thermal testing by cold. These teeth require full coverage to Sclerosing osteitis (dense bone island) differs prevent fracture. with vital pulps due to desiccation. D. fabricate a metal-ceramic crown. Symptomatic irreversible pulpitis. A post provides strength for these teeth. the patient is diabetic. content. C. periodontal disease causes inadequate A. tooth. osteitis is A. C. fabricate a porcelain veneer. B. Reversible pulpitis. Pericementitis. and translation. when orthograde endodontics is impractical. by orthograde endodontics. B. Periostitis. D. the D. The NDEB periodically reviews the bank to improve its quality. B. These teeth are more brittle than teeth D. C. D.A patient presents complaining of spontaneous A retrograde filling during endodontic apical pain from a tooth. B. the format of some items is not currently used. B. perform endodontic treatment and nonvital bleaching. C. A periapical radiograph can be used to D. B. locate the buccal bone level. Radiographs can differentiate between a granuloma and a cyst. reversible pulpitis. The electric pulp tester is the most suitable test Which of the following is consistent with a in cases of teeth with diagnosis of pulpal necrosis? A. C. C. frequently found in elderly patients. Negative response to electric pulp test. accompanied by severe pain. Poorly localized spontaneous pain. diagnosis is B. a vital tooth. B. obliterated canals. Analgesics. the format of some items is not currently used. pulp necrosis. C. D. gingival irritation. concussion injury. E. full cuspal coverage. D. C. A. Ice water. Recent trauma. Note: Some of the items in the Released Test Item Bank may have been discontinued due to outdated science or errors. Air jet. cracked tooth syndrome. B. Extreme pain elicited by palpation and percussion tests. In addition. galvanic shock. B. a chronic apical abscess (chronic loss of pulp vitality. Ice stick. The most likely A. a proliferative reaction of the pulp. D. D. Positive response to thermal tests.Which of the following agents is most effective Two weeks following the placement of a in cold testing? restoration. D. A. Which of the following may affect the results of electric pulp testing? Pulp polyp is A. Periapical radiolucencies are indicative of C. Ethyl chloride. a patient complains of pain to hot and cold in the restored tooth. D. periradicular abscess). an acute pathological lesion. A. The NDEB periodically reviews the bank to improve its quality. Emotional factors. C. C. A definitive diagnosis of an apical lesion B. A. open apices. Pain threshold. All of the above. an acute apical abscess (acute cannot be made using radiography alone. D. C. content. B. and translation. ©The National Dental Examining Board of Canada 2017 . Which statement is true? A positive pulp response to the application of cold indicates A. B. periradicular abscess). Asymptomatic irreversible pulpitis. reversibly inflamed. Examination reveals a fluctuant alone. the periapical area A child has a carious exposure of the pulp in is involved if there is pain the first primary molar. B. chronic apical abscess (chronic periradicular abscess). to electric pulp testing. Which clinical sign might be associated with an acute apical abscess (acute periradicular Which of the following clinical signs/symptoms abscess)? is characteristic of symptomatic irreversible pulpitis? A. B. The cavity is filled with pink tissue which bleeds when punctured by the A. explorer. C. content. D. Acute apical abscess (acute periradicular A. B. most likely diagnosis is C. necrotic. Pulp diagnosis cannot be made on a B. B. normal. swelling over tooth 2. In addition. D. Which of the following statements regarding periapical radiographs is true? A patient is in intense pain with a left A. B. Simple bone cyst/traumatic bone cyst. C. Note: Some of the items in the Released Test Item Bank may have been discontinued due to outdated science or errors. D. symptomatic apical periodontitis (acute periapical radiograph. D. which is beginning to can be made on a periapical radiograph affect the eye. The true dimension of the lesion can be and is tender to palpation and percussion. A. Presence of a sinus tract. A definitive diagnosis between granuloma versus cyst can be established from a A. when the patient is lying down. A positive reaction to the electric pulp tester. A. asymptomatic apical periodontitis periapical radiograph alone. In teeth with pulp necrosis.2. Reversible pulpitis. Swelling. C. D. What is the pulpal diagnosis? B. D. The predicted from a periapical radiograph. Pus formation. periradicular abscess). periradicular periodontitis). A definitive diagnosis of an apical lesion hemifacial swelling. C. (chronic periradicular periodontitis). acute apical abscess (acute periradicular abscess). B. abscess).Which of the following conditions is most A positive and prolonged reaction to a heat likely to be associated with a sinus tract? stimulus indicates that the pulp is A. C. the format of some items is not currently used. and translation. which has deep caries. Lingering thermal pain. D. Presence of a sinus tract. A positive reaction to percussion. Hypercementosis. The NDEB periodically reviews the bank to improve its quality. to thermal stimuli. on percussion. cold. Symptomatic irreversible pulpitis. ©The National Dental Examining Board of Canada 2017 . Chronic apical abscess (chronic C. A positive reaction of short duration to C. irreversibly inflamed. Normal pulp. D. Spontaneous pain. Internal root resorption. E. In addition. C. C. Overfilling refers to the incomplete filling abscess). pulpitis? D. a cracked tooth. (chronic periradicular periodontitis). C. symptomatic apical periodontitis (acute A. asymptomatic apical periodontitis B. Steam autoclave. ©The National Dental Examining Board of Canada 2017 . Alcohol autoclave. The pain the associated soft tissues appear normal. C. D. its completeness of obturation.5. B. B. but leaving voids in the pulp chamber for possible recontamination or infection. Asymptomatic apical periodontitis (chronic periradicular periodontitis). of the canal space. All of the above. B. D. Chronic apical abscess (chronic material extruding beyond the apical periradicular abscess). Acute apical abscess (acute periradicular A. Tenderness to pressure. foramen. There is a large cold liquids. Ethylene oxide method. C. periradicular periodontitis). and translation. E. Which of the following methods of instrument sterilization uses the lowest temperature? A. Vertical root fracture. is most likely caused by What is the most likely diagnosis? A. internal resorption. Electric pulp tests and restoration but the tooth is asymptomatic and radiographic appearance are normal. Acute apical abscess (acute periradicular D. D. What is the most likely diagnosis of a tooth presenting spontaneous pain of rapid onset and Which one of the following statements is true swelling of associated soft tissues? as defined in endodontics? A. C. the format of some items is not currently used. The NDEB periodically reviews the bank to improve its quality. Underextension refers to the vertical extent of the filling material regardless of A. Overextension refers to the extrusion of Which of the following signs or symptoms is filling material through an accessory characteristic of a symptomatic irreversible canal. Note: Some of the items in the Released Test Item Bank may have been discontinued due to outdated science or errors. Chronic apical abscess (chronic periradicular abscess). Underfilling refers to the complete filling D. content. Palpation sensitivity.A patient complains of pain in a mandibular A radiograph reveals a radiolucency associated molar when chewing hard foods and drinking with the apex of tooth 1. of the canal system with a surplus of B. Non-lingering thermal pain. Dry heat oven. Symptomatic apical periodontitis (acute periradicular periodontitis). B. Glass bead sterilizer. abscess). B. inadequate periodontal support. pulp stones. place tooth under the tongue and come to your office immediately. intact. Pulpotomy. B. E. and translation. The NDEB periodically reviews the bank to improve its quality. two pulp canals are most commonly found in the A. A. D. 1. B. D. distal root of mandibular first molars. spongy bone. used instead of a shoulder because a bevel D. masticatory efficiency. improves marginal adaptation. ©The National Dental Examining Board of Canada 2017 . keeps the teeth from having any movement during function. C. 3. constricted root canals. All of the above. E. Anachoresis.Endodontic therapy is CONTRAINDICATED The lamina dura is in teeth with A. B. insures proper cusp form and increases office at the end of the day. D. D. For a cast gold restoration. mesial root of mandibular first molars. Which one of the following is the initial 4. B. accessory canals. curved roots. (4) only D. treatment for internal resorption? A. B. Firm contact between approximating teeth is important because it A patient telephones and tells you he has just knocked out his front tooth but that it is still A. content. A. C. wrap the tooth in tissue and come to your D. cribriform plate. hypercalcified bone. C. In addition. protection to the gingival papillae. stabilizes the dental arches and gives office in a week's time. compact bone. distobuccal root of maxillary molars. Pulpectomy. C. place the tooth in milk and come to your the presence of microorganisms in the office immediately. D. increases the thickness of gold. protects the enamel. In permanent teeth. Apicoectomy. Which of the following is/are associated with E. All of the above. Your instructions should be to B. locates the marginal ridges of each tooth. Note: Some of the items in the Released Test Item Bank may have been discontinued due to outdated science or errors. increases retention. Cavernous sinus thrombosis. Bacteremia. (1) (2) (3) A. a gingival bevel is C. palatal root of maxillary first premolars. 2. put the tooth in alcohol and come to your office immediately. (1) and (3) B. Pulp capping. (2) and (4) C. put the tooth in water and come to your C. bloodstream? A. C. the format of some items is not currently used. the format of some items is not currently used. The temporalis muscle attaches to the pulpitis). Note: Some of the items in the Released Test Item Bank may have been discontinued due to outdated science or errors. A. ©The National Dental Examining Board of Canada 2017 . 70% ethyl alcohol.In restoring occlusal anatomy. Staphylococcus aureus. E. All of the above. A lingual approach for a conservative Class III Which of the following microorganisms are preparation for a composite resin requires most frequently found in infected root canals? A. The NDEB periodically reviews the bank to improve its quality. parallelism of the incisal and gingival B. In addition. D. content. None of the above. Severe throbbing tooth pain which increases 3. pulp hyperemia. chronic pulpitis (chronic ulcerative B. Lactobacilli. C. Streptococcus viridans. vertical and horizontal planes. 1. 2. C. C. D. B. histiocyte. E. The cell of the dental pulp most capable of transforming into other cells is the Which of the following statements are true concerning the adult mandible? A. cusp height. A. undifferentiated mesenchymal cell. D. mesial inclines of maxillary cusps and D. (1) and (4) pulpitis). Staphylococcus albus. C. and translation. C. a pulp polyp (chronic hyperplastic 4. B. mesial inclines of mandibular cusps and distal inclines of maxillary cusps. The genial tubercles are attachments for when the patient lies down is a symptom of the anterior bellies of the digastric muscles. (1) (3) (4) E. B. B. maintenance of the incisal contact area. D. a retentive internal form. B. the protrusive Sterilization of carious dentin without pulp condylar path inclination has its primary injury is assured by the application of influence on the morphology of A. walls. (1) and (3) D. absolute alcohol. chronic apical abscess. of the mandibular ramus both in the D. fibroblast. phenol. odontoblast. late stage of acute pulpitis (acute suppurative pulpitis). The mandibular foramen lies in the centre C. A. The angle formed by the junction of the ramus and the body of the mandible is an acute one. Enterococci. chlorhexidine. lateral surface of the coronoid process. (1) and (2) C. A. distal inclines of mandibular cusps. anterior teeth only. C. D. A. In addition. (4) only E. large buccal cusp. there is pain on percussion. C. secondary cementum. radiolucency. D. primary dentin. All of the above. 2. they are replaced by The most important principle dictating location A. secondary dentin. ©The National Dental Examining Board of Canada 2017 . All of the above. gemination. prominent transverse ridge. the tooth throbs when the patient is lying B. (1) (2) (3) B. B. 4. twinning. A. the occlusal In teeth with complete pulp necrosis. Buccinator. A. D. small lingual lobe. (2) and (4) external irritation or caries is the formation of D. The NDEB periodically reviews the bank to improve its quality. B. B. Lateral (external) pterygoid. ameloblasts. (1) (2) (3) C. (1) and (3) D. there is pain to thermal stimuli. removal of all pulp horns. portion through cemental union is known as 3. (1) and (3) A protective mechanism of the dental pulp to C. Note: Some of the items in the Released Test Item Bank may have been discontinued due to outdated science or errors. Superior constrictor. and size of access to the root canal system is B. 4. When odontoblasts are destroyed or undergo degeneration. large buccal pulp horn. the format of some items is not currently used. multinucleated giant cells. concrescense. pulp stones. preservation of tooth structure. Palatoglossus.In the mandibular first premolar. (4) only E. straight line access to the canal. The joining together of two teeth in the root 2. osteoblasts. the radiograph shows an apical D. undifferentiated mesenchymal cells. fusion. B. C. A. (2) and (4) D. C. A. 3. content. C. removal of all caries. Which of the following muscles comprise the retromolar pad? 1. the dovetail of an ideal disto-occlusal amalgam periapical area is involved if preparation is usually not extended into the mesial fossa because of the 1. and translation. A. down. 4. The greatest single factor in reducing radiation exposure in dentistry is A. Note: Some of the items in the Released Test Item Bank may have been discontinued due to outdated science or errors. fluorosis. proper filtration. unusual position of the contact area. restoration being in the esthetic zone. high speed film. Anterior superior alveolar. papillofibroma. endothelioma. C. B. and translation. All of the above. dysplasia. marginal ridge. This patient should have A. E. 3. The deciduous enamel is thinner and appears whiter. (1) (2) (3) B. (4) only C. B. Which of the following nerves should be B. Nasociliary. 1. intact basal cell layer and a maxillary first premolar because of the chronic inflammatory cells are histologic features that may be found in A. D. no additional therapy. the format of some items is not currently used. D. radiotherapy to site of biopsy. The NDEB periodically reviews the bank to improve its quality. The deciduous root trunk is shorter. lateral incisor? D. was diagnosed as a fibroma. vitamin D therapy. buccolingual width of the tooth's mesial D. re-excision with wider margins. higher kVp. B. content. carcinoma in situ. collimation of the X-ray beam. concavity in the cervical third of the mesial surface of the crown. following statement(s): C. E. hemisection of the tongue.Special attention is given to matrix adaptation Hyperkeratosis. ©The National Dental Examining Board of Canada 2017 . In addition. Sphenopalatine. acanthosis. permanent teeth isçare best described by the B. D. C. Secondary dentin formation may be stimulated by The difference(s) between deciduous and A. Nasopalatine. (2) and (4) B. squamous cell carcinoma. A. pulp necrosis. D. size of the interproximal gingival embrasure. attrition. C. The deciduous molar roots flare more. (A) and (C) diameter on the lateral border of the tongue E. anesthetized for extraction of a maxillary C. for the insertion of amalgam in a MO cavity in increased mitosis. 2. C. An excisional biopsy of a nodule 5mm in D. (1) and (3) A. All of the above. radium implantation around biopsy site. E. A. cardiac arrhythmia. E. C. E. Note: Some of the items in the Released Test Item Bank may have been discontinued due to outdated science or errors. rheumatic heart disease. content. impacted maxillary third molar. A. (B) and (C) E. asthma. B. In addition. high blood pressure. D. The term applied to a low white blood cell Untreated diabetes mellitus characteristically count is demonstrates A. most frequently affects molars. D. thrombocytopenia. B. hyperlipidemia. (A) and (C) D. C. E. dysuria. Serum phosphorus. E. When a patient experiences continuous pain in the maxillary premolar and molar areas and there is no evidence of dental infection. and translation. coronary artery disease. Serum calcium. D. C. trigeminal neuralgia. glossopharyngeal neuralgia. leukocytosis. the most likely diagnosis is A. microcytic hypochromic anemia. C. acute maxillary sinusitis. D. affects non vital teeth in the majority of B. cases. ©The National Dental Examining Board of Canada 2017 . macrocytic hyperchromic anemia. supernumerary incisors. Serum alkaline phosphatase. D. A. blue sclerae. Fordyce's granules. D. B. peg lateral incisors. is most often confined to the apical half of called a the root. fatty degeneration. Thyroid activity. B. hypoglycemia. B. embolism. the format of some items is not currently used. impacted maxillary canine. C. C. B. thrombocythemia. D. D. pigmented teeth. B.Hypercementosis (cemental hyperplasia) An anemia in which the red blood cells are smaller and less intense in color than normal is A. thrombosis. C. A. macrocytic hypochromic anemia. A. microcytic hyperchromic anemia. hypophagia. B. infarction. hyperglycemia. E. C. None of the above. A patient who uses nitroglycerine has Erythroblastosis fetalis may be a cause of A. C. A decrease of which of the following is Coronary artery occlusion can lead to indicative of hypoparathyroidism? A. leukopenia. The NDEB periodically reviews the bank to improve its quality. C. 4. type of timer. (1) (2) (3) B. and translation. 4. (1) (2) (3) D. In addition. B. A. line voltage fluctuation. (2) and (4) D. C. Hertwig's root sheath. The apical region of a non-vital tooth with a deep carious lesion may radiographically show Selection of the appropriate kilovoltage for 1. (2) and (4) frequently impacted? D. lipoma. diameter of the primary beam of membrane. the result of chronic inflammation. lymphadenopathy. a normal anatomical structure. radiographic contrast. A. 4. secondary radiation. loss of lamina dura. papilloma. All of the above. 5. squamous epithelium is called a/an 3. Mandibular second molar. palatine petechiae. leukopenia. A. The NDEB periodically reviews the bank to improve its quality. The benign neoplasm that originates from 2. a positive Paul Bunnel test. B. B. caused by premature degeneration of 2. C. All of the above. C. Maxillary first premolar. (4) only E. D. A. Mandibular cuspid. dental films is influenced by 2. 3. Maxillary cuspid. (4) only E. All of the above. adenoma. content. C. ©The National Dental Examining Board of Canada 2017 . choriocarcinoma. (2) (3) (4) (5) D. B. tissue density. intensity of central beam. (1) and (3) E. (1) (3) (4) (5) E. (1) and (3) Which one of the following teeth is most C. a defective cementoid substance. Proper collimation of the useful beam for the film size and target-film distance will reduce 1. the format of some items is not currently used. B. chondroma. 1. (2) and (5) E. calcification of the periodontal B. Note: Some of the items in the Released Test Item Bank may have been discontinued due to outdated science or errors. image definition. radiation received by patient. widening of the periodontal space. radiation. C. A. a circumscribed radiolucency. 3. A. D. filter thickness. (1) (2) (3) (5) D.Acellular cementum on a root is In infectious mononucleosis you are most likely to find A. (4) only B. C. C. The NDEB periodically reviews the bank to improve its quality. None of the above. and translation. carrying on development. (2) and (3) D. the posterior region of the maxilla.The fixing solution serves the purpose of In the presence of an acute bacterial infection. A. associated with a root penetrating cavity. (1) only C. hypoparathyroidism. pain. polymorphonuclear leukocytes. (1) and (3) E. A. D. occlusion. eosinophils. ©The National Dental Examining Board of Canada 2017 . C. caused by exposure to radiation. resulting in a submerged tooth. E. increased density of tissue. type of exudate. D. more radiation affecting the silver halide crystals. decreased density of tissue. hyperthyroidism. (1) and (3) B. Myasthenia gravis. laboratory tests will show an increase in 1. lymphocytes. near the junction of the D. All of the above. B. response of pulp to electrical stimulation. in the anterior region of the mandible near the midline. associated with a non-vital pulp. A patient presents with apparent paralysis of one side of the face which appeared the day before. C. an auto-immune disease. 2. the anterior region of the maxilla. Trigeminal neuralgia. Ankylosis is commonly An ameloblastoma is most frequently found in A. plasma cells. E. Bell's palsy. hardening the emulsion. increased exposure time. (2) and (3) A. A. B. tenderness to percussion. C. the result of a root fracture. 3. E. the mandible. removing unexposed silver salts. E. hypothyroidism. None of the above. (1) and (2) D. D. 2. C. D. 1. the format of some items is not currently used. radiographic examination. D. In addition. D. out of body and the ramus. 4. Glossodynia. a viral infection. a drug reaction. C. B. What is the most likely diagnosis? A radiolucent area in a radiograph occurs as a result of A. B. hyperparathyroidism. monocytes. found in permanent teeth. A. E. B. content. a genetic disease. Note: Some of the items in the Released Test Item Bank may have been discontinued due to outdated science or errors. C. Sickle cell anemia is A. In the early stage. B. 3. B. a periapical abscess can be differentiated from a lateral periodontal abscess Multiple giant cell lesions of the bone are by associated with A. Multiple supernumerary teeth are most Which gingival manifestation(s) would be commonly found in expected in a patient with a blood dyscrasia? A. the format of some items is not currently used. 3. B. B. 1. pulp calcifications. C. Down's syndrome. Muco-epidermoid carcinoma. supernumerary teeth. In addition. ©The National Dental Examining Board of Canada 2017 . A. (2) and (4) A draining fistula of short duration related to a D. soft palate. C. Atrophy. amelogenesis imperfecta. which may be associated C. dentinogenesis imperfecta. irrigation of canals. Hypercementosis. Excessive orthodontic forces. Ulceration. Pleomorphic adenoma. D. E. tooth mobility. A. hypothyroidism. (1) (2) (4) C. Traumatic injury. Note: Some of the items in the Released Test Item Bank may have been discontinued due to outdated science or errors. C. All of the above. no special treatment. anterior teeth. and translation. common tumour of the salivary glands? A. A. lower lip. B. All of the above. E. Adenocystic carcinoma. C. 5. B. Cementoma. cretinism. surgical excision. D. (1) (2) (3) (5) E. D. gingival recession. with root resorption? D. (1) (2) (5) E. osteogenesis imperfecta. dysfunction syndrome.A common clinical sign of occlusal traumatism "Dens in dente" (dens invaginatus) is is associated with A. Bleeding. B. 1. pocket formation. Periapical granuloma. B. A. 3. 2. (4) only tooth undergoing endodontic therapy requires E. content. Which one of the following is the most D. Enlargement. A. antibiotics. cherubism. upper lip. 2. E. Adenoma. Among the following. buccal mucosa. (1) (2) (3) B. temporomandibular joint pain . 4. (1) and (3) C. The NDEB periodically reviews the bank to improve its quality. (1) (2) (4) (5) D. B. C. Mucoceles are most commonly found in the 4. cleidocranial dysplasia. D. tongue. Radiographic changes of the joint. B. (1) and (3) D. Streptococcus pyogenes. All of the above. ©The National Dental Examining Board of Canada 2017 . apex. D. E. (2) and (4) C. angle technique as opposed to a bisecting B. and translation. the format of some items is not currently used. lymphatics drain superiorly in this region. infection has passed into the angular vein space. B. 3. vesicles. Muscle tenderness. 2. B. In addition. greater entrance dosage. C. D. E. 4. epithelial hyperplasia. ankylosis. content. C. Excessive formation of scar tissue beyond the wound margin is called In radiography. a fibro-epithelial polyp. Pain. bullae. A. Diplococcus pneumoniae. D. Mycobacterium tuberculosis. B. a fibroma. angle technique will result in C. nodules. The most logical explanation for causing The earliest radiographic sign of occlusal swelling beneath the eye caused by an trauma is abscessed maxillary canine is that the A. a more heterogenous beam of X-rays. Neisseria gonorrhoeae. The NDEB periodically reviews the bank to improve its quality. Note: Some of the items in the Released Test Item Bank may have been discontinued due to outdated science or errors. "Clicking" or "popping" noise in the joints. widening of the periodontal ligament C. which has no valves. (2) and (3) B. papules. E. D. B. the root apex lies superior to the attachment of the caninus and levator labii superioris muscles. less gonadal radiation. E. a keloid. hypercementosis. a parallel technique or right A. D. 1. (1) (2) (4) A. C. less dimensional distortion. A. macules. root resorption. Limitation of jaw motion.Which of the following is NOT a sign or The finding of “acid-fast” microorganisms in symptom of the myofascial pain dysfunction sputum suggests the presence of syndrome? A. alteration of the lamina dura. D. Petechiae are A. bone is less porous superior to the root C. D. dysplasia of cells confined within the A. shortness of breath. epithelium. Petechiae of the palate. B. A. Which one of the following would be of greatest value in determining the etiology of an oral ulceration? Which of the following is NEVER associated with an impacted tooth? A. maximum OFD (object -film distance) E. Pharyngitis. B. Salivary glands. metaplasia. Laboratory tests. Ludwig's angina. a blockage of Wharton's duct. (2) and (4) D. a blockage of Stensen's duct. C. 4. Fingernails. 3. D. Teeth. early invasion of malignant cells through mealtime is indicative of the basement membrane. Cytological smear. pitting edema of the ankles. (4) only E. The NDEB periodically reviews the bank to improve its quality. minimum FFD (focal-film distance). a ranula. A. (1) (2) (3) D. C. D. The term "carcinoma in situ" implies that the lesion shows Intermittent painful swelling in the A. B. B. C. D. Adeno-ameloblastoma. epistaxis. All of the above. maximum OFD (object-film distance) and minimum FFD (focal-film distance). D. content. an epidemic parotitis. Pindborg's tumor. distant metastasis of a malignant tumour. A. C. C. Fatigue. ©The National Dental Examining Board of Canada 2017 . B. In addition. C. B. C. maximum FFD (focal-film distance). and translation. Systemic evaluation. History of the oral lesion. minimum OFD (object-film distance) and A. Primordial cyst. B. the format of some items is not currently used. minimum magnification and Which of the following is NOT associated with maximum definition are achieved by infectious mononucleosis? A. (1) and (3) E. E. Ameloblastoma. minimum OFD (object-film distance) and C. E. A patient with congestive heart failure may have Which of the following is/are NOT usually 1.In radiography. Sweat glands. exophthalmos. and maximum FFD (focal-film distance). affected by hereditary ectodermal dysplasia? 2. Note: Some of the items in the Released Test Item Bank may have been discontinued due to outdated science or errors. submandibular region that increases at B. Odontogenic myxoma. Lymphadenopathy. D. Hair. Gingival enlargement. submandibular space. A. Note: Some of the items in the Released Test Item Bank may have been discontinued due to outdated science or errors. B. submental space. Recurrent pericoronitis.000. Poor flap design. (1) and (3) C. patient’s religious beliefs. All of the above. A periapical infection of a mandibular third 4. (1) and (3) 3. (4) only 1. (1) (2) (3) A pontic replacing a mandibular first molar B. Incipient syncope. (2) and (4) D. (2) and (4) D. conceals the porcelain to metal junction on its gingival surface. A. after an injection of 4ml of lidocaine 2% with epinephrine l:l00. Pain. patient’s infectious disease status. What is the most probable Which of the following is NOT an indication diagnosis? for the removal of impacted mandibular third molars? A. 3. 2. A. (1) and (3) should be designed so that it(s) C. parapharyngeal space. D. (4) only A patient suddenly becomes pale and sweaty E. Resorption of the distal aspect of the E. and translation. 2. B. patient’s physical handicap. The NDEB periodically reviews the bank to improve its quality. ©The National Dental Examining Board of Canada 2017 . (1) (2) (3) B. The blood pressure is 80/60. The respiration is slow. D. B. has open gingival embrasures. it is ethical for a dentist to refuse to Which of the following will impede healing treat a patient on the basis of the following the surgical closure of an oro-antral fistula? A. (2) and (4) 4. 1. 3. second molar.In Canada. An impending adrenal insufficiency. 1. The radial pulse is slow and steady. C. pterygomandibular space. closely to the ridge. content. All of the above. An allergic reaction to the local B. In addition. complexity of the required treatment. D. Sinus infection. A toxic reaction to lidocaine. Excessive tissue tension. C. All of the above. Crowding of incisors. C. the format of some items is not currently used. A. gingival surface is convex in all molar may spread by direct extension to the directions. Blowing the nose. (4) only E. (1) (2) (3) 2. gingival surface is concave and adapts E. D. A toxic reaction to epinephrine. anesthetic. 4. C. B. C. A. C. Central giant cell granuloma. thyroid membrane. heart failure. D. the format of some items is not currently used. Cylindroma. D. The NDEB periodically reviews the bank to improve its quality. B. paralysis of muscles of respiration. exophthalmos. asphyxia. prolonged bleeding. body of the mandible. D. maxillary tuberosity. made at the B. postoperative infection. platelet count. a dry socket. white blood cell count. D. In addition. C. B. Squamous cell carcinoma. All of the above. C. alkalinity of the infected tissue inhibits C. the nerves. It is difficult to obtain satisfactory anesthesia in the presence of infection near the injection site Bacterial infection may be confirmed by because 1. E. first tracheal ring. D. B. acidity of the infected tissue inhibits A. B. B. allergic reaction to the anesthetic solution. fracture of the malar ridge. nasal bleeding and swelling. thyroid notch. Which of the following is best removed by curettage? In an acute upper airway obstruction. a possible the extraction of an isolated maxillary second complication is molar is A. hemoglobin level. neck of the condyle. Ameloblastoma. E. D. Ludwig's angina may cause death by A patient presenting with diplopia. syncope or shock. A. Pleomorphic adenoma. increased blood supply carries the 4. erythrocyte sedimentation rate. (1) and (3) D. C. anesthetic solution away too fast. (4) only E. A. fracture of the tuberosity. cricothyroid membrane.The most likely complication associated with In a patient with liver disease. (2) and (4) action of the anesthetic agent. cricoid cartilage. Note: Some of the items in the Released Test Item Bank may have been discontinued due to outdated science or errors. 3. A. C. and translation. the entry to the airway on an emergency basis should be A. E. content. may suffer from a fracture of the B. pyemia. convulsions. ©The National Dental Examining Board of Canada 2017 . A. zygomatic bone. the swelling causes increased pressure on 2. (1) (2) (3) action of the anesthetic agent. C. nerve damage. D. A. E. A. The NDEB periodically reviews the bank to improve its quality. are safe to use during pregnancy. Sodium. Note: Some of the items in the Released Test Item Bank may have been discontinued due to outdated science or errors. altering the formation of histamine. A. increasing the action of histaminase. (4) only E. All of the above. D. have a wide spectrum of antibacterial activity. (2) and (4) D. decrease the immune response. A. B. (1) and (3) C. Mitral. low percentage of blood carbon dioxide. C. D. 4. embolus. A. C. Potassium. Fluoride. may increase susceptibility to superinfections. None of the above. Which valve is most commonly affected by B. high percentage of blood carbon dioxide. 2. blocking the actions of histamine by competitive inhibition. cause retention of sodium and fluid. B. (1) (2) (3) B. D. Meperidine. Chloral hydrate. D. Which of the following does NOT relieve pain? A therapeutic advantage of penicillin V over A. C. hematoma. B. low blood pressure. 3. A. and translation. 1. histamine. C. Bicarbonate. Tetracyclines D. penicillin G is B. Pulmonary. syncope. D. (1) (2) (3) C. the format of some items is not currently used. Hydromorphone. B. ©The National Dental Examining Board of Canada 2017 . Antihistamines act by A.Which of the following is directly involved in Adrenal corticosteroids the conversion of prothrombin to thrombin? 1. In addition. Aortic. 2. A. Methadone. E. C. have no side effects. slower renal excretion. greater absorption when given orally. interfering with the degradation of B. high percentage of blood oxygen. greater resistance to penicillinase. heighten the immune response. C. E. (4) only The major stimulator of respiration is E. Tricuspid. Calcium. The most common complication of a 3. rheumatic heart disease? C. B. venipuncture is 4. (1) and (3) D. thrombophlebitis. content. broader antibacterial spectrum. All of the above. Codeine. (2) and (4) D. cause diabetes. pancreas. Diazepam. an anxious dental patient with a peptic ulcer? A. teratogenicity. C. liver. Note: Some of the items in the Released Test Item Bank may have been discontinued due to outdated science or errors. tolerance. 1. C. Which of the following would you prescribe for 4. ©The National Dental Examining Board of Canada 2017 . retards platelet function. 2. D.Trismus is most frequently caused by Which of the following does NOT influence the rate of induction during inhalation A. idiosyncrasy. The appearance of a rash. The NDEB periodically reviews the bank to improve its quality. (1) (2) (3) A. Pulmonary ventilation. B. tetanus. itching. 1. Silica gel. B. In addition. spleen. extreme malaise. anesthesia? B. 2. E. has antipyretic properties. content. the format of some items is not currently used. cerebral hypoxia. and translation. 3. B. electrolyte imbalance. A. C. A. All of the above. infection. A. allergy. (2) and (4) D. broncho- constriction and fever after the administration Acquired Immune Deficiency Syndrome of a drug are the result of (AIDS) may be characterized by A. Concentration of the anesthetic in the inspired mixture. produces CNS stimulation. E. Scopolamine. Solubility of the anesthetic in blood. B. C. mandibular fracture. D. 4. muscular dystrophy. Calcium carbonate. (2) and (4) C. candidiasis. Acetaminophen in therapeutic doses E. D. Hemoglobin content of the blood. D. C. (1) and (3) C. cerebral hyperemia. Reserpine. Short-acting barbiturates are metabolized Unconsciousness in syncope results from mainly in the A. a smooth and red tongue. kidneys. B. D. (1) (2) (3) B. D. (4) only E. has strong anti-inflammatory properties. Blood supply to the lungs. fever or chills. (4) only D. small intestine. (1) and (3) B. neurogenic shock. E. C. All of the above. rapid weight loss and night sweats. 3. Superior constrictor. (4) only A.0 mg. Nasociliary. Buccinator. C. (1) (2) (3) In a standard inferior alveolar nerve block. 1. E. allow placement of teeth over the residual 3. stimulation of the central nervous system. increase the supporting surface area. cause C. Sphenopalatine. 0. (2) (3) (4) D. 3. B. (2) and (4) E. High plasma levels of local anesthetics may B. B. Nasopalatine. active metabolites. Masseter.8 mg. C.000.8ml 2% lidocaine with epinephrine 1/100. magnesium. Medial (internal) pterygoid. B. A. facilitate reliable impression making. after intramuscular administration. Mylohyoid. E. D. the amount of vasoconstrictor is Which of the following nerves should be anesthetized for extraction of a maxillary A. potassium. It does not produce antianxiety effects space. Its sedative effect can be reversed by naloxone. The inorganic ion that is implicated in primary hypertension is A. In addition. E. 18. fluoride. (1) and (3) which muscle is penetrated by the needle? C. and translation. All of the above. (3) and (4) Note: Some of the items in the Released Test Item Bank may have been discontinued due to outdated science or errors. Anterior superior alveolar.Which of the following statements is/are true Vestibuloplasty is a preprosthetic surgical regarding diazepam? procedure used to 1. 2. reliable than oral. (1) and (2) B. sodium. lateral incisor? B. the format of some items is not currently used. 4. D. B. D. (1) (3) (4) C. A.018 mg. Intravenous administration is more ridge. inhibition of peristalsis. A. D. depression of the central nervous system. 1. 0. C. The NDEB periodically reviews the bank to improve its quality. 180.18 mg.0 mg. inhibition of the vagus nerve to the heart. ©The National Dental Examining Board of Canada 2017 . content. Its long duration of action is partly due to A. C. (2) and (4) D. D. In a standard dental cartridge (carpule) containing 1. provide adequate posterior inter-arch 2. 4. (1) (2) (3) B. C. He becomes unresponsive to D. In addition. and his respirations are E. leaving the tuberosity in place and suture. (4) only verbal stimuli. The NDEB periodically reviews the bank to improve its quality. The tooth with the D. D. The chief mechanism by which the body D. 4. drains in the D. allergy. hydroxylation and oxidation. diphenhydramine. depressed to 10 per minute. All of the above. pterygomandibular space. You should A. inability to bring the molars into contact. content. C. and translation. buccal vestibule. hydroxyzine. mandibular third molar generally points and C. Increase in arterial oxygen. ©The National Dental Examining Board of Canada 2017 . if possible. sequestration in the body fats. During extraction of a maxillary third molar. fill the defect with Gelfoam and suture. force the patient to drink coffee. Note: Some of the items in the Released Test Item Bank may have been discontinued due to outdated science or errors. Decrease in arterial carbon dioxide. E. C. the format of some items is not currently used. Decrease in arterial oxygen. the first drug that should be administered is A. alkalosis. Appropriate treatment is to A. remove both. remove the tooth. 2. A. leave both and stabilize. epinephrine. D. 1. Death from barbiturates is the result of Bilateral dislocated fractures of the mandibular condyles result in A. Decrease in arterial pH. A. the tuberosity is fractured. irreversible hypotension. E. anterior open bite. B. B. submandibular space. D. Increase in arterial carbon dioxide. A. 3.Which of the following is the strongest In the treatment of an acute anaphylactic stimulus to respiration? reaction. buccal space. depression of the centres of respiration. remove both and suture. C. B. support respiration with oxygen. tuberosity remains attached to the surrounding soft tissue. inability to protrude the mandible. anesthesia of the mental nerves. toxic effects on the liver. D. B. observe the patient. (1) and (3) A 57 year old man received 10mg of diazepam C. hydrocortisone. C. oxidation. C. administer ephedrine. reflect the mucoperiosteum. reduction. B. metabolizes short-acting barbiturates is A. (2) and (4) intravenously. B. An acute periapical abscess originating from a B. A. 4. (1) (3) (5) D. 3. E. All of the above. (2) (4) (5) What is the maximum number of cartridges After an inferior alveolar nerve block injection. Naso palatine. pterygoid plexus of veins. Greater palatine. (1) (2) (4) B. (2) and (3) D. 6. 5. B. retromandibular vein. (1) (2) (3) B. (2) and (4) C. the format of some items is not currently used. The NDEB periodically reviews the bank to improve its quality. 2. D. (1) and (2) C. 2. In addition. 4. internal maxillary artery. C. 2. B. content. (4) only C. A. 3. 3. placed over firm bone where possible. hemostasis. retroparotid space. Note: Some of the items in the Released Test Item Bank may have been discontinued due to outdated science or errors. All of the above. atrial fibrillation. (4) only D. Anterior superior alveolar. (1) and (3) A. (1) and (3) B. Acetaminophen. without blanching. (3) and (4) E. Posterior superior alveolar. (1. ©The National Dental Examining Board of Canada 2017 . Middle superior alveolar.Which of the following lower(s) the body Hydrochlorothiazide (Hydrodiuril) is used to temperature increased by bacterial pyrogens? treat 1. E. (1) (3) (4) C. C. 4. (2) (3) (5) E.8ml) of a 2 local anesthetic solution that a patient would develop seventh nerve paralysis can be administered without exceeding a total if the injection was made into the dose of 300mg? A. 4. angina pectoris. 15mm apart. they should be anesthetized for the removal of a maxillary first molar? 1. D. Codeine. hypertension. A. and translation. Acetylsalicylic acid. 1. (1) (2) (3) A. congestive heart failure. 10. 8. ventricular fibrillation. interrupted. E. (1) and (5) When sutures are used to reposition tissue over Which of the following nerves should be extraction sites. tight enough to produce immediate 4. D. Bradykinin. 1. 3. firm enough to approximate tissue flaps 2. 2. (2) and (4) B. internal pterygoid muscle. 5. C. a supernumerary tooth. The eruption of a permanent central incisor may be delayed by The radiographic appearance of internal A. B. 1. The NDEB periodically reviews the bank to improve its quality. D. 3. (4) only E. radiopacity around the apex of the root. and translation. (1) (2) (3) C. as viewed A radiographic examination of a 10 year old on successive cephalograms.During normal growth. All of the above. A 3-year old with a 4mm overjet. An 8-year old with a previous thumb habit. A. radiolucency around the apex of the root. growing skull is the C. a retained deciduous incisor. in the furcation area. Bolton point. A. 2. the gnathion. alteration of arch length. In addition. Broadbent's point. diastema between maxillary central incisors? D. A 14-year old with no abnormal oral An ankylosed deciduous molar can cause habits. radiolucent enlargement of the pulp D. teeth. backward and upward. dentinogenesis imperfecta. B. localized radiopacities in the pulp cavity. D. referred for orthodontic treatment to close a C. osteoblasts. downward and backward. forward only. ectodermal dysplasia. The cells responsible for root resorption are A. D. the format of some items is not currently used. radiographic bony changes from an infection are initially seen A. cementoblasts. sella turcica. A. dense fibrous tissue. This is characteristic of B. B. An 8-year old with no abnormal oral habits. In primary molars. D. osteoclasts. C. C. the most stable point in a B. Which of the following patients should be B. C. Note: Some of the items in the Released Test Item Bank may have been discontinued due to outdated science or errors. will move child reveals retention of deciduous teeth and presence of many unerupted supernumerary A. at the base of the developing tooth. All of the above. (1) and (3) D. content. A. radiolucency on the surfaces of the root. In cephalometry. All of the above. D. nasion. ©The National Dental Examining Board of Canada 2017 . E. congenital hypothyroidism. cleidocranial dysplasia. premolar. delayed eruption of the succeeding 4. at the apices. B. cavity. fibroblasts. resorption is B. at the alveolar crest. B. C. C. A. difficulty with extraction. downward and forward. (2) and (4) D. In addition. D. delaying the eruption timetable. mandibular second molar. Note: Some of the items in the Released Test Item Bank may have been discontinued due to outdated science or errors. maxillary canine. mandibular canine. and translation. have 2 arch orthodontic treatment. median palatine suture. A. have erupted. is sustained over a longer period of time in girls. pterygopalatine suture. in the anterior region. dentition. the last primary B. occurs two years earlier in boys than in A. Later than 12 years. A. All of the above. usually shortened skull. (1) and (3) The developing permanent tooth C. C. is tall and thin. All of the above. causing sclerotic bone to form over the occlusal surface of erupting teeth. occurs at the same chronologic age in who both sexes. matures early. D. content. persistence of deciduous teeth.Roots of the permanent maxillary central The principal growth sites of the maxilla in a incisors are completed by what age? downward and forward direction include the A. D. delayed suture closure. zygomaticomaxillary suture. C. C. A. E. B. An endomorph is characterized as a person C. (1) (2) (3) B. D. C. be treated with a removable appliance. B. 1. A. 4. be observed and treated when the cuspids D. D. (2) and (4) D. ©The National Dental Examining Board of Canada 2017 . (4) only. B. interfering with jaw growth. frontomaxillary suture. self-correct. 8 years. 12 years. maxillary first molar. E. A. 10 years. C. clavicles absent or maldeveloped. be treated in the complete permanent B. girls. The NDEB periodically reviews the bank to improve its quality. Cleidocranial dysostosis is distinguished by B. In a normal eruption pattern. has a more protrusive path of eruption in Mandibular growth the anterior region. All of the above. may show deviated eruption times if the primary tooth is lost prematurely. B. is muscular. lies apically and lingually to primary teeth E. D. 3. matures late. maxillary second molar. Hypothyroidism affects the dental developmental pattern by A. A single tooth anterior crossbite found in a 9 year old should A. E. B. E. 2. tooth to be lost is the C. the format of some items is not currently used. is short and fat. is sustained over a longer period of time in boys. C. accelerating the eruption timetable. D. appositional growth. 4. C. thirteen years. and D. on the posterior border of the ramus. and translation. (1) and (3) C.The roots of the first permanent molar should The primary stimulus for growth of the be completely formed by the age of mandible is A. All of the above. on the alveolar margins. ©The National Dental Examining Board of Canada 2017 . until the end of mixed dentition. may remain for years with no significant resorption. tetracycline therapy. The NDEB periodically reviews the bank to improve its quality. E. A. A single hypoplastic defect located on the 3. D. content. In addition. B. labial surface of a maxillary central incisor is 4. throughout life. 3. both appositional and interstitial growth. B. interstitial growth. Alveolar bone is undergoing remodeling A. dietary deficiency. (2) and (4) C. (1) and (3) B. on all surfaces of the mandible. As the mandible grows downward and forward. six years. D. postextraction bleeding. on the anterior border of the ramus. The roots of primary molars in the absence of C. B. 1. until the complete eruption of permanent their permanent successors teeth. E. 2. 2. eleven years. nine years. E. may remain for years partially resorbed. seven years. A patient who is jaundiced because of liver bone deposition takes place disease has an increased risk of A. B. B. (4) only D. 1. C. cardiac arrest. D. Note: Some of the items in the Released Test Item Bank may have been discontinued due to outdated science or errors. (1) (2) (3) B. the format of some items is not currently used. C. E. C. sometimes are partially resorbed and become ankylosed. A. D. high fluoride intake. postoperative infection. environmental. (1) (2) (3) A. epigenetic. (4) only E. (2) and (4) Cartilage grows by D. most likely due to a/an A. endocrine deficiency. incisor. anaphylactic shock. C. functional. B. B. genetic. D. and D. trauma to the maxillary primary central E. None of the above. A. D. A. All of the above. through the primary dentition. are always resorbed. and translation. Alveolar crest. area relocation. C. 3. state of dental maturation. translatory growth. D. growth period is called 3. In addition. 4. content. All of the above. heredity. 1. is normal if there is a flush terminal plane. ©The National Dental Examining Board of Canada 2017 . B. an infra-orbital block. None of the above. A. 2. may aid in the prediction of permanent tooth malocclusion.The term "dental age" refers to the The interocclusal relationship of the primary second molars A. Circular. determines the amount of leeway space. C. remodeling. All of the above. (1) (2) (3) B. (1) and (3) anesthetic technique of choice is C. thumbsucking. mouth breathing. tooth erupted. Which of the following is/are essential when using spherical rather than admix alloy for a routine amalgam restoration? The mechanism of adjustment to maintain the 1. 2. The local B. 1. shape and proportions of bone throughout its 2. ectopic eruption. The NDEB periodically reviews the bank to improve its quality. does not affect the resultant permanent C. A. 4. A. a dead soft matrix band. a larger diameter condenser. A 3 year old requires the extraction of a A. cortical drift. decreased condensing pressure. (2) and (4) D. (4) only. (1) (2) (3) C. a tuberosity block plus subperiosteal infiltration of the mesio-buccal root. (1) and (3) D. eruption time of a given tooth. Apical. a posterior superior alveolar block. A. B. (4) only E. 4. The most frequent cause of malocclusion is A. (1) (2) (3) deciduous maxillary second molar. the format of some items is not currently used. (1) and (3) C. E. Oblique. (4) only. B. (2) and (4) D. Note: Some of the items in the Released Test Item Bank may have been discontinued due to outdated science or errors. D. (2) and (4) D. number of years elapsed since a given molar relationship. B. an anatomical wedge. 3. All of the above. E. buccal and palatal infiltration. attached to bone? C. D. Which of the following fibre groups are B. B. data obtained from sickness surveys.The oral mucosa covering the base of the Which of the following foods is the most alveolar bone cariogenic? A. Oral malignancies. and translation. Pharyngitis. B. D. bone. the A. dental caries? A. is normally non-keratinized but can A. content. Toffee. B. All of the above A. Stomatitis medicamentosa. which of the following within a population. the format of some items is not currently used. materials gradually decreases in concentration C. Calcium. (1) and (3) B. The NDEB periodically reviews the bank to improve its quality. (2) and (4) consumption. physiological stimulation. is closely bound to underlying muscle and D. Dental caries. has a tightly woven dense collagenous occurs in the position of corium. Chloride. (1) (2) (3) A. B. adjustments? 1. Daily calcium intake. merges with the keratinized gingiva at the mucogingival junction. Fluoride. following information for the prevention of 4. Frequency of fermentable carbohydrate C. (4) only C. In addition. Protein. Dark chocolate. C. Note: Some of the items in the Released Test Item Bank may have been discontinued due to outdated science or errors. E. Cleft lip and palate. Cheese. Carbonate. study of disease patterns in a population. C. Xerostomia. B. D. protrusive excursion. C. Jam. In chewing. 2. Which of the following oral diseases are D. All of the above. control of disease. largely preventable through lifestyle E. C. from the enamel surface ? D. Epidemiology of disease is best described as D. B. A diet survey can provide which of the 3. become keratinized in response to B. ©The National Dental Examining Board of Canada 2017 . does not contain elastic fibres. Leukoplakia. Periodontal disease. lateral excursion on the non-working side. C. maximum contact between teeth E. Which oral condition predisposes to caries? D. D. habitual occlusion. Total carbohydrate consumption. A. A. usual low level of disease normally found After tooth eruption. Total protein consumption. spirochetes. adjacent teeth. the second is false. a gingival pocket. All of the above. Dental plaque. gram-negative organisms. and translation. the second is incision is influenced by the true. . causes inflammation. C. melanin pigmentation. In addition. E. Polymorphonuclear leukocytes. D. B. the crest of the alveolar bone. D. Mast cells. Lymphocytes. horizontal bone loss. Note: Some of the items in the Released Test Item Bank may have been discontinued due to outdated science or errors. Radiographically. B. Calculus. (2) and (4) E. B. A. thickness of the epithelium. A gingivectomy may be performed when there is/are In health. epithelial keratinization. depth of the vestibule. permit eruption or elongation of teeth. A. A. frenum attachment. A. C. Plasma cells and monocytes. The colour of normal gingiva is affected by the 1. E. (4) only E. prevent pulpitis. Acquired pellicle. A. ©The National Dental Examining Board of Canada 2017 . B. D. microorganisms? 3. the sulcus Which cells migrate into the gingival sulcus in becomes predominantly populated by the largest numbers in response to the accumulation of plaque? A. (1) and (3) D. B. the normal alveolar crest C. Which of the following contains 2. should parallel an imaginary line drawn D. The first statement is false. is difficult to remove. content. an adequate zone of attached gingiva. takes 24 hours to establish. no intrabony defects. D. All of the above. and C. is composed of salivary glycoproteins. gram-positive organisms. C. C. Occlusal (night) guards are used to C. D. The NDEB periodically reviews the bank to improve its quality. D. 4. the design of the B. reduce pocket formation. as seen in a radiograph. D. The first statement is true. C. the format of some items is not currently used. diplococcal organisms.sp 1 B. B. between the cemento-enamel junction of E. Both statements are true. All of the above. the cemento-enamel junction. is situated 1~to~2mm apical to A. Acquired pellicle A. C. amount of attached gingiva.With the development of gingivitis. treat bruxism. presence of infrabony defects. vascularity of the gingiva. In periodontal flap surgery. Both statements are false. All of the above. B. Macrophages. (1) (2) (3) C. A. B. premolars. All of the above. D. fibrotic. A. bacteria and their products. All of the above. 3. B. associated with exudate formation. D. deficiency may result in 2. affected by D. Distal surfaces of mandibular second B. E. A. epithelial attachment. osteogenesis imperfecta. Mesial surfaces of mandibular incisors. fibroedematous. desquamated epithelial cells. friction at the tooth surface from the diet. A. The NDEB periodically reviews the bank to improve its quality. C. B.Maximum shrinkage after gingival curettage Dental plaque is composed of can be expected from tissue that is A. D. B. edematous. horizontally. abnormal formation of osteoid. vertically. peg-shaped teeth. 4. Which of the following root surfaces are most E. In addition. B. cuticle or pellicle. C. Paget's disease. obliquely. and translation. D. The periodontium is best able to tolerate forces As gingival inflammation progresses to directed to a tooth marginal periodontitis. vitamin A premolars. Plaque accumulation on tooth surfaces is C. C. Hutchinson's incisors. E. B. position and surface characteristics of the teeth. osteitis fibrosa cystica. Resorption of the alveolar crest. the associated changes are: A. None of the above. ©The National Dental Examining Board of Canada 2017 . enamel hypoplasia. partial anodontia (hypodontia). (1) and (4) D. (3) and (4) Dietary deficiency of vitamin D can result in A. (1) and (2) E. myositis ossificans. dentinogenesis imperfecta. (1) and (3) C. C. formed within an infrabony pocket. Apical migration and disintegration of the C. A. the anatomy. (2) and (4) E. likely to have concavities that will make root planing difficult? 1. A. Destruction of the alveolar crest and periodontal ligament fibres. content. Mesial surfaces of maxillary first During tooth development. D. B. All of the above. Mesial surfaces of maxillary incisors. D. laterally. the format of some items is not currently used. components from oral secretions. lips and tongue. C. B. Note: Some of the items in the Released Test Item Bank may have been discontinued due to outdated science or errors. E. the architecture of the gingival tissues and their relationship to the teeth. at the dentino-enamel junction. A. C. increasing plaque retention. C. caries susceptibility. dentin. promoting bacterial colonization. on the surface of the clinical crown. Root surfaces.Calculus contributes to gingival inflammation Irregularly distributed shallow to moderate by craters in the interseptal bone are best eliminated by A. B. gingivoplasty. D. C. D. repair of hypoplastic defects of the enamel. deep scaling. Which treatment procedure is indicated for a patient with asymptomatic age related gingival In a young patient living in an area with recession? communal water fluoridation. Note: Some of the items in the Released Test Item Bank may have been discontinued due to outdated science or errors. D. fibronectin. D. B. D. B. B. osteoid. A. Connective tissue graft. Lateral sliding flap. D. A. and translation. B12. causing traumatic occlusion. B. ©The National Dental Examining Board of Canada 2017 . cementum. K. formation of collagen. D. chamber. Pits and fissures. A. bone grafting. A. calcium absorption. B. reticulin. B. B. all of the above. Overhangs on restorations initiate chronic inflammatory periodontal disease by The principal component of the fibres of the periodontal ligament is A. C. causing pressure atrophy. at the layer of dentin nearest the pulp E. C. evenly throughout the enamel. C. C. D. C. D. No treatment. In addition. Incisal dentin. Gingivoplasty. the fluoride concentration of an erupted tooth is greatest A. C. Gingival graft. having cytotoxic bacterial products. having a porous surface. Proximal enamel. D. calculus formation. E. content. C. osteoplasty. increasing food retention. The NDEB periodically reviews the bank to improve its quality. B. elastin. Abnormalities in blood clotting may be associated with a deficiency of vitamin Vitamin D is a factor in A. the format of some items is not currently used. C. E. All of the above. collagen. Caries in older persons is most frequently found on which of the following locations? Vitamin C is essential for A. B. content. two-wall infrabony pockets. (1) and (3) E. prevent temporomandibular joint C. Abrasion is most commonly seen on the 4. C. occlusal surface of posterior teeth. fibroblasts. B. plaque on his teeth. Note: Some of the items in the Released Test Item Bank may have been discontinued due to outdated science or errors. neutrophils. lymphocytes. D. (3) and (4) C. dental caries. A. rods. (1) and (2) B. improve oral hygiene by preventing food impaction. extent of dental neglect. Regarding dental caries. D. three-wall infrabony pockets. suprabony periodontal pocket formation. incisal edges. D. More frequent consumption of carbohydrates increases the risk. C. Carious lesions are most likely to develop if a patient has In periodontics. spirochetes. lactic acid in his mouth. (1) only D. active periodontitis are D. suprabony pockets. and translation. A. B. one-wall infrabony pockets. C. tooth mortality. A. increased tooth mobility. a high lactobacillus count. syndrome. B. facial surfaces of teeth. cocci. D. A. achieve a more favorable direction and distribution of forces of occlusion. B. D. plasma cells. motile rods. A clenching habit may be a factor in D. 3. lingual surface of posterior teeth. C. A. saliva with low buffering capacity. B. the format of some items is not currently used. Increased dietary fat increases the risk. marginal gingivitis. The NDEB periodically reviews the bank to improve its quality. B. C. 2. is correct? C. A. All of the above. All carbohydrates are equally cariogenic. the best prognosis for bone regeneration follows the surgical treatment of A. C. generalized recession. dental needs. B.DMF-S is an index for expressing The most important objective of occlusal adjustment of a natural dentition is to A. An increase of immunoglobulins is consistent with increased numbers of A. 1. The rate of carbohydrate clearance from The predominant organisms associated with the oral cavity is not significant. which of the following B. increase the shearing action in mastication. ©The National Dental Examining Board of Canada 2017 . In addition. D. E. The instrument best suited for root planing is The higher modulus of elasticity of a a/an chromium-cobalt-nickel alloy. file. The NDEB periodically reviews the bank to improve its quality. a reduction in gold content. D. B. A. content. C. None of the above. sickle scaler. C. A. epithelial hyperplasia. permit eruption or elongation of teeth. apically repositioned flap. a shallower undercut. The coronal collagen fibres of the The absence of adequate drainage in a periodontium are periodontal pocket may result in A. C. coronally positioned flap. C. C. a reduction in platinum content. B. location of the lesions. D. reduce unfavorable forces on teeth. E. the format of some items is not currently used. means that chromium- A. D. curette. ©The National Dental Examining Board of Canada 2017 . D. D. double-papilla pedicle graft. temperature of the patient. B. cobalt-nickel partial denture clasp will require B. lymphadenopathy. C. B. D. C. D. E. abscess formation. All of the above. are dynamic rather than static. C. free gingival graft.The epithelial attachment does not migrate A removable full-arch occlusal splint is used to apically in A. B. cyst formation. D. B. and translation. A. Note: Some of the items in the Released Test Item Bank may have been discontinued due to outdated science or errors. a shorter retentive arm. C. allow for individual tooth movement. A. The tissues of the epithelial attachment From Type I to Type IV gold alloys there is A. compared to a Type IV gold alloy. transeptal. pain. a heavier cross section for a clasp arm. supraperiosteal. more taper. an increase in gold content. ultrasonic scaler. B. D. reduce pocket formation. A. chronic periodontitis. increased calculus formation. juvenile periodontitis. All of the above. hyperplastic gingivitis. circular. no change in the gold content. B. In addition. C. exhibit a high rate of biologic turnover. rapidly progressive periodontitis. laterally positioned sliding flap. can be reconstituted by repair. hoe. B. D. Correction of an inadequate zone of attached Necrotizing ulcerative gingivitis (NUG) and gingiva on several adjacent teeth is best acute herpetic gingivostomatitis can be accomplished with a/an differentiated clinically by (the) A. All of the above. (1) and (3) C. 1. C. (2) and (4) D. it conceals the porcelain to metal junction the powder particles. The NDEB periodically reviews the bank to improve its quality. 4. E. it has opened gingival embrasures. provide retention for a cast crown. All of the above. exothermic heat of setting. A. (4) only E. 4. producing a systemic reaction. the format of some items is not currently used. 2. strengthen a weakened tooth. hardness. (2) and (4) A. Note: Some of the items in the Released Test Item Bank may have been discontinued due to outdated science or errors. C. In addition. A. and translation. B. During the setting phase. (1) (2) (3) B. its gingival surface is convex in all directions. C. melting point. discolouring gingival tissue. a dental stone mixture will exhibit A cast post and core is used to A. (1) and (3) C. 2. 4. redirect the forces of occlusion. gain in moisture content. 3. strength. interfering with the setting of the D. (4) only impression material. 3. (4) only 1. D. difference in particle shape and density. B. E. A. (1) and (3) results in increased C. ©The National Dental Examining Board of Canada 2017 . causing tissue necrosis. on its gingival surface. An epinephrine-containing retraction cord has A. expansion. provide intraradicular venting. difference in the chemical composition of 3. All of the above. (1) (2) (3) the potential of B.The gypsum material used for fabrication of A pontic replacing a mandibular first molar dies exhibits a higher crushing strength than should be designed so that does regular stone because of 1. (2) and (4) D. (1) (2) (3) The addition of platinum to a dental gold alloy B. content. loss in compressive strength. it seals the muco-gingival field. D. contraction. resistance to corrosion. B. 2. D. (1) (2) (3) B. impingement on the buccal frenum. A. D. dentures. The NDEB periodically reviews the bank to improve its quality. The proximal contacts. (1) and (4) 2. which of the following can be 4. upon delivery of the denture. (3) and (4) 4. at a right angle to the occlusal plane. C. C. C. D. A. at the gingival margin. D. D. The taper of the preparation. should be completed B. high muscle attachments. A. (2) and (4) generalized soreness over the entire mandibular D. In addition. During the fabrication of new complete 3. excess border thickness. 0. after the teeth have been set on the trial D. D. on the enamel. denture. The orientation of the occlusal plane. B. and translation. Decrease in ductility. immediately after making the final casts. In the preparation of gypsum products. In the design of a removable partial denture. C. Increase in ductility. a C. All of the above. The impression used to pour the cast.The gingival margin of the preparation for a Which of the following should be checked first full crown on a posterior tooth. B. Increase in resistance to corrosion. ©The National Dental Examining Board of Canada 2017 . (4) only alveolar ridge can be caused by E. at least 1mm supragingivally. (1) and (2) 1. parallel to the long axis of the tooth. B. C. an increase in the water/powder ratio will Adjustment of the occlusal plane of natural teeth opposed by a complete or partial denture A. guiding planes are made A. increase the surface hardness.5mm subgingivally. Note: Some of the items in the Released Test Item Bank may have been discontinued due to outdated science or errors. modified to achieve the desired occlusion? A. E. The occlusal contacts. C. accelerate the setting reaction. C. working a metal? 1. 2. B. B. with a clinical when a cast gold crown that fits on its die crown that satisfies the requirements for cannot be seated on its abutment? retention and resistance. The compensating curve. should be placed A. after the diagnosis and treatment plan has Which of the following are effects of cold been established. at a right angle to the major connector. None of the above. The cusp inclination. (1) and (3) Following the insertion of complete dentures. at the cemento-enamel junction. content. (2) and (3) 3. The condylar inclination. parallel to the path of insertion. Increase in hardness. the format of some items is not currently used. B. Inadequate interocclusal distance. increase the compressive strength. A. it is biocompatible with the pulp. E. high tensile strength. C.A removable partial denture rest should Upon examination of an edentulous patient. provide the necessary clearance. B. ©The National Dental Examining Board of Canada 2017 . Zygomatic process. C. be located on a centric contact. A. (4) only E. low compressive strength. it interacts with setting amalgam to form a weak chemical union. D. D. its pH stimulates secondary dentin 3. reduce the retromolar pads surgically to D. All of the above. The main reason for adding copper to a dental amalgam alloy is to Which of the following structures affects the A. thickness of the flange of a maxillary complete B. upward only. Coronoid process. increase expansion. interferences. B. The treatment of choice is to C. reduce the tuberosities surgically to provide the necessary clearance. increase lustre. retromolar pads at the correct occlusal vertical B. Note: Some of the items in the Released Test Item Bank may have been discontinued due to outdated science or errors. D. it is compressible when set. reduce tarnish resistance. upward and facially. 2. proceed with construction of the denture A. C. (1) and (3) D. and reduce the posterior extension of the B. D. be extended for retention. Dental porcelain has polycarboxylate cement can be used as a base material because 1. mandibular denture to eliminate C. the format of some items is not currently used. and translation. Genial tubercle. B. 4. dimension. (2) and (4) D. content. upward and palatally. A. high hardness. low impact strength. B. increase retention of the partial denture. E. the direction of occlusal vertical dimension to gain the resorption of the alveolar ridge is necessary clearance. reduce the tin-mercury phase. formation. it is observed that the tuberosities contact the A. direct forces parallel to the long axis of the abutment. denture? C. make amalgamation easier. uniform in all directions. A. The NDEB periodically reviews the bank to improve its quality. Mylohyoid ridge. Where cavity preparations are extensive. Malar process. construct new dentures at an increased In an edentulous maxilla. In addition. A. (1) (2) (3) C. (4) only B. (2) and (4) D. Corrosion resistance. D. D. C. B. retard the final set. increased strength. better colour. 2. C. is not subject to as much muscular displacement. and translation. incorporates a posterior palatal seal. is completely surrounded by soft tissue.The polishing of an amalgam restoration The prime advantage of vacuum firing of porcelain is A. the format of some items is not currently used. removes the mercury rich surface layer of D. clinical crown length. (1) (2) (3) 4. protect the cement from ultraviolet light. dissolve all the alloy particles in the mercury. locations where esthetics are important. should not be performed before 24 hours after insertion. coat the alloy particles with mercury. A. Increased strength. presence of caries. C. covers a greater area. Lowered specific gravity. After initial setting. completely prevents tarnish from B. B. D. All of the above. presence of an existing restoration. (1) (2) (3) D. ©The National Dental Examining Board of Canada 2017 . 1. Increased hardness. 3. esthetic requirements. E. The NDEB periodically reviews the bank to improve its quality. A. C. Note: Some of the items in the Released Test Item Bank may have been discontinued due to outdated science or errors. occurring. A. create a smooth finish. (1) and (3) C. hasten the final set. as they form. 3. reduce the size of the crystals as rapidly B. B. Class II lesions in adults. The principal purpose of amalgam trituration is to Glass ionomer cement restorations are A. less shrinkage. more translucency. C. 2. root caries. a chemically cured glass ionomer cement restoration should have a coating agent applied to Gold contributes which of the following properties to a gold-copper alloy? A. protect the cement from moisture. The location of a crown margin is determined A maxillary complete denture exhibits more by retention and stability than a mandibular one because it 1. A. indicated for B. C. (2) and (4) A. content. C. D. the amalgam. (4) only E. is more permanent if the surface is heated during the procedure. B. 4. All of the above. reduce mercury content of the restoration. D. (1) and (3) E. In addition. incisal edge fractures. The NDEB periodically reviews the bank to improve its quality. Severe enamel hypoplasia. (2) (3) (4) The success of remineralization of "white spot" D. has a triple helical structure. forms insoluble high tensile strength prothrombin. All of the above. (1) and (3) A lowering of serum calcium is the stimulus for C. In addition. (1) (2) (3) B. Vitamin E is an important antioxidant. Frequent ingestion of polysaccharides. All of the above. Buccinator. adrenalin.Which of the following muscles has two Chronic alcoholism will separate functions in mandibular movement? 1. Collagen 3. the format of some items is not currently used. is most common in hard tissues. (1) (2) (3) B. content. (4) only D. 3. 5. decrease the effectiveness of local D. (1) and (3) containing foods. C. B. D. fibres. pH of the saliva. 2. Frequent ingestion of high sucrose- B. D. A. (4) only E. C. External (lateral) pterygoid. (1) (4) (5) C. delay healing. Vitamin A is important in the formation of visual purple. increase tendency to hemorrhage. anesthetics. Which of the following is the greatest risk 3. A. All of the above. A. factor for rampant caries in children? 4. mucous membranes. Vitamin D promotes intestinal calcium and phosphate absorption. Masseter. All of the above. 4. ©The National Dental Examining Board of Canada 2017 . Vitamin A maintains the integrity of C. frequency of the cariogenic challenge. (2) and (4) the endogenous release of D. 4. A. D. 2. insulin. The main functions of fat soluble vitamins are: E. E. A. A. Note: Some of the items in the Released Test Item Bank may have been discontinued due to outdated science or errors. B. availability of mineral ions in saliva. 2. parathyroid hormone. viscosity of the saliva. Deficiency of vitamin D. cause impairment of liver function. adrenocortical hormone. C. 1. Vitamin K catalyzes the synthesis of B. (1) (2) (3) B. and translation. thyroid hormone. Geniohyoid. (2) and (4) C. A. lesions is dependent upon the 1. 4. A. characterized by abnormally large pulp chambers? A. the format of some items is not currently used. (1) and (4) an inlay preparation is to A. The NDEB periodically reviews the bank to improve its quality. C. location of the carious lesion. pulp chamber. C. preventing formation of thromboplastin. kilovoltage controls by 1. All of the above. Reversible pulpitis. D. insufficient fluoride. fibrin. C. Enlargement of the thyroid gland can be caused In X-ray equipment. 1. depth of the pulpal floor. D. excess iodine. amount of carious material removed. thickness of the remaining dentin. excess sodium. amount of radiation produced. speed of electrons. B. (1) (3) (5) The principal reason for a cavosurface bevel on D. penetrating power of radiation. B. D. (1) and (3) A. 3. Using pins to retain amalgam restorations C. and translation. Note: Some of the items in the Released Test Item Bank may have been discontinued due to outdated science or errors. Internal resorption. improve marginal adaptation. In addition. increase resistance and retention forms. the pin holes should be parallel to the A. content. C. D. 2. A. is when deciding whether or not to use pulp most likely to result in a periapical lesion? protection? A. pulp exposures. B. E. Chronic hyperplastic pulpitis. D. the liver. D. long axis of the tooth. ©The National Dental Examining Board of Canada 2017 . excess calcium. Dentinogenesis imperfecta. Which of the following conditions is 4. Diffuse calcification of the pulp. insufficient iodine. increases the risk of D. D. (2) and (4) B. decrease marginal percolation. Acute suppurative pulpitis. A. E. Dentinal dysplasia Type I. if left untreated. cracks in the teeth. nearest external surface. axial wall. thermal sensitivity.Warfarin (Coumadin®) acts by In pin-retained restorations. 5. periodontal ligament invasion. C. incorporating ionic calcium. Regional odontodysplasia. Amelogenesis imperfecta. (1) (2) (4) C. 2. (1) (2) (3) B. B. E. (1) and (2) B. remove undermined enamel. contrast. 3. inhibiting the synthesis of prothrombin in C. What is the most important factor to consider Which of the following. B. temperature of the cathode filament. preventing fibrinogen conversion to A. B. (4) only C. twisting and rocking. C. C. establish the location of the posterior palatal 3. Anterior disc dislocation. (2) and (4) B. the radiopacity that can obliterate the apices of maxillary molars is the A. and translation. B. thick layer of bone forming the inner B.Which of the following has the highest rate of In the bisecting angle principle of intraoral recurrence? radiography. The most likely origin of a metastic carcinoma of the mandible is a primary lesion of the 1. (4) only posterior nasal spine. thin radiopaque line around the roots of the teeth. D. D. B. Prominent fungiform papillae. C. seal of a maxillary complete denture include 4. zygoma and the zygomatic process of the maxilla. Median rhomboid glossitis. prostate. maxillary sinus. All of the above. breast. Note: Some of the items in the Released Test Item Bank may have been discontinued due to outdated science or errors. Odontogenic keratocyst. The NDEB periodically reviews the bank to improve its quality. orbital process of the zygomatic bone. (1) (2) (3) A. In addition. the format of some items is not currently used. B. Median palatal cyst. content. D. anterior border of the tuberosities. the A. The anatomical landmarks used to help 2. ©The National Dental Examining Board of Canada 2017 . Which of the following is the most frequent cause of ankylosis of the temporomandibular joint? Radiographically. the lamina dura is a A. E. the palatine raphe and the posterior border of Hyperplastic lingual tonsils may resemble the palatine bone. E. a quick snap. Nasoalveolar cyst. pterygomaxillary notches and the D. A. pterygomaxillary notches and the fovea B. Lingual varicosities. C. B. which of the following? A. nasopharynx. C. having the patient create a positive pressure. Squamous cell carcinoma. Incisive canal cyst. Intra-articular injection of steroids. thin radiolucent line around the roots of D. D. Trauma. C. D. thick layer of cortical bone. palatine bone and the zygoma. removed from the mouth by D. Irreversible hydrocolloid materials are best C. posterior border of the tuberosities and the posterior border of the palatine bone. B. a slow teasing motion. surface of the alveolus. the teeth. C. lung. A. Epulis fissuratum. A. Chronic subluxation. (1) and (3) palatinae. All of the above. hypophosphatasia. C. mylohyoid. buccinator and genioglossus muscles. B. A. an increase in congenitally missing teeth. buccinator. Epithelial cells are united to the basal E. In addition. The NDEB periodically reviews the bank to improve its quality. buccinator and styloglossus muscles. Coagulation time. lingual to the crest of the mandibular E. Note: Some of the items in the Released Test Item Bank may have been discontinued due to outdated science or errors. A. over the buccal shelf area. 1. ridge. 4. less marginal breakdown. styloglossus and geniohyoid muscles. mylohyoid. less corrosion. (1) and (3) Which of the following statements is correct? C. ridge. can bring the alveolar ridge to the level of the D. cleidocranial dysostosis. Bleeding time. a mixture of plaster of Paris and water will exhibit A. In patients with cleft palates there may be Upon setting.To improve denture stability. ©The National Dental Examining Board of Canada 2017 . ectodermal dysplasia. a higher incidence of crown defects. (4) only A. D. buccal to the crest of the mandibular B. 3. trauma. Prothrombin time. D. mylohyoid and buccinator muscles. lamina by hemidesmosomes. over the crest of the mandibular ridge. mandibular molar The absence of a pulp chamber in a deciduous teeth should normally be placed maxillary incisor is most likely due to A. (1) (2) (3) B. and translation. D. C. C. an increase in supernumerary teeth. superior constrictor. gain in moisture content. B. B. B. All of the above. loss in compressive strength. B. the format of some items is not currently used. C. attachment of the A. Clotting time. content. (2) and (4) D. The basal lamina is a product of epithelial Extreme resorption of an edentulous mandible cells. B. D. All of the above. contraction. Before performing surgery on a patient who is taking warfarin. D. amelogenesis imperfecta. lower creep. A. higher 1 hour compressive strength. 2. C. C. D. C. which of the following should High copper amalgam alloys are superior to be evaluated? conventional alloys in that they have A. expansion. The basal lamina is divided into lamina lucida and lamina densa. is associated with gram-negative anaerobic flora. is associated with root caries. ©The National Dental Examining Board of Canada 2017 . (2) and (4) D. (1) and (3) C. A 70-year old female has consumed optimal B.Normal sulcular epithelium in man is Gingival inflammation may result from 1. has a definite predilection toward males. cementum. All of the above. A. osteoporosis. 1. 0. The NDEB periodically reviews the bank to improve its quality. fibres of the periodontal ligament are modified by A. (4) only E. D. hyperparathyroidism. (4) only E. alveolar bone. All of the above. dental caries. nausea. B. a potent inflammatory agent. B. A. and translation. levels of fluorides all her life. A. 3. Note: Some of the items in the Released Test Item Bank may have been discontinued due to outdated science or errors. A. (1) (2) (3) B. a cell wall component of gram-negative bacteria. 3.0ppm. B. tooth morphology. mental confusion. is associated with gram-positive anaerobic flora. present in diseased root cementum. occlusal function. Juvenile periodontitis C. (4) only The lowest level of fluoride in drinking water E.0ppm. plaque. (2) and (4) 4. nonpermeable.0ppm. 2. D. (1) (2) (3) 2. C. content.5ppm. D. (1) (2) (3) B. squamous. (1) and (3) C. the format of some items is not currently used. 3. which will cause enamel mottling is A. Paget's disease of bone. A. 4. 1. Hypoglycemia is characterized by 4. All of the above. nonkeratinized. trauma. 1. 5. C. tachycardia. All of the above. C. The arrangement and character of the principal D. (2) and (4) Endotoxin is D. All of the above. B. chemical irritation. sweating. 2. expect to find a decreased incidence of D. B. A. C. D. You would C. In addition. stratified. (1) and (3) 3. increase mental alertness and decrease fatigue. be placed in 2. increasing target-film distance. ©The National Dental Examining Board of Canada 2017 . and translation. are analeptics. using an aluminum filter. Acetominophen. (2) and (4) D. Short object-film distance. C. A. (1) and (3) B. milliamperage. A. Long source-film distance. C. The amount of radiation to a patient can be D. Periapical cyst. None of the above. The NDEB periodically reviews the bank to improve its quality. A. kilovoltage. exposure time. None of the above. A. 3. 4. 3. All of the above. have no effect on psychomotor activity. reduced by E. D. (1) and (3) C. decreasing target-film distance. B. (2) (3) (4) E. Bradykinin. (1) (3) (5) D. (4) only E. A. B. Which of the following has/have analgesic. C. A. (4) only Filters are placed in the path of the x-ray beam D. to A. 2. Acetylsalicylic acid. 4. Hyperplastic pulpitis. increase contrast. All of the above. 100% relative humidity. 2. are useful in controlling arrhythmias. Note: Some of the items in the Released Test Item Bank may have been discontinued due to outdated science or errors. Internal resorption. A 1% aqueous calcium sulfate solution. and B. D. density of the object. (2) and (4) C. the format of some items is not currently used.Amphetamines Which of the following will increase image sharpness on radiographs? 1. B. (1) (2) (3) D. A. 1. it should 1. C. (2) (3) (5) The depth of penetration of any object by x- rays is determined by If an alginate impression must be stored for a few minutes before the cast is poured. reduce film density. reduce patient radiation dose. antipyretic and anti-inflammatory effects? D. content. Periapical cementoma. 3. Small focal spot. B. A. In addition. with a non-vital tooth? 5. B. C. (1) (2) (4) C. water. B. using a high speed film. All of the above. Which of the following is most often associated 4. reduce exposure time. using low kVp. (1) (2) (3) (4) B. (1) and (3) 3. (4) only E. (1) and (3) C. be chamfered. B. induces bone formation throughout the B. amalgam restoration the cavosurface angle E. (1) and (3) implanted material. is highly biocompatible. the margins of a restoration 3. 4. drifting of the teeth. (2) and (4) 4. periodontal disease is the most likely cause of C. be bevelled. 2. (4) only E. burning gingivae.30 years. painful. (1) (2) (3) E. supragingivally. In addition.20 years. B. C. marginal gingivitis. (1) (2) (3) 2. approach 90 degrees. A. content. (1) (2) (3) 4. 30 . when placed Desquamative gingivitis is seen in subperiostially. mucous membrane pemphigoid. 1. A characteristic sign of aggressive periodontitis in an adolescent (juvenile periodontitis) is A. Location of the tumor. and translation. following factors? 1. should A. World epidemiological data indicates that B. on enamel. B. Prognosis for a patient with oral squamous cell C. (1) (2) (3) B. subgingivally.Particulate hydroxyapatite.75 years. Symptoms. To ensure maximum marginal strength for an D. The NDEB periodically reviews the bank to improve its quality. Lymph node involvement. (4) only E. approach 45 degrees. should be placed 4. All of the above. A. All of the above. C. D. 1. B. 3. over 75 years. tooth loss in the following age group: D. hyperplastic gingivitis. (2) and (4) D. pemphigus vulgaris. Whenever possible. All of the above. on cementum. 2. (1) and (3) C.50 years. (2) and (4) D. 2. A. 1. the format of some items is not currently used. All of the above. (2) and (4) D. 65 . Note: Some of the items in the Released Test Item Bank may have been discontinued due to outdated science or errors. A. C. (4) only A. erythema multiforme. infection following surgery. carcinoma depends upon which of the D. has a low incidence of secondary 3. 20 . erosive lichen planus. has a tendency to migrate following insertion. ©The National Dental Examining Board of Canada 2017 . 10 . Size of the tumor. not interfere with lateral forces. Pre-extraction profile records. B. (1) (2) (3) D. should result in a long bevel. the restoration is more difficult to polish. should remove unsupported enamel which D. (1) (2) (3) B. The NDEB periodically reviews the bank to improve its quality. excessive vertical dimension and poor retention. C. ©The National Dental Examining Board of Canada 2017 . excessive occlusal vertical dimension. is unnecessary since the tooth structure in B.A bevel is CONTRAINDICATED on the The retention form of a full crown preparation cavosurface margins of a Class I amalgam can be improved by cavity preparation because 1. utilizing grooves or boxes. reduced vertical dimension and guide(s) in determining a patient’s occlusal improperly balanced occlusion. (2) and (4) In patients wearing complete dentures. D. A. the bevel would be worn away. use of too large a posterior tooth and too 2. and translation. should result in a sharp gingivoproximal line angle. little horizontal overlap. the major connector A. improper relation of teeth to the ridge and 4. this area is strong. may fracture. frequent cause of tooth contact (clicking) E. is contraindicated because of the low edge strength of amalgam. (4) only E. dissipate vertical forces. incorrect centric relation position. A. (4) only. a thin flange of the amalgam restorative material is prone to fracture. (1) and (3) C. content. A. the format of some items is not currently used. C. during speaking is A. this type of margin would tend to leak. nervous tension. In addition. All of the above. should B. reducing its taper. as the tooth undergoes natural attrition. unbalanced occlusion. the most D. D. C. Observation of the rest position. (1) and (3) C. B. act as a stress-breaker. E. B. C. Phonetics. (2) and (4) D. The most common causes are Which of the following is/are (a) useful A. vertical dimension? B. E. All of the above. 3. D. 3. A. lack of vertical overlap. polishing the preparation. Appearance. excessive anterior vertical overlap. C. rigidly connect the bilateral components. increasing its length. Note: Some of the items in the Released Test Item Bank may have been discontinued due to outdated science or errors. Planing the enamel at the gingival cavosurface of a Class II amalgam preparation on a permanent tooth In partial denture design. 2. A patient with complete dentures complains of clicking. 1. 4. phenothiazines. vancomycin. (1) and (3) C. palpitations. (1) (2) (3) malignancy include B. alveolar ridge resorption. (2) and (4) A. increased heart rate. 3. Intravenous administration of epinephrine A. 2. The NDEB periodically reviews the bank to improve its quality. (1) (4) (5) results in B. Mitral valve prolapse with regurgitation. penicillin V. benzodiazepines. All of the above. tricyclic antidepressants. cephalexin. D. A. B. add potassium sulfate. 5. incision and drainage. E. add a small amount of borax. D. E. D. In addition. Note: Some of the items in the Released Test Item Bank may have been discontinued due to outdated science or errors. Cardiac pacemaker. C. (1) and (3) C. (1) (2) (3) B. 3. the format of some items is not currently used. (2) and (4) D. D. All of the above. D. atrophic thinning of the oral mucosa. leukoplakia. Prosthetic heart valves. 1. tetracycline. All of the above. ulceration. use cold water. ©The National Dental Examining Board of Canada 2017 . denture induced hyperplasia. B. C. spontaneous bleeding. (4) only E. necrosis. Antibiotic prophylaxis is recommended for patients with which of the following? The surgical procedure indicated for 1. the drug of choice is The use of complete dentures by older people most frequently causes A. B. angular cheilitis. E. (1) (2) (3) (4) 1. add additional water. D. Side effects of chemotherapeutic treatment for A. 4. enucleation. 2. All of the above. (2) (3) (5) C. extend spatulation time. 4. C. 3. stomatitis.The best means of extending the working time The most appropriate oral drugs for control of of an irreversible hydrocolloid impression anxiety in a dental patient are material is to A. content. 4. B. odontogenic cysts is 2. barbiturates. C. and translation. increased systolic pressure. A. In an infection caused by non-penicillinase producing staphylococcus. C. cauterization. (4) only B. A. respiratory depression. All heart murmurs. wound dehiscence. Tooth-to-bone. base will result in D. C. Floor of the mouth. and translation. (2) and (4) D. (1) and (3) C. A. B. C. A. slower healing. gingival irritation. light. Bone-to-bone. Note: Some of the items in the Released Test Item Bank may have been discontinued due to outdated science or errors. (4) only E. ankylosed permanent canines. The most likely cause is 1. Cephalometrics is used in orthodontics to A. D. B. heavy. ©The National Dental Examining Board of Canada 2017 . D. content. (1) (2) (3) B.Water irrigation devices have been shown to The undesirable side-effect most commonly associated with use of a finger spring to tip the A. ankylosed primary canines. 1. severe mobility of the tooth. necrosis of bone. D. disinfect pockets for up to 18 hours. All of the above. dislodge food particles from between teeth. the format of some items is not currently used. C. B. 4. Cephalometrics is useful in assessing which of D. squamous cell carcinoma in the elderly? E. 3. crown of an anterior permanent tooth is B. primary site for the development of oral D. 2. A. permanent Forces for orthodontic tooth movement ideally canines and primary canines all in contact and should be anterior to the premolars. tendency for the tooth to intrude. B. Tooth-to-tooth. (2) and (4) D. All of the above. 3. A. continuous. (4) only E. The NDEB periodically reviews the bank to improve its quality. and C. the following relationships? A. 2. impacted permanent lateral incisors. 4. treat malocclusions. In addition. eliminate plaque. congenitally missing permanent lateral A. Which of the following is the LEAST likely C. A surgical flap not repositioned over a bony C. study growth changes. C. B. Clinical examination of a 15 year old girl shows permanent central incisors. Dorsum of the tongue. foreign body inflammatory reaction. All of the above. aid in diagnosis and case analysis. (1) (2) (3) incisors. All of the above. pain. Lateral border of the tongue. B. B. intermittent. Tonsillar fossa. (1) and (3) C. prevent calculus formation. In addition. D. and translation. B. C. intrusion of the central incisor. Note: Some of the items in the Released Test Item Bank may have been discontinued due to outdated science or errors. act as enzymes. B. All of the above. and B. D. keratinized squamous epithelium. grow slowly. (1) and (3) D. intrusion of the central incisor and lingual A. B. All of the above. Which of the following are mechanisms of Attached gingival tissue is primarily composed growth of the naso-maxillary complex? of A. lingual movement of the crown and 4. All of the above. B.Benign neoplasms Wolff's Law states that bone elements 1. (1) (2) (3) stress. are generally painless. A. and C. Appositional. ©The National Dental Examining Board of Canada 2017 . (1) (2) (3) movement of the crown. 3. Sutural. producing a light force on the labial of a proclined 1. A. stress. lingual movement of the crown and labial C. the format of some items is not currently used. B. is located at the cemento- enamel junction. Cartilaginous. C. collagenous fibres. act as chemotactic products. E. elastic fibres. D.. E. can be managed conservatively. B. (4) only E. B. (4) only D. The epithelial attachment A. The inflammatory response in periodontal pathology is caused by bacterial products from plaque which A removable orthodontic appliance. decrease their mass to reflect functional A. A. activate the kallikrein system. initiate an immune response. is composed of stratified squamous non- keratinized epithelium. A. maxillary central incisor will cause 2. A. can metastasize. content. 3. increase their mass to reflect functional 4. moves apically along the root surface as periodontal disease progresses. C. All of the above. with periodontitis. rearrange themselves in the direction of 2. functional pressures. C. (2) and (4) movement of the root apex. C. B. D. lingual movement of the root apex. (1) and (3) C. All of the above. The NDEB periodically reviews the bank to improve its quality. (2) and (4) D. in health. hypothyroidism. Metabolism is decreased by a reduced B. underdevelopment. C. 4. cementoenamel junction. 2. (1) (2) (3) D. 1. normal occlusal function. content. (1) and (3) E. (4) only E. 4. (1) and (3) C. Codeine. Oxycodone. All of the above. Absorption is altered by a decrease in the A. B. soft condition. C. C. junction. backward placement of the film. base of the pocket to the cementoenamel gastric pH. When using the periodontal probe to measure Which of the following pharmacokinetic pocket depth. free gingival margin to the liver mass. (2) and (4) D. ©The National Dental Examining Board of Canada 2017 . 2. hyperparathyroidism. gingiva. stress than the same alloy in a softened B. Paget's disease. (1) (2) (3) B. base of the pocket to the crest of the free total body fat. Distribution is altered by a decrease in C. (Coumadin®)? 2. 3. Acetylsalicylic acid. (4) only B. All of the above. less plastic deformation per unit of stress Hypercementosis at the root apex is often than the same alloy in a softened associated with condition. excessive milliamperage. A hardened gold alloy will exhibit A. In addition. junction. B. per unit of stress of the alloy in hard or E.Excessively dark radiographs will result from Which of the following should NOT be prescribed for a patient receiving warfarin 1. base of the pocket to the mucogingival renal blood flow. (1) (2) (3) 4. no difference in the plastic deformation D. 3. The NDEB periodically reviews the bank to improve its quality. (4) only E. the measurement is taken from change(s) occur(s) with aging? the 1. A. (2) and (4) D. Excretion is reduced because of lessened D. and translation. Ketorolac. overexposure. the format of some items is not currently used. (2) and (4) A. All of the above. A. orthodontic tooth movement. greater plastic deformation per unit of A. 3. Note: Some of the items in the Released Test Item Bank may have been discontinued due to outdated science or errors. condition. (1) and (3) C. (4) only E. horizontal condylar inclination. at the symphysis. D. 3. 3. (4) only E. There is a differential between girls and boys with respect to the age at which the growth velocity reaches its peak. 3. Apical migration of junctional epithelium. centric relation. along the lower border. varnish. C. impair tooth movement. the format of some items is not currently used. A. A. D. increased strength. B. (1) (2) (3) B. Bleeding upon probing. growth of the approximates the pulp. you would protect the mandible is greatest pulp with A. All of the above. (1) (2) (3) B. Bennett angle. girls six months ahead of boys. crush the periodontal ligament. In a cavity preparation which closely After the age of 6 years. (4) only D. E. B. B. All of the above. ©The National Dental Examining Board of Canada 2017 . Increased depth upon probing. a calcium hydroxide cement liner and a glass ionomer cement base. boys six months ahead of girls. Note: Some of the items in the Released Test Item Bank may have been discontinued due to outdated science or errors. cause hyalinization. reduced polymerization shrinkage. (1) and (3) C. The NDEB periodically reviews the bank to improve its quality. E. Excessive orthodontic force used to move a tooth may 1. (2) and (4) C. A. B. D. C. and translation. That difference is A hinge axis facebow records A. better polishability. 4. (1) (2) (3) A. girls one year ahead of boys. C. (2) and (4) D. opening and closing axis of the mandible. Fibrotic enlargement of marginal tissue. cause root resorption. lateral condylar inclination. 2. a calcium hydroxide cement base.Which of the following is/are common to both Compared to unfilled resins. 4. A. reduced thermal dimensional changes. (1) and (3) C. posterior to first molars. composite resins gingival and periodontal pockets? have 1. girls two years ahead of boys. a calcium hydroxide wash and cavity D. a zinc phosphate cement base. C. In addition. (1) and (3) B. 2. (2) and (4) D. All of the above. 4. content. 2. between canines. 1. Division 2. Which of the following drug groups can cause xerostomia? The best space maintainer to prevent the lingual collapse that often occurs following the 1. content. Anemia. A. tipping. 1. density of the cortical bone. III. I. C. E. (1) (2) (3) B. (1) and (3) A. Cardiac failure. C. 4. D. which of the following changes older patient short of breath on mild exertion? are likely to occur? 1. 2. ©The National Dental Examining Board of Canada 2017 . and translation. free gingival and transseptal fibres. D. A. None of the above. (1) (2) (3) D. Mouth breathing is most commonly associated with Angle's malocclusion Class A. 3. Diuretics. B. C. Non-steroidal anti-inflammatory agents. early loss of a mandibular primary canine is a 2. All of the above. Antidepressants. (1) and (3) D. All of the above. No movement of second permanent molar. A. B. oblique fibres of the periodontal ligament. (2) and (4) D. 4.The predominant type of movement produced Recurring tooth rotations occur most frequently by a finger spring on a removable appliance is after orthodontic correction due to A. Note: Some of the items in the Released Test Item Bank may have been discontinued due to outdated science or errors. All of the above. (2) and (4) E. (1) and (3) C. B. (2) and (4) B. (4) only C. lingual arch. Mesial drift of second permanent molar. (4) only E. rotation. 4. persistence of tongue and finger habits. Nance expansion arch. (4) only E. Antibiotics. The NDEB periodically reviews the bank to improve its quality. D. (1) (2) (3) C. B. In addition. torque. Division 1. Distal drift of second premolar. Osteoarthritis. 3. Obesity. A. D. C. II. band and loop space maintainer. translation. distal shoe space maintainer. B. No movement of second premolar. 2. Following loss of a permanent mandibular first Which of the following conditions can make an molar at age 8. II. 3. the format of some items is not currently used. (2) and (4) E. D. routine amalgam restoration 2. Mesio-angular. D. 3. B. Epinephrine.The design of a mucoperiosteal flap should Which of the following is/are essential when using spherical rather than admix alloy for a 1. A. A. D. A. (1) and (2) B. toxicity of local anaesthetic. An alveoplasty is performed to Which one of the following describes the 1. possibility of intravascular administration. permit repositioning over a solid bone 1. an anatomical wedge. (2) and (3) D. Note: Some of the items in the Released Test Item Bank may have been discontinued due to outdated science or errors. Disto-angular. Anterior to the pterygomandibular raphe. Hydrochloric acid. Metabisulfite. Medial to the medial pterygoid muscle. (1) and (3) C. In the surgical removal of an impacted mandibular third molar. nerve block? 3. Lidocaine. 2. D. toxicity of vasoconstrictor. All of the above. content. (1) (2) (3) B. The NDEB periodically reviews the bank to improve its quality. All of the above. (2) and (4) B. facilitate removal of teeth. possibility of paraesthesia. D. a larger diameter condenser. 4. Vertical. 4. provide for visual access. the format of some items is not currently used. a dead soft matrix band. 3. ©The National Dental Examining Board of Canada 2017 . which of the following Which of the following constituents of a local would be considered to be the most difficult? anesthetic cartridge is most likely to be allergenic? A. (1) and (3) A. provide for instrument access. In addition. correct irregularities of alveolar ridges of local anesthetic for the inferior alveolar following tooth removal. B. prepare the residual ridge for dentures. (1) (2) (3) C. C. Superior to the lateral pterygoid muscle. B. (1) and (3) D. base. Aspiration is carried out prior to a local anaesthetic injection in order to reduce the A. position of the needle tip during administration 2. C. (4) only E. Horizontal. C. decreased condensing pressure. be semilunar in shape. A. B. Lateral to the sphenomandibular ligament. and translation. C. All of the above. (4) only C. A. Ethoxy benzoic acid cement. space for an appropriate thickness of the 1. contaminated with latex. 3. (4) only 2. B. A thinner matrix band. B. definite finish line. margin well below the gingival crest. elimination of post-operative sensitivity. veneering material. (2) and (4) 1. enamel results in D. (1) and (3) amalgam restoration. E. (1) (2) (3) resins? B. restoration is in supraocclusion. 4. Shade of composite resin. Intensity of the light source. It is most likely E. 3. Which of the following affect(s) polymerization of visible light cured composite A. 2. (2) and (4) impression material is considered to be good D. (1) (2) (3) Twenty-four hours after placement of a Class II B. C. and translation. In addition. improved wear resistance of the composite. restoration is leaking. C.The tooth preparation for a porcelain veneer In order to achieve a proper interproximal must create a/an contact when using a spherical alloy. decreased crack formation in the enamel. Glass ionomer cement. Zinc phosphate cement. C. 1. rough surface for improved bonding. Polycarboxylate cement. Proximity of light source. (1) and (3) The dimensional stability of polyether C. 3. Resin bonding of composites to acid-etched C. content. A. An anatomical wedge. Note: Some of the items in the Released Test Item Bank may have been discontinued due to outdated science or errors. decreased polymerization shrinkage of the resin. ©The National Dental Examining Board of Canada 2017 . D. 2. (1) and (3) C. (4) only There are no other symptoms. B. E. Which of the following cements can chemically D. Use of mechanical condensation. which of the following is/are essential? A. 4. that the A. reduced microleakage. A. distorted by rapid removal of the bond to enamel? impression from the mouth. A larger sized condenser. B. Thickness of composite resin. pulp was exposed. All of the above. (4) only EXCEPT if the material is E. A. D. All of the above. dehydrated. E. 4. (2) and (4) complaining of discomfort when "biting". The NDEB periodically reviews the bank to improve its quality. allowed to absorb water after setting. D. All of the above. (1) (2) (3) B. the format of some items is not currently used. used in uneven thickness. D. amount of base material is inadequate. a patient returns C. Anticariogenicity.8 . The NDEB periodically reviews the bank to improve its quality. 2. place a provisional (temporary) crown. a calcium hydroxide cement. when used as a luting D. 0. (1) (2) (3) B. 90. C. following properties? 1. caries removal. pulp regeneration. C. and sensitive to cold. 2. (1) (2) (3) incisal third of tooth 2. 3. All of the above.1. All of the above. B. (4) only tooth but the patient complains that it is rough E. The fracture occurred one C.1 exposing a small B. well defined axiogingival line D. it is ethical to refuse to and temporarily close with zinc oxide and treat a patient on the basis of eugenol. smooth the surrounding enamel and apply 3. The most appropriate emergency treatment is to A.100 cavosurface margin. recognition of lack of skill or knowledge. religious beliefs. glass ionomer cement. (4) only agent for cast restorations. angle. infectious disease. Chemical adhesion. finish line that is 2mm subgingival. (2) and (4) hour previously. 4. 4. A. (1) (2) (3) B. D. physical handicap. Mechanical retention. clean the canal As a dentist in Canada. All of the above. A. 2. a consideration in porcelain fused to metal restoration with a porcelain butt joint margin requires a A. smooth the surrounding enamel and apply 1. Note: Some of the items in the Released Test Item Bank may have been discontinued due to outdated science or errors. dentin bonding. A 22 year old presents with a fracture of the A. Insolubility. and translation. has which of the E. (4) only E. 4. the format of some items is not currently used.2mm shoulder. open the pulp chamber. 3. In addition. B. (1) and (3) C. content. plaque accumulation. sharp. There is no mobility of the D. 1. ©The National Dental Examining Board of Canada 2017 . (1) and (3) amount of dentin. (1) and (3) C.The "smear layer" is an important To achieve optimum strength and esthetics. (2) and (4) D. (2) and (4) Zinc phosphate cement. 4. (2) and (4) 2. maximum intercuspation. E. Inadequate knowledge of the specific 4. Diabetes mellitus. protrusive position. E. normal rest position. pain. ©The National Dental Examining Board of Canada 2017 . (4) only E. a dentist may periodontitis are refuse to treat a patient with HIV infection for a specific procedure for the following reason(s). respect the patient's decision. The periodontium is sound. D. The most appropriate management is to 1. content. inflammation. (2) and (4) D. 2. 2. B. All of the above. He has C. Cyclic neutropenia. (1) and (3) D. (1) (2) (3) B. vesicle formation. procedure. pseudomembranes. A. A. (4) only E. (1) (2) (3) procedure. (4) only 3. 3. Hereditary hypohydrotic ectodermal dysplasia. (1) and (3) 1. 1. Inadequate experience in the specific pathogens. D. The NDEB periodically reviews the bank to improve its quality. 2. D. Lack of instruments or equipment for this A. A.According to the principles of ethics that are The features of aggressive (rapidly progressive) generally accepted in Canada. A. All of the above. B. All of the above. (2) and (4) carious lesions involving the dentin on all D. the format of some items is not currently used. Note: Some of the items in the Released Test Item Bank may have been discontinued due to outdated science or errors. onset before the age of 35. All of the above. and translation. maximum opening. B. 1. C. taken with the patient in 4. refuse to refer or to treat this patient since it is unethical. suspected periodontal microbial 1. rapid attachment loss. Acquired immunodeficiency syndrome. advise the patient to consult a specialist. (2) and (4) designed for infectious patients. 3. (1) and (3) extracted and dentures fabricated. (4) only Aphthous ulcers are characterized by E. ulcerations of the gingiva. 2. (1) (2) (3) A patient wants all his remaining teeth B. C. (1) (2) (3) B. discuss all of the appropriate treatment with a 3mm anterior functional shift should be options. In addition. (4) only remaining teeth. (2) and (4) E. (1) and (3) C. All of the above. (1) and (3) 4. Which of the following systemic diseases 4. procedure. (1) (2) (3) C. initial contact. does/do NOT predispose a patient to periodontitis? A. A lateral cephalometric radiograph for a patient 3. Infection control procedures that are not C. 3. tender. nontender. A. Facial nerve VII. pericoronally. the format of some items is not currently used. B. fixed. nontender. This is most likely the result of A. changes in dietary pattern. Benign (reactive) lymphoid hyperplasia of B. matted. C. content. Levator labii superioris and zygomaticus major. Psoriatic arthritis. (2) and (4) D. mobile. All of the above. D. B. A patient with a tumor in the right infratemporal fossa shows a significant shift of the mandible to the right when opening. 3. tender. (1) and (3) C. Risorius and buccinator. Trigeminal nerve V. affects the gingiva. Rheumatoid arthritis. periapically. D. age related decrease in salivary flow. C. contains melanin. is macular. cervical lymph nodes is typically C. Which two muscles are involved in sucking? A. age related changes in cementum composition. A. C. 4. A healthy 78 year old patient presents with C. (4) only Melanoma differs from physiologic E. E. Hypoglossal nerve XII. three new carious lesions on root surfaces. 2. In addition. D. B. Which Dentigerous cysts are usually found nerve is involved? A. changes clinically. D. B. D. interradicularly. ©The National Dental Examining Board of Canada 2017 . the architecture at the CEJ. to eliminate toxins from the patient. B. Lupus arthritis. Glossopharyngeal nerve IX. and translation. Note: Some of the items in the Released Test Item Bank may have been discontinued due to outdated science or errors. fixed. Suppurative arthritis. D. to relieve symptoms of multiple sclerosis. chronic periodontal disease. Buccinator and orbicularis oris.Which articular disease most often It is ethical to replace amalgam restorations accompanies Sjögren’s syndrome? 1. C. C. on request from an informed patient. pigmentation because it A. B. A. Caninus and depressor angularis. Degenerative arthrosis. (1) (2) (3) B. in highly esthetic areas of the mouth. A. E. mid-root. The NDEB periodically reviews the bank to improve its quality. D. A. D. Free nerve endings. B. “guided tissue thermal sensitivity. ulcerative periodontitis (NUP) in a patient with D. A. All of the above. ©The National Dental Examining Board of Canada 2017 . A. 2. antibiotic therapy. 3. (1) and (3) wife complains that he snores. Note: Some of the items in the Released Test Item Bank may have been discontinued due to outdated science or errors. fabricate restorations to increase the patient’s vertical dimension of occlusion. E. midline deviation. This pain is associated regeneration” is most successful in treating with which of the following? 1. All of the above. and translation. The most common clinical characteristic/s of a 3. (4) only 4. Golgi receptor. involvement is A. Cementoblasts. for a mandibular Class III furcation D. (1) (2) (3) A 45 year old. oral hygiene instruction. The most likely cause of tooth loss following a C. asymmetrical arches. A. buccolingual functional crossbite is/are 4. In addition. (4) only. B. the format of some items is not currently used. content. horizontal bone loss. 2.Following root planing. topical steroid therapy. refer for a sleep assessment. 1. B. refer for an orthognathic surgery tunneling procedure to provide complete access consultation. The most appropriate treatment of necrotizing C. fabricate an appliance to reduce snoring. no fever and no lymphadenopathy is 1. mandibular shift from initial contact to A. (2) and (4) D. D. a 3-walled infrabony defect. root sensitivity. (1) (2) (3) maximum intercuspation. (2) and (4) management of the patient’s snoring problem is D. to E. (1) (2) (3) B. several missing teeth. The initial C. overweight man reports that his B. (4) only E. pulpal involvement. involvement. C. All of the above. a mandibular Class III furcation C. a mandibular Class II furcation involvement. The NDEB periodically reviews the bank to improve its quality. periodontal debridement. a patient experiences In periodontal therapy. Odontoblastic processes. (1) and (3) C. B. 4. (2) and (4) 3. (1) and (3) 2. recurrent pocketing. root caries. reticulin. osseous contours. localized periodontal abscess. Note: Some of the items in the Released Test Item Bank may have been discontinued due to outdated science or errors. D. composed of C. ©The National Dental Examining Board of Canada 2017 . C. D. (1) (2) (3) B. the format of some items is not currently used. the normal alveolar crest is A. healing by secondary intention. A. (4) only For an otherwise healthy patient. adequate access to correct irregular 2. (1) (2) (3) Continued smoking will impair wound healing B. contraction of peripheral blood vessels. 4. gingiva. D. The predominant organism(s) associated with B. Gingival connective tissue fibres are primarily B. (1) and (3) following a surgical procedure because of C. Lymphocytes. (2) and (4) D. E. All of the above. In addition. at the cementoenamel junction. oxytalin. Porphyromonas gingivalis. Macrophages. initial treatment must include A. Neutrophils. increased rate of plaque formation. C. collagen. occlusal adjustment. E. A. stain development. superficial irritation to tissues by smoke. elastin. On bite-wing radiographs of adults under the age of 30. 3. 2. increased rate of calculus formation. junction. C. with an acute E. D.Which of the following cells are involved in an Eliminating periodontal pockets by immune response to plaque in the periodontal gingivectomy results in pocket? A. Prevotella intermedia. Plasma cells. 1-2mm apical to the cementoenamel chronic (adult) periodontitis is/are junction. 1. (2) and (4) D. prescription of an antibiotic. scaling and root planing. C. not clearly distinguishable. All of the above. The NDEB periodically reviews the bank to improve its quality. 3-4mm apical to the cementoenamel 1. D. B. prescription of an analgesic. (1) and (3) C. A. 3. and translation. Heliobacter pilori. B. B. content. retention of all or most of the attached 4. (4) only A. Pseudomonas aeruginosa. healing by primary intention. 25 lower arches and 70% overbite. 50 . A. 10mm or more in each of the upper and America are caries free? lower arches and 35% overbite. D. nocturnal bruxism. They discharge into the internal jugular opening muscles. soft. B. B. content. the format of some items is not currently used. A 23 year old female complains of bilateral A. B. Note: Some of the items in the Released Test Item Bank may have been discontinued due to outdated science or errors. They drain the anterior palatine pillar. 4. Swallowing and respiration neuronal 2. posterior third of the tongue. All of the above. applied to patients with Class I occlusions with D. Reflexes between jaw closing and 1. as soon as the malocclusion is diagnosed. 3.75 The best time to begin interceptive orthodontic treatment for a patient with a skeletal Class II malocclusion is A. serial extraction is best C.17 year olds in North B. mandibular subluxation. 65 . activity. Periodontal receptor stimulation. several months prior to the pre-pubertal growth spurt. D. In addition. soft palate. well C. They are found medially to the mandible. and translation. 15 . nontender. The NDEB periodically reviews the bank to improve its quality.Chewing “automatism” is dependent on which Which of the following statements applies to mechanism? the submandibular lymph nodes? A. less than 10mm in each of the upper and lower arches and 35% overbite. (1) and (3) region. they are enlarged. D. crowding of A. defined and movable. C. fibrous ankylosis of the temporomandibular joints. In its classic form. immediately following complete eruption of the deciduous dentition. C. What percentage of 5 . ©The National Dental Examining Board of Canada 2017 . Her symptoms have been present for C. (4) only. 10mm or more in each of the upper and C. Reticular formation neuronal activity. When draining an area of acute infection. immediately following complete eruption of the first permanent molars. (2) and (4) the past week and are most pronounced in the D. after skeletal maturity. 30 . nodes. (1) (2) (3) stiffness and soreness in the preauricular B. less than 10mm in each of the upper and A. early osteoarthritis.45 D. E. The most likely cause is E. B. morning.60 lower arches and 70% overbite. C. 1. smear layer. . and translation. D. B. content. history of spontaneous pain. E. (4) only E. α2. The layer of intermingled collagen and resin located beneath a restoration is called the If a patient loses a permanent maxillary first molar before the age of 11. D. premolar drifts distally. the A. C. (1) and (3) C. All of the above. chance of clinical success when there is a 4. A. Lingual vein.The maxillary cast partial denture major The presurgical assessment of a permanent connector design with the greatest potential to maxillary first molar reveals roots which are cause speech problems is trifurcated and widely divergent. mesially. trifurcation. done during extraction of this tooth? C. Reduce the palatal bone to the level of the trifurcation. a thin broad palatal strap. (1) (2) (3) mandibular second molar? B. 4. Submandibular duct. apical lesion. α1. 2. (2) and (4) D. (2) and (4) A. Which of the following phases in the setting reaction of dental amalgam is weak and corrosion-prone? A. Amputate the crown and section the roots. a thick narrow major connector. 3. (1) and (3) C. Reflect a flap. 1. D. All of the above. E. A. In addition. (4) only B. C. The maxillary antrum encroaches into the area of A. C. an anterior and a posterior bar. 2. 1. Which of the following should be B. The NDEB periodically reviews the bank to improve its quality. vital pulp. D. narrow horseshoe shaped. the format of some items is not currently used. B. overbite increases. D. ©The National Dental Examining Board of Canada 2017 . (1) (2) (3) B. Lingual artery. decalcification layer. prolonged response to cold stimulus. Lingual nerve. 3. B. opposing tooth erupts into the space created. Reduce the buccal bone to the level of the A direct or indirect pulp cap has the greatest trifurcation. hybrid layer. Weil layer. Note: Some of the items in the Released Test Item Bank may have been discontinued due to outdated science or errors. 2. maxillary second molar erupts and moves D. Which of the following structures lies inferior to the mylohyoid muscle at the level of the A. the teeth were not individually ligated. highly esthetic. D. A laboratory-fabricated composite resin inlay compared to a direct composite resin restoration has increased A. Asthmatic reaction to acetylsalicylic acid. C. Müller’s fibres. Lateral tongue. D. C. Soft palate. Condensation silicone. Polysulfide rubber. ©The National Dental Examining Board of Canada 2017 . the holes were placed too far apart. B. E. premalignant. C. B. present for C. highly translucent. control of polymerization shrinkage. B. Concurrent use of alcohol. present for a few Periapical cemental dysplasia is days. Ventral tongue. The NDEB periodically reviews the bank to improve its quality. content. oxytalan fibres. Occasionally tender normal coloured nodule. the holes were placed too close together. weeks. several months. A. painful. D. Which of the following mucosae is normally B. Burning red plaque. Sore shallow ulcer. C. A. A. A gastric ulcer. surface smoothness. unaffected by moisture during the setting reaction. Note: Some of the items in the Released Test Item Bank may have been discontinued due to outdated science or errors. a fluoride releasing material. present for several years. D. expansile. Sharpey’s fibres. Asymptomatic gray macule. C. Irreversible hydrocolloid. D. The reason for this is A. B. associated with vital teeth. In addition. C. C. the format of some items is not currently used. Hard palate. D. D. and translation. The collagen fibres of the periodontal ligament that insert into bone are called The major advantage of glass ionomer cement as a restorative material is that it is A. D. a light weight dam was used. keratinized? C. reticular fibres. Which one of the following is NOT a contraindication to ibuprofen? A. Polyvinylsiloxane. B. colour stability. Which of the following is most likely to be a squamous cell carcinoma? A. bondability to tooth structure. present for several A. a rubber dam frame was used.Which of the following impression materials After placement of the rubber dam you notice has the best dimensional stability? that the interdental papilla is protruding from beneath the rubber dam. that B. B. B. D. An allergy to acetaminophen. Fibrous dysplasia of the jaws Following root canal therapy. Streptococcus mutans. B. All of the above. C. pain. epithelium proliferation from the periodontal ligament. cementum deposition. (1) (2) (3) B. A. D. 1. (2) and (4) C. traced to source with a gutta-percha point on a radiograph. dentures are D. A. A. ©The National Dental Examining Board of Canada 2017 . All of the above. the format of some items is not currently used. Lactobacillus acidophilus. β-hemolytic streptococci. evaluate esthetics. (1) and (3) C. and translation. Hirsutism. Cushing’s disease? 2. xerostomia. 4. 2. C. 4. connective tissue capsule formation. (2) and (4) D. Note: Some of the items in the Released Test Item Bank may have been discontinued due to outdated science or errors. spontaneous hemorrhage of the gingiva. has monostotic and polyostotic forms. A. (1) (2) (3) B. Osteoporosis. E. verify the maxillomandibular records. E. A. (4) only D. 3. the most desirable form of tissue response at the apical A. Staphylococcus aureus. All of the above. E. Hypertension.Oral signs and/or symptoms of advanced The micro-organisms of dental caries are vitamin C deficiency include 1. All of the above. Buffalo hump. (1) and (3) C. In addition. therapy. angular cheilitis. foramen is B. is premalignant. (4) only tried in to E. cauterized to remove the epithelium. Diabetes insipidus. (1) (2) (3) A. D. (2) and (4) While the teeth are set in wax. content. is bilaterally symmetrical. (1) and (3) B. is of known etiology. C. 2. nonsurgical root canal therapy. 4. A fistula found in association with a non-vital tooth should be Which of the following is NOT associated with 1. C. dentin deposition. (4) only E. 3. treated with nonsurgical root canal D. 3. The NDEB periodically reviews the bank to improve its quality. verify the vertical dimension of occlusion. B. begins in the fifth decade. D. treated with combined surgical and B. D. B. Mandibular block anesthesia. blunted roots and abnormal Which one of the following luting agents has dentin are indicative of been shown to reduce the incidence of fracture in an all-ceramic restoration? A. nonsurgical root canal therapy only. advanced adult periodontitis. D. C. C. periodontal surgical therapy only. calculus. Crohn’s disease. Subgingival restoration. before procedure only. C. diffuse widening of the periodontal ligament. Resin modified glass ionomer cement. D. Enamel hypoplasia. A. C. In addition. plaque. the format of some items is not currently used. She is losing weight and has A.5g six hours after initial diffuse brown macular pigmentation of the oral dose. The NDEB periodically reviews the bank to improve its quality. clindamycin 300mg orally one hour A. erythromycin stearate. D. B. hypoparathyroidism. dentin. Which of the following dental procedures could be performed with minimal risk for a 35- year old patient with a severe bleeding The primary objective of periodontal disorder? debridement is removal of A. mucosa. C. then 150mg six hours B. The patient has loss of attached gingiva and multiple areas of gingival A. E. D. 2g orally two hours before procedure only. C. content. B. hyperthyroidism. clindamycin 600mg orally one hour D. Addison’s disease. E. The current recommended regimen of antibiotic prophylaxis for a patient with a prosthetic heart valve and an allergy to penicillin is A patient complains of irritability. hypothyroidism. erythema multiforme. D. nonsurgical root canal therapy before A. Supragingival calculus removal. cementum. after initial dose. acute myeloid leukemia. then 1. periodontal surgical therapy before non. Zinc phosphate cement. The most likely diagnosis is surgical endodontic treatment. A. iron deficiency anemia. C. before procedure. B. B. amoxicillin 2g orally one hour before The most likely diagnosis is procedure only. The clinical examination shows procedure. Composite resin cement. cicatricial pemphigoid. fatigue and weakness. C. periodontal therapy.The most appropriate treatment of a true A 52 year old patient presents with a limitation combined endodontic-periodontal lesion is of mouth opening. B. A panoramic radiograph shows B. ©The National Dental Examining Board of Canada 2017 . hyperparathyroidism. recession. D. Incisional biopsy. and translation. amoxicillin 3g orally one hour before diarrhea. Note: Some of the items in the Released Test Item Bank may have been discontinued due to outdated science or errors. Glass ionomer cement. hyperparathyroidism. The pigmentation appeared recently. scleroderma. Hypothyroidism. located in the midline of the pain. A. benign migratory glossitis. B. B. Hypoparathyroidism. The most prevalent inflammatory cells found in gingival tissue 24 hours following flap surgery are A. D. E. polymorphonuclear leukocytes. A patient has a displaced right subcondylar Which of the following “in office” preventive fracture of the mandible. a fibroma. elevated. This occurs due to which of the following events? A.Which of the following presents with high An 8 year old patient with all primary molars serum calcium levels. Which muscle is an uncooperative 4-year old patient from a non- prevented from functioning appropriately? compliant family? A. the format of some items is not currently used. Fluoride varnish every six months. Stress-induced sympathetic activity the circumvallate papillae is indicative of causes vasodilatation of pulpal blood vessels. median rhomboid glossitis. Temporalis. content. Local mediators of pulpal pain are not B. C. place patient on appropriate recall disappears when a patient visits a dental office. Dietary counseling. C. Oral hygiene instruction. C. should be to B. Stress-induced sympathetic activity A smooth. The NDEB periodically reviews the bank to improve its quality. released when a stress-induced rise in C. D. E. Hyperparathyroidism. ©The National Dental Examining Board of Canada 2017 . On opening. use a cervical headgear to reposition maxillary molars. lymphocytes. D. Pulpal pain receptors undergo fatigue. macrophages. D. The pain associated with pulpitis often D. B. dorsum of the tongue immediately anterior to B. C. Pit and fissure sealants on all primary molars. C. In addition. B. iron deficiency anemia. refer for orthodontic consultation. D. Masseter. blood pressure occurs. Lateral pterygoid. C. D. and translation. the procedures is most practical and effective for mandible deflects to the right. The management of this patient A. Medial pterygoid. schedule. thinning of cortical bone still present exhibits a cusp-to-cusp relationship and giant cell osteoclasts in the jaw and drifting of permanent maxillary and mandibular first teeth? molars and good alignment of the lower incisors. Hyperthyroidism. Note: Some of the items in the Released Test Item Bank may have been discontinued due to outdated science or errors. disk the distal surfaces of primary mandibular second molars. red patch devoid of inhibits pulpal sensory fibres that cause filiform papillae. A. monocytes. A. Fluoride supplements. a granular cell tumor. (4) only E. more resistant than HIV. how 3. C. B. Medial pterygoid muscles. which of the sugarless gum is most effective in preventing following has/have maximum EMG activity caries? when the teeth are in maximum intercuspation? A. (1) (2) (3) B. All of the above. Sympathetic fibres constrict the bronchioles. Mannitol. All of the above. (1) (2) (3) fall into the right primary bronchus. ©The National Dental Examining Board of Canada 2017 . (4) only D. D. (1) and (3) C. The trachea is membranous posteriorly to 2. minute? A. in higher numbers in blood than HIV. (2) and (4) C. many external chest compressions are given per 4. An aspirated foreign body would likely A. Sorbitol. Digastric muscles. C. D. 1. the format of some items is not currently used. A. secondary bronchi. (1) and (3) B. (1) (2) (3) A. C. B. 3. resuscitation (CPR) on an adult patient. 3. The NDEB periodically reviews the bank to improve its quality. 60. Note: Some of the items in the Released Test Item Bank may have been discontinued due to outdated science or errors. In addition. 40. 2. D. Lateral pterygoid muscles. All of the above. The right primary bronchus forms two 4. transmissible through saliva. content. (2) and (4) D. B. Glycerol. and translation. Inflammation of the facial nerve. Masseter muscles. Which of the following are possible causes of Bell’s Palsy? 1. Xylitol.The risk of transmission of Hepatitis B Virus Which of the following statements concerning (HBV) is greater than that of Human the airway is correct? Immunodeficiency Virus (HIV) because HBV is A. Incorrect injection technique. (2) and (4) D. (1) and (3) C. Viral infection. B. 100. 80. Which of the following sweeteners used in During a normal chewing cycle. Surgical trauma. accommodate the pulsations of the aorta. 4. E. 1. With two rescuers performing cardiopulmonary 2. (4) only E. autoclave resistant. after the crown appears calcified radiographically. D. Improper safety light. (1) (2) (3) B. Vitamin A. The NDEB periodically reviews the bank to improve its quality. Vitamin D. scanty. the format of some items is not currently used. Which of the following is a possible cause for a 3. hands are moist. A. Cold developer. reduces the number of skin bacteria which multiply and cause irritation. fair hair. as soon as possible. fine. formed. All of the above. Vitamin E. (2) and (4) D. C. allows gloves to slide on easier when the B. 2. incorrect vertical angulation of the x-ray B. C. 3. and an intolerance to hot weather. A. cystic fibrosis. Over exposure. In addition. content. The washing of hands must be performed B. 1. (1) and (3) C. malignant hyperthermia. minimizes the transient bacteria which low density radiograph (light film)? could contaminate hands through small pinholes. conical E. malalignment of teeth. completely eliminates skin bacteria. ectodermal dysplasia. Vitamin C.The most appropriate time to remove a Overlapping contacts on a bitewing radiograph supernumerary tooth that is disturbing the result from eruption of a permanent tooth is 1. closed. A. 2. (1) (2) (3) B. patient movement during the exposure. The most likely diagnosis is A. E. D. after two thirds of the permanent root has beam. All of the above. D. Excessive developing time. incorrect horizontal angulation of the x- C. after the apex of the permanent root has ray beam. before putting on and after removing gloves C. A. (4) only A patient presents with hypodontia. (4) only Which of the following is necessary for E. (1) and (3) C. 4. 4. Note: Some of the items in the Released Test Item Bank may have been discontinued due to outdated science or errors. and translation. ©The National Dental Examining Board of Canada 2017 . teeth. Vitamin K. collagen formation? A. B. (2) and (4) D. because it D. achondroplasia. D. An excessive occlusal vertical dimension. negative effects of the additional load applied 2. (1) and (3) abutment teeth. splinting abutments with adjacent teeth. A reduced horizontal overlap. (1) and (3) D. 3. determine the path of insertion of the removable partial denture. (1) and (3) C. using cast clasps on distal abutments. 2. (4) only E. D. keeping a light occlusion on the distal extensions. (2) and (4) E. (4) only E. 4. (4) only C. (2) and (4) B. content. 2. 4. A. phenytoin sodium. evaluate the retention potential of the B. increased exposure to latex. C. and translation. evaluate the modifications needed to D. Aging pulps show a relative increase in The survey of the diagnostic cast that is done as 1. to them is by 3. In addition. 1. placing distal rests on distal abutments. vascularity. B. A. Note: Some of the items in the Released Test Item Bank may have been discontinued due to outdated science or errors. ©The National Dental Examining Board of Canada 2017 . B. fibrous elements. All of the above. 3. (1) (2) (3) B. (2) and (4) E. cell numbers. (1) (2) (3) C. part of a removable partial denture design 2. (1) and (3) A. A. regular relining of the distal extensions. (1) (2) (3) B. cyclosporine. history of spina bifida. A. 4.The risk of latex allergy increases with a/an The best way to protect the abutments of a Class I removable partial denture from the 1. locate the height of contour abutment teeth. All of the above. (4) only create parallel guiding planes. chestnuts or avocado. All of the above. The NDEB periodically reviews the bank to improve its quality. procedure is needed to 3. (2) and (4) 3. nifedipine. A. E. the format of some items is not currently used. Which of the following could cause clicking sounds during speech in denture wearers? Gingival enlargement may result from the administration of 1. 4. (1) (2) (3) C. A nonbalanced occlusion. prednisolone. A lack of denture retention. (1) and (3) D. 1. C. (1) (2) (3) 2. history of allergy to bananas. (2) and (4) D. All of the above. history of eczema. calcification. (4) only E. All of the above. 4. C. B. lateral incisors. changes in dietary pattern. D. use of an inappropriate luting material. Absence of permanent maxillary lateral D. contains elastic fibers. The NDEB periodically reviews the bank to improve its quality. Distal of the 3. A. facial height.6 pontic. C. C. subgingival deposits. increased vasodilation. In addition. C. Where following clinical observations? should the male part of the attachment ideally be located? A. 6 years. E. bicuspid can be confirmed? This is most likely the result of A. 8 years. 2 years. 4. (1) and (3) C. A. mandibular angle. 3. decreased salivary flow. Mesial of the 3. (2) and (4) A. Mesial of the 3. the format of some items is not currently used. maxillary protrusion. Hypertrophied labial frenum. inadequate ceramic thickness. D. exposure of the cementoenamel junctions. Delayed exfoliation of primary maxillary B. Dental diastema. Note: Some of the items in the Released Test Item Bank may have been discontinued due to outdated science or errors. incisors. 2. A semi-precision attachment is chosen commonly associated with which of the rather than preparing the teeth again.5 retainer. overbite. is closely bound to underlying periosteum. A survey of the master cast shows that the 3.7 abutments for a fixed partial denture An unerupted supernumerary tooth between the have different paths of insertion with respect to permanent maxillary central incisors is 3. 1. D. has no rete pegs. A. changes in cementum composition. C.Healthy attached gingiva A fracture in an all ceramic crown may be A. All of the above. E. D.7. (4) only B.7 retainer.5 and 3. Distal of the 3. the degree of keratinization. caused by B. D. What is the earliest age that the diagnosis of a After many caries free years a 78 year old congenitally missing mandibular second patient develops multiple root surface caries. The angle SNA can be used to evaluate the A. and translation. ©The National Dental Examining Board of Canada 2017 . has no basal cell layer. C. sharp line angles in the tooth preparation. D. 4 years. increased collagen fiber density. (1) (2) (3) The redness of inflamed gingiva is due to B. excessive occlusal load. D. C. B. upper incisor inclination.6 pontic. content. B. B. Increased overjet. A. 4. (1) and (3) C. D. 3. All of the above. (1) and (3) C. arthritis. Corticosteroids may be used for the C.0mm A.A Bolton relationship has determined a Trauma from occlusion may  maxillary “12” excess of 3. Addison's disease. What effect(s) could this Bolton relationship C. diastemas and/or a poorly fitting denture? A. Reduced overjet. A. affect the blood supply to gingiva. Rickets. E. 2. initiate periodontitis. (1) (2) (3) B. B. A larger diameter condenser tip. D. Mumps. (4) only E. asthma. (2) and(4) D. fever and unilateral parotid Which of the following is/are essential when swelling following a general anesthetic most using a spherical rather than an admix alloy for likely has a routine amalgam restoration? A. acute bacterial sialadenitis. In addition. An anatomical wedge. systemic corticosteroids (Prednisone). allergy. Osteoporosis. All of the following are well documented 3. (4) only enlargement of the jaws. 2. B. 1. A serrated condenser tip. 1. affect the progression of periodontitis. Paget’s disease. B. (1) (2) (3) B. mouth rinses. (1) (2) (3) B. Sjögren’s syndrome. 2. the format of some items is not currently used. initiate marginal gingivitis. 4. (2) and (4) Which of the following diseases may cause an D. (1) and (3) C. (2) and (4) D. management of D. A. Hypophosphatasia. (4) only E. development of E. Note: Some of the items in the Released Test Item Bank may have been discontinued due to outdated science or errors. A. 3. A thinner matrix band. All of the above. have on a Class I malocclusion? D. radiotherapy to the head and neck. The NDEB periodically reviews the bank to improve its quality. B. A patient with pain. antibiotics.5mm  maxillary “6” excess of 3. Maxillary crowding. sarcoidosis. C. E. initiating factors of hairy tongue EXCEPT 4. Deeper overbite. Phantom bone disease. E. ©The National Dental Examining Board of Canada 2017 . content. candidiasis. All of the above. 1. C. sialolithiasis. and translation. All of the following are common features of a Which conditions are associated with Acquired malignant neoplasm of the jaws EXCEPT Immunodeficiency Syndrome (AIDS)? A. multiple jaw biopsies. parathyroid biopsy. E. 4mm in diameter should be treated by A. tuberculosis. creatine clearance and BUN.Multiple osteomas and supernumerary teeth In mucous membrane pemphigoid. paresthesia and tooth loosening. B. ©The National Dental Examining Board of Canada 2017 . Gardner’s syndrome. C. D. E. The test(s) diagnostic for primary The most likely diagnosis of a non-healing hyperparathyroidism in a patient with multiple indurated ulcer on the lateral border of the brown tumours is/are tongue in a 60 year old patient is A. a positive may be associated with Nikolsky sign is the result of a/an A. squamous cell carcimona. Cleidocranial dysplasia. separation at the basement membrane. a pigmented macule measuring E. Acute marginal periodontitis. B. radiographic skeletal survey. well-circumscribed periapical 4. C. C. D. A. A. (1) (2) (3) B. carbon dioxide laser. B. A. (4) only In general. a traumatic ulcer. The NDEB periodically reviews the bank to improve its quality. radiolucencies. B. content. E. E. major aphthous ulcer. 1. excisional biopsy. Rubenstein – Taybi syndrome. intraepithelial bulla formation. 3. C. (2) and (4) D. electrocautery. (1) and (3) C. In addition. widening of periodontal ligament spaces. Ectodermal dysplasia. D. C. All of the above. Note: Some of the items in the Released Test Item Bank may have been discontinued due to outdated science or errors. D. irregular radiopacities. the format of some items is not currently used. C. moth-eaten radiolucencies. and translation. B. separation of the lamina propria and D. submucosal. D. a deep fungal infection. intraepithelial separation. Gorlin’s syndrome. 2. Candidiasis. serum calcium and PTH level. E. exfoliative cytology. B. Geographic tongue. incisional biopsy. Hairy leukoplakia. force the alloy particles together. (1) (2) (3) A. and translation. 3. penetrating power of radiation. A. The NDEB periodically reviews the bank to improve its quality. 2. All of the above. express excess mercury content. (2) and (4) D. epigenetic. (4) only D. All of the above. 4. using an aluminum filter. 3. 3. ©The National Dental Examining Board of Canada 2017 . (1) and (3) C. further break down the alloy particles. complete the trituration process. E. 4. functional. B. (1) (2) (3) A. (1) and (3) B.Proper collimation of the useful beam for the The primary stimulus for growth of the film size and target-film distance will reduce mandible is 1. 2. A. amount of radiation produced. (2) and (4) D. speed of electrons. (4) only E. (4) only Alginate impression material E. is a thermoplastic material. eliminate the need for internal retention. B. radiation received by patient. 4. sets by condensation polymerization. reduced by the cavosurface margin of the cavity can be bevelled to 1. (1) and (3) C. A. All of the above. kilovoltage controls Amalgam is condensed to 1. 2. D. 4. (1) (2) (3) B. environmental. is a rigid material. All of the above. radiographic contrast. A. genetic. aid in finishing. C. All of the above. content. secondary radiation. (1) and (3) C. (1) (2) (3) B. is a reversible hydrocolloid. (4) only D. image definition. 1. the format of some items is not currently used. contrast. (1) (2) (3) B. 2. Note: Some of the items in the Released Test Item Bank may have been discontinued due to outdated science or errors. 3. (1) and (4) C. is an irreversible material. In x-ray equipment. D. improve convenience form. (2) and (4) C. C. (2) and (4) D. The amount of radiation to a patient can be For an acid etched Class III composite resin. increase the surface area for etching. 1. using a high speed film. (1) and (3) B. 4. 2. increasing target-film distance. E. 3. E. (4) only E. A. In addition. using low kVp. central incisor. such as preventing the recirculation of the drug through the enterohepatic circulation. (2) and (4) B. All of the above. A. The principles of closed fracture management are After initiating preventive management for a 16 year old patient with multiple extensive 1. 4. retrusion. Delay any treatment until the hygiene D. (1) and (3) restorations within the next few weeks. mandibular 3. Restore all teeth with composite resin over the next few months. (2) and (4) D. Middle face cellulitis. division 1 chemotherapeutic agents can interfere with malocclusion may describe a combination of reabsorption of a contraceptive steroid thus skeletal problems. (4) only E. incision and debridement at fracture site. Note: Some of the items in the Released Test Item Bank may have been discontinued due to outdated science or errors. (4) only C. A. (1) (2) (3) C. (4) only E. 2. maxillary protrusion. C. normal mandibular following can interfere with this mechanism? position. reduction of fracture. A. maxillary protrusion. restoration of occlusion. C. Which of the 1. Penicillin V. carious lesions. (1) (2) (3) B. Displacement of fractures is influenced by Which of the following clinical conditions is 1. Tetracycline. Acetaminophen. a Class II. E. Chronic periapical abscess of a B. 3. direction of the blow. which of the following 2.Alteration of the intestinal flora by some In Angle's classification. Codeine. (1) and (3) A. hemorrhage. restorative treatments is most appropriate? 3. (2) and (4) C. with the maxilla more protrusive than the mandible. mandibular 1. All of the above. (1) and (3) D. (1) (2) (3) C. A. A. Acute periapical abscess of a mandibular 4. 4. 4. content. In addition. and translation. All of the above. (4) only improves. (1) (2) (3) B. direction of fracture line. Place amalgam restorations over the next few months. Infected dentigerous cyst. B. 3. retrusion. All of the above. The NDEB periodically reviews the bank to improve its quality. Excavate caries and place temporary B. the most serious? 2. normal maxillary position. (2) and (4) E. bimaxillary protrusion. immobilization of fracture. D. 2. D. D. the format of some items is not currently used. (1) and (3) mandibular third molar. muscle attachments. ©The National Dental Examining Board of Canada 2017 . B. periapical osseous dysplasia (periapical 3. A larger sized condenser. C. and translation. All of the above. content. His daughter. (1) (2) (3) B. enhance the bond between a porcelain C. ©The National Dental Examining Board of Canada 2017 . (4) only D. predisposing to dental diseases. A. D. All of the above. which of the following is/are essential? 1. excessive orthodontic forces. A. syneresis. imbibition. D. traumatic injury. (4) only E. contain pulp tissue. who is his legal guardian. All of the above. reduce the surface tension when investing a wax pattern. Obtain written consent from the patient. Use of mechanical condensation. (2) and (4) C. Obtain written consent from the personal C. low tear strength. A properly placed wedge. B. drug induced. 1. (2) and (4) D. Xerostomia can be A patient with Alzheimer’s dementia presents with his personal care worker. E. periapical granuloma. The NDEB periodically reviews the bank to improve its quality. is unavailable. daughter. D. D. 1. Note: Some of the items in the Released Test Item Bank may have been discontinued due to outdated science or errors. Obtain consent from the patient’s E. B. cemento-osseous dysplasia). (1) and (3) B. is required to obtain informed consent for an 3. (2) and (4) care worker. elective invasive procedure? 4. develop from excess dental follicle cells. veneer and the resin cement. (1) (2) (3) A. the format of some items is not currently used. (1) (2) (3) B. 3. A thinner matrix band. viscoelasticity. A silane coupling agent is used to Cementicles A. control polymerization shrinkage in A. 2. What 2. facilitate the soldering of gold castings. 4. 4. composite resins. A. E. In addition. Hydrocolloid impressions are removed from the mouth with a snap because they exhibit A. (4) only D. The patient’s presence implies consent. found in the elderly. are a response to trauma. associated with diabetes. (1) and (3) C. B.Root resorption of permanent teeth may be In order to achieve a proper interproximal associated with contact when using a spherical alloy. low elastic recovery. 2. (1) and (3) C. C. have the same composition as cementum. Dentigerous cyst. D. C. polyalkenoic acid cement. (4) only E. All of the above. adhesive resin cement. 4. sodium phosphate (Na3PO4) is the A. Radicular cyst. ε metal significantly higher than εceramic. terminator. In alginate impression materials. radiation therapy for oral cancer. cerebrospinal rhinorrhea. D. 3. Lateral periodontal cyst. B. thirteen years. E. A clinical sign of unilateral fracture of the body B. be completely formed by the age of C. of the zygoma is C. D. and translation. ε metal significantly lower than εceramic. cross linking agent. In addition. E. E. activator. D. C. Which of the following is most likely to B. displace the adjacent teeth? C. phosphate cement. phenolic cement. C. D. B. impaired hearing. Sjögren’s syndrome. ε metal equal to εceramic. B. resin modified glass ionomer cement. D. C. catalyst. disinfectant. antidepressant drug therapy. A. C. ©The National Dental Examining Board of Canada 2017 . (2) and (4) D. A. E. reactor. Periapical abscess. E. catalyst.The desirable relationship between the Xerostomia can result from coefficients of thermal expansion of an alloy (ε metal ) and a ceramic (εceramic) used for a metallo. Light is the A. seven years. ε metal slightly lower than εceramic. Periapical cemental dysplasia. B. retarder. Note: Some of the items in the Released Test Item Bank may have been discontinued due to outdated science or errors. B. A. ceramic restoration is 2. anticholinergics (Atropine). content. ε metal slightly higher than εceramic. the format of some items is not currently used. A. (1) and (3) E. 1. six years. Light-cured dental composites set when exposed to light. subconjunctival haemorrhage. otorrhea. nine years. The NDEB periodically reviews the bank to improve its quality. reactor. The roots of the first permanent molar should B. eleven years. A. (1) (2) (3) D. initiator. Zinc oxide eugenol cement is a/an A. D. mastoid. 40% more sensitive than ANSI D speed film. When compared to dental amalgams made To ensure maximum marginal strength for an from lathe cut particles. A.Which of the following principle fibre groups A rubber dam should be used in of the periodontal ligament is the most numerous and provides the main support for A. E. The purpose of a post and core restoration is to D. C. The NDEB periodically reviews the bank to improve its quality. retain the crown. prevent root discolouration. Transseptal. D. Ibuprofen. C. ANSI F speed film emulsion is approximately A. 60% more sensitive than ANSI D speed film. seal the root canal treatment. Horizontal. ©The National Dental Examining Board of Canada 2017 . B. In addition. D. lesions. Cyclosporine. C. 80% more sensitive than ANSI D speed patient’s risk for intraoral candidiasis? film. C. zygomatic. B. B. the tooth? B. All of the following are parts of the temporal In comparing ANSI D and ANSI F speed intra- bone EXCEPT the oral radiographic films. be beveled. dental amalgams made amalgam restoration the cavosurface angle from spherical particles should A. Pilocarpine. are more difficult to adapt to the cavity C. composite placement. D. Gingival. removing carious dentin from deep B. Oblique. B. Warfarine (Coumadin®). be chamfered. hamulus. amalgam placement. D. content. C. require higher condensation forces. Pentobarbital. B. approach 90 degrees. pulp capping procedures. Note: Some of the items in the Released Test Item Bank may have been discontinued due to outdated science or errors. Which of the following medications increases a D. set more quickly. approach 45 degrees. film. 20% more sensitive than ANSI D speed C. C. and translation. A. D. B. reinforce the remaining tooth structure. E. preparation. A. A. require more mercury. the format of some items is not currently used. all of the above. tympanic. A. resistance of the tooth to fracture. the symptoms reported by the patient. disinfection of the operatory. B. A. content. based on D.6 appears close to A. odontoblastic processes branch considerably at this region. protection from fluid contamination. can be stored for up to 7 days if placed in sterile bags after sterilization. A. the function of The single most important measure to reduce the post is to insure the risk of transmitting organisms to patients is A. B. C. introduction of single use instruments and disposables. proximity of the radiolucency to the pulp. A. the pulp on the bitewing radiograph. Unbagged sterilized instruments C. placement of a composite resin restoration is to C. D. distribution of forces along the long axis. ameloblasts make synaptic connections with odontoblasts at this junction. C. retention of the definitive restoration. must not be stored after sterilization. Bundle. contrast between the tooth and the operating field. Lymphocyte. ©The National Dental Examining Board of Canada 2017 . C. use of personal protective barriers: masks. odontoblastic tubules help convey hydrostatic forces to the pulp cells. sealing of the root canal. D. Woven. B. provide D. eyewear. handwashing. the format of some items is not currently used. Which of the following is the dominant C. access to the operating field. In addition. A. diagnosis of irreversible pulpitis can be made C. D. B. B. interproximal gingival retraction. Lamellar. D. free nerve endings terminate on odontoblasts at this region. Plasma cell. B. A B. The dentino-enamel junction is the most C.In a post-endodontic restoration. A. The NDEB periodically reviews the bank to improve its quality. a lower electric pulp test reading compared to the control. The principal use of the rubber dam during B. can be stored for up to 24 hours if placed in an airtight container after sterilization. inflammatory cell type in the initial lesion of gingivitis? A. and translation. Which of the following types of bone contain the insertions of the periodontal ligament fibres? A carious lesion on tooth 1. Macrophage. sterilization of instruments and D. Neutrophil. can be stored for up to 1 year if wrapped sensitive portion of a tooth because after sterilization. outerwear and gloves. Note: Some of the items in the Released Test Item Bank may have been discontinued due to outdated science or errors. Cortical. D. cardiac insufficiency. D. B. activates nociceptors. Uncured composite resins. C. D. Dental impression materials. C. 7th left intercostal space. will undergo normal root resorption. B. D. 9th left intercostal space. oxygen improves polymerization of the surface. Trituration of amalgam. lubrication function. the format of some items is not currently used. D. When light cured composite resin restorations B. influences the strength of metals more C. which of the following? B. should be extracted. determines a material’s fracture A. decreases with cyclic or fatigue loading. In addition. toughness. The apex of the heart lies deep to the A. The NDEB periodically reviews the bank to improve its quality. content. D. B. C. are polymerized C. antimicrobial effect. is not typically a function of a material’s processing. An enameloma is C. an odontoma. emphysema. should be treated endodontically to prevent root resorption. D. shrinkage decreases as conversion rate increases. causes vasoconstriction. cross-linking increases the stiffness of the A primary molar. fluoride concentration. rhinophyma. B.A drug inhibiting ATP release at a site of injury The size of pores or cracks in a material could be a potential analgesic because ATP A. 5th left intercostal space. A. B. You would suspect A. E. E. an ameloblastoma. is more susceptible to dental caries. A. A. asthma. 2nd left intercostal space. permanent successor. Saliva is most effective in minimizing an acid challenge by its Wettability is NOT a desirable property for A. inhibits nociceptors. E. C. D. ©The National Dental Examining Board of Canada 2017 . B. A. Note: Some of the items in the Released Test Item Bank may have been discontinued due to outdated science or errors. A patient has a history of shortness of breath and ankle edema. may remain for years with no significant resorption. 3rd left intercostal space. C. and translation. C. in the absence of its material. than ceramics. Hand instruments for placing resins. D. Acrylic resin for denture bases. B. buffering action. a pearl of enamel. prevents vasoconstriction. an enamel hypocalcification. a conversion rate of 80-95% is achieved. D. B. B. 2. D. 4. (4) only Dentinal pain is explained by E. molar sagittal relationship. heat. all of the above. B. (4) only Aspiration prior to a local anesthetic injection E. The NDEB periodically reviews the bank to improve its quality. A patient presents with a dislocated mandible D. removing carious dentin from deep 4. E. the format of some items is not currently used. (2) and (4) D. Which of the following statements is/are true when using forceps for extraction of a maxillary first molar? The occlusal parameter that is most useful to differentiate between an overbite of dental or 1. Palatal bone is thinner than buccal bone. hydrodynamic theory. In addition. C. reduces the A. cold. angular cheilitis. C. pain. (1) (2) (3) B. proprioception. D. C. mandibular anterior lack of space. All of the above. content. E. glossitis. the buccal direction. inject the joint with hydrocortisone. skeletal origin is the 2. A. sensation of C. (1) and (3) C. hydraulic pressure theory. Forcep movement should be principally in C. (1) (2) (3) B. erythematous oral mucosa. recommend mandibular movement be A. minimized. advise vigorous exercise of the mandible. composite placement. After reduction. the palatal direction. (1) and (3) C. All of the above.Oral signs and/or symptoms of vitamin B2 A rubber dam should be used in (riboflavin) deficiency may include A. A. pulp capping procedures. 3. lesions. 3. mandibular curve of Wilson. Buccal bone is easier to expand. B. A. the management of this patient should be to A. toxicity of local anesthetic. maxillary curve of Wilson. osmotic pressure theory. and translation. amalgam placement. mandibular curve of Spee. possibility of paresthesia. C. Note: Some of the items in the Released Test Item Bank may have been discontinued due to outdated science or errors. toxicity of vasoconstrictor. mechanical deformation theory. ©The National Dental Examining Board of Canada 2017 . Forcep movement should be principally in A. D. (2) and (4) D. possibility of intravascular administration. refer for joint surgery. 1. All afferent impulses from the pulp result in the B. pain. B. after an accident. D. The NDEB periodically reviews the bank to improve its quality. vigorous curettage of the socket. placement of a dressing in the socket. The extraction should be treat a patient due to delayed and A. to reduce the risk of developing Alzheimer’s disease. water coolant will B. C. ©The National Dental Examining Board of Canada 2017 . A. parotitis. reduce debris accumulation. Malocclusion. increase frictional heat. Diplopia. D. be indicative of A. C. B. an evaluation of liver function performed. B. a patient being mentally challenged.A patient who is a hepatitis B carrier presents It is ethical for a dentist in Canada to decline to for an extraction. Management of a “dry socket” should include Using a high speed handpiece WITHOUT A. on request from an informed patient. gingivitis. the patient referred to a hospital dental E. C. infection control. It is ethical to replace amalgam restorations The subgingival microbial flora isolated from A. C. an antibiotic prescribed prophylactically. and translation. rescheduled at the end of the day for B. hydrogen peroxide irrigation of socket. C. a prescription for antibiotics. flora of C. periradicular abscess. Spontaneous hemorrhage from the gingiva may E. a difference in religious beliefs. the dentist’s lack of skill or knowledge. a patient being physically challenged. In addition. B. reduce clogging of dental bur. a fracture of the mandible? D. D. the format of some items is not currently used. sites of peri-implantitis is most similar to the B. department. D. to eliminate toxins from the patient. Note: Some of the items in the Released Test Item Bank may have been discontinued due to outdated science or errors. C. D. B. the patient’s infectious status. Hodgkin’s disease. A. produce a smoother surface. diabetes. periodontitis. Bleeding from the nose. Which of the following may be associated with C. A. leukemia. D. decrease pulpal damage if used with light pressure. Swelling of the orbit. A. content. D. B. to relieve symptoms of multiple sclerosis. A 70 year old insulin-dependent patient has just Mercury content in an amalgam can be reduced completed a 7 day course of ampicillin for a by using respiratory infection. He presents with signs and symptoms consistent with a diagnosis of A. fast setting amalgam. oral candidiasis. Which of the following drugs B. adequate condensation. is/are appropriate to manage this condition? C. high zinc content alloy. D. low copper content alloy. 1. Fluconazole. 2. Nystatin. 3. Ketoconazole. 4. Clindamycin. Pins for cusp replacement should ideally be placed A. (1) (2) (3) B. (1) and (3) A. within enamel. C. (2) and (4) B. at the dentino-enamel junction. D. (4) only C. a minimum of 0.5-1.0mm from the E. All of the above. dentino-enamel junction. D. a minimum of 1.5-2.0mm from the dentino-enamel junction. Plaque accumulation is an etiologic factor in caries as it When light-cured composite resins are placed A. is a good source of fermentable carbohydrates. A. surface polymerization is inhibited by B. is an acidic environment. carbon dioxide (CO2). C. is composed of bacteria and their by- B. the degree of conversion is 85-95%. products. C. increments of resin should not exceed D. allows only S. mutans species to flourish. 2mm. D. polymerization shrinkage increases with filler content. Tooth 1.1 has a small fractured mesioincisal corner into dentin. Which of the following is true with respect to the preparation for the The protective role of junctional epithelium is restoration? aided by its increased number of A. Pins are usually necessary to achieve A. cell layers. adequate retention. B. intercellular spaces. B. A lingual dovetail is usually required for C. cell-cell adhesions. retention. D. collagen fibres. C. An enamel bevel of 1mm is placed where enamel thickness allows. D. Dentin coves are the primary form of retention for the restoration. Note: Some of the items in the Released Test Item Bank may have been discontinued due to outdated science or errors. In addition, the format of some items is not currently used. The NDEB periodically reviews the bank to improve its quality, content, and translation. ©The National Dental Examining Board of Canada 2017 In the keyhole model of the enamel prism One of the mechanisms of bacterial adherence to the dental pellicle is through A. crystallites in the tail are angled compared to the head. A. positively charged bacteria with the B. the water is located at the prism edges. pellicle. C. proteins are only present in the tail. B. interaction of cations such as calcium. D. crystallites have an identical molecular C. hydrophilic interactions. structure. D. interaction of salivary anions. The periodontal ligament is constantly GABA is a/an remodeled due to the activity of A. drug binding to benzodiazepine receptor. A. osteoclasts. B. neurotrophic factor. B. fibroblasts. C. inhibitory neurotransmitter. C. macrophages. D. excitatory neurotransmitter. D. mesenchymal cells. In neurons, glutamate is an amino acid that Odontoblast gap junctions binds only to A. adhere the cells to one another. A. nuclear receptors. B. attach the cells to the basement membrane. B. ionotropic receptors. C. seal off the dentin from the pulp. C. metabotropic receptors. D. permit lateral cell-cell communication. D. ionotropic and metabotropic receptors. When odontoblasts are destroyed as a result of The descending (or repolarizing) phase of the cavity preparation action potential is caused by A. adjacent fibroblasts differentiate into new A. opening potassium channels. odontoblasts. B. closing potassium channels. B. a permanent defect results in the C. opening chloride channels. odontoblast layer. D. closing calcium channels. C. surrounding odontoblasts are stimulated to divide. D. new cells differentiate from pulpal mesenchyme cells. Which source delivers the highest dose of radiation to humans? A. Cosmic. B. Medically-related. C. Radon. D. Terrestrial. Note: Some of the items in the Released Test Item Bank may have been discontinued due to outdated science or errors. In addition, the format of some items is not currently used. The NDEB periodically reviews the bank to improve its quality, content, and translation. ©The National Dental Examining Board of Canada 2017 Objects that absorb x-ray radiation are referred A 70 year old insulin-dependent patient has just to on a radiographic image as being completed a 7 day course of ampicillin for a respiratory infection. He presents with signs A. radiodense. and symptoms consistent with a diagnosis of B. radiolucent. oral candidiasis. Which of the following is the C. radiopaque. most appropriate management for this patient? A. Double the patient’s insulin dose. B. Reduce the patient’s insulin dose. Which form of hepatitis does NOT have a C. Prescribe a topical steroid. known carrier state? D. Prescribe clindamycin. E. Prescribe nystatin. A. Hepatitis A. B. Hepatitis B. C. Hepatitis C. D. Hepatitis D. The location and extent of subgingival calculus is most accurately determined clinically by A. radiopaque solution used in conjunction Which of the following describes the radiation with radiographs. produced by high voltage? B. disclosing solution. C. probing with a fine instrument. A. Short wavelength, low frequency. D. visual inspection. B. Short wavelength, high energy. C. Long wavelength, high frequency. D. Long wavelength, low energy. In an infrabony pocket, the epithelial attachment is located A 50 year old woman has a history of A. within basal bone. rheumatoid arthritis, bilateral enlargement of B. coronal to alveolar bone crest. one or more salivary glands and lacrimal C. apical to alveolar bone crest. glands, as well as dryness of the eyes, nose, mouth and throat. The diagnosis is A. erythema multiforme. The most likely diagnosis for a child with a B. Reiter's syndrome. painful, fiery-red, diffuse gingivitis is C. Gardner's syndrome. D. Sjögren's syndrome. A. primary herpetic gingivostomatitis. E. Plummer-Vinson syndrome. B. aggressive periodontitis. C. idiopathic fibromatosis. D. aphthous stomatitis. Note: Some of the items in the Released Test Item Bank may have been discontinued due to outdated science or errors. In addition, the format of some items is not currently used. The NDEB periodically reviews the bank to improve its quality, content, and translation. ©The National Dental Examining Board of Canada 2017 Which of the following is/are clinical signs of A protective mechanism of the dental pulp to gingivitis? external irritation or caries is the formation of 1. Loss of stippling. A. pulp stones. 2. Gingival hyperplasia. B. tertiary dentin. 3. Decreased pocket depth. C. secondary cementum. 4. Bleeding on probing. D. primary dentin. A. (1) (2) (3) B. (1) and (3) C. (2) and (4) Which of the following drugs is used in the D. (4) only treatment of mild allergic reactions? E. All of the above. A. Isoproterenol. B. Meperidine hydrochloride. C. Diphenhydramine hydrochloride. Regarding dental caries, which of the following D. Propoxyphene. is correct? A. All carbohydrates are equally cariogenic. B. More frequent consumption of When a radiographic examination is warranted carbohydrates increases the risk. for a 10 year old child, the most effective way C. The rate of carbohydrate clearance from to decrease radiation exposure is to the oral cavity is not significant. D. Increased dietary fat increases the risk. A. use a thyroid collar and lead apron. B. apply a radiation protection badge. C. use high speed film. D. decrease the kilovoltage to 50kVp. Lidocaine (Xylocaine®) is an example of a E. take a panoramic film only. local anesthetic which is chemically classified as an A. amide. If an alginate impression must be stored for a B. ester. few minutes before the cast is poured, it should C. aldehyde. be placed in D. ethamine. E. aminide. A. water. B. 100% relative humidity. C. a 1% aqueous calcium sulfate solution. Procaine (Novocaine®) is an example of a local anesthetic which is chemically classified as an An end result of ionizing radiation used to treat oral malignancies is A. amide. B. ester. A. deformity of the jaws. C. aldehyde. B. reduced vascularity of the jaws. D. ethamine. C. increased vascularity of the jaws. E. aminide. D. increased brittleness of the jaws. Note: Some of the items in the Released Test Item Bank may have been discontinued due to outdated science or errors. In addition, the format of some items is not currently used. The NDEB periodically reviews the bank to improve its quality, content, and translation. ©The National Dental Examining Board of Canada 2017 Which of the following is most often associated A 4 year old child has a normal complement of with a nonvital tooth? deciduous teeth, but in appearance they are grayish and exhibit extensive occlusal and A. Chronic periradicular periodontitis. incisal wear. Radiographic examination B. Internal resorption. indicates some extensive deposits of secondary C. Periapical cemento-osseous dysplasia. dentin in these teeth. This condition is typical D. Hyperplastic pulpitis. of A. cleidocranial dysplasia. B. amelogenesis imperfecta. Myxedema is associated with C. neonatal hypoplasia. D. dentinogenesis imperfecta. A. insufficient parathyroid hormone. B. excessive parathyroid hormone. C. insufficient thyroid hormone. D. excessive thyroid hormone. Root resorption of permanent teeth may be associated with 1. excessive orthodontic forces. Condensing osteitis in the periapical region is 2. chronic periradicular periodontitis. indicative of a/an 3. traumatic injury. 4. periapical cemento-osseous dysplasia. A. acute inflammation of the pulp. B. pulpal abscess. A. (1) (2) (3) C. chronic inflammation of the pulp. B. (1) and (3) D. early apical abscess formation. C. (2) and (4) D. (4) only A 15 year old presents with hypoplastic enamel E. All of the above. on tooth 1.5. All other teeth are normal. This was most probably caused by a/an A. vitamin D deficiency. Which of the following results from a necrotic B. generalized calcium deficiency. pulp? C. high fever encountered by the patient when he had measles at age 3. A. Dentigerous cyst. D. infection of tooth 5.5 during the B. Lateral periodontal cyst. development of tooth 1.5. C. Chronic periradicular periodontitis. E. hereditary factor. D. Pulp polyp. Which of the following features would be most indicative of a cracked tooth? A. Periapical radiolucency. B. Hypersensitivity to thermal stimuli. C. Pain upon biting pressure. D. Absent vitalometric response. Note: Some of the items in the Released Test Item Bank may have been discontinued due to outdated science or errors. In addition, the format of some items is not currently used. The NDEB periodically reviews the bank to improve its quality, content, and translation. ©The National Dental Examining Board of Canada 2017 Which of the following is/are associated with An ameloblastoma can develop from the an unerupted tooth? epithelial lining of which of the following cysts? 1. Odontogenic adenomatoid tumor. 2. Periapical cemento-osseous dysplasia. A. Periradicular. 3. Calcifying epithelial odontogenic tumor. B. Dentigerous. 4. Cementoblastoma. C. Residual. D. Lateral periodontal. A. (1) (2) (3) B. (1) and (3) C. (2) and (4) D. (4) only Radiographically, the opening of the incisive E. All of the above. canal may be misdiagnosed as a 1. branchial cyst. 2. nasopalatine cyst. An ankylosed tooth is usually 3. nasolabial cyst. 4. periradicular cyst. A. nonvital. B. associated with a root fracture. A. (1) (2) (3) C. infraerupted. B. (1) and (3) D. found in the permanent dentition. C. (2) and (4) D. (4) only E. All of the above. For which of the following pathological conditions would a lower central incisor tooth be expected to respond to heat, cold and On a bite-wing radiograph of posterior teeth, electric pulp test? which of the following is most likely to be misdiagnosed as proximal caries? A. Apical cyst. B. Acute apical abscess. A. Cemento-enamel junction. C. Periapical cemento-osseous dysplasia. B. Marginal ridge. D. Chronic apical periodontitis. C. Carabelli cusp. D. Calculus. E. Cemental tear. The microscopic appearance of the central giant cell granuloma of the jaws is similar to that of lesions which occur in A well circumscribed 3mm radiolucent lesion is present in the apical region of the mandibular A. hyperparathyroidism. second premolar. The tooth responds normally B. Paget's disease. to vitality tests. The radiolucency is most C. cleidocranial dysplasia. likely D. hyperpituitarism. A. a periradicular periodontitis. B. a dentigerous cyst. C. a rarefying osteitis. D. the mental foramen. Note: Some of the items in the Released Test Item Bank may have been discontinued due to outdated science or errors. In addition, the format of some items is not currently used. The NDEB periodically reviews the bank to improve its quality, content, and translation. ©The National Dental Examining Board of Canada 2017 A patient complains of acute pain 24 hours One week after an amalgam restoration is after the insertion of a restoration in a tooth placed in the mandibular first premolar, the with no preexisting periapical pathology. The patient returns complaining of a sharp pain of tooth is vital and tender to percussion. The short duration when eating or drinking radiograph will show something cold. Teeth respond normally to electric pulp testing and heat and the A. an apical radiolucency. radiographs are normal. The most likely B. acute osteitis. diagnosis is C. root resorption. D. condensing osteitis. A. hypercementosis. E. normal lamina dura. B. reversible pulpitis. C. pulpal microabscess. D. acute periradicular periodontitis. A cold stimulus applied to a tooth will produce a hypersensitive response if the tooth A 6 year old patient has a larger than average A. is nonvital. diastema between the maxillary central B. has a periodontal pocket. incisors. The radiographic examination shows a C. has a hyperemic pulp. mesiodens. In order to manage the diastema, D. has chronic proliferative pulpitis. you should extract the mesiodens A. after its complete eruption. B. once the patient has reached the age The most appropriate radiographic examination of 12. for a 4 year old without visible or clinically C. only if it develops into a cystic lesion. detectable caries or anomalies, and with open D. as soon as possible. proximal contacts is A. maxillary and mandibular anterior occlusals. In children, the most common cause of a fistula B. a pair of posterior bite-wings. is a/an C. maxillary and mandibular posterior periapicals. A. acute periradicular abscess. D. no radiographic examination. B. suppurative periradicular periodontitis. C. acute periodontal abscess. D. dentigerous cyst. Note: Some of the items in the Released Test Item Bank may have been discontinued due to outdated science or errors. In addition, the format of some items is not currently used. The NDEB periodically reviews the bank to improve its quality, content, and translation. ©The National Dental Examining Board of Canada 2017 A large carious exposure occurs on a Which of the following will impede healing permanent first molar of a 7 year old. There is following the surgical closure of an oroantral no periapical involvement and the tooth is vital. fistula? The treatment should be to 1. Poor flap design. A. cap the exposure with calcium hydroxide 2. Excessive tissue tension. and place zinc-oxide and eugenol. 3. Blowing the nose. B. perform a pulpotomy and place calcium 4. Sinus infection. hydroxide. C. perform a pulpectomy. A. (1) (2) (3) D. extract the tooth and place a space B. (1) and (3) maintainer. C. (2) and (4) D. (4) only E. All of the above. In a 4 year old child, the primary central incisor has discoloured following a traumatic injury. The treatment of choice is An 8 year old patient with all primary molars still present exhibits a cusp-to-cusp relationship A. pulpotomy. of permanent maxillary and mandibular first B. pulpectomy. molars. The management of this patient should C. observation. be to D. extraction. A. plan serial extractions for more normal adjustment of the occlusion. B. refer the patient to an orthodontist for The most appropriate treatment following the consultation. extraction of a first primary molar in a 4 year C. place a cervical headgear to reposition old child is maxillary molars. D. disk the distal surfaces of primary A. regular assessment of arch development. mandibular second molars to allow B. to perform space analysis. normal adjustment of permanent molars. C. insertion of a space maintainer. E. observe. D. extraction of the contra-lateral molar. E. extraction of the opposing molar. To prevent mesial drift of a permanent first molar, the ideal time to place a distal extension A Le Fort I or Guerin fracture is a space maintainer is A. fracture of the zygomatic arch. A. as soon as the tooth erupts through the B. horizontal fracture of the maxilla. gingival tissue. C. fracture of the malar complex involving B. after the permanent second molar has the floor of the orbit. erupted. D. pyramidal fracture of the maxilla. C. immediately after extraction of the E. craniofacial dysjunction. primary second molar. D. as soon as the extraction site of the primary second molar has completely healed. Note: Some of the items in the Released Test Item Bank may have been discontinued due to outdated science or errors. In addition, the format of some items is not currently used. The NDEB periodically reviews the bank to improve its quality, content, and translation. ©The National Dental Examining Board of Canada 2017 Epinephrine should NOT be used as a Which of the following conditions would NOT vasoconstrictor for patients with uncontrolled require antibiotic premedication before endodontic therapy? A. hyperthyroidism. B. hyperparathyroidism. A. Valvular heart disease. C. myxedema. B. Cardiac prosthesis. D. asthma. C. Persistent odontogenic fistula. D. Immunosuppressive therapy. E. Organ transplant. A known insulin-dependent diabetic patient feels unwell following the administration of a local anesthetic and becomes pale and sweaty. Under normal conditions, the most definitive This condition does not respond to placing the test to confirm the loss of pulp vitality is patient in a supine position. The most likely cause is A. applying warm gutta percha to the crown. B. cutting into the dentin without anesthetic. A. syncope. C. applying ethyl chloride to the crown. B. adrenal insufficiency. D. performing a radiographic examination of C. hyperglycemia. the tooth. D. hypoglycemia. E. performing an electric pulp test. E. carotid sinus reflex. A radiopaque area within the alveolar process Generally, glass ionomer cements contain containing several rudimentary teeth suggests a/an A. zinc oxide and distilled water. B. zinc oxide and polyacrylic acid. A. periapical cemento-osseous dysplasia. C. fluoroaluminosilicate powder and B. ameloblastoma. orthophosphoric acid. C. compound odontoma. D. fluoroaluminosilicate powder and D. complex odontoma. polyacrylic acid. E. Pindborg tumor. A fracture in an all-ceramic crown may be The recommended cavity access for a Class III caused by carious lesion is from the lingual because it 1. inadequate ceramic thickness. A. permits less removal of intact enamel. 2. sharp line angles in the tooth preparation. B. preserves the labial enamel for esthetic 3. excessive occlusal load. purposes. 4. use of an inappropriate luting material. C. is easier than from the vestibular. D. permits the use of a larger bur. A. (1) (2) (3) B. (1) and (3) C. (2) and (4) D. (4) only E. All of the above. Note: Some of the items in the Released Test Item Bank may have been discontinued due to outdated science or errors. In addition, the format of some items is not currently used. The NDEB periodically reviews the bank to improve its quality, content, and translation. ©The National Dental Examining Board of Canada 2017 Mesiobuccal cusp of the first molar. Distobuccal cusp of the second molar. A. The minimum time to wait before placing 2. Distobuccal cusp of the first molar. C. malalignment of teeth. (4) only E. 4. the format of some items is not currently used. 1 to 2 weeks. In an ideal Class I occlusion. incorrect vertical angulation of the x-ray composite restorations after the completion of a beam. 3mm. B. (2) and (4) D. D. osteoblasts. osteoclasts. the cusp of which mandibular tooth is in contact with the central fossa of the maxillary second molar? Elevated serum parathyroid hormone levels A. D. Mesiobuccal cusp of the second molar. composite resins can be used as a core material for endodontically treated teeth to A. ©The National Dental Examining Board of Canada 2017 .Composite resins bond to tooth structure Assuming there is adequate tooth structure through remaining. Van der Waals forces. A. A. the resin has a high contrast colour with tooth structure. (1) and (3) C. 4 to 5 weeks. chondroblasts. there is an adequate ferrule. The NDEB periodically reviews the bank to improve its quality. 2mm. 1mm. B. Overlapping contacts on a bitewing radiograph result from 1. content. the resin is autopolymerizing. A. 24 to 48 hours. B. chemical bonding. Note: Some of the items in the Released Test Item Bank may have been discontinued due to outdated science or errors. B. incorrect horizontal angulation of the x- ray beam. The minimal occlusal reduction for a porcelain D. C. micromechanical retention. be crowned provided B. C. C. (1) (2) (3) D. All of the above. C. and translation. A. B. subsequent crown margins are not located onlay is on cementum. patient movement during the exposure. osteocytes. bleaching (whitening) treatment is 3. result in Ca2+ being released from bone through B. In addition. receptor-mediated actions on C. 1 to 2 hours. B. Dismissal of the patient to have his breakfast. hypercementosis.Which of the following is characteristic of a Which of the following statements about maxillary sinus retention cyst/antral alternate sweeteners is INCORRECT? pseudocyst? A. the format of some items is not currently used.m. C. Saccharin alone is not carcinogenic. 175ml of diet cola. The NDEB periodically reviews the bank to improve its quality. the number of pulp stones found in first sun. difficulty getting to and from the dental A. In addition. intestinal mucosa upon absorption. B. C. office. his individuals from seeking dental treatment is speech becomes slurred and he is less alert than usual. A 50 year old obese patient with type 2 diabetes takes oral hypoglycemics. Which of the following is the most A. C. A. 15g of glucose as tablets or in solution. neurotransmitters in pulpal tissues. During his 8:30 a. B. B. and glutamic acid. Pain and soreness of the face. appointment. D. 20-35% 25-35% C. C. ©The National Dental Examining Board of Canada 2017 . the deposition of secondary and tertiary C. He is frequently skipping meals in order to reduce his The most common barrier preventing elderly weight. molars with advancing age. skin upon ultraviolet radiation from the A. 25-40% 20-30% Note: Some of the items in the Released Test Item Bank may have been discontinued due to outdated science or errors. 25-35% 20-35% D. D. Xylitol is a polyol with the same sweetness B. fear of being “hurt”. B. D. liver upon hydroxylation. a lack of perceived need for treatment. the cost associated with receiving appropriate management of this patient? treatment. 20-30% 20-30% B. radiograph. dentin. Cyclamate is not approved as a food additive. Aspartame is a dipepitide of aspartic acid C. Buccal expansion of the maxillary sinus. D. Dome-shaped appearance on a as sucrose. The Dietary Reference Intake (DRI) recommendation for fat is % Fat of Total Energy Intake Children (4-18 yr) Adult A. It may be possible to prepare a tooth for a restoration without anesthesia in an elderly Vitamin D is activated in the patient due to an increase in A. 175ml unsweetened orange juice. kidney upon hydroxylation. D. and translation. content. increased root sensitivity. Undercontoured restorations on the vestibular D. B. C. Mandibular block anesthesia. the format of some items is not currently used. needs to end within 1mm of the apex. A. disorder? D. have liver function tests performed to the post assess liver damage from a previous hepatitis B infection. ©The National Dental Examining Board of Canada 2017 . Fibrous dysplasia. worker should 4. B. and lingual surfaces of posterior teeth can immediately lead to A. value required for immunity. D. refrain from performing any exposure- prone procedures for a period of 3-6 months followed by a full series of hepatitis B vaccinations. retains the core. C. (2) and (4) vaccinations followed by post- D. gingival recession. Temporalis. (1) and (3) B. D. informed that their anti-HBsAg is less than the 2. deflection of food particles. A. D. C. structures on periapical radiographs EXCEPT C. Osteosarcoma. C. A. anterior nasal spine. Medial pterygoid. content. (1) (2) (3) followed by post-immunization testing. low post operative sinsitivity. C. receive one additional vaccination A. When restoring an endodontically treated tooth. The advantage of a high copper amalgam is A. All of the following appear as midline B. B. 35 year old patient with a severe bleeding C. increased polishability. Masseter.Which of the following bone lesions of the Post-immunization serological test results for a mandible is/are malignant? health care worker who has completed the series of vaccinations against hepatitis B is 1. nasal septum. zygomatic process of the maxilla. nasopalatine/incisive canal. high formation of the 2 phase. Osteochondroma. B. Note: Some of the items in the Released Test Item Bank may have been discontinued due to outdated science or errors. The health care 3. and translation. Incisional biopsy. Supragingival calculus removal. Which of the following muscles is a depressor of the mandible? Which of the following dental procedures A. could be performed with minimal risk for a B. B. All of the above. In addition. Subgingival restoration. A. undergo the full series of hepatitis B C. The NDEB periodically reviews the bank to improve its quality. Lateral pterygoid. D. high early strength. food impaction. E. Ewing's tumor. (4) only immunization testing. reinforces the root. gloves. With respect to the use of gloves in dentistry. gloves may be reused on the same patient What is the purpose of having a radiometer in a if they can be removed without dental office? contaminating the operator. utility gloves are appropriate for patient A. Osteopetrosis. sterile gloves should be used during a machine. C. appropriate restorative material for this lesion B. vestibular restoration. D. ©The National Dental Examining Board of Canada 2017 . The NDEB periodically reviews the bank to improve its quality. Primary personal protective equipment includes all of the following EXCEPT Which of the following is NOT a characteristic A. lip. masks. C. glove liners. Osteoporosis. and the patient does not want any mercury in their mouth. is a/an C. They are used as a dentin replacement. The most A. palate. A. and translation. microfill resin composite. B. risks in that patient’s medical history. D. gloves may be used on more than one B. E. gingiva. the format of some items is not currently used. Osteogenesis imperfecta. D. They are placed with minimal thickness. To track the number of x-rays taken in a care when no other options are available. D. D. C. C. They provide some type of therapeutic E. An 86 year old patient with poor oral hygiene has a cavitated lesion with active caries. A. B. buccal mucosa. resin-modified glass ionomer. content. tongue. occlusal restoration. To measure the output of the visible light. direct veneer restoration. B. E. C. month. Skeletal hyperostosis. In addition. mesiocclusal restoration. D.The effects of polymerization shrinkage of A patient has been taking a systemic composite resin must be taken into account for corticosteroid for 10 years. hybrid resin composite. of cavity liners? B. protective eyewear. To measure the wavelength of the x-ray D. A. A. B. benefit. surgical procedure. Which skeletal all the following EXCEPT the placement of disorder would the patient most likely have as a a/an result of this medication? A. They promote pulpal health. Osteoarthritis. patient as long as there are no identified curing unit. amalgam. C. protective clothing. D. C. Note: Some of the items in the Released Test Item Bank may have been discontinued due to outdated science or errors. B. To measure the wavelength of the visible light-curing unit. The A syphilitic gumma is most commonly found lesion is on the vestibular root surface of a on the maxillary posterior tooth. D. B. In addition. Early detection of a noncavitated smooth surface caries lesion allows for a A. A. steam. The NDEB periodically reviews the bank to improve its quality. dietary and insulin regimen. Insulin should be decreased prior to following operative procedures. Enamel staining. virus. The sterilization process effectively A. handled with latex gloves. bacteria. Hepatitis B.When managing patients with true latex A well-controlled type 1 diabetic requires an allergies. C. preventive regimen to be implemented to arrest the demineralization and remineralize. Creutzfeldt-Jacob disease is caused by (a) B. ambient air for a maximum of 10 minutes C. heated air. For sterilization to occur in an autoclave. D. Hepatitis C. C. and translation. Note: Some of the items in the Released Test Item Bank may have been discontinued due to outdated science or errors. conservative preparation for an amalgam restoration. A. C. B. the format of some items is not currently used. Hepatitis A. D. Latex protein antigens can exist in the treatment. content. fungus. Insulin should be increased prior to B. C. B. treatment. The patient should follow their normal contain latex-free materials. D. Which of the following is NOT associated with cancer chemotherapy in an adult? Which virus is the most likely to cause an A. Gingival bleeding. What is the most true? appropriate management for this patient? A. packaged instruments are subjected to pressurized A. Patients should be scheduled at the end of the day to avoid any exposure to latex products in the dental office. conservative preparation for a resin- modified glass ionomer restoration. Emergency kits should be available that D. conservative preparation for a composite resin restoration. Afternoon appointments should be removes latex proteins from instruments scheduled. infection in a healthcare worker following B. exposure to blood from an individual infected C. which of the following statements is MOD amalgam restoration. C. ©The National Dental Examining Board of Canada 2017 . boiling water. chemical vapour. prion. Human immunodeficiency virus. Candida infection. B. with the virus? D. the D. Dysgeusia. repeat the full series of hepatitis B vaccinations followed by post. curved roots. a prosthetic heart valve. content. size of the preparation. office immediately. D. Endodontic therapy is CONTRAINDICATED A dentist infected with Hepatitis C virus but in teeth with without disease symptoms should A. Crohn's disease. B. burnishing technique. the format of some items is not currently used. months followed by a full series of B. type of amalgam used. intact. B. D. constricted root canals. E. The health care B. accessory canals. and translation. place tooth under the tongue and come to patient with your office immediately. amount of amalgam used. hepatitis B vaccinations. value required for immunity. C. Note: Some of the items in the Released Test Item Bank may have been discontinued due to outdated science or errors. put the tooth in water and come to your there is compliance with standard office at the end of the day. E. place the tooth in milk and come to your A. worker should C. be allowed to practice but should be excluded from performing exposure prone procedures after assessment and agreement by an expert review panel and if there is A patient telephones and tells you he has just compliance with standard precautions knocked out his front tooth but that it is still (routine practices). An acute oral infection is of most concern in a D. C. Autoclave. D. otitis media. Your instructions should be to D. A. receive one additional vaccination followed by post-immunization testing. B. The NDEB periodically reviews the bank to improve its quality. pulp stones. precautions (routine practices). put the tooth in alcohol and come to your office immediately. refrain from performing any exposure- prone procedures for a period of 3-6 A. hepatitis B infection. B. A. C. have liver function tests performed to D. not be allowed to practice.Post-immunization serological test results for a The residual mercury content of an amalgam health care worker who has completed the restoration is significantly affected by series of vaccinations against hepatitis B reveals that their anti-HBsAg is less than the A. wrap the tooth in tissue and come to your office in a week's time. C. In addition. Flame sterilization. disinfect gutta-percha points? C. be allowed to practice after assessment and agreement by an expert review panel and if A. Chemical solutions. ©The National Dental Examining Board of Canada 2017 . Which of the following may be used to immunization testing. Glass bead sterilizer. excluded from performing exposure prone D. assess liver damage from a previous E. procedures. Dry heat sterilization. inadequate periodontal support. B. be allowed to practice but should be C. pemphigus. improves marginal adaptation. bifurcation area. chlorhexidine. 3. adapt the matrix to the gingival margin. 4. Note: Some of the items in the Released Test Item Bank may have been discontinued due to outdated science or errors. 1. 3. D. (1) and (3) distal inclines of maxillary cusps. 4. A. 2. capsules. (2) and (4) D. ©The National Dental Examining Board of Canada 2017 . None of the above. 1. endotoxins and enzymes. (4) only E. content. aid in the creation of a contact. (1) and (3) C. Microbial virulence factors A. preparation. (4) only A. (4) only E. diagnosis of acute dento-alveolar abscess? C. (1) (2) (3) D. mesial inclines of maxillary cusps and distal inclines of mandibular cusps. increases the thickness of gold. C. are caused only by Gram-positive tester. trifurcation area. (1) (2) (3) Accessory canals in permanent teeth are most B. B. All of the above. A positive reaction to percussion. are produced by non-pathogenic microbes. mesial inclines of mandibular cusps and B.For a cast gold restoration. (2) and (4) D. phenol. the format of some items is not currently used. C. A. a gingival bevel is In restoring occlusal anatomy. include exotoxins. 3. A. E. C. (1) (2) (3) B. separate the teeth. What clinical evidence would support a B. B. E. 4. absorb moisture. the protrusive used instead of a shoulder because a bevel condylar path inclination has its primary influence on the morphology of 1. are always pathogenic. A. apical third of the root. Sterilization of carious dentin without pulp injury is assured by the application of During matrix placement for a Class II cavity A. A negative reaction to the electric vitality D. In addition. 2. microbes. All of the above. 2. cervical third of the root. absolute alcohol. (1) and (3) commonly found in the C. C. and translation. 70% ethyl alcohol. cusp height. middle third of the root. anterior teeth only. increases retention. (2) and (4) D. E. a wedge is placed to B. The NDEB periodically reviews the bank to improve its quality. D. All of the above. A positive reaction of short duration to cold. protects the enamel. Presence of a draining fistula. C. (4) only B. B. a reversible pathologic condition. All of the above. Note: Some of the items in the Released Test Item Bank may have been discontinued due to outdated science or errors. small lingual lobe. fibrous elements. (1) and (5) E. (4) and (5) 1. (2) and (4) A. Lactobacilli. (1) (2) (3) D. following operative trauma. 3. B. D. C. prominent transverse ridge. zygomatic arch. multinucleated giant cells. Plasma cells. B. Eosinophils. A. more frequent following the use of air coolant rather than water coolant. (1) and (2) E. 5. vascularity. due to increased intrapulpal tissue pressure. A. due to contraction of collagen fibres. 2. Which of the following cells are characteristic 3. (1) (4) (5) D. undifferentiated mesenchymal cells. Lymphocytes. 1. of chronic inflammation of the dental pulp? 4. the occlusal dovetail of an ideal disto-occlusal amalgam preparation is usually not extended into the The masseter muscle originates from the mesial fossa because of the A. Streptococcus viridans. nerve tissue. (1) (2) (3) B. one of the first histological changes 2. When odontoblasts are destroyed or undergo degeneration. (1) (2) (3) 5. B. C. Neutrophils. Macrophages. large buccal cusp. and translation. D. ameloblasts. 4. 2. E. A. 1. content. Staphylococcus albus. (3) (4) (5) In the mandibular first premolar. angle of the mandible. pterygoid fossa. osteoblasts. large buccal pulp horn. In addition. C. (3) (4) (5) dentinal tubuli is thought to be D. D. 5.Which of the following microorganisms are Aging of the pulp is evidenced by a relative most frequently found in infected root canals? increase in A. 3. cell numbers. Staphylococcus aureus. coronoid process. the format of some items is not currently used. B. D. they are replaced by A. (1) and (3) C. calcification. ©The National Dental Examining Board of Canada 2017 . C. (1) and (3) Odontoblast nuclei displacement into adjacent C. The NDEB periodically reviews the bank to improve its quality. 4. Enterococci. its thermal diffusivity is similar to dentin and enamel. preservation of tooth structure. removal of all caries at the enamel-dentin junction.The physiologic rest position of the mandible is A tooth with a mild pulpitis should be sedated with 1. a fairly constant position throughout life. zinc-oxide-eugenol cement. D. B. D. D. used when making a centric interocclusal D. D. The most important principle dictating location and size of access to the root canal system is A. silicophosphate cement. 2. The NDEB periodically reviews the bank to improve its quality. In addition. C. Which of the following muscles comprise the B. A. All of the above. 4. B. (1) and (3) C. 2. removal of all pulp horns. Geniohyoids. the format of some items is not currently used. All of the above. Superior constrictor. dentin repair. (1) (2) (3) 1. 4. Temporalis. a position determined by the musculature. B. B. ©The National Dental Examining Board of Canada 2017 . C. A. (2) and (4) material beneath a metallic restoration because D. removal of all caries. Lateral pterygoids. polycarboxylate cement. A. D. C. use of calcium hydroxide. dimension. E. All of the above. All of the above. (1) (2) (3) B. content. Medial pterygoids. straight line access to the canal. composite resin. Note: Some of the items in the Released Test Item Bank may have been discontinued due to outdated science or errors. (4) only dependent upon E. (1) (2) (3) B. A. and translation. circulatory. Lateral (external) pterygoid. A. 3. Which of the following muscles contribute to C. (4) only 4. its thermal conductivity is similar to dentin and enamel. (1) and (3) Polycarboxylate cement may be used as a base C. a well sealed restoration. (2) and (4) The success of indirect pulp capping is D. Palatoglossus. Function(s) of the dental pulp include(s) A. All of the above. defensive. (4) only E. its compressive strength when set will the protrusion of the mandible? resist forces of condensation. 3. (1) and (3) 2. A. 1. record. C. Buccinator. used in determining occlusal vertical B. retromolar pad? C. sensory. (2) and (4) 3. content. B. (1) and (3) C. is best described as A. has a Factor VIII deficiency. C. 4. dysplasia. 3. tonsil. anesthetized for extraction of a maxillary increased mitosis. D. in the dental office for a patient who B. A. E. Nasopalatine. Sphenopalatine. The greatest single factor in reducing radiation C. This patient should have Extraction of a tooth is CONTRAINDICATED A. idiopathic fibromatosis. sphenopalatine neuralgia. D. B. had a myocardial infarct two months ago. (2) and (4) pharynx is most likely diagnostic of D.Which of the following nerves should be Hyperkeratosis. is 4 months pregnant. Ménière's disease. A. B. All of the above. Note: Some of the items in the Released Test Item Bank may have been discontinued due to outdated science or errors. The NDEB periodically reviews the bank to improve its quality. In addition. Anterior superior alveolar. squamous cell carcinoma. re-excision with wider margins. 4. hemisection of the tongue. C. endothelioma. C. 2. 2. diameter on the lateral border of the tongue was diagnosed as a fibroma. degeneration. D. radiotherapy to site of biopsy. C. is hypothyroid. intact basal cell layer and lateral incisor? chronic inflammatory cells are histologic features that may be found in 1. no additional therapy. D. (2) and (4) D. carcinoma in situ. B. 1. All of the above. hyperplasia. higher kVp. inflammation. acanthosis. high speed film. A. (1) and (3) part of the tongue. and translation. described as E. neoplasia. trigeminal neuralgia. psychotic glosso pyrosis. (1) (2) (3) B. 3. (4) only E. ©The National Dental Examining Board of Canada 2017 . nasopharynx and C. radium implantation around biopsy site. exposure in dentistry is D. (1) (2) (3) Lancinating paroxysmal pain in the posterior B. A. glossopharyngeal neuralgia. Enlargement of the gingiva. collimation of the X-ray beam. Nasociliary. papillofibroma. (4) only An excisional biopsy of a nodule 5mm in E. the format of some items is not currently used. A. proper filtration. cardiac arrhythmia. the most likely diagnosis is 1. The NDEB periodically reviews the bank to improve its quality. A. Retromolar area. asthma. B. When a patient experiences continuous pain in In hyperparathyroidism. C. typical features of the maxillary premolar and molar areas and bone involvement are there is no evidence of dental infection. C. B. B. A. mandibular division of cranial V. long buccal nerve. 4. trigeminal neuralgia. Hard palate. Serum phosphorus. Serum alkaline phosphatase. C. adenoma. D. C. E. D. renal stones. impacted maxillary canine. peg lateral incisors. Fordyce's granules. impacted maxillary third molar. Gingiva. In addition. rheumatic heart disease. (1) (3) (4) E. B. pathological fractures. content. (2) and (4) D. C. A. and translation. supernumerary incisors. blue sclerae. The benign neoplasm that originates from Erythroblastosis fetalis may be a cause of squamous epithelium is called a/an A. Thyroid activity. osteopetrosis. (1) and (3) C. ©The National Dental Examining Board of Canada 2017 . choriocarcinoma. 3. Note: Some of the items in the Released Test Item Bank may have been discontinued due to outdated science or errors. the format of some items is not currently used. acute maxillary sinusitis. E. Loss of taste to the anterior two thirds of the tongue and a lack of secretion of submandibular glands indicates nerve damage A decrease of which of the following is to the indicative of hypoparathyroidism? A. subperiosteal erosion of the phalanges. All of the above. D. Buccal mucosa. pigmented teeth. A. D. C. cranial VII. E. 2.Which of the following sites for squamous cell A patient who uses nitroglycerine has carcinoma has the best prognosis? A. B. A. C. B. high blood pressure. D. B. Lower lip. coronary artery disease. E. glossopharyngeal neuralgia. lipoma. chorda tympani nerve. Serum calcium. chondroma. B. papilloma. D. D. The apical region of a non-vital tooth with a In the presence of an acute bacterial infection, deep carious lesion may radiographically show laboratory tests will show an increase in 1. widening of the periodontal space. A. polymorphonuclear leukocytes. 2. loss of lamina dura. B. plasma cells. 3. a circumscribed radiolucency. C. lymphocytes. 4. calcification of the periodontal membrane. D. monocytes. E. eosinophils. A. (1) (2) (3) B. (1) and (3) C. (2) and (4) D. (4) only A patient presents with apparent paralysis of E. All of the above. one side of the face which appeared the day before. What is the most likely diagnosis? A. Glossodynia. An increased heart rate may be associated with B. Bell's palsy. C. Myasthenia gravis. A. hypothyroidism. D. Trigeminal neuralgia. B. prolonged corticosteroid therapy. C. hyperthyroidism. D. Down syndrome. Sickle cell anemia is A. a genetic disease. Selection of the appropriate kilovoltage for B. caused by exposure to radiation. dental films is influenced by C. a viral infection. D. a drug reaction. A. line voltage fluctuation. E. an auto-immune disease. B. diameter of the primary beam of radiation. C. type of timer. D. tissue density. Metastasis is most likely to occur in E. filter thickness. A. squamous cell carcinoma. B. basal cell carcinoma. C. ameloblastoma. In the early stage, a periradicular abscess can D. complex odontoma. be differentiated from a lateral periodontal E. odontogenic fibroma. abscess by A. pain. B. type of exudate. An ameloblastoma is most frequently found in C. tenderness to percussion. D. response of pulp to electrical stimulation. A. the anterior region of the maxilla. E. radiographic examination. B. the mandible, near the junction of the body and the ramus. C. the posterior region of the maxilla. D. in the anterior region of the mandible near the midline. Note: Some of the items in the Released Test Item Bank may have been discontinued due to outdated science or errors. In addition, the format of some items is not currently used. The NDEB periodically reviews the bank to improve its quality, content, and translation. ©The National Dental Examining Board of Canada 2017 Which gingival manifestation(s) would be The most logical explanation for causing expected in a patient with a blood dyscrasia? swelling beneath the eye caused by an abscessed maxillary canine is that the 1. Enlargement. 2. Bleeding. A. lymphatics drain superiorly in this region. 3. Ulceration. B. bone is less porous superior to the root 4. Atrophy. apex. C. infection has passed into the angular vein A. (1) (2) (3) which has no valves. B. (1) and (3) D. the root apex lies superior to the C. (2) and (4) attachment of the caninus and levator labii D. (4) only superioris muscles. E. All of the above. The finding of “acid-fast” microorganisms in Mucoceles are most commonly found in the sputum suggests the presence of A. upper lip. A. Mycobacterium tuberculosis. B. lower lip. B. Diplococcus pneumoniae. C. tongue. C. Streptococcus pyogenes. D. buccal mucosa. D. Neisseria gonorrhoeae. E. soft palate. Excessive formation of scar tissue beyond the Multiple supernumerary teeth are most wound margin is called commonly found in A. a fibroma. A. cherubism. B. a keloid. B. cretinism. C. a fibro-epithelial polyp. C. hypothyroidism. D. epithelial hyperplasia. D. cleidocranial dysplasia. E. Down's syndrome. A clinical finding common to alcoholism, poorly-controlled diabetes mellitus, uremia and Which of the following is NOT a sign or liver disease is symptom of the myofascial pain dysfunction syndrome? A. smooth tongue. B. increased blood pressure. A. Pain. C. coated tongue. B. Muscle tenderness. D. labial fissures. C. Limitation of jaw motion. E. halitosis. D. "Clicking" or "popping" noise in the joints. E. Radiographic changes of the joint. Note: Some of the items in the Released Test Item Bank may have been discontinued due to outdated science or errors. In addition, the format of some items is not currently used. The NDEB periodically reviews the bank to improve its quality, content, and translation. ©The National Dental Examining Board of Canada 2017 In radiography, minimum magnification and Which of the following is/are NOT usually maximum definition are achieved by affected by hereditary ectodermal dysplasia? A. minimum OFD (object-film distance) and A. Salivary glands. minimum FFD (focal-film distance). B. Teeth. B. minimum OFD (object-film distance) and C. Sweat glands. maximum FFD (focal-film distance). D. Hair. C. maximum OFD (object -film distance) E. Fingernails. and maximum FFD (focal-film distance). D. maximum OFD (object-film distance) and minimum FFD (focal-film distance). The term used to describe epithelial changes including nuclear hyperchromatism, loss of increased nuclear to cytoplasmic ratio and The cells responsible for antibody production abnormal mitoses is are called A. acanthosis. A. polymorphonuclear leukocytes. B. hyperkeratosis. B. mast cells. C. dysplasia. C. plasma cells. D. parakeratosis. D. macrophages. E. hyperparakeratosis. E. megakaryocytes. Which one of the following would be of Intermittent painful swelling in the greatest value in determining the etiology of an submandibular region that increases at oral ulceration? mealtime is indicative of A. History of the oral lesion. A. a ranula. B. Cytological smear. B. a blockage of Wharton's duct. C. Systemic evaluation. C. Ludwig's angina. D. Laboratory tests. D. a blockage of Stensen's duct. E. an epidemic parotitis. A patient suddenly becomes pale and sweaty after an injection of 4ml of lidocaine 2% with A patient with a white blood cell count of epinephrine l:l00,000. The radial pulse is slow 34,000/mm3 has a differential of and steady. The respiration is slow. The blood lymphocytes 62%, lymphoblasts 4% and pressure is 80/60. What is the most probable polymorphonuclear leukocytes 34%. The most diagnosis? likely form of leukemia is A. A toxic reaction to lidocaine. A. aleukemic. B. A toxic reaction to epinephrine. B. granulocytic. C. An allergic reaction to the local C. monocytic. anesthetic. D. lymphocytic. D. Incipient syncope. E. plasma cell. E. An impending adrenal insufficiency. Note: Some of the items in the Released Test Item Bank may have been discontinued due to outdated science or errors. In addition, the format of some items is not currently used. The NDEB periodically reviews the bank to improve its quality, content, and translation. ©The National Dental Examining Board of Canada 2017 A periapical infection of a mandibular third Bacterial infection may be confirmed by molar may spread by direct extension to the 1. white blood cell count. 1. parapharyngeal space. 2. hemoglobin level. 2. submandibular space. 3. erythrocyte sedimentation rate. 3. pterygomandibular space. 4. platelet count. 4. submental space. A. (1) (2) (3) A. (1) (2) (3) B. (1) and (3) B. (1) and (3) C. (2) and (4) C. (2) and (4) D. (4) only D. (4) only E. All of the above. E. All of the above. A 20-year old male presents with a three-day The most likely complication associated with history of an acute generalized gingivitis. He the extraction of an isolated maxillary second has malaise, fever and bilateral cervical molar is lymphadenopathy. A blood examination reveals A. a dry socket. Hb: 8.9g/100ml B. nerve damage. Platelets: 82,000/mm3 C. fracture of the malar ridge. Red blood cell count: 3,900,000/mm3 D. fracture of the tuberosity. White blood cell count: 870,000/mm3 Normal Values: Hb: 14-18g/100ml A patient presenting with diplopia, Platelets: 150,000-400,000/mm3 exophthalmos, nasal bleeding and swelling, Red blood cell count: 4-5million/mm3 may suffer from a fracture of the White blood cell count: 5,000-10,000/mm3 A. neck of the condyle. The most likely diagnosis is B. body of the mandible. C. zygomatic bone. A. thrombocytopenic purpura. D. maxillary tuberosity. B. acute myelogenous leukemia. C. infectious mononucleosis. D. acute necrotizing ulcerative gingivitis. Reduced mobility of the temporomandibular joint is called Ludwig's angina may cause death by A. Charcot's arthritis. B. osteoarthritis. A. heart failure. C. ankylosis. B. asphyxia. D. arthrosis. C. convulsions. D. paralysis of muscles of respiration. E. pyemia. Note: Some of the items in the Released Test Item Bank may have been discontinued due to outdated science or errors. In addition, the format of some items is not currently used. The NDEB periodically reviews the bank to improve its quality, content, and translation. ©The National Dental Examining Board of Canada 2017 If an odontogenic infection involves the The most common complication of a pterygomandibular space, the most obvious venipuncture is clinical sign will be A. syncope. A. trismus. B. hematoma. B. facial swelling. C. thrombophlebitis. C. swelling in the submandibular area. D. embolus. D. rise in body temperature above 39C (102ºF). Antihistamines act by It is difficult to obtain satisfactory anesthesia in A. increasing the action of histaminase. the presence of infection near the injection site B. altering the formation of histamine. because C. blocking the actions of histamine by competitive inhibition. A. the swelling causes increased pressure on D. interfering with the degradation of the nerves. histamine. B. increased blood supply carries the anesthetic solution away too fast. C. acidity of the infected tissue inhibits action of the anesthetic agent. Tetracyclines D. alkalinity of the infected tissue inhibits action of the anesthetic agent. 1. have no side effects. 2. may increase susceptibility to superinfections. 3. are safe to use during pregnancy. Which of the following drugs are classic 4. have a wide spectrum of antibacterial antagonists for curare over-dosage? activity. A. Anticholinesterases. A. (1) (2) (3) B. Ganglionic stimulants. B. (1) and (3) C. Ganglionic blocking agents. C. (2) and (4) D. Alpha adrenergic blocking agents. D. (4) only E. Beta adrenergic blocking agents. E. All of the above. The major stimulator of respiration is Which valve is most commonly affected by rheumatic heart disease? A. low blood pressure. B. high percentage of blood oxygen. A. Aortic. C. low percentage of blood carbon dioxide. B. Pulmonary. D. high percentage of blood carbon dioxide. C. Tricuspid. D. Mitral. Note: Some of the items in the Released Test Item Bank may have been discontinued due to outdated science or errors. In addition, the format of some items is not currently used. The NDEB periodically reviews the bank to improve its quality, content, and translation. ©The National Dental Examining Board of Canada 2017 A therapeutic advantage of penicillin V over All of the following are possible effects of penicillin G is acetylsalicylic acid except A. greater resistance to penicillinase. A. reduction of fever. B. broader antibacterial spectrum. B. shortening of bleeding time. C. greater absorption when given orally. C. suppression of inflammatory response. D. slower renal excretion. D. bleeding from the gastrointestinal tract. E. None of the above. Which of the following does NOT influence Trismus is most frequently caused by the rate of induction during inhalation anesthesia? A. tetanus. B. muscular dystrophy. A. Pulmonary ventilation. C. infection. B. Blood supply to the lungs. D. mandibular fracture. C. Hemoglobin content of the blood. D. Concentration of the anesthetic in the inspired mixture. E. Solubility of the anesthetic in blood. Acetaminophen in therapeutic doses 1. retards platelet function. 2. has strong anti-inflammatory properties. Which of the following would you prescribe for 3. produces CNS stimulation. an anxious dental patient with a peptic ulcer? 4. has antipyretic properties. A. Reserpine. A. (1) (2) (3) B. Scopolamine. B. (1) and (3) C. Silica gel. C. (2) and (4) D. Diazepam. D. (4) only E. Calcium carbonate. E. All of the above. Unconsciousness in syncope results from Short-acting barbiturates are metabolized mainly in the A. electrolyte imbalance. B. neurogenic shock. A. liver. C. cerebral hyperemia. B. kidneys. D. cerebral hypoxia. C. small intestine. D. pancreas. E. spleen. The inorganic ion that is implicated in primary hypertension is A. sodium. B. fluoride. C. potassium. D. magnesium. Note: Some of the items in the Released Test Item Bank may have been discontinued due to outdated science or errors. In addition, the format of some items is not currently used. The NDEB periodically reviews the bank to improve its quality, content, and translation. ©The National Dental Examining Board of Canada 2017 In a standard dental cartridge (carpule) The chief mechanism by which the body containing 1.8ml 2% lidocaine with metabolizes short-acting barbiturates is epinephrine 1/100,000, the amount of vasoconstrictor is A. oxidation. B. reduction. A. 18.0 mg. C. hydroxylation and oxidation. B. 0.018 mg. D. sequestration in the body fats. C. 1.8 mg. D. 0.18 mg. E. 180.0 mg. Early anoxia is characterized by 1. cyanosis. Vestibuloplasty is a preprosthetic surgical 2. bradycardia. procedure used to 3. tachycardia A. facilitate reliable impression making. A. (1) only B. provide adequate posterior inter-arch B. (1) and (2) space. C. (1) and (3) C. allow placement of teeth over the residual D. All of the above. ridge. D. increase the supporting surface area. An acute periapical abscess originating from a mandibular third molar generally points and In a standard inferior alveolar nerve block, drains in the which muscle is penetrated by the needle? A. submandibular space. A. Buccinator. B. pterygomandibular space. B. Mylohyoid. C. buccal vestibule. C. Superior constrictor. D. buccal space. D. Masseter. E. Medial (internal) pterygoid. When sutures are used to reposition tissue over extraction sites, they should be A 57 year old man received 10mg of diazepam intravenously. He becomes unresponsive to 1. placed over firm bone where possible. verbal stimuli, and his respirations are 2. interrupted, 15mm apart. depressed to 10 per minute. Appropriate 3. firm enough to approximate tissue flaps treatment is to without blanching. 4. tight enough to produce immediate A. administer ephedrine. hemostasis. B. observe the patient. C. force the patient to drink coffee. A. (1) (2) (3) D. support respiration with oxygen. B. (1) and (3) C. (2) and (4) D. (4) only E. All of the above. Note: Some of the items in the Released Test Item Bank may have been discontinued due to outdated science or errors. In addition, the format of some items is not currently used. The NDEB periodically reviews the bank to improve its quality, content, and translation. ©The National Dental Examining Board of Canada 2017 What is the maximum number of A radiographic examination of a 10 year old cartridges (1.8ml) of a 2 local anesthetic child reveals retention of deciduous teeth and solution that can be administered without presence of many unerupted supernumerary exceeding a total dose of 300mg? teeth. This is characteristic of A. 2. A. cleidocranial dysplasia. B. 4. B. ectodermal dysplasia. C. 6. C. dentinogenesis imperfecta. D. 8. D. congenital hypothyroidism. E. 10. In cephalometry, the most stable point in a After an inferior alveolar nerve block injection, growing skull is the a patient would develop seventh nerve paralysis if the injection was made into the A. sella turcica. B. nasion. A. internal maxillary artery. C. Broadbent's point. B. retroparotid space. D. Bolton point. C. internal pterygoid muscle. D. retromandibular vein. E. pterygoid plexus of veins. The radiographic appearance of internal resorption is In primary molars, radiographic bony changes A. radiolucent enlargement of the pulp from an infection are initially seen cavity. B. radiolucency around the apex of the root. A. at the apices. C. radiolucency on the surfaces of the root. B. in the furcation area. D. localized radiopacities in the pulp cavity. C. at the alveolar crest. E. radiopacity around the apex of the root. D. at the base of the developing tooth. Roots of the permanent maxillary central The highest incidence of congenitally missing incisors are completed by what age? lateral incisors is most likely seen in a patient with A. 8 years. B. 10 years. A. unilateral cleft lip and palate. C. 12 years. B. congenital heart disease. D. Later than 12 years. C. Down's syndrome. D. hyperthyroidism. The organisms associated with a carious pulpitis are A. streptococci. B. staphylococci. C. spirochetes. D. viruses. Note: Some of the items in the Released Test Item Bank may have been discontinued due to outdated science or errors. In addition, the format of some items is not currently used. The NDEB periodically reviews the bank to improve its quality, content, and translation. ©The National Dental Examining Board of Canada 2017 Mandibular growth The roots of primary molars in the absence of their permanent successors A. is sustained over a longer period of time in girls. 1. sometimes are partially resorbed and B. is sustained over a longer period of time become ankylosed. in boys. 2. may remain for years with no significant C. occurs at the same chronologic age in resorption. both sexes. 3. may remain for years partially resorbed. D. occurs two years earlier in boys than in 4. are always resorbed. girls. A. (1) (2) (3) B. (1) and (3) C. (2) and (4) Hypothyroidism affects the dental D. (4) only developmental pattern by E. All of the above. A. interfering with jaw growth. B. delaying the eruption timetable. C. causing sclerotic bone to form over the A Class II dental occlusion in the mixed occlusal surface of erupting teeth. dentition will likely D. accelerating the eruption timetable. A. develop into a Class I occlusion after normal exfoliation of the primary molars. B. worsen with forward growth of the A single tooth anterior crossbite found in a maxilla. 9 year old should C. develop into a Class I occlusion with late mandibular growth. A. self-correct. D. develop into a skeletal malocclusion with B. be treated with a removable appliance. growth of the maxilla and mandible. C. have 2 arch orthodontic treatment. E. not change as the maxilla and mandible D. be treated in the complete permanent grow. dentition. E. be observed and treated when the cuspids have erupted. Alveolar bone is undergoing remodeling A. through the primary dentition. Mandibular condylar region grows by B. until the end of mixed dentition. C. until the complete eruption of permanent A. sutural and interstitial proliferation. teeth. B. interstitial and appositional proliferation. D. throughout life. C. appositional and sutural proliferation. D. interstitial proliferation only. E. appositional proliferation only. Note: Some of the items in the Released Test Item Bank may have been discontinued due to outdated science or errors. In addition, the format of some items is not currently used. The NDEB periodically reviews the bank to improve its quality, content, and translation. ©The National Dental Examining Board of Canada 2017 molars. The NDEB periodically reviews the bank to improve its quality. relationship of the maxilla to the hinge deciduous maxillary second molar. A. C. (4) only A facebow is used to record the E. premolars. affected by fluoride. D. A 3 year old requires the extraction of a 4. The An exchange of calcium ions between saliva likely cause is and enamel is A. 4. buccal and palatal infiltration. C. D. 3. ankylosis of the mandibular second 1. B. translatory growth. anesthetic technique of choice is A. E. (1) and (3) B. E. D. the permanent first molars are in contact with the first premolars and the crowns of the second premolars have erupted lingually. tetracycline therapy. 3. heredity. 1. teeth too large for the dental arch. important in maintenance of tooth D. thumbsucking. Note: Some of the items in the Released Test Item Bank may have been discontinued due to outdated science or errors. E. a posterior superior alveolar block. cortical drift. a component of remineralization and B. The local axis. (2) and (4) C. inter-occlusal relationship. demineralization. 2. All of the above. area relocation. 2. ©The National Dental Examining Board of Canada 2017 . vertical dimension of occlusion. All of the above. C. B. (1) (2) (3) B. B. endocrine deficiency. (1) and (3) C. (4) only. C. A. ectopic eruption. trauma to the maxillary primary central incisor. a tuberosity block plus subperiosteal D. remodeling. The mechanism of adjustment to maintain the shape and proportions of bone throughout its growth period is called A. (1) (2) (3) A.A single hypoplastic defect located on the The most frequent cause of malocclusion is labial surface of a maxillary central incisor is most likely due to a/an A. In the mandibular dental arch of a 12-year old boy. faulty lingual eruption of the second premolars. infiltration of the mesio-buccal root. (2) and (4) D. mouth breathing. and translation. In addition. B. dietary deficiency. content. the format of some items is not currently used. premature loss of deciduous second structure. C. an infra-orbital block. D. high fluoride intake. horizontal condylar inclination. lack of space. pH dependent. Which of the following foods is the most C. A. is normally non-keratinized but can become keratinized in response to physiological stimulation. interprismatic substances. is closely bound to underlying muscle and bone. Lymphocytes. epithelial keratinization. Ulceration. spirochetes. alveolar bone D. A. A. study of disease patterns in a population. 2. (4) only The enamel structures most resistant to the E. (1) (2) (3) B. 4. The oral mucosa covering the base of the C. and translation. (2) and (4) D. Atrophic thinning. lamellae. diplococcal organisms. cuticles. All of the above. Polymorphonuclear leukocytes. Mast cells. B. The NDEB periodically reviews the bank to improve its quality. E. B. Macrophages. Cheese. (1) (2) (3) B. the sulcus E. mucogingival junction. vascularity of the gingiva. Note: Some of the items in the Released Test Item Bank may have been discontinued due to outdated science or errors. cariogenic? D. D. merges with the keratinized gingiva at the 3. rods. thickness of the epithelium. gram-positive organisms. 1. (2) and (4) D. control of disease. gram-negative organisms. A. C. the format of some items is not currently used. within a population. has a tightly woven dense collagenous corium. B. Jam. Necrosis. C. Spontaneous bleeding. C. D. the largest numbers in response to the D. content. (1) and (3) C. D. becomes predominantly populated by A. accumulation of plaque? A. usual low level of disease normally found 3. action of acids are A. The colour of normal gingiva is affected by the B. E. (1) and (3) C. Dark chocolate.Which oral mucosa changes are possible side Epidemiology of disease is best described as effects of chemotherapy? the 1. does not contain elastic fibres. B. (4) only With the development of gingivitis. ©The National Dental Examining Board of Canada 2017 . Which cells migrate into the gingival sulcus in C. melanin pigmentation. All of the above. A. In addition. data obtained from sickness surveys. Toffee. 2. B. 4. Plasma cells and monocytes. D. Incisal dentin. Pits and fissures. increasing the resistance of dentin to C. horizontally. dentinogenesis imperfecta. vitamin A Caries in older persons is most frequently deficiency may result in found on which of the following locations? A. D. D. B. D. Hutchinson's incisors. pellicle. providing a more favorable pulpal blood supply. B. vertically. is derived from mast cells. C. to demineralization. Dietary deficiency of vitamin D can result in A. C. B. E. the format of some items is not currently used. partial anodontia (hypodontia). causing tooth enamel to be more resistant E. and translation. increasing plaque retention. B. Maximum shrinkage after gingival curettage can be expected from tissue that is Overhangs on restorations initiate chronic inflammatory periodontal disease by A. C. D. fibrotic. myositis ossificans. A. enamel hypoplasia. materia alba. osteitis fibrosa cystica. ©The National Dental Examining Board of Canada 2017 . During tooth development. A. causing pressure atrophy. In addition. D. A. obliquely. D. osteogenesis imperfecta. B. laterally. C. abnormal formation of osteoid. C. peg-shaped teeth. C. E. Paget's disease. and D. content. C. plaque. bacterial penetration. B. directed to a tooth B. B. Proximal enamel. causing traumatic occlusion. E. primary cuticle. is a transudate. D. associated with exudate formation. Nasmyth's membrane. All of the above. Note: Some of the items in the Released Test Item Bank may have been discontinued due to outdated science or errors. increasing food retention.Gingival crevicular fluid Fluorides are effective in the prevention of dental caries by A. never varies in volume. edematous. C. fibroedematous. formed within an infrabony pocket. A. B. the new mucoprotein coating which forms on the surface is called The periodontium is best able to tolerate forces A. is an exudate. The NDEB periodically reviews the bank to improve its quality. Root surfaces. E. B. After a tooth surface has been completely cleaned. D. Cheese. patient has D. on the surface of the clinical crown. D. Canned fruit. A. marginal gingivitis. The NDEB periodically reviews the bank to improve its quality. In addition. B. dental needs. C. A. White bread. osteoplasty. Abrasion is most commonly seen on the D. eliminated by D. craters in the interseptal bone are best C. Potatoes. dental caries. a high lactobacillus count. In patients with advanced periodontitis. C. Gingivoplasty. tooth mortality. Connective tissue graft. Gingival graft. B. D. generalized recession. periodontal surgery. lactic acid in his mouth. E. mobile DMF-S is an index for expressing teeth should be splinted in order to A. Fruit jello. E. C. Irregularly distributed shallow to moderate B. and translation. saliva with low buffering capacity. enhance formation of a new connective tissue attachment after surgery. A. Which of the following foods is LEAST D. A. A clenching habit may be a factor in A. at the dentino-enamel junction. cariogenic? A. incisal edges. B. extent of dental neglect. at the layer of dentin nearest the pulp D. facial surfaces of teeth. C. occlusal surface of posterior teeth. D. B. content. bone grafting. ©The National Dental Examining Board of Canada 2017 . None of the above. B. A. D. Lateral sliding flap. No treatment. B. the format of some items is not currently used. B. chamber. gingivoplasty. Note: Some of the items in the Released Test Item Bank may have been discontinued due to outdated science or errors. suprabony periodontal pocket formation. C. increased tooth mobility. lingual surface of posterior teeth. evenly throughout the enamel. B. reduce gingival inflammation. plaque on his teeth. the fluoride recession? concentration of an erupted tooth is greatest A. C. deep scaling.Which treatment procedure is indicated for a In a young patient living in an area with patient with asymptomatic age related gingival communal water fluoridation. accelerate epithelialization after C. Carious lesions are most likely to develop if a C. free gingival graft. laterally positioned sliding flap. C. a panoramic film. D. pain. B. impaction. location of the lesions. means that chromium- cobalt-nickel partial denture clasp will require A. B. D. double-papilla pedicle graft. A. B. neutrophils. D. A. C. In addition. prevent temporomandibular joint syndrome. ultrasonic scaler. and translation. abscess formation. lymphocytes. compared to a periodontal pocket may result in Type IV gold alloy. C. sickle scaler. B. improve oral hygiene by preventing food D. increased calculus formation. C. D. lymphadenopathy. The higher modulus of elasticity of a The absence of adequate drainage in a chromium-cobalt-nickel alloy. E. The instrument best suited for root planing is a/an An increase of immunoglobulins is consistent A. A. hoe. fibroblasts. a periapical film . Correction of an inadequate zone of attached gingiva on several adjacent teeth is best Crown-root ratio and residual bone support can accomplished with a/an best be seen radiographically in A. cyst formation. C. C. a shallower undercut. technique. achieve a more favorable direction and distribution of forces of occlusion. a bite-wing film. A. B.bisecting angle D. D. apically repositioned flap. ©The National Dental Examining Board of Canada 2017 . file. D. temperature of the patient. content. C.The most important objective of occlusal Necrotizing ulcerative gingivitis (NUG) and adjustment of a natural dentition is to acute herpetic gingivostomatitis can be differentiated clinically by (the) A. with increased numbers of B. Note: Some of the items in the Released Test Item Bank may have been discontinued due to outdated science or errors. B. The NDEB periodically reviews the bank to improve its quality. the format of some items is not currently used. mastication. more taper. a periapical film . a shorter retentive arm. increase the shearing action in B. a heavier cross section for a clasp arm.paralleling technique. curette. E. coronally positioned flap. plasma cells. epithelial hyperplasia. C. The gingival margin of the preparation for a full crown on a posterior tooth. 4. D. a dental stone mixture will exhibit A. interfering with the setting of the A. hardness. D. Adhesion to calcium. redirect the forces of occlusion. 3. at least 1mm supragingivally. ©The National Dental Examining Board of Canada 2017 . provide retention for a cast crown. tooth migration. E. (2) and (4) E. Ease of manipulation. 2. 0. (1) and (3) D. 1. (1) and (3) A. resistance to corrosion. 4. dissolve surface oxides and lower the melting range. occlusal prematurities. A. increased tooth sensitivity. D. (4) only E. growth. expansion. In addition.Which of the following physical properties An epinephrine-containing retraction cord has would be_least_important for an impression the potential of material for partially edentulous patients? A. C. 2. provide intraradicular venting. All of the above. at the cemento-enamel junction. E. Note: Some of the items in the Released Test Item Bank may have been discontinued due to outdated science or errors. B. Elasticity. B. should be placed A. (1) (2) (3) C. producing a systemic reaction. on the enamel. A. All of the above. B. with a clinical The main purpose of flux in soldering is to crown that satisfies the requirements for retention and resistance. The NDEB periodically reviews the bank to improve its quality. the format of some items is not currently used. A cast post and core is used to The addition of platinum to a dental gold alloy results in increased 1. Improper temporary coverage of bridge abutments can cause During the setting phase. D. A. 3. strength. B. D. contraction.5mm subgingivally. causing tissue necrosis. strengthen a weakened tooth. Biocompatibility. C. (2) and (4) B. impression material. at the gingival margin. All of the above. prevent oxidation and lower the melting D. (1) (2) (3) B. and translation. range of the solder. prevent recrystallization and grain B. B. C. loss in compressive strength. C. C. gain in moisture content. discolouring gingival tissue. C. (4) only C. content. Dimensional stability. gingival recession. E. D. melting point. dissolve surface oxides and prevent further oxidation. Which of the following should be checked first A. C. B. In addition. impingement on the buccal frenum. The cusp inclination. 3. (1) and (3) cannot be seated on its abutment? C. parallel to the path of insertion. Dental porcelain has B. 2. at a right angle to the occlusal plane. increased. At his first post insertion appointment. expand when stored in water. The orientation of the occlusal plane. The proximal contacts. low compressive strength. 2. (1) (2) (3) Following the insertion of complete dentures. D. high tensile strength. All of the above. The occlusal contacts. B. C. a B. 1. The most likely cause is A metal in the wrought condition differs from the same metal in the cast condition in that A. Inadequate interocclusal distance. B. Note: Some of the items in the Released Test Item Bank may have been discontinued due to outdated science or errors. B. (4) only A. inadequate polishing of the framework. which of the following can be modified to achieve the desired occlusion? A. C. the yield strength and hardness are D. overextended borders of the partial. excess border thickness. show syneresis. low impact strength. 4. (1) (2) (3) when a cast gold crown that fits on its die B. D. C. high muscle attachments. (4) only E. improper path of insertion. 1. ©The National Dental Examining Board of Canada 2017 . C. All of the above. a patient with a new removable partial denture complains of a tender abutment tooth. if heated sufficiently. D. C. at a right angle to the major connector. parallel to the long axis of the tooth. The compensating curve. content. A. and translation. All of the above. The condylar inclination. the occlusion. 4. the format of some items is not currently used. The taper of the preparation. D. A. shrink when stored in air. 3. (2) and (4) D. guiding planes are made A. (2) and (4) alveolar ridge can be caused by D. E. In the design of a removable partial denture. (1) and (3) generalized soreness over the entire mandibular C.During the fabrication of new complete Alginate hydrocolloids dentures. B. high hardness. the grains are deformed and elongated. D. recrystallization can occur. The impression used to pour the cast. The NDEB periodically reviews the bank to improve its quality. A. All of the above. A. Surface hardness. better colour. C. (2) and (4) is observed that the tuberosities contact the D. Mylohyoid ridge. the format of some items is not currently used. more translucency. (4) only retromolar pads at the correct occlusal vertical E. B. The NDEB periodically reviews the bank to improve its quality. Melting point. and translation.Which of the following structures affects the A maxillary complete denture exhibits more thickness of the flange of a maxillary complete retention and stability than a mandibular one denture? because it A. content. C. The treatment of choice is to A. covers a greater area. The purpose of relining a distal saddle of a B. 4. displacement. C. increased strength. In addition. 2. Lowered specific gravity. 3. (2) and (4) D. (1) and (3) C. C. dimension. E. and reduce the posterior extension of the mandibular denture to eliminate A. it C. D. (1) and (3) Upon examination of an edentulous patient. porcelain is C. Zygomatic process. Elastic strength. ©The National Dental Examining Board of Canada 2017 . All of the above. construct new dentures at an increased 1. Coronoid process. Increased hardness. E. (4) only E. function. incorporates a posterior palatal seal. Ductility. A. Gold casting alloys are classified as Type I-IV according to which of the following physical properties? A. reduce the tuberosities surgically to removable partial denture is to improve provide the necessary clearance. B. is not subject to as much muscular D. reduce the retromolar pads surgically to provide the necessary clearance. D. B. D. Malar process. Note: Some of the items in the Released Test Item Bank may have been discontinued due to outdated science or errors. occlusion. (1) (2) (3) B. D. Gold contributes which of the following properties to a gold-copper alloy? A. Increased strength. occlusal vertical dimension to gain the 2. (1) (2) (3) interferences. proceed with construction of the denture 4. 1. 3. All of the above. is completely surrounded by soft tissue. necessary clearance. Corrosion resistance. Genial tubercle. tissue adaptation. The prime advantage of vacuum firing of B. less shrinkage. B. fit of the framework. Percentage of gold present in the alloy. insulin. create a smooth finish. Dental plaque developing on tooth surfaces will Heat treatment alters a gold alloy's result in A. 3. B. esthetic requirements. Streptococcus mutans. hardness. B. Streptococcus salivarius. B. periodontal disease. C. Masseter. content. External (lateral) pterygoid. A. (1) (2) (3) B. B. D. Hyaluronidase. separate functions in mandibular movement? 4. Actinomyces viscosus. ©The National Dental Examining Board of Canada 2017 . (2) and (4) Which of the following bacterial products have D. (1) (2) (4) D. (3) (4) (5) Note: Some of the items in the Released Test Item Bank may have been discontinued due to outdated science or errors. B. clinical crown length. Lactobacillus acidophilus. a chemically cured glass A lowering of serum calcium is the stimulus for ionomer cement restoration should have a the endogenous release of coating agent applied to A. E. All of the above. parathyroid hormone. C. A. demineralization of enamel. The NDEB periodically reviews the bank to improve its quality. amelogenesis imperfecta. Geniohyoid. C. D. Desxyribonuclease. (1) and (3) C. All of the above. the format of some items is not currently used. (4) only been implicated in the initiation of E. Protease. D. protect the cement from ultraviolet light. presence of an existing restoration. E. A. Which of the following muscles has two 3. and translation. C. 4. D. inflammatory periodontal disease? 1.After initial setting. A. adrenocortical hormone. A. Buccinator. In addition. thyroid hormone. (1) and (2) C. D. percentage elongation. proportional limit. protect the cement from moisture. and B. B. C. Endotoxin. 2. hasten the final set. A. B. (2) (3) (4) D. A. retard the final set. presence of caries. C. adrenalin. (2) (3) (5) E. Neuraminidase. 2. 5. The location of a crown margin is determined The microorganism most commonly associated by with root surface caries is 1. Diffuse calcification of the pulp. Internal resorption. (2) and (4) susceptibility to dental caries? D. (1) and (3) C. ©The National Dental Examining Board of Canada 2017 . Deficiency of vitamin D. All of the above. Hereditary fructose intolerance. B.In long-standing gingivitis. (1) (2) (3) B. C. 1. C. E. nearest external surface. Severe enamel hypoplasia. E. the subgingival Using pins to retain amalgam restorations microflora shifts toward increases the risk of A. 2. Frequent ingestion of polysaccharides. (4) only E. is A. (4) only E. Vitamin K deficiency during tooth development. All of the above. insufficient iodine. Acute suppurative pulpitis. A. Chronic hyperplastic pulpitis. B. None of the above. 2. B. 4. Hyposalivation. B. content. Vitamin D deficiency during tooth development. B. A. Gram-positive bacteria. In pin-retained restorations. D. excess calcium. provide thermal insulation to the pulp. aerobic bacteria. reduce sensitivity of freshly cut dentin. (1) and (3) Which of the following condition(s) increase(s) C. the pin holes 3. A. provide a barrier to the passage of most likely to result in a periapical lesion? irritants from restorative materials. long axis of the tooth. Frequent ingestion of high sucrose. The NDEB periodically reviews the bank to improve its quality. periodontal ligament invasion. Reversible pulpitis. 3. Note: Some of the items in the Released Test Item Bank may have been discontinued due to outdated science or errors. C. Cements used as cavity liners Which of the following. D. Enlargement of the thyroid gland can be caused by Which of the following is the greatest risk factor for rampant caries in children? A. axial wall. pulp exposures. insufficient fluoride. A. if left untreated. Gram-negative anaerobic bacteria. C. (1) (2) (3) B. the format of some items is not currently used. C. thermal sensitivity. All of the above. (2) and (4) D. excess iodine. pulp chamber. C. D. D. D. A. containing foods. 1. In addition. excess sodium. B. D. and translation. should be parallel to the 4. cracks in the teeth. content. pterygomaxillary notches and the fovea palatinae. over the crest of the mandibular ridge. B. A. thick layer of cortical bone. Irreversible hydrocolloid materials are best removed from the mouth by In the bisecting angle principle of intraoral A. In addition. E. palatine bone and the zygoma.Which of the following conditions is Radiographically. C. mandibular molar teeth should normally be placed A. Amelogenesis imperfecta. C. A. D. C. Epulis fissuratum. posterior border of the tuberosities and which of the following? the posterior border of the palatine bone. having the patient create a positive A. D. and translation. a slow teasing motion. the format of some items is not currently used. Regional odontodysplasia. D. twisting and rocking. Squamous cell carcinoma. palatine raphe and the posterior border of B. Prominent fungiform papillae. the lamina dura is a characterized by abnormally large pulp chambers? A. over the buccal shelf area. maxillary sinus. C. anterior border of the tuberosities. D. Median palatal cyst. Odontogenic keratocyst. Incisive canal cyst. Dentinogenesis imperfecta. the palatine bone. zygoma and the zygomatic process of the maxilla. B. the C. thick layer of bone forming the inner surface of the alveolus. the radiopacity that can obliterate B. B. pterygomaxillary notches and the posterior nasal spine. D. To improve denture stability. Dentinal dysplasia Type I. lingual to the crest of the mandibular ridge. the teeth. orbital process of the zygomatic bone. Lingual varicosities. The NDEB periodically reviews the bank to improve its quality. Which of the following has the highest rate of recurrence? The anatomical landmarks used to help establish the location of the posterior palatal A. pressure. buccal to the crest of the mandibular ridge. ©The National Dental Examining Board of Canada 2017 . the apices of maxillary molars is the C. Median rhomboid glossitis. seal of a maxillary complete denture include B. thin radiolucent line around the roots of B. Note: Some of the items in the Released Test Item Bank may have been discontinued due to outdated science or errors. thin radiopaque line around the roots of the teeth. Nasoalveolar cyst. D. radiography. C. a quick snap. D. B. D. the A. Hyperplastic lingual tonsils may resemble C. A. Coagulation time. buccinator and genioglossus muscles. A. superior constrictor. A. resistance of the host. Clotting time. ©The National Dental Examining Board of Canada 2017 . Tachycardia is a term which describes a pulse C. expansion. contraction. is associated with root caries. B. The NDEB periodically reviews the bank to improve its quality. nonkeratinized. The severity of the course of periodontal disease depends upon the Before performing surgery on a patient who is taking warfarin. In addition. D. ensuring maximum distribution of rate of occlusal forces. C. buccinator and styloglossus B. Prothrombin time. B. B. less than 60. (4) only connector of a removable partial denture is E. D. (1) (2) (3) B.Upon setting. virulence of the organism. anaerobic flora. reducing the size of the connector. Extreme resorption of an edentulous mandible can bring the alveolar ridge to the level of the attachment of the Juvenile periodontitis A. mylohyoid and C. squamous. D. B. and translation. C. buccinator. and the gingiva. D. providing relief between the connector A. E. Note: Some of the items in the Released Test Item Bank may have been discontinued due to outdated science or errors. less than 50. A. (2) and (4) Prevention of gingival irritation by a major D. styloglossus and geniohyoid A. D. has a definite predilection toward males.. mylohyoid. mylohyoid. D. is associated with gram-negative muscles. less than 70. is associated with gram-positive anaerobic muscles. A. 3. buccinator muscles. (1) and (3) C. D. 2. nonpermeable. which of the following should A. accomplished by A. B. loss in compressive strength. and B. C. 4. using split palatal bars. flora. C. number of organisms present. more than 70. stratified. be evaluated? B. C. All of the above. the format of some items is not currently used. gain in moisture content. content. Bleeding time. B. and C. a mixture of plaster of Paris and Normal sulcular epithelium in man is water will exhibit 1. expands. has a tendency to migrate following insertion. when placed to subperiostially. 1. reduce film density. The NDEB periodically reviews the bank to improve its quality. 1. reduce patient radiation dose. Chemical bonding to etched dentin. Decreasing the water/powder ratio by a small amount. C. burning gingivae. in an adolescent (juvenile periodontitis) is A. In addition. When gold or a gold alloy changes from a liquid to a solid state it A. be chamfered. (1) and (3) B. 3. painful. D. is highly biocompatible. D. D. Adding a small amount of salt to the E. induces bone formation throughout the implanted material. increase contrast. Which of the following properties apply to C. approach 45 degrees. reduce exposure time. the format of some items is not currently used. C. has a low incidence of secondary infection C. A. marginal gingivitis. C. (4) only A. Decreasing the mixing time. B. (1) only B. B.Filters are placed in the path of the x-ray beam Particulate hydroxyapatite. (1) (2) (3) products will increase the compressive strength B. Which of the following modifications to the standard procedure for mixing gypsum A. To ensure maximum marginal strength for an 3. ©The National Dental Examining Board of Canada 2017 . water before mixing. B. All of the above. amalgam restoration the cavosurface angle should A. All of the above. (1) and (3) of the set material? C. C. E. work hardens. (1) and (2) A. 4. 2. drifting of the teeth. D. hyperplastic gingivitis. Non-irritating to pulpal tissue at moderate depth. following surgery. A characteristic sign of aggressive periodontitis D. content. becomes brittle. be bevelled. 2. and translation. glass ionomer cements? D. Using warmer water. corrodes. contracts. (2) and (4) D. Anticariogenic because of fluoride release. Note: Some of the items in the Released Test Item Bank may have been discontinued due to outdated science or errors. approach 90 degrees. B. C. should result in a sharp gingivoproximal line angle.Planing the enamel at the gingival cavosurface A patient with complete dentures complains of of a Class II amalgam preparation on a clicking. the major connector C. Appearance. excessive occlusal vertical dimension. is unnecessary since the tooth structure in C. should remove unsupported enamel which D. D. (2) and (4) D. incorrect centric relation position. E. D. (4) only. B. Pre-extraction profile records. add a small amount of borax. tetracycline. The NDEB periodically reviews the bank to improve its quality. cementoenamel junction. the most frequent cause of tooth contact (clicking) The area of the tooth that is most sensitive during speaking is during cavity preparation is A. In partial denture design. the format of some items is not currently used. dentinoenamel junction. E. cephalexin. use cold water. should D. In an infection caused by non-penicillinase E. C. 4. little horizontal overlap. C. C. (1) and (3) C. add additional water. B. Observation of the rest position. 3. dissipate vertical forces. ©The National Dental Examining Board of Canada 2017 . All of the above. In patients wearing complete dentures. is contraindicated because of the low edge B. E. (1) (2) (3) B. A. unbalanced occlusion. reduced vertical dimension and A. use of too large a posterior tooth and too this area is strong. Which of the following is/are (a) useful The best means of extending the working time guide(s) in determining a patient’s occlusal of an irreversible hydrocolloid impression vertical dimension? material is to 1. In addition. A. C. excessive vertical dimension and poor strength of amalgam. penicillin V. rigidly connect the bilateral components. D. lack of vertical overlap. Phonetics. cementum. D. vancomycin. content. excessive anterior vertical overlap. the drug of choice is A. not interfere with lateral forces. extend spatulation time. should result in a long bevel. B. A. B. 2. act as a stress-breaker. dentin. and translation. The most common causes are permanent tooth A. D. improperly balanced occlusion. nervous tension. A. B. Note: Some of the items in the Released Test Item Bank may have been discontinued due to outdated science or errors. retention. improper relation of teeth to the ridge and may fracture. B. add potassium sulfate. producing staphylococcus. 2. content. Tonsillar fossa.Intravenous administration of epinephrine The most common cause of malocclusion with results in a Class I molar relationship is 1. 4. A. crossbite in the posterior segments. Cardiac pacemaker. D. A. 3. (1) (2) (3) B. (2) and (4) D. C. Dorsum of the tongue. the format of some items is not currently used. commonly be treated during the mixed dentition stage? A. Maxillary incisor rotation. Floor of the mouth. caries. D. a thumbsucking habit. Prosthetic heart valves. ©The National Dental Examining Board of Canada 2017 . (1) (2) (3) B. disinfect pockets for up to 18 hours. increased systolic pressure. tooth size and jaw size discrepancy. Anterior cross-bite. Lateral border of the tongue. Note: Some of the items in the Released Test Item Bank may have been discontinued due to outdated science or errors. C. improper eruption of permanent first molars. respiratory depression. B. A. very thin mix of cement. D. C. In addition. patients with which of the following? C. All of the above. All heart murmurs. C. Water irrigation devices have been shown to Which of the following is the LEAST likely A. (4) only Which of the following conditions should NOT E. palpitations. 4. (4) only C. B. rapid and heavy application of pressure until the inlay is seated. D. dislodge food particles from between squamous cell carcinoma in the elderly? teeth. Mitral valve prolapse with regurgitation. Class II molar relationship. continuous firm pressure on the inlay until the cement is set. increased heart rate. Antibiotic prophylaxis is recommended for B. and translation. C. All of the above. Posterior cross-bite. thick mix of cement. D. periodontal disease. primary site for the development of oral B. 3. B. The NDEB periodically reviews the bank to improve its quality. bruxism. eliminate plaque. 1. The most frequent cause of tooth loss in the elderly is A. the best procedure to ensure accurate seating is a A. (1) and (3) A. When cementing an inlay. (1) and (3) C. extraoral trauma. (2) and (4) B. D. E. prevent calculus formation. 2. E. D. use of a removable partial denture. supportive therapy. drainage. All of the above. are generally painless. 1. (2) and (4) D. lingual movement of the crown and 2. A. B. crown-root ratio of the abutments. D. (2) and (4) D. slower healing. A. necrosis of bone. grow slowly. D. E. Note: Some of the items in the Released Test Item Bank may have been discontinued due to outdated science or errors. C. 2. can be managed conservatively. there is an increase in the A surgical flap not repositioned over a bony A. C. (1) and (3) C. A. E.In the management of a patient with an acute A removable orthodontic appliance. (2) and (4) D. (1) and (3) D. 2. and B. 3. diameter of the wire segment. In addition. Benign neoplasms 1. base will result in B. can metastasize. fixed partial denture is influenced by the 4. the format of some items is not currently used. The NDEB periodically reviews the bank to improve its quality. (1) and (3) function of C. intrusion of the central incisor and lingual 4. All of the above. (1) (2) (3) In clinical dentistry. length of the span of the fixed partial A. number of plasma cells. wound dehiscence. (4) only E. intrusion of the central incisor. All of the above. foreign body inflammatory reaction. and translation. A. A. and C. 4. the treatment should a light force on the labial of a proclined include: maxillary central incisor will cause 1. number of mast cells. (1) (2) (3) denture. content. B. In acutely inflamed gingival tissue. B. E. length of the wire segment. (1) and (3) 2. All of the above. All of the above. C. (2) and (4) 3. amount of periodontal support of the D. stiffness of wire is a B. level of histamine. producing odontogenic infection. position of the abutments in the arch. (4) only A. None of the above. 4. (1) (2) (3) movement of the root apex. alloy composition. tetanus immunization. (1) (2) (3) B. The choice and number of abutments for a 3. lingual movement of the root apex. movement of the crown. ©The National Dental Examining Board of Canada 2017 . C. B. E. 3. elimination of the cause. C. (4) only E. (4) only abutments. 1. lingual movement of the crown and labial A. A. Metabolism is decreased by a reduced 3. E.Which of the following pharmacokinetic Compared to unfilled resins. (2) and (4) D. 4. 3. B. (1) (2) (3) E. All of the above. Bennett angle. Excretion is reduced because of lessened B. base of the pocket to the cementoenamel junction. premature eruption of a maxillary incisor. C. (2) and (4) D. In addition. Note: Some of the items in the Released Test Item Bank may have been discontinued due to outdated science or errors. All of the above. the measurement is taken from C. D. base of the pocket to the crest of the free permanent dentition that is characterized by gingiva. once it has migrated apically. B. unexplainable genetic factors. (1) and (3) C. prolonged retention of a primary incisor. (1) and (3) renal blood flow. C. pocket depth. the D. desmosomes. liver mass. B. junction. C. oxytalan fibres. and translation. A. increased strength. Absorption is altered by a decrease in the 1. hemidesmosomes. lingually situated supernumerary teeth. C. content. The junctional epithelium. 2. D. E. base of the pocket to the mucogingival A. anterior crowding. lateral condylar inclination. D. opening and closing axis of the mandible. affected side. An anterior cross-bite of a permanent maxillary incisor in a mixed dentition is often associated with A. molar usually produces a malocclusion in the C. attaches to the cementum by means of A. D. B. The NDEB periodically reviews the bank to improve its quality. composite resins change(s) occur(s) with aging? have 1. free gingival margin to the Premature loss of a primary maxillary second cementoenamel junction. (4) only A. labially displaced maxillary canines. (4) only E. reduced thermal dimensional changes. reduced polymerization shrinkage. the format of some items is not currently used. ©The National Dental Examining Board of Canada 2017 . collagen fibres. delayed eruption of the permanent first molar. a functional shift. centric relation. Distribution is altered by a decrease in total body fat. a Class II molar relationship on the A hinge axis facebow records affected side. a Class III molar relationship on the A. 2. gastric pH. better polishability. When using the periodontal probe to measure B. E. B. horizontal condylar inclination. (1) (2) (3) 4. (1) (2) (3) 1. All of the above. C. No movement of second premolar. 3. A. C. 2. rotation. B. molar at age 8. A. be semilunar in shape. (1) and (3) C. (1) (2) (3) B. Note: Some of the items in the Released Test Item Bank may have been discontinued due to outdated science or errors. which of the following changes 4. D. (1) (2) (3) B. In addition. Mesial drift of second permanent molar. A. (4) only Recurring tooth rotations occur most frequently E. B. free gingival and transseptal fibres. A. Horizontal. (4) only 4. impair tooth movement. The predominant type of movement produced C. (1) and (3) 2. (2) and (4) mandibular third molar. 4. B. B. crush the periodontal ligament. provide for instrument access. 2. A. No movement of second permanent E. The NDEB periodically reviews the bank to improve its quality. D. provide for visual access. oblique fibres of the periodontal ligament. torque. band and loop space maintainer. content. density of the cortical bone. by a finger spring on a removable appliance is D. cause root resorption. C. tipping. 1. D. The design of a mucoperiosteal flap should D. translation. Mesio-angular. All of the above. 3. Distal drift of second premolar. Nance expansion arch. molar. are likely to occur? A. (2) and (4) 3. Vertical. and translation. ©The National Dental Examining Board of Canada 2017 . (1) and (3) In the surgical removal of an impacted C. Disto-angular. which of the following D. after orthodontic correction due to A. lingual arch.Excessive orthodontic force used to move a The best space maintainer to prevent the tooth may lingual collapse that often occurs following the early loss of a mandibular primary canine is a 1. B. persistence of tongue and finger habits. the format of some items is not currently used. cause hyalinization. All of the above. (2) and (4) D. C. permit repositioning over a solid bone Following loss of a permanent mandibular first base. (4) only would be considered to be the most difficult? E. distal shoe space maintainer. (1) (2) (3) A. All of the above. Anterior to the pterygomandibular raphe. have diabetes mellitus (controlled). 1. Zinc phosphate cement. of local anesthetic for the inferior alveolar 4. position of the needle tip during administration 3. (2) and (4) streptococcus. A. Polycarboxylate cement. 2.Titanium implants in the oral cavity are Which of the following affect(s) CONTRAINDICATED for patients who polymerization of visible light cured composite resins? A. All of the above. Proximity of light source. Use of mechanical condensation. nerve block? A. (1) (2) (3) 4. During dental treatment. are over age 75. contact when using a spherical alloy. Lateral to the sphenomandibular ligament. 4. Intensity of the light source. have a terminal disease. Ethoxy benzoic acid cement. C. C. A. In order to achieve a proper interproximal D. Penicillin. A. (4) only drug of choice? E. Tetracycline. E. Superior to the lateral pterygoid muscle. (1) and (3) the infection is caused by beta hemolytic C. 3. (1) and (3) C. A thinner matrix band. The NDEB periodically reviews the bank to improve its quality. and translation. administer 100 oxygen. (2) and (4) A. monitor the patient. ©The National Dental Examining Board of Canada 2017 . (1) and (3) E. Which one of the following describes the 2. retrosternal pain. B. (4) only E. Erythromycin. An anatomical wedge. D. which of the following is/are essential? 1. C. Cloxacillin. D. Shade of composite resin. a 62 year old insulin. C. B. stop treatment and 2. administer 50 dextrose intravenously. suddenly complains of bond to enamel? severe. the format of some items is not currently used. 1. crushing. 3. Medial to the medial pterygoid muscle. In addition. (2) and (4) D. Which of the following is the D. 3. All of the above. (4) only B. B. Which of the following cements can chemically dependent diabetic. A larger sized condenser. 2. are on thyroid replacement therapy. B. (1) (2) (3) D. 4. (4) only D. (2) and (4) C. Glass ionomer cement. content. Thickness of composite resin. (1) and (3) B. (1) (2) (3) Cultures made from a dental abscess indicate B. All of the above. administer sublingual nitroglycerin. Note: Some of the items in the Released Test Item Bank may have been discontinued due to outdated science or errors. The appropriate initial management would be to 1. C. tooth but the patient complains that it is rough and sensitive to cold. a calcium hydroxide cement. 4. A. A. Lower first bicuspid. In addition. dentin bonding. All of the above. 2. has which of the periodontitis? following properties? 1. Diabetes mellitus. religious beliefs. Upper lateral incisor. Chemical adhesion. C. ©The National Dental Examining Board of Canada 2017 . Mechanical retention. glass ionomer cement. caries removal. pulp regeneration. and translation. Anticariogenicity. E. Which of the following systemic diseases Zinc phosphate cement. (1) (2) (3) B. place a provisional (temporary) crown. (2) and (4) C. plaque accumulation. Lower canine. Note: Some of the items in the Released Test Item Bank may have been discontinued due to outdated science or errors. 1. All of the above. B. C. Insolubility. D. 1. 3. The fracture occurred one D. the format of some items is not currently used. Upper canine. 2. The NDEB periodically reviews the bank to improve its quality. (1) (2) (3) A 22 year old presents with a fracture of the B. (2) and (4) amount of dentin. Cyclic neutropenia. 3. (4) only D. There is no mobility of the E. clean the canal used during extraction? and temporarily close with zinc oxide and eugenol. Hereditary hypohydrotic ectodermal 4. E. All of the above. it is ethical to refuse to consideration in treat a patient on the basis of A. 4. (4) only E. open the pulp chamber. 3. physical handicap. (1) (2) (3) A. (2) and (4) D. smooth the surrounding enamel and apply B. (1) and (3) B. The most appropriate emergency treatment is to For which of the following teeth is the risk of root fracture increased if a rotational force is A. A. when used as a luting does/do NOT predispose a patient to agent for cast restorations. B. Acquired immunodeficiency syndrome. 2. smooth the surrounding enamel and apply D. recognition of lack of skill or knowledge.1 exposing a small C.The "smear layer" is an important As a dentist in Canada. content. (4) only hour previously. (1) and (3) incisal third of tooth 2. dysplasia. infectious disease. Upper first bicuspid. D. (1) and (3) C. the format of some items is not currently used. affects the gingiva. initial contact. All of the above. B. Glossopharyngeal nerve IX. E. 2. (4) only E. D. is macular. A patient with a tumor in the right infratemporal fossa shows a significant shift of the mandible to the right when opening. D. A 45 year old. C. E. undergoes clinical changes. overweight man reports that his Which articular disease most often wife complains that he snores. maximum opening. antibiotic therapy. Golgi receptor. 4. The NDEB periodically reviews the bank to improve its quality. B. contains melanin. D. Lupus arthritis. (1) and (3) C. patient’s vertical dimension of occlusion. Levator labii superioris and zygomaticus C. fabricate restorations to increase the D. periodontal debridement. (1) (2) (3) B. D. Cementoblasts. fabricate an appliance to reduce snoring.A lateral cephalometric radiograph for a patient Which two muscles are involved in sucking? with a 3mm anterior functional shift should be taken with the patient in A. (2) and (4) D. content. A. Hypoglossal nerve XII. C. 3. This pain is associated Physiologic (racial) pigmentation differs from with which of the following? melanoma because melanoma A. Which The most appropriate treatment of necrotizing nerve is involved? ulcerative periodontitis (NUP) in a patient with no fever and no lymphadenopathy is A. Degenerative arthrosis. C. A. B. protrusive position. Odontoblastic processes. A. Suppurative arthritis. refer for an orthognathic surgery consultation. A. C. Psoriatic arthritis. normal rest position. C. Facial nerve VII. Risorius and buccinator. D. ©The National Dental Examining Board of Canada 2017 . B. refer for a sleep assessment. Following root planing. B. The initial accompanies Sjögren’s syndrome? management of the patient’s snoring problem is to A. a patient experiences thermal sensitivity. maximum intercuspation. Rheumatoid arthritis. Note: Some of the items in the Released Test Item Bank may have been discontinued due to outdated science or errors. D. major. oral hygiene instruction. 1. topical steroid therapy. B. Trigeminal nerve V. Free nerve endings. and translation. Caninus and depressor angularis. In addition. B. C. Buccinator and orbicularis oris. 1-2mm apical to the cementoenamel junction. prescription of an analgesic. (2) and (4) preparation on tooth 1. D. a mandibular Class II furcation D. prescription of an antibiotic. D. Her symptoms have been present for C. None of the above. stiffness and soreness in the preauricular B. C. axiopulpal. (1) and (3) A line angle NOT present on a Class I cavity C. 3. root sensitivity. All of the above. B. root caries. nocturnal bruxism. linguopulpal. localized periodontal abscess. “guided tissue Which of the following structures may be regeneration” is most successful in treating associated with the role of the central nervous system in sleep (nocturnal) bruxism? 1. morning. C. A delta and C nerves. early osteoarthritis. B. 3-4mm apical to the cementoenamel junction. the normal alveolar crest is D. Sphenopalatine ganglion. The most likely cause is A. must include D. B. In addition. (4) only. 2. mandibular subluxation. scaling and root planing. mesiopulpal. Lettuce. buccopulpal. involvement. the format of some items is not currently used. Which of the following would maximize D. vitamin E intake following osseous surgery? A. horizontal bone loss. The most likely cause of tooth loss following a E. B. A. not clearly distinguishable. a mandibular Class III furcation B. Petrous nerves. A. with an acute B. Note: Some of the items in the Released Test Item Bank may have been discontinued due to outdated science or errors. E. content. region. For an otherwise healthy patient. tunneling procedure to provide complete access for a mandibular Class III furcation involvement is A 23 year old female complains of bilateral A. a 3-walled infrabony defect. fibrous ankylosis of the temporomandibular joints. A.6 is D. (1) (2) (3) B. age of 30. Fish. Wheat germ. C. the past week and are most pronounced in the D. initial treatment C. A. 4. C. Eggs. pulpal involvement. involvement. and translation. A. Basal ganglia (nigrostriatal). recurrent pocketing. occlusal adjustment. at the cementoenamel junction. ©The National Dental Examining Board of Canada 2017 . The NDEB periodically reviews the bank to improve its quality.In periodontal therapy. On bite-wing radiographs of adults under the C. to fracture is/are: B. 1. apply a cold compress. C. Symphysis. C. shift of the mandible from initial contact 4. 10mm or more in each of the upper and lower arches and 35% overbite.3mg epinepherine bite. The NDEB periodically reviews the bank to improve its quality. lower arches and 70% overbite. D. (1) and (3) C. the most likely region(s) of the mandible lower arches and 35% overbite. a thin broad palatal strap. be to B. The immediate treatment should anterior open bite. apply pressure. (sublingually). All of the above. narrow horseshoe shaped. Class II Division 1 associated with an the left side. Anterior open bite associated with a lip or A. administer 0. striking the A. A. Class II Division 2 associated with an increased anterior overbite. (4) only E. All of the above. (1) (2) (3) digit sucking habit. refer for immediate medical treatment. C. (1) and (3) C. In its classic form. Cross-bite associated with a functional 3. D. 10mm or more in each of the upper and 4. lower arches and 70% overbite. an anterior and a posterior bar. less than 10mm in each of the upper and 2. a thick narrow major connector. the patient’s face becomes quickly and visibly swollen on A. Note: Some of the items in the Released Test Item Bank may have been discontinued due to outdated science or errors. D.Which type of malocclusion should be Immediately following a left posterior superior corrected as early as possible? alveolar nerve block injection. 1. Angles of the mandible. less than 10mm in each of the upper and chin. content. B. Condylar necks. and translation. the format of some items is not currently used. B. (2) and (4) The maxillary cast partial denture major D. ©The National Dental Examining Board of Canada 2017 . Class III associated with an anterior open 2. serial extraction is best applied to patients with Class I occlusions with crowding of If a person falls from a bicycle. (4) only connector design with the greatest potential to E. Mid-body. E. to maximum intercuspation. 3. (1) (2) (3) B. (2) and (4) D. cause speech problems is A. In addition. dentures are D. D. connective tissue capsule formation. all of the following should be D. E. 3. maxillary second molar erupts and moves A. Hypothyroidism in adults is associated with B. verify the maxillomandibular records. the format of some items is not currently used. A. the A lower molar requiring a crown has an patient complains of being lightheaded and amalgam restoration extending 1. E. All of the above. hyperventilation. D. weight loss.If a patient loses a permanent maxillary first Following root canal therapy. mental defects. steam sterilization. All of the above. hyperglycemia. C. ©The National Dental Examining Board of Canada 2017 . A. C. 1mm apical to the amalgam margin. mesially. tachycardia. The crown is most appropriately most likely experiencing placed A. staff education. E. (2) and (4) While the teeth are set in wax. dentin deposition. Note: Some of the items in the Released Test Item Bank may have been discontinued due to outdated science or errors. on the existing amalgam. (4) only tried in to E. 2. C. (1) and (3) C. Sweating is observed. C. and translation. hypoglycemia. B. During the examination. the most molar before the age of 11. B. B. opposing tooth erupts into the space C. overbite increases. B. C. chemical sterilization. D. medical history. In addition. 2mm apical to the amalgam margin. B. (1) (2) (3) B.0mm weak. In a dental office. at the amalgam/tooth junction. premolar drifts distally. cementum deposition. periodontal ligament. generalized edema. the desirable form of tissue response at the apical foramen is 1. standard precautions. A 45 year old with insulin dependent diabetes mellitus comes for a morning dental appointment. content. A. D. A. used to reduce the risk of Hepatitis B infection for staff and patients EXCEPT A. The patient is subgingivally. epithelium proliferation from the created. D. cerebrovascular accident. verify the vertical dimension of occlusion. syncope. 4. The NDEB periodically reviews the bank to improve its quality. exophthalmos. evaluate esthetics. A. iron deficiency anemia. 3. (1) and (3) A. 4. Root caries risk in adults is 1. 2. A. greater in adults who have high streptococcus mutans counts. The most incisors. 2. elevated. decreased blood supply to the pulp. 1. molars and good alignment of the lower spontaneous ache relieved by cold. red patch devoid of experience. All of the above. Three still present exhibits a cusp-to-cusp relationship days ago the symptoms changed to moderate of permanent maxillary and mandibular first pain on biting combined with a dull. All of the above. All of the above. a cracked tooth. The NDEB periodically reviews the bank to improve its quality. use a cervical headgear to reposition C. institutionalized patients. Enlargement of the thyroid gland. (2) and (4) E. the format of some items is not currently used. In addition. content. C. associated with previous enamel caries A smooth. continuous deposition of dentin. 4. B. A 20 year old female patient is suspected of having bulimia. pulpal necrosis. (1) and (3) C. reduced in communities with fluoridated dorsum of the tongue immediately anterior to water. (1) (2) (3) B. the circumvallate papillae is indicative of 3. C.A patient presents with a 3 week history of An 8 year old patient with all primary molars prolonged tooth pain to hot and cold. refer for orthodontic consultation. a granular cell tumor. B. filiform papillae. B. ©The National Dental Examining Board of Canada 2017 . benign migratory glossitis. (1) and (3) D. similar in institutionalized and non. D. Note: Some of the items in the Released Test Item Bank may have been discontinued due to outdated science or errors. A. A. located in the midline of the 2. (4) only C. 4. place patient on appropriate recall schedule. chronic apical abscess. continuous deposition of cementum. Calluses on the dorsum of the fingers. B. D. Which of the following signs will help confirm the diagnosis? Normal aging changes in the hard tooth tissues 1. (2) and (4) B. 3. maxillary molars. a fibroma. The management of this patient likely diagnosis is should be to A. D. a vertical root fracture. (2) and (4) D. median rhomboid glossitis. disk the distal surfaces of primary E. (4) only E. (4) only E. Bulky clothing to disguise weight loss. (1) (2) (3) E. (1) (2) (3) C. Enamel erosion of maxillary anterior include teeth. reversible pulpitis. mandibular second molars. D. and translation. increased porosity of enamel. B. (4) only E. Submandibular. Submaxillary. a disinfection solution is to A. the nerve’s association with which ganglion? C. the maxillary nerve. Submandibular. A. In addition. Pterygopalatine. (1) and (3) C.Upon stimulation of salivary flow. increase the concentration of the product. and an intolerance could contaminate hands through small to hot weather. The washing of hands must be performed C. Which of the following sweeteners used in sugarless gum is most effective in preventing Lacrimation and flushing of the face are caries? autonomic phenomena that are occasionally associated with trigeminal neuralgia involving A. completely eliminates skin bacteria. which gland Which of the following is necessary for is the main source of salivary volume? collagen formation? A. The best way for a dentist to ensure efficacy in E. B. 1. ©The National Dental Examining Board of Canada 2017 . A. D. C. D. make a fresh solution every day. Vitamin A. Trigeminal. malignant hyperthermia. Which of the following is a possible cause for a low density radiograph (light film)? A. fair hair. B. hands are moist. conical 3. Excessive developing time. Sorbitol. C. B. Note: Some of the items in the Released Test Item Bank may have been discontinued due to outdated science or errors. Nasociliary. Vitamin C. Glycerol. Sublingual. fine. A. The NDEB periodically reviews the bank to improve its quality. the format of some items is not currently used. (2) and (4) D. allows gloves to slide on easier when the A. achondroplasia. minimizes the transient bacteria which teeth. E. Over exposure. C. B. This can be explained by B. Otic. scanty. B. Vitamin E. before putting on and after removing gloves D. 4. D. C. ectodermal dysplasia. 2. Mannitol. and translation. cystic fibrosis. Vitamin K. follow the manufacturer’s instructions. B. Cold developer. (1) (2) (3) D. The most likely diagnosis is pinholes. reduces the number of skin bacteria which multiply and cause irritation. Xylitol. D. A patient presents with hypodontia. content. C. All of the above. Improper safety light. D. Vitamin D. increase the time of contact with the because it product. Parotid. ideally be located? D. C. C. chosen rather than preparing the teeth again. enlargement of the jaws.7. acute bacterial sialadenitis.5 retainer. sarcoidosis. and translation. a 1:10 dilution of sodium hypochlorite. regular relining of the distal extensions. Where should the male part of the attachment C. mandibular angle. D. A. a complex phenolic. Healthy attached gingiva A.6 pontic. A. D. B. Phantom bone disease. splinting abutments with adjacent teeth. content. using cast clasps on distal abutments. facial height. C. rickets. maxillary protrusion. swelling following a general anesthetic most B. Distal of the 3. diastemas and/or a poorly fitting denture? E. Hutchinson’s incisors and mulberry molars are B. development of D. The best way to protect the abutments of a D. overbite. D. A. fever and unilateral parotid A. C. E. has no rete pegs. congenital syphilis. Mesial of the 3. E. congenital porphyria. Mesial of the 3. Mumps. E. contains elastic fibers. has no basal cell layer. ©The National Dental Examining Board of Canada 2017 .5 and 3. placing distal rests on distal abutments. Rickets. Distal of the 3. A. upper incisor inclination. Hypophosphatasia. D.7 abutments for a fixed partial impression for 10 minutes in denture have different paths of insertion with respect to 3. B. E. fluorosis. Paget’s disease. 2% glutaraldehyde. cleidocranial dysplasia.7 retainer. In addition. is closely bound to underlying associated with periosteum. Osteoporosis. B. C. A semi-precision attachment is A. B. Which of the following diseases may cause an C. Class I removable partial denture from the negative effects of the additional load applied to them is by A patient with pain.A recommended method for disinfecting A survey of the master cast shows that the alginate impressions is to immerse the 3. keeping a light occlusion on the distal likely has extensions. The NDEB periodically reviews the bank to improve its quality. 10% ethyl alcohol. sialolithiasis. D. B. The angle SNA can be used to evaluate the A. C.6 pontic. Note: Some of the items in the Released Test Item Bank may have been discontinued due to outdated science or errors. Sjögren’s syndrome. the format of some items is not currently used. B. D. (1) (2) (3) A. (4) only D. C. D. 2. erythema multiforme. The NDEB periodically reviews the bank to improve its quality. C. D.Multiple “punched-out” radiolucencies of the In composite resin restorations. and translation. preventing the recirculation of the drug through B. periodontitis? A. scaling and root planing (debridement). multiple myeloma. the enterohepatic circulation. (1) and (3) B. the lips in a patient with target-like skin lesions the cavosurface margin of the cavity can be are typical of bevelled to A. open flap curettage. Smoking. C. 3. A. lupus erythematosus. B. content. improve convenience form. E. B. aid in finishing. glass ionomer skull and jaws are most commonly seen with cements can be used as a base because they are A. Behçet’s syndrome. A. Patients who have gingival enlargements surgically removed should be forewarned that there is a high incidence of A. guided tissue regeneration. Reiter’s syndrome. plasmacytoma. composite resins. B. post-operative swelling. C. occlusal adjustment. Crusted hemorrhagic and ulcerative lesions of For an acid-etched Class III composite resin. sedative to a hyperemic pulp. increase the surface area for etching. All of the above. Tetracycline. 1. Codeine. chondrosarcoma. following can interfere with this mechanism? D. Acetaminophen. altered taste sensation. compatible with the expansion of E. In addition. metastatic carcinoma. reoccurrence of gingival enlargement. Alteration of the intestinal flora by some Soft tissue pockets CANNOT be reduced by chemotherapeutic agents can interfere with reabsorption of a contraceptive steroid thus A. eliminate the need for internal retention. ©The National Dental Examining Board of Canada 2017 . Poorly controlled diabetes. B. pemphigus vulgaris. Poor oral hygiene. C. Which of the C. D. osteosarcoma. neutral in colour. B. biocompatible. Which of the following is NOT a risk factor for 4. Note: Some of the items in the Released Test Item Bank may have been discontinued due to outdated science or errors. Coronary heart disease. D. the format of some items is not currently used. Penicillin V. E. (2) and (4) C. D. B. dentinal hypersensitivity. C. surgical emphysema. ameloblastoma. D. failure of the union of the median nasal C. Delay any treatment until the hygiene from tooth-forming regions. B. few months. of radiation received by the patient is best The most likely diagnosis is reduced by A. D. a herniation of the buccal fat pad. The NDEB periodically reviews the bank to improve its quality. content. a hematoma. multiple myeloma. In addition. unreacted monomer evaporating from the process with the lateral nasal and set material. collimation. the amount left cheek and left cheek swelling is observed.After initiating preventive management for a During tooth development. hypophosphatasia. restorations within the next few weeks. ©The National Dental Examining Board of Canada 2017 . D. C. primary bonds replacing secondary bonds. polymerization reaction. the patient complains of a tense sensation in the When exposing radiographic film. B. Hypercementosis may be associated with Enamel pearls form when A. which of the following restorative treatments is most appropriate? A. A. an immediate allergic reaction. C. Note: Some of the items in the Released Test Item Bank may have been discontinued due to outdated science or errors. milliamperage. set material. Excavate caries and place temporary preodontoblasts to start producing dentin. failure of proper union of the median and B. A. Paget’s disease. maxillary processes. C. low kVp correlated with high D. the format of some items is not currently used. D. epithelial rests transform into ameloblast vesicles. decreased target-object distance. only with epithelium and mesenchyme C. Polymerization shrinkage associated with the setting of composite resins is a result of Cleft lip and palate usually result from A. epithelial 16 year old patient with multiple extensive mesenchymal interactions function carious lesions. A.8. D. decreased object-film distance. and translation. ameloblasts migrate apically down the C. Shortly after the administration of a local anesthetic for the removal of tooth 2. D. cells of the epithelial root sheath do not migrate away from the dentin. anhidrotic ectodermal dysplasia. through the exchange of small molecules. only at the initial stages when tooth positions are being laid down. B. Restore all teeth with composite resin over the next few months. improves. cells of the dental follicle fail to develop. reaction by-products evaporating from the lateral nasal processes. A. when preameloblasts signal B. root. temperature changes occurring during the C. Place amalgam restorations over the next B. C. B. B. D. failure of development of both the lateral nasal and maxillary processes. for caries prevention. refer him to a qualified maxillary sinus border. 3 to 4 Sv. refer him back to his physician requesting a more thorough assessment. a calcium sulfate dihydrate nucleating C. E. The the nasal fossa. D. Note: Some of the items in the Released Test Item Bank may have been discontinued due to outdated science or errors. 3 to 4 milli. C. D.1/2.(µ) Sv. Mandibular.3/2.A 75 year old male patient whose wife died Which anatomical structures form the inverted 10 months ago presents for his recall Y (Y line) in maxillary periapical radiographs? appointment. (4) only C. recommend that he drink 3 cans of a sinus border. to reach sound tooth structure. (1) (2) (3) B. (1) and (3) A. he confirms he has lost about 6kg in the last A. Mesial of teeth 1. to increase retention. B. Mesial of teeth 1. content. a sodium sulfate nucleating agent. B. A. ©The National Dental Examining Board of Canada 2017 . D. The most practical method to significantly The mean annual radiation dose equivalent for reduce the setting time of stone and plaster is to human populations on earth is approximately use A. radiolucency in the mandibular premolar region? A. most appropriate management for this patient is B. All of the above. Mesial of teeth 1. Which foramen presents as an apical 4. 3 to 4 micro.2/2. 2. B. dietician/nutritionist and follow up after his appointment. 3 to 4 k Sv. C. nutritional supplement each day.1. warm mixing water.3. provide him with a copy of Canada’s when required Food Guide to Healthy Eating. C. A crown margin can be extended subgingivally D.4/2. Zygomatic process of the maxilla and B. and translation. agent. the format of some items is not currently used. a longer mixing time. Incisive. for esthetics. Which of the following root surfaces are most likely to have flutings/concavities that will make thorough root debridement difficult? A. Mesial of teeth 1. C. Floor of the nasal fossa and maxillary A.4.(m) Sv. D. (2) and (4) B. Mental. Anterior nasal spine and to nasopalatine/incisive canal. Nasopalatine/incisive canal and floor of 8 months but is otherwise in good health. D. In addition. D. 1. C. Lingual. Looking wasted and fatigued. The NDEB periodically reviews the bank to improve its quality. 3.2. A. occurs with premature loss of primary D. the mechanical equivalent of the Curve of B. content. there is a high. A. guide table represents A. B. at the cervical line. the incisal D. molars. The full palatal strap major connector is indicated where The leeway space in an 8 year old child A. 15-20% maleic acid. undercut palatal torus is permanent incisors. survey line. D. contact in lateral excursion. The NDEB periodically reviews the bank to improve its quality. terminal line. allows accommodation of premolars that are larger than the primary molars. D. and dentin is A. axis of rotation/fulcrum line. and translation. rotation is located C. Wilson.The maximum recommended thickness of an The characteristics of "Group function incremental composite resin to be light cured is occlusion" are: A. palatal tissue is soft and compressible. A. B. The teeth on the working side make D. present. C. D. within the apical half of the root. will provide space for eruption of the B. B. the anterior equivalent of condylar crown of a single-rooted tooth. contact in lateral excursion. very few teeth remain in a flat or U. Note: Some of the items in the Released Test Item Bank may have been discontinued due to outdated science or errors. a well-defined. C. narrow palatal vault. shaped arch. D. The posterior teeth on both sides make The most effective agent used to etch enamel contact in lateral excursion. In addition. 4mm. 5-10% tartaric acid. mandibular arch. line of greatest torque. 3mm. The teeth on the non-working side make B. is greater in the maxillary arch than in the C. 30-40% phosphoric acid. The line drawn through the occlusal rests of C. C. 2mm. the format of some items is not currently used. When a simple tipping force is applied to the B. at the apex. A. C. 5mm. within the cervical one third of the root. ©The National Dental Examining Board of Canada 2017 . C. is approximately 3. the centre of guidance. two principal abutments is D. B. a mechanical equivalent of the horizontal and vertical overlap of the anterior teeth. 40-50% citric acid. On a semi-adjustable articulator.5mm in the mandibular arch. E. Only canine and lateral incisors make contact in lateral excursion. a reference point for the establishment of occlusal vertical dimension. opened by the tensor veli palatine muscle. D. bacteria. D. which ligament? B. Jugulo-omohyoid. pacinian corpuscles. yeast. B. High carbohydrate diet. content. B. C. B. nociceptors. C. High protein diet. a virus.Which of the following may result in acetone Pain caused by trauma to the oral mucosa breath? would activate A. In addition. B. low-threshold mechanoreceptors. odontoblastic tubules help convey hydrostatic forces to the pulp cells. D. Submandibular. E. and translation. Mastoid. ©The National Dental Examining Board of Canada 2017 . closed by the superior constrictor muscle. Stylomandibular. D. D. merkel disks. D. Lymphatic spread from a carcinoma on the tip of the tongue would initially involve which Immediately following an inferior alveolar nodes? nerve block. C. A. A. A. B. immediately after making the final casts. Prolonged fasting. C. Pterygomandibular. The needle has penetrated through A. closed by the levator veli palatine muscle. opened by the tensor tympani muscle. Reduced thyroid hormone level in a child is associated with The dentino-enamel junction is the most sensitive portion of a tooth because A. after the diagnosis and treatment plan has C. free nerve endings terminate on C. C. C. Deep parotid. A. E. C. odontoblasts at this region. Poor oral hygiene. muscle spindles. the auditory A. been established. the format of some items is not currently used. B. Note: Some of the items in the Released Test Item Bank may have been discontinued due to outdated science or errors. D. delayed tooth eruption. D. A. upon delivery of the denture. lack of tooth eruption. the patient exhibits facial paralysis. Sphenomandibular. B. ameloblasts make synaptic connections with odontoblasts at this junction. B. The NDEB periodically reviews the bank to improve its quality. Stylohyoid. mycoplasma. closed by the tensor tympani muscle. Submental. odontoblastic processes branch considerably at this region. Adjustment of the occlusal plane of natural teeth opposed by a complete or partial denture should be completed During the act of swallowing. supernumerary teeth. early tooth eruption. E. after the teeth have been set on the trial (pharyngotympanic) tube is denture. Whooping cough is caused by D. lymphocytes. macrophages. 1. The washing of hands must be performed B. Note: Some of the items in the Released Test Item Bank may have been discontinued due to outdated science or errors. malleability. decreased by work hardening. eosinophils. higher than the proportional limit. hands are moist. neural crest cells. 4. the same as the proportional limit. In addition. A. middle third of the root. before putting on and after removing gloves C. contact there are bilateral posterior crossbites E. completely eliminates skin bacteria. D. content. Hardening of Type IV cast gold dental alloys by heat treatment increases A. two ideal occlusions. 2. B. existing odontoblasts. The most likely cause of this finding is A. (1) and (3) C.A patient with multiple small bruises (purpura) When odontoblasts are destroyed. reduces the number of skin bacteria which multiply and cause irritation. Accessory canals in permanent teeth are most 3. platelets. erythrocytes. midline central diastema. C. C. C. T-cells. All of the above. shows a B. The NDEB periodically reviews the bank to improve its quality. the format of some items is not currently used. ©The National Dental Examining Board of Canada 2017 . allows gloves to slide on easier when the B. At initial D. A patient. A. elastic modulus. and coincident midlines. occlusal interference and functional shift. minimizes the transient bacteria which commonly found in the could contaminate hands through small pinholes. midline that is deviated to the right. lateral incisors. true unilateral crossbite. E. severe temporomandibular dysfunction. undifferentiated cells. B. the same as the stress at fracture. when in full intercuspation. (4) only E. (2) and (4) D. D. yield strength. cervical third of the root. delayed exfoliation of primary canines. ductility. apical third of the root. and translation. because it D. right side posterior crossbite and a lower C. B. D. C. D. B. A clinical diagnostic indication of palatal The yield strength of an orthodontic wire is impaction of maxillary permanent canines does NOT include A. proclined and laterally flared permanent C. coring. new most likely has a low count of odontoblasts are derived from A. lack of canine buccal bulges in a 10 year old patient. A. A. (1) (2) (3) B. a self. gram-negative microorganisms. viruses. threading pin will B. cause vasodilation. consists primarily of C. C. Prevotella. infraorbital. D. the format of some items is not currently used. WITHOUT water coolant will A. Porphyromonas. T cells. In addition. technique would be B. D. Adrenal corticosteroids A. D. The NDEB periodically reviews the bank to improve its quality. C. D. D. intraligamentary. increase heart rate. Streptococcus. C. A. A. reduce clogging of dental bur. produce a smoother surface. B. increase frictional heat. increase tensile strength of an amalgam D. reduce debris accumulation. middle superior alveolar. A. aerobic microorganisms. reduce inflammation. the most appropriate local anesthetic A. plasma cells. Note: Some of the items in the Released Test Item Bank may have been discontinued due to outdated science or errors. increase the retention of an amalgam Using a high speed dental handpiece restoration. The tooth lies must be extracted.There is an acute apical abscess (acute In the normal periodontium. In addition to a palatal injection. Subgingival plaque in deep periodontal pockets B. D. gram-positive microorganisms. A. 1 to 2mm coronal to the CEJ. 1 to 2mm apical to the CEJ. restoration. ©The National Dental Examining Board of Canada 2017 . increase protein synthesis. C. Actinobacillus. B. cause pulpal inflammation. at the CEJ. B cells. vestibular infiltration. and translation. The cell-mediated immune response to bacterial plaque in chronic periodontitis is regulated by When placed into sound dentin. A. strengthen the amalgam. tooth surface are associated with the genus C. content. B. neutrophils. E. the alveolar crest periradicular abscess) on tooth 1. B. C. B. C. decrease pulpal damage if used with light The microorganisms that initially colonize the pressure.3. features of an arrested carious lesion in an C. subsequent crown margins are not located An amalgam coronal-radicular core build-up on cementum. B. (2) and (4) D. C. preparation with 3. (1) (2) (3) C. subgingival proximal extension. desensitizes the pulp. the resin is autopolymerizing. B. outline form. luting porcelain (all ceramic) crowns. B. there is an adequate ferrule. E. and eugenol cements. bond mechanically to calcium in enamel temporary restoration is that it and dentin. a stepped buccal or lingual wall. have a lower modulus of elasticity than The most important advantage of using zinc phosphate cements. Conventional glass ionomer cements C. is most likely due to a 2. retention form. C. C. sublingual space. The placement of a reverse curve in a Class II amalgam preparation aids in Composite resin is a satisfactory core material A. A brown spot lesion with a hard surface. A. D. and translation. occlusal fissure system). In addition. B. B. inadequate isthmus depth. Note: Some of the items in the Released Test Item Bank may have been discontinued due to outdated science or errors. a pulp chamber. A small cavitated lesion with exposed dentin. such as calcium hydroxide in a deep preparation. A white spot lesion with a frosty surface. C.Periapical infection from a mandibular second Isthmus fracture during function in a recently molar may spread by direct extension to the placed proximal-occlusal silver amalgam restoration (with occlusal extension through the 1. occludes dentinal tubules. A. elicit less pulp response than zinc-oxide D. the resin has a high contrast colour with D. stimulates dentin repair. resistance form. for endodontically treated molar teeth requires A. (1) and (3) D. the format of some items is not currently used. submandibular space. do not require a protective liner. reinforced zinc oxide eugenol cement as a D. All of the above. The NDEB periodically reviews the bank to improve its quality. B. A. the presence of a post. B. inadequate isthmus width. tooth structure. A lesion extending into dentin on a bite- wing radiograph. ©The National Dental Examining Board of Canada 2017 . buccal space. ferrule and amalgam Which of the following is consistent with the bonding. occlusal fissure? D. A. for endodontically treated teeth provided B. convenience form. an adequate pulp chamber and ferrule. (4) only E. are superior to zinc phosphate cement for A. C. 4. chelates to tooth structure. the use of retentive threaded pins. content. buccal vestibule. A. D. Trigeminal. enhance the progression of caries by treatment in the dental office? metabolizing sucrose. Trochlear. act symbiotically with S. D. faster for myelinated fibres than for Postoperative sensitivity associated with the unmyelinated fibres. D. C. increasing filler content decreases A. B. convert lactate to acetic and propionic Which of the following conditions acid. content. Note: Some of the items in the Released Test Item Bank may have been discontinued due to outdated science or errors. galvanic conduction. CONTRAINDICATES routine dental C. Astrocytes are cells found in the Most of the problems associated with direct A. polymerization shrinkage. Glossopharyngeal. independent of myelination. and translation. the format of some items is not currently used. independent of the axon diameter. D. A. mutans to decrease pH and promote caries activity. increasing filler content increases the C. polymerization shrinkage. B.In the formulation of composite resins Most of the somatosensory information from the oral cavity reaches the brain through which A. The NDEB periodically reviews the bank to improve its quality. D. B. A. B. peripheral nerves. B. D. excessive thickness of a glass ionomer Veillonella species in supragingival plaque base. In addition. C. placement of direct posterior composite resin B. Insulin-dependent diabetes. degree of conversion. Recent (15 days) myocardial infarct. D. decreasing filler content increases radiopacity of the restoration. slower for myelinated fibres than for restorations is most often the result of unmyelinated fibres. C. Hypothyroidism. brain. D. inadequate etching of the dentin. A. related to C. Second trimester pregnancy. C. ©The National Dental Examining Board of Canada 2017 . blood. B. large particle size of macrofilled composite resins. tongue. D. Hypoglossal. C. act as pioneer microorganisms in the A. poor shade selection. decreasing filler content increases the nerve? modulus of elasticity. posterior composite resin restorations are B. polymerization shrinkage. high thermal conductivity. development of plaque. The speed of propagation of action potentials along axons is A. The location and extent of subgingival calculus is most accurately determined clinically by Which of the following is NOT commonly used by general dentists to treat temporomandibular A. C. content. D. apical to alveolar bone crest. C. The NDEB periodically reviews the bank to improve its quality. C. cephalosporin. and translation. probing with a fine instrument. diffuse gingivitis is A. erythromycin. Hard acrylic splints. fiery-red. painful. B. 6-30%. sensitivity. Corticosteroids. has been shown to be in the range of B. In addition. inadequate base thickness.The risk of contracting hepatitis B following a Aging pulps show a relative increase in percutaneous injury from an occupational exposure to the contaminated blood of a patient A. In an infrabony pocket. D. placement of a conservative posterior D. C. aphthous stomatitis. unpolymerized resin. A patient complains of sensitivity following C. A. A. idiopathic fibromatosis. dental abscess is B. cell numbers. 72-90%. B. C. primary herpetic gingivostomatitis. composite resin restoration. calcification. C. metronidazole. B. disclosing solution. within basal bone. The most probable cause is A. B. acid etching. Nonsteroidal anti-inflammatory drugs D. E. (NSAIDs). ©The National Dental Examining Board of Canada 2017 . Muscle relaxants. D. 35-66%. polymerization shrinkage. the epithelial attachment is located The antibiotic of choice for a periradicular A. B. penicillin V. ampicillin. vascularity. coronal to alveolar bone crest. D. prolonged application of the curing light. The most likely diagnosis for a child with a E. A. Note: Some of the items in the Released Test Item Bank may have been discontinued due to outdated science or errors. B. radiopaque solution used in conjunction disorders and bruxism? with radiographs. the format of some items is not currently used. visual inspection. C. aggressive periodontitis. Regarding dental caries. increased brittleness of the jaws. B. D. Gingival hyperplasia. More frequent consumption of When a radiographic examination is warranted carbohydrates increases the risk. C. and translation. use high speed film. secondary cementum. D. is correct? A. C. Propoxyphene. aldehyde. A. A. A. ester. All carbohydrates are equally cariogenic. content. (2) and (4) Which of the following drugs is used in the D. deformity of the jaws. tertiary dentin. water. (1) and (3) C. Loss of stippling. take a panoramic film only. (4) only treatment of mild allergic reactions? E. E. the format of some items is not currently used. reduced vascularity of the jaws. All of the above. The NDEB periodically reviews the bank to improve its quality. A. B. B. 4. aminide. be placed in D. Procaine (Novocaine®) is an example of a local anesthetic which is chemically classified as an An end result of ionizing radiation used to treat oral malignancies is A. C. use a thyroid collar and lead apron. D.Which of the following is/are clinical signs of A protective mechanism of the dental pulp to gingivitis? external irritation or caries is the formation of 1. C. D. Meperidine hydrochloride. C. 100% relative humidity. a 1% aqueous calcium sulfate solution. 3. A. Lidocaine (Xylocaine®) is an example of a E. Isoproterenol. for a 10 year old child. Diphenhydramine hydrochloride. Increased dietary fat increases the risk. Note: Some of the items in the Released Test Item Bank may have been discontinued due to outdated science or errors. The rate of carbohydrate clearance from to decrease radiation exposure is to the oral cavity is not significant. apply a radiation protection badge. A. amide. the most effective way C. primary dentin. amide. increased vascularity of the jaws. In addition. it should C. B. C. B. decrease the kilovoltage to 50kVp. 2. B. If an alginate impression must be stored for a B. aldehyde. few minutes before the cast is poured. B. aminide. Decreased pocket depth. ethamine. D. E. ethamine. local anesthetic which is chemically classified as an A. pulp stones. ester. (1) (2) (3) B. which of the following D. ©The National Dental Examining Board of Canada 2017 . Bleeding on probing. neonatal hypoplasia. Pain upon biting pressure. E. Chronic periradicular periodontitis. All of the above. hereditary factor.5. C. cleidocranial dysplasia. and translation.Which of the following is most often associated A 4 year old child has a normal complement of with a nonvital tooth? deciduous teeth. B. The NDEB periodically reviews the bank to improve its quality. excessive thyroid hormone. (1) (2) (3) C. B. development of tooth 1. Dentigerous cyst. C. (1) and (3) D. C. acute inflammation of the pulp. generalized calcium deficiency. Which of the following features would be most indicative of a cracked tooth? A. (2) and (4) D. Periapical cemento-osseous dysplasia. (4) only E. 4. Condensing osteitis in the periapical region is 2. content. D. D. A. dentin in these teeth. B. Chronic periradicular periodontitis. of A. Myxedema is associated with C. B. Radiographic examination B. but in appearance they are grayish and exhibit extensive occlusal and A. All other teeth are normal. infection of tooth 5. pulpal abscess. C. ©The National Dental Examining Board of Canada 2017 . Hyperplastic pulpitis. Note: Some of the items in the Released Test Item Bank may have been discontinued due to outdated science or errors. excessive orthodontic forces. A.5. Internal resorption. amelogenesis imperfecta. excessive parathyroid hormone. D. In addition. This condition is typical D. B. periapical cemento-osseous dysplasia. high fever encountered by the patient B. A 15 year old presents with hypoplastic enamel on tooth 1. when he had measles at age 3. Absent vitalometric response. chronic inflammation of the pulp. B. D.5 during the D. vitamin D deficiency. incisal wear. indicates some extensive deposits of secondary C. C. dentinogenesis imperfecta. indicative of a/an 3. the format of some items is not currently used. insufficient thyroid hormone. Root resorption of permanent teeth may be associated with 1. Pulp polyp. early apical abscess formation. Hypersensitivity to thermal stimuli. traumatic injury. insufficient parathyroid hormone. A. chronic periradicular periodontitis. Lateral periodontal cyst. This was most probably caused by a/an Which of the following results from a necrotic pulp? A. Periapical radiolucency. A. D. Note: Some of the items in the Released Test Item Bank may have been discontinued due to outdated science or errors. Marginal ridge. (2) and (4) D. C. Periapical cemento-osseous dysplasia. cold and On a bite-wing radiograph of posterior teeth. The microscopic appearance of the central giant cell granuloma of the jaws is similar to that of lesions which occur in A well circumscribed 3mm radiolucent lesion is present in the apical region of the mandibular A. Odontogenic adenomatoid tumor. (1) (2) (3) C. 3. Calcifying epithelial odontogenic tumor. Lateral periodontal. B. In addition. 2. B. (1) (2) (3) B. (4) only Radiographically.Which of the following is/are associated with An ameloblastoma can develop from the an unerupted tooth? epithelial lining of which of the following cysts? 1. A. B. C. Calculus. C. (1) and (3) C. All of the above. Apical cyst. A. 2. nasolabial cyst. a periradicular periodontitis. cleidocranial dysplasia. Dentigerous. Periapical cemento-osseous dysplasia. E. to vitality tests. found in the permanent dentition. canal may be misdiagnosed as a 1. The NDEB periodically reviews the bank to improve its quality. A. Carabelli cusp. the format of some items is not currently used. (2) and (4) D. and translation. B. 4. Residual. associated with a root fracture. ©The National Dental Examining Board of Canada 2017 . The tooth responds normally B. hyperparathyroidism. branchial cyst. a dentigerous cyst. the mental foramen. C. content. second premolar. Cementoblastoma. infraerupted. nonvital. C. D. Paget's disease. A. nasopalatine cyst. a rarefying osteitis. All of the above. electric pulp test? which of the following is most likely to be misdiagnosed as proximal caries? A. Acute apical abscess. A. periradicular cyst. Cemental tear. hyperpituitarism. 4. Periradicular. An ankylosed tooth is usually 3. For which of the following pathological conditions would a lower central incisor tooth be expected to respond to heat. D. (4) only E. the opening of the incisive E. Chronic apical periodontitis. Cemento-enamel junction. D. likely D. A. (1) and (3) D. The radiolucency is most C. B. B. before birth. D. The most appropriate radiographic examination for a 4 year old without visible or clinically detectable caries or anomalies. A. D. C. a pair of posterior bite-wings. hypercementosis. is nonvital. A. ©The National Dental Examining Board of Canada 2017 . C. cap the exposure with calcium hydroxide periapicals. The most likely old child is diagnosis is A. has a periodontal pocket. B. In addition. There is A. D. B. condensing osteitis. The treatment should be to B. 4 years. The mandibular incisors and permanent first molars. extraction of the opposing molar. B. maxillary and mandibular posterior A. B. D. maxillary and mandibular anterior no periapical involvement and the tooth is vital. A. acute osteitis. The The probable age at which this child received radiograph will show tetracycline therapy was A. once the patient has reached the age of 12. E. C. to perform space analysis. normal lamina dura. the patient returns complaining of a sharp pain of short duration when eating or drinking something cold. an apical radiolucency. A 6 year old patient has a larger than average A cold stimulus applied to a tooth will produce diastema between the maxillary central a hypersensitive response if the tooth incisors. tooth is vital and tender to percussion. E. Note: Some of the items in the Released Test Item Bank may have been discontinued due to outdated science or errors. and place zinc-oxide and eugenol. content. C. after its complete eruption. and with open proximal contacts is A large carious exposure occurs on a permanent first molar of a 7 year old. acute periradicular periodontitis. has a hyperemic pulp. In order to manage the diastema. perform a pulpotomy and place calcium hydroxide. B. extract the tooth and place a space One week after an amalgam restoration is maintainer. D. C. D. The radiographic examination shows a mesiodens. only if it develops into a cystic lesion. 6 years. The NDEB periodically reviews the bank to improve its quality. Teeth respond normally to The most appropriate treatment following the electric pulp testing and heat and the extraction of a first primary molar in a 4 year radiographs are normal. the format of some items is not currently used. 1 year. extraction of the contra-lateral molar. A. placed in the mandibular first premolar. root resorption. reversible pulpitis. C. you should extract the mesiodens B. no radiographic examination. insertion of a space maintainer. D. has chronic proliferative pulpitis. and translation. regular assessment of arch development. B. C.A patient complains of acute pain 24 hours A 12 year old child presents with characteristic after the insertion of a restoration in a tooth tetracycline discoloration of the maxillary and with no preexisting periapical pathology. C. D. occlusals. pulpal microabscess. perform a pulpectomy. as soon as possible. hyperthyroidism. place a cervical headgear to reposition C. myxedema. C. space maintainer is B. erupted. hyperparathyroidism. (4) only B. The management of this patient should be to Epinephrine should NOT be used as a A. consultation. disk the distal surfaces of primary mandibular second molars to allow normal adjustment of permanent molars. C. pyramidal fracture of the maxilla. administer 100mg hydrocortisone intravenously. refer the patient to an orthodontist for A. immediately after extraction of the primary second molar. Note: Some of the items in the Released Test Item Bank may have been discontinued due to outdated science or errors. refer the patient to a hospital. the patient's face 4. after the permanent second molar has E. the format of some items is not currently used. maxillary molars. B. D. asthma. craniofacial dysjunction. fistula? 1. Excessive tissue tension. B. and translation. E. Poor flap design. alveolar block injection. D.A Le Fort I or Guerin fracture is a To prevent mesial drift of a permanent first molar. use hot packs over the swelling. 2. observe. Sinus infection. the ideal time to place a distal extension A. fracture of the zygomatic arch. administer diphenhydramine An 8 year old patient with all primary molars hydrochloride (Benadryl®) 50mg still present exhibits a cusp-to-cusp relationship intravenously. horizontal fracture of the maxilla. fracture of the malar complex involving A. use pressure followed by cold packs over C. (1) (2) (3) B. D. All of the above. plan serial extractions for more normal vasoconstrictor for patients with uncontrolled adjustment of the occlusion. The NDEB periodically reviews the bank to improve its quality. gingival tissue. as soon as the extraction site of the Which of the following will impede healing primary second molar has completely following the surgical closure of an oroantral healed. ©The National Dental Examining Board of Canada 2017 . E. as soon as the tooth erupts through the the floor of the orbit. C. of permanent maxillary and mandibular first molars. content. D. C. E. (1) and (3) A. B. becomes quickly and visibly swollen. In addition. D. (2) and (4) the swelling. Immediately following a posterior superior 3. D. Blowing the nose. The immediate treatment should be to A. 2-5mg. is initiated primarily on the tooth surface beyond the restoration. sharp line angles in the tooth preparation. occurs in the pits of tooth structure. 500-1000mg. the tooth. the most definitive feels unwell following the administration of a test to confirm the loss of pulp vitality is local anesthetic and becomes pale and sweaty. Organ transplant. This condition does not respond to placing the A. C. compound odontoma. Pindborg tumor. B. The usual adult dosage of codeine administered orally is A fracture in an all-ceramic crown may be caused by A. ©The National Dental Examining Board of Canada 2017 . fluoroaluminosilicate powder and polyacrylic acid. C. B. (1) and (3) C. 3.A known insulin-dependent diabetic patient Under normal conditions. E. cause is C. 4. Valvular heart disease. hyperglycemia. 1. B. content. All of the above. D. performing a radiographic examination of A. Persistent odontogenic fistula. adrenal insufficiency. is caused by voids within the restoration entirety. and translation. (2) and (4) A. cutting into the dentin without anesthetic. orthophosphoric acid. applying ethyl chloride to the crown. Cardiac prosthesis. E. C. In addition. D. fluoroaluminosilicate powder and D. is caused by microleakage at the D. A. D. the format of some items is not currently used. B. performing an electric pulp test. The most likely B. D. inadequate ceramic thickness. (1) (2) (3) Recurrent caries B. complex odontoma. (4) only restoration-tooth interface. Immunosuppressive therapy. A radiopaque area within the alveolar process containing several rudimentary teeth suggests a/an Generally. 30-60mg. syncope. zinc oxide and polyacrylic acid. E. applying warm gutta percha to the crown. use of an inappropriate luting material. C. A. periapical cemento-osseous dysplasia. B. D. The NDEB periodically reviews the bank to improve its quality. 2. 250-500mg. C. B. patient in a supine position. C. excessive occlusal load. hypoglycemia. glass ionomer cements contain A. zinc oxide and distilled water. require antibiotic premedication before endodontic therapy? A. Note: Some of the items in the Released Test Item Bank may have been discontinued due to outdated science or errors. E. E. ameloblastoma. Which of the following conditions would NOT D. carotid sinus reflex. of the B. An anterior endodontically treated tooth has been restored with a carbon fibre. C. Class IV restoration. atrophy of the posterior pituitary. cigarette smoking. A. weak amalgam at the margin. Unmyelinated nerves found in the pulp that respond to thermal. Direct veneer restoration. and translation. C. structure. When compared to permanent teeth. A. Note: Some of the items in the Released Test Item Bank may have been discontinued due to outdated science or errors. acquired xerostomia. outline form. B. B. D. hyperplasia of the anterior pituitary. C. a greater thickness of dentin. A. B. C. a greater thickness of enamel. chemical and inflammatory stimuli are called A. crown size. D. B. A. C. Amount of remaining coronal tooth C. ©The National Dental Examining Board of Canada 2017 . C. Presence of enamel on the entire periphery. What is the most important factor margin of a Class II amalgam cavity influencing the prognosis of this tooth? preparation will result in A. poor oral hygiene. more prominent cervical constriction. Preventive resin restoration. convenience form. D. Size of the restoration. unsupported enamel at the margin. D. content. a hyperactive thyroid. D. development of root caries is B. C fibres. D. B. Type of luting cement. A delta fibres. primary teeth have The primary etiological factor for the A. the format of some items is not currently used. In addition. poor retention. Alloy composition of the post. D. retention form. hyperplasia of the parathyroids. gingival recession.In which of the following will the effects of Adjustments made to Class II amalgam polymerization shrinkage be greatest? preparations to ensure the tooth structure does not fracture under masticatory forces is a part A. poor esthetics. D. The NDEB periodically reviews the bank to improve its quality. mechanical. Gigantism is caused by What is the best predictor of success for a composite resin restoration? A. C. sympathetic fibres. a direct restorative core and a porcelain fused to metal Excessive flare of the distobuccal cavosurface crown. D. B. pulps which are smaller in relation to B. dentinal tubules. Type of core material. Presence of flat dentinal walls. Class I occlusal restoration. C. resistance form. Depth of the restoration. Subjects in this trial are protected by the B. A. can remineralize early root carious lesions. the format of some items is not currently used. C. exaggerated due to an altered host immune response. Note: Some of the items in the Released Test Item Bank may have been discontinued due to outdated science or errors. A subpoena is required for the dentist to and lactation. Which of the following statements is correct? A. In addition. Drugs Act. A. Calcitonin gene related peptide. women over 70 years require more than men over 70 years. Ethyl chloride. more calcium is required during pregnancy C. Prostaglandin E2. C. Which of the following statements is correct? Regarding the Dietary Reference Intake (DRI) for calcium. D. ©The National Dental Examining Board of Canada 2017 . Treatment inflammation and pain when released by pulpal included a pulpotomy and a prescription for fibres? 30 tabs of Percocet®.Which of the following substances causes A new patient had an acute pulpitis. a police officer that it was very easy to get B. The NDEB periodically reviews the bank to improve its quality. the same in all ages. plaque-free teeth. under the age of 18. decreased due to an altered host immune legislated to follow the Tri-Council Policy response. a police officer attends the office D. B. B. D. exaggerated due to a change in the type of clinical trial regulations of the Food and infecting microorganisms. A. content. narcotics from his new dentist. C. B. present relevant documents. causes unsightly stain on exposed roots. C. explaining that he is conducting an investigation into health professionals complicit in narcotic trafficking. Dentists will receive a $500 finder’s fee for each subject Gingival response to plaque microorganisms in recruited. boys require more calcium than girls. Acetylcholine. Known to local police as a drug user. Dentists in a prosthetic study club have been asked to recruit subjects for a clinical trial of a new dental implant system. and translation. Ice water. B. Subjects will receive the treatment elderly patients is for free. C. Free treatment for subjects is justified due Duraflor® to the time and inconvenience. upon request. The finder’s fee is justified because of the time required to identify suitable subjects. Ethics review of clinical trials in Canada is D. later. is only effective on dry. Statement. Patient files must be released to police amount. Which of the following agents is most effective D. D. Air jet. should only be used on individuals in in cold testing? unfluoridated areas. A. There is a legal duty to assist in any criminal investigation. adult men and women require the same B. Neuraminase. CO2 (dry ice). A. Several weeks C. the patient subsequently bragged to A. who are elderly. taking systemic antibiotics. D. D. 2. Subgingival restoration. C. (4) only E. simultaneous placement of adjacent proximal restorations. Supragingival calculus removal. light condensing forces. 2. B. All of the above. C. Hypoparathyroidism. B. and translation. Note: Some of the items in the Released Test Item Bank may have been discontinued due to outdated science or errors. C. B. strength. (1) (2) (3) B. A. tightening the matrix band. In addition. appropriate wedge selection. 4. (2) and (4) C. Neutrophils. B. 4. Mandibular block anesthesia. The NDEB periodically reviews the bank to improve its quality. A delta and C nerves. acne rosacea. D. the format of some items is not currently used. Which of the following structures may be associated with the role of the central nervous Which of the following presents with high system in sleep (nocturnal) bruxism? serum calcium levels.A Vitamin B2 (riboflavin) deficiency usually Which of the following cells are characteristic arises in patients of chronic inflammation of the dental pulp? 1. Incisional biopsy. consuming a high protein or fat diet. resistance to corrosion. lupus erythematosus. C. Plasma cells. A. Macrophages. ductility. 3. pemphigus vulgaris. Hyperparathyroidism. All of the above. (1) and (3) C. Hyperthyroidism. C. (4) only D. teeth? B. 3. A "butterfly-rash" of the face can sometimes be An open proximal contact on an amalgam found in restoration can be prevented by A. D. Hypothyroidism. Which of the following dental procedures could be performed with minimal risk for a 35 year old patient with a severe bleeding Cold working a metal increases its disorder? A. A. with acute infection. ©The National Dental Examining Board of Canada 2017 . Petrous nerves. content. hardness. D. Sphenopalatine ganglion. (2) and (4) D. Lymphocytes. (1) and (3) B. D. erythema multiforme. B. thinning of cortical bone and giant cell osteoclasts in the jaw and drifting A. (1) (2) (3) A. A. E. Basal ganglia (nigrostriatal). 1. C. the format of some items is not currently used. enamel. It bonds on its own to tooth structure. Gingival inflammation. C. In addition. materia alba. Which of the following indicates a failure of a dental implant? Streptococcus mutans selectively colonizes A. age and gender of the patient. ©The National Dental Examining Board of Canada 2017 . C. Porins. B. biofilm. herpangina. tongue. B. A. plaque. A thin layer can be left to seal the A. Hypercementosis. Increased probing depths. D. Curve of Wilson. optimal results are obtained by considering all the following EXCEPT A. the new glycoprotein coating which forms on the surface is called A. content. Teichoic acid. lower lip line. C. D. papilloma. Mobility. Multiple odontogenic keratocysts. B. Demineralization of teeth. midline orientation of the incisors. implant length. verruca vulgaris.What is the most significant radiographic After a tooth surface has been completely finding in hyperparathyroidism? cleaned. D. Generalized loss of lamina dura. Lipopolysaccharide. In providing maxillary anterior esthetic Coxsackie A virus is the etiologic agent in dentistry. The NDEB periodically reviews the bank to improve its quality. C. C. condyloma accunimatum. C. D. B. C. C. cementum. structure. B. Which of the following bacterial components is a potent inducer of proinflammatory cytokines Which statement is true regarding amalgam? and can cause fever and shock? A. It is time-consuming to place compared to composite resin. buccal mucosa. B. cavosurface margins. Horizontal bone loss of one third of the A. B. D. Note: Some of the items in the Released Test Item Bank may have been discontinued due to outdated science or errors. B. pellicle. D. Peptidoglycan. It wears at a rate similar to that of tooth D. and translation. D. Rampant caries. A. E. C. Patient’s gender. effectiveness of the irrigation of the socket. For an avulsed. be placed on the hands before tying on a C. C. when used as a luting A. fine tactile discrimination. Reduced polymerization shrinking effect. A. the format of some items is not currently used. Anticariogenicity. maxillary and ethmoidal sinuses. Chemical adhesion. D. Insolubility. E. use of an appropriate antibiotic. Which of the following would indicate a right A. mask. The NDEB periodically reviews the bank to improve its quality. buccal superficial fascial and canine B. C. D. pressure. B. be washed with mild soap and warm water after a dental procedure before treating the next patient. Amount of bodily fluid involved in the injury. temperature. D. Mechanical retention. submandibular and sublingual regions. orbital floor fracture? B. B.Which of the following does NOT need to be Which of the following is NOT an advantage of considered when assessing whether exposure of using an etchant and bonding system before bodily fluids to a dental health care personnel insertion of a composite resin restoration? is significant? A. For sterilization to occur in an autoclave. Ludwig’s angina initially involves a massive infection of the Zinc phosphate cement. Type of bodily fluid. D. B. all of the following must be monitored EXCEPT A patient is hit in the right eye with a baseball. Note: Some of the items in the Released Test Item Bank may have been discontinued due to outdated science or errors. ©The National Dental Examining Board of Canada 2017 . and translation. Right periorbital edema. D. parapharyngeal and retropharyngeal agent. spaces. fully developed permanent B. prognosis is most affected by the C. be removed after use and replaced with tooth that has been reimplanted. B. not be used if the patient reports a latex allergy. B. not be used in dental procedures requiring mouth. rigidness of the splint. Improved retention. D. Limited upward movement of the right eye. Infectious disease status of the patient. D. Greater strength of the restoration. In addition. A. content. C. C. A. C. has which of the following properties? spaces. thoroughness of the curettage of the root surface. time. Premature occlusal contact on the Gloves should maxillary right. A. A. Right periorbital ecchymosis. length of time the tooth has been out of the D. chemical levels. a favourable new gloves before treating another patient. E. Decreased marginal leakage. D. Note: Some of the items in the Released Test Item Bank may have been discontinued due to outdated science or errors. Milk. Atropine. the porcelain thermal expansion coefficient should be slightly lower than the metal. friend or acquaintance. (factor V Leiden) are at risk for B. Which of the following drugs to sintering should be avoided. and translation. Saliva. thrombocytopenia. fusing should be followed by quenching. osteocytes. a patient becomes apneic and B. content. In the absence of “Hanks balanced salt solution”. B. D. osteoclasts. the cells that generate new bone tissue are Patients with resistance to activated protein C A. D. fibrinolysis. mobility and displacement. In addition. C. The NDEB periodically reviews the bank to improve its quality. C. should be administered immediately? C. C. A. When producing a metal-ceramic crown restoration A. is a/an B. D. C. D. acetaminophen. Naloxone. Splinting of a permanent maxillary incisor following trauma is required in the management of a The most common abuser of an elderly person A. hypercoagulation. osteophytes. Propranolol. D. no mobility and no displacement. luxation injury. benzodiazepine. mobility and no displacement. the body-porcelain layer should be rich in Following the IV administration of a opacifying oxides. hydrocodone and acetaminophen. C. B. A. the format of some items is not currently used.A subluxated tooth will have The most appropriate analgesic for a pregnant patient is A. D. Class II fracture. adult child. acetylsalicylic acid. B. spouse. osteoblasts. concussion injury. oversizing the body-porcelain layer prior unresponsive. sibling. B. E. erythrocytosis. Tap water. A. C. B. A. Saline. E. bleeding diatheses. Flumazenil. ibuprofen. what is the most appropriate media to transport an avulsed tooth? A. Amphetamine. C. During healing following implant placement. Class IV fracture. C. D. B. ©The National Dental Examining Board of Canada 2017 . B. Justice. D. B. Veracity. D. lateral periodontal cyst. paradental cyst. Mandibular central incisor. C. Note: Some of the items in the Released Test Item Bank may have been discontinued due to outdated science or errors. A panoramic radiograph and bite-wings are What is the most likely diagnosis for a patient required as part of an examination for a new who exhibits elevated blood pressure. Justice. Bronchitis. Mandibular first premolar. Autonomy. E. stylohyoid ligament. a new partner in the victim’s life. the highest risk for The most likely condition to mimic periodontal victim fatality is associated with disease is A. B. D. The most appropriate management is to A. The most common soft tissue calcification or ossification found on a panoramic radiograph is Which ethical principle is violated when a (a) dentist increases fees solely based on the fact that the patient is covered by dental insurance? A. lymph node. C. request the existing radiographs. phlebolith. and translation. C. Veracity. ©The National Dental Examining Board of Canada 2017 . B. A. C. B. Autonomy. make a panoramic radiograph and bite- B. D. dentist practices while impaired? B. B.In domestic violence cases. Beneficence. Nonmaleficence. Hypertension. swollen patient. A. disuse atrophy (ridge resorption). D. severe depression of the abuser. sialolith. Langerhans cell histiocytosis. C. The NDEB periodically reviews the bank to improve its quality. make a panoramic radiograph only. D. Congestive heart failure. C. In addition. prior threats to kill the victim. C. Nonmaleficence. distended neck veins and difficulty been taken in the last year by another dental breathing when lying down? office. a history of abuse of the abuser as a child. The patient states that radiographs had ankles. D. make bite-wings radiographs only. wings. A. D. the format of some items is not currently used. End stage renal disease. C. content. Maxillary lateral incisor. Maxillary first molar. an actual or pending separation. A. E. E. Beneficence. Which permanent tooth is most likely to have an anomaly? Which ethical principle is violated when a A. Lichen planus. A. NUG can be cured through modification of diet. a deficiency of certain nutrients causes A 70 year old female has consumed optimal NUG. Illustrating. C. C. Acetaminophen with codeine. because of all the sugar in them you’ll get D. The NDEB periodically reviews the bank to improve its quality. Bacteria in your mouth are the main cause of caries. Veracity. The patient’s diet is important to investigate further because A. cavities. Not brushing your teeth means the sugar from your snack attacks your teeth for A 20 year old student presents with clinical about twenty minutes. Candidiasis. levels of fluorides all her life. Autonomy. D. White sponge nevus. D. Leukoedema. ©The National Dental Examining Board of Canada 2017 . E. Explain that treatment cannot be initiated B. She could be B. D. and translation. Note: Some of the items in the Released Test Item Bank may have been discontinued due to outdated science or errors. Paget’s disease. patients with NUG lose interest in eating. Linea alba. Explaining how his present habits increase his caries risk. The ‘bugs’ in your mouth eat the sugar in (NUG). B. C. orthodontic treatment to straighten his teeth. He has continued to experience one to A. Justice. two carious lesions per year and now wants B. for this patient? A. symptoms of necrotizing ulcerative gingivitis D. Food intake for the last 24 hours the food you eat. A. When you eat doughnuts and soft drinks. management of myofascial pain syndrome? C. Hyperparathyroidism. vegetables. Which of the following analgesics is B. with the aid of drawings the CONTRAINDICATED for the long-term basic caries process. B. Acetylsalicylic acid with codeine. Ibuprofen. B. Oxycodone. Which of the following is the most appropriate D. In addition. NUG may be limiting the food choices the expected to have a reduced incidence of which patient is making. initial step in addressing behavior modification E. Beneficence. the format of some items is not currently used.Which ethical principle is involved with a A 28 year old male patient works long days that dentist’s duty to recognize signs of abuse and typically include numerous soft drinks and neglect and to know appropriate interventions? candy bars. and change it into acid indicates a soft diet lacking in fruits and which can make holes in your teeth. Osteoporosis. A. but no opportunity to brush his teeth. Dental caries. content. C. until he can keep his teeth clean. What is the most likely diagnosis of a white diffuse lesion with a wrinkled appearance on Which of the following explanations of the the buccal mucosa which disappears upon caries process is most appropriate for a stretching? 10 year old patient who snacks frequently on soft drinks and doughnuts? A. D. Non maleficence. Providing a new toothbrush and sample tube of dentifrice. C. of the following? C. E. A. content. Which of the following is increased after stimulating α1-adrenergic receptors? Which disease is associated with a higher A. D. ©The National Dental Examining Board of Canada 2017 . Rigor mortis is A. rest in a temperate climate is approximately A. caused by the disruption of D. 250ml. Muscle spindles associated with stretch C. proportion of Fusobacterium. B. Brush your teeth well after snacks. are composed of extrafusal muscle fibres. 5. D. elevated vagal tone. due to intracellular Ca2+ ion depletion. Root caries.5L. D. 500ml. C. 1. Early childhood caries. Aggressive periodontitis. Daily fluid loss from a healthy individual at E. In addition. D. Defects in the outflow tract of the heart in C. B. possess fibres that are capable of produced in contraction. Prevotella. hyperkalemia. the format of some items is not currently used. SA node activity. Blood vessel constriction. The NDEB periodically reviews the bank to improve its quality. neurulation. C. A. mechanistically identical to a tetanic individuals with craniofacial malformations are contraction. Chronic periodontitis. E. D. Bronchial dilation. characterized by spastic paralysis. acetylcholine. reflexes D. A piece of fruit and plain yogurt is an E. A. and spirochetes? C. Necrotizing ulcerative periodontitis. mediate autogenic inhibition of muscle contraction. C. Eat a bigger lunch and not snack. due to cellular ATP depletion. B. acceptable snack. E. epithelial cells of the small intestine.Which of the following dental health Heart rate is increased by recommendations is consistent with the general nutrition guidelines? A. an isotonic contraction. osteoblasts. D. neural crest development. Note: Some of the items in the Released Test Item Bank may have been discontinued due to outdated science or errors. Angiotensin II converting enzyme is primarily D. and translation. folding of the embryo. mesoderm development. atropine. C. B. hepatocytes.0L.0L. Contractility of the heart. 2. reduced α1–adrenergic receptor activity. B. B. properties of muscle. Porphyromonas. Diet soft drinks and potato chips are an C. acceptable snack. vascular endothelial cells of the lung. E. C. A. B. B. Kupffer cells. B. are innervated by alpha motor neurons. contribute to the main force production A. dentist to take for a female patient experiencing B. In addition. the format of some items is not currently used. 6. be associated with disease. A. augmented dentin sensitivity. be capable of causing disease in B.In the embryo which of the following could be To initiate caries. Which of the following fibre groups are attached to bone? What is the most appropriate action for a A. D. The physiologic rest position of the mandible is On the basis of “Socransky’s Modifications of Koch’s Postulates”. demonstrate virulence factors responsible for enabling the organisms to cause periodontal tissue destruction.9. Inner ear. D. Note: Some of the items in the Released Test Item Bank may have been discontinued due to outdated science or errors. B. Normal aging changes in the tooth tissues C. contains minimal amounts of collagen. Maxillary sinus. not humoral. B. Offer resource materials and referrals to community agencies to the patient. D. Apical. Counsel the patient to leave the abuser. A. experimental animal models.9. Speak to the alleged abuser to assess the level of threat to the patient. ©The National Dental Examining Board of Canada 2017 . affected by the time of measurement. Thymus. E. continuous deposition of cementum. Circular. increased porosity of enamel. following EXCEPT B. bacterial plaque pH must missing if pharyngeal pouch development is reach disrupted? A.4. B. D. 5. a position unrelated to tonicity of periodontal disease must possess all of the mandibular muscles. by increases in the number of organisms at D. A. A. Focus on dental matters and consider the include abuse issues in the future. spousal abuse? C. C. 6. C. Interradicular. Thyroid. in the form of Cementum alteration in the host cellular. as evidenced vertical dimension. has areas devoid of matrix-forming cells. C. C. B. be eliminated or decreased in sites that demonstrate clinical resolution of disease with treatment. and translation. the primary means of determining occlusal A. vascularized at its periphery. demonstrate a host response. record. immune response. Transseptal. used when making a centric interocclusal diseased sites. B. The NDEB periodically reviews the bank to improve its quality. the potential pathogens in A. content.4. D. C. C. 5. displacement of the gingiva for future impression making. the format of some items is not currently used. accumulation of plaque and gingival 3. Rhabdomyosarcoma. A. Adenocarcinoma. A. C. C. Masseter. In addition. C. C. Note: Some of the items in the Released Test Item Bank may have been discontinued due to outdated science or errors. nausea. A fracture through the angle of the mandible may result in an upward displacement of the proximal fragment. the tongue? D. Argues with the abuser in front of their A. 2. a hard nylon bristle brush. C. Masseter. Side effects of therapeutic doses of codeine can Overcontouring in the gingival third of a include provisional restoration contributes to the 1. B. Masseter. Adenoma. (2) and (4) D. E. D. The NDEB periodically reviews the bank to improve its quality. which of the following should be used to 2mm thicknesses of composite resin. Prescribe radiographs every 3 years.In a teenage patient with normal gingiva. Confides in friends and asks for help. A. incremental placement of no more than child. (1) (2) (3 D. (1) and (3) restoration. Reports the abuse to law enforcement. D. Which of the following Polymerization shrinkage in a composite resin groups of muscles produce this movement? is reduced by A. Squamous cell carcinoma. D. A. temporal and lateral pterygoid. doubling the curing time of the resin in preparations that are deep. C. ©The National Dental Examining Board of Canada 2017 . What is the most frequent malignant tumour of C. An abused woman is most at risk of being interdental plaque removal is best murdered when she accomplished with A. resin. inflammation. Use of high speed film. placing a glass ionomer liner on all B. 4. B. respiratory depression. a regular toothbrush. dental floss. B. content. B. using a flowable composite on the gingival floor of Class II preparations. digastric and lateral pterygoid. Application of a radiation protection badge. marginal integrity of the provisional B. B. retention of the provisional restoration. temporal and medial pterygoid. exposed dentin before placing composite C. When taking radiographs on a 10 year old D. D. E. B. an interdental wood stick. constipation. (4) only E. Digastric and geniohyoid. drowsiness. A decrease of the Kilovoltage to 50kVp. Separates from her abusive spouse. and translation. an interproximal brush. children. decrease radiation exposure? A. All of the above. Granular cell myoblastoma. Parotid. keratinocytes. bluish-grey discolouration C. Sublingual. fibrous dysplasia. Gingival enlargement associated with use of B. A. Mikulicz's disease. associated with pulpal inflammation. generalized moderate chronic periodontitis is B. increase of D. pulpal hemorrhage. D. chromogenic bacteria. associated with fever. phenytoin is primarily characterized by an C. content. keratinocytes. collagen. expansile. edema. osteitis deformans. Gingival enlargement associated with use of C. A. osteogenesis imperfecta. 31-40 days. ©The National Dental Examining Board of Canada 2017 . collagen. increase of A. sialolithiasis. plasma cells. Condensing osteitis differs from idiopathic osteosclerosis because it is A. 21-30 days. C. plasma cells. B. Submandibular. C.Blue sclera is characteristic of Which of the following is used to verify the effectiveness of instrument sterilization? A. D. B. The most common cause of bilateral swelling D. edema. discoloured composite restoration. osteopetrosis. C. A. mumps or acute infectious parotitis. B. of the parotid glands in children is A. C. In addition. C. and translation. Oral microorganisms. A. D. of the crown of an anterior tooth is due to D. painful. D. B. Anerobic bacteria. cyclosporine is primarily characterized by an D. Gram-negative bacteria. 1-10 days. C. the format of some items is not currently used. The NDEB periodically reviews the bank to improve its quality. external resorption. B. B. In which salivary gland is a pleomorphic adenoma most frequently found? The minimum time to re-evaluate tissue response after initial therapy for a patient with A. B. 11-20 days. Note: Some of the items in the Released Test Item Bank may have been discontinued due to outdated science or errors. Following trauma. mixed salivary tumours. Spores. root planning. apical migration of dentogingival 3. In addition. the relative C. B. A. skin: 3. C.The most appropriate suture combination for a Histologically. A. Over the lifetime of a patient. When will infiltrating new blood vessels be E.0 silk.0 gut. C. the format of some items is not currently used. content. quaternary ammonium compound. B. epithelium. D. 2 to 3 days. constant.0 vicryl. B. muscle: 3. skin: 3. mucosa: 3. gingival graft? A. gingival connective tissue. mucosa: histologically detectable following a free 3.0 vicryl.0 gut. periodontitis is distinguished through laceration of the lower lip would be from gingivitis by A. C.0 nylon. D. muscle: 3.0 gut. periodontal ligament. mucosa: C. B. Note: Some of the items in the Released Test Item Bank may have been discontinued due to outdated science or errors. Chlorhexidine is a B. increased gingival inflammation. posterior. E. D. A. mandible is A.0 silk. phenol. skin: 5. D. the minimum thickness of bone between the implant and the inferior alveolar A. muscle: 3. D. antibiotics. plant extract. muscle: 3. 6mm. The NDEB periodically reviews the bank to improve its quality. muscle: 3. 8mm.0 silk. nerve should be B. position of the midface in comparison to the D. C. skin: 3. mucosa: 3. and translation. alveolar bone. irrigation. ©The National Dental Examining Board of Canada 2017 . bisbiguanide compound. 2 to 3 months.0 silk. the curetted C.0 vicryl.0 gut. 2 to 3 hours. Initial therapy for periodontal pockets that are deeper than 5mm must include supragingival When placing an implant in the posterior scaling and mandible.0 silk.0 nylon. anterior. cementum. 2 to 3 weeks.0 gut. C. 2mm. polishing. root surface is repopulated by cells derived from all of the following tissues EXCEPT A. increased collagen destruction. skin: 5. B. 4mm. Following periodontal surgery. epithelium. B. mucosa: 3. structure and function. A. structure. not be washed. ©The National Dental Examining Board of Canada 2017 . B. 3. Note: Some of the items in the Released Test Item Bank may have been discontinued due to outdated science or errors. B.0 chromic gut. A. fetid odor but no radiographic evidence of bone loss. When used as a direct pulp capping material. cigarette smoking. Glass-ionomer. B. Necrotizing ulcerative gingivitis. C. The facial growth spurt has severe pain and discomfort in the mouth. Zinc-oxide eugenol.0 nylon. A 22 year old patient. starts on average at 7 years for females. psychological stress. Localized aggressive gingivitis. be washed with plain soap before initial use. C. healing? D.0 silk. D. structure and function. bacterial plaque. Clinical findings are gingival ulceration in the A. A. The NDEB periodically reviews the bank to improve its quality. of lost tissue with one that is similar in C. Necrotizing ulcerative periodontitis. 4. B.0 polyglycolic acid. ulcerated papillae. content. function. B. structure. which of the following produces the best pulp C. and translation. gingiva covered B. C. 6. Latex gloves should D. C. a D. C. Resin-modified glass ionomer. What is the most probable diagnosis? A. anterior region of both arches. the format of some items is not currently used. C. In addition.Which of the following resorbable sutures has Regeneration of periodontal tissue is the the smallest thread size? replacement of lost tissue with one that is similar in A. starts on average at 13 years for males. Localized aggressive periodontitis. A. be washed with plain soap when used Repair of periodontal tissues is the replacement between patients. periodontitis is E. 5. gingival bleeding upon slight provocation. occurs in males before females. be washed with a disinfectant solution only. by a yellow-grey slough. A. B. parallels body growth. B. who is otherwise healthy. Adhesive resin. function. D. The most significant modifiable risk factor for D. Calcium hydroxide. dental calculus. E. D. A. aphthous ulcers. Non maleficence. Renal excretion of drug metabolites is B. Which of the following is NOT a recognized ethics-based principle? A. unless a preoperative antimicrobial rinse is used in addition to standard precautions. unless there is adherence to standard C. Veracity. traumatic ulcers. Atorvastatin. Note: Some of the items in the Released Test Item Bank may have been discontinued due to outdated science or errors. precautions. Non maleficence. even if standard precautions are used. C. patients Which of the following has been implicated in who are actively infectious with the development of oral pigmentation? Mycobacterium tuberculosis CANNOT be safely treated A. recurrent herpes. D. In addition. ©The National Dental Examining Board of Canada 2017 . glomerular filtration. E. metabolites? B. C. gastric emptying rate. decreased cardiac output. The NDEB periodically reviews the bank to improve its quality. B. Triazolam. Scholarship. Autonomy. E. content. C. pemphigus vulgaris. plasma protein binding. C. Midazolam. volume of distribution. A. The most likely diagnosis is Which of the following has clinically active A. influenced by all of the following EXCEPT C. Omeprazole. B. D. Valsartan. B. A. Justice. D. Citizenship. D. round ulcers. Fluoxetine. A. and translation. Diazepam. plasma protein binding. Which of the following is NOT a recognized ethics-based principle? Metabolism of ibuprofen is NOT affected by A. 3-5mm in diameter. B. Alprazolam. D. hepatic enzyme activity. Chloroquine. D. E. unless preoperative prophylactic antibiotic coverage and standard precautions are A 32 year old patient presents with several used. C. B. D. B.In a general dental practice setting. shallow. Lorazepam. Justice. the format of some items is not currently used. tissue redistribution. C. on the left and right buccal mucosa. Beneficence. Autonomy. content. Consumer. Alveolar ridge keratosis. C.To advance a patient’s welfare. ©The National Dental Examining Board of Canada 2017 . B. A. harmful contact. C. agents because B. C. D. C. Common. determined by the shape of the lesion. National Dental Examining Board of A. the collagen matrix is maintained. Intention. intentional touching. B. In Canada. Causation. multifactorial in nature. occlusal therapy. dentists should Which of the following laws is the LEAST refer to a specialist considering all of the relevant to the dental patient record as a legal following EXCEPT document? A. The NDEB periodically reviews the bank to improve its quality. treatment without consent. indicates ‘mature minor’ status. Criminal. Canada. D. D. determined by an occlusal analysis. correlated to the dominant hand used for brushing. D. fixed prosthodontics. C. B. applies universally. orthodontic treament. B. fees. Provincial Regulatory Authorities. D. Commision on Dental Accreditation. A. Which type of legal action can result in the payment of damage? The disease-control phase of treatment includes A. enamel bonding is improved. presumes capacity. Squamous cell carcinoma. Provincial. C. skills. Note: Some of the items in the Released Test Item Bank may have been discontinued due to outdated science or errors. The following violations provide grounds for a criminal action in battery EXCEPT Moistened dentin is prefrerred over dry dentin prior to the application of dentin bonding A. and translation. D. experience. D. In addition. the format of some items is not currently used. D. D. The legally set age for authorization of health What is the most likely diagnosis of a white care treatment lesion on the retromolar pad opposing a non- functional molar? A. B. C. C. A. C. the smear layer is removed by the application of water. Candidiasis. knowledge. E. is 16 years of age. D. pit and fissure sealants. Lichen planus. Discipline. A. curing time is reduced. B. defamation causing harm. scaling and root planning. the practice of dentistry is regulated through the A. C. B. Canadian Dental Association. The etiology of noncarious cervical lesions is B. Tort. B. hypoadrenalism. Prepare the lesion with a ball diamond Patients with a history of ankle swelling.7? D. A. Platysma. constrictive pericarditis. C. Sublingual and submandibular. Accessory. In addition. hypoparathyroidism. Geniohyoid. Prepare and restore with a mesio-occlusal B. E. decreased vertical dimension. non-cavitated lesion is lubrication? observed on the 4. of a radiolucency extending into the dentin. Dorsum of tongue. What is the most appropriate management of C. vitamin B deficiency. likely suffering from C. Perform no operative treatment and apply fluoride. D. Floor of mouth. Digastric. The NDEB periodically reviews the bank to improve its quality.6 DO are most likely responsible for the lack of restoration. hyperthyroidism. and translation. D. ©The National Dental Examining Board of Canada 2017 . high incidence of clefts. C. micrognathia. and restore with a flowable composite shortness of breath and orthopnea are most resin. (a) B. decreased salivary flow. Exophthalmia may be a sign of Angular cheilitis in older patients who wear A. C. E. Gingiva. D. multiple supernumerary and unerupted teeth. the format of some items is not currently used. Note: Some of the items in the Released Test Item Bank may have been discontinued due to outdated science or errors. associated high caries index. Stylohyoid. B. slot composite resin. Labial. D. B. E. emphysema. which salivary glands At the preparation stage for a 4. A. A.7 mesial proximal surface. What is the most common site for intraoral squamous cell carcinoma? A. hyperadrenalism. content. D. A. Buccal mucosa. Mylohyoid. C. B. Which of the following is the principal muscle of the floor of the mouth? Cleidocranial dysostosis can be associated with A. There is no evidence on a bitewing radiograph A. B. Prepare and restore with a mesio-occlusal slot resin modified glass ionomer. B. congestive heart failure.In a xerostomic patient. hypothyroidism. antibiotic therapy. C. Parotid. a brown. the mesial surface of tooth 4. asthma. with C. D. complete dentures is most likely associated B. premature loss of teeth. D. the principle of do-no-harm overrides the side. content. C. C. ©The National Dental Examining Board of Canada 2017 . Warthin’s tumor. elevator muscles is B. an absence of contact on cingulums during suboptimal treatment because protrusion. the format of some items is not currently used. D. mitral valve prolapse. posterior tooth contact on the non-working C. C. to refuse recommended beneficial B. C. C. B. alcoholism. D. Ethics is most concerned with A. observation with dark-field microscopy. 60%. D. treatment voluntarily. a patient’s informed choice must always be protrusion. the law protects a patient’s right to make working side. increased. 40% A.A 60 year old patient presents with lingual The incidence of a second mesiobuccal canal in erosion of the maxillary incisors. patient’s personal choice. and translation. provincial codes of conduct. B. staining with methylene blue. A. 100%. C. B. ulcerative colitis. poor decisions. to accept recommended beneficial treatment voluntarily. C. D. Note: Some of the items in the Released Test Item Bank may have been discontinued due to outdated science or errors. In the context of informed consent. legal liability. The most the maxillary first molar is likely cause is A. 80%. C. Pleomorphic adenoma. contact between posterior teeth during B. treatment with an understanding of forseeable consequences. to refuse recommmended beneficial D. decreased. patient rights. choice means the ability Which of the following is the most common tumor of the parotid gland? A. B. Spirochete activity is seen by During an incisal clench. the activity of the A. honoured. standard of right and wrong. An ideal occlusion has A dentist must be prudent in deciding how far to follow a patient’s informed choice for A. Oncocytoma. observation with conventional B. Canalicular adenoma. unaffected. D. E. microscopy. rheumatoid arthritis. In addition. diabetes mellitus. canine guidance or group function on the A. The NDEB periodically reviews the bank to improve its quality. staining with Gram's method. A. B. molar. content. Temporalis. and translation. A. C. B. the forceps movement should be C. The NDEB periodically reviews the bank to improve its quality. B.The Controlled Drugs and Substances Act Patients with thrombocytopenia have an controls the distribution and use of which of the increased risk of following? A. fibroma. prolonged bleeding. D. Celecoxib. Gardner syndrome? D. pyogenic granuloma. C. B. A. the format of some items is not currently used. controls the distribution and use of which of the B. pulp polyp. squamous cell carcinoma. erosive lichen planus. C. risk for sinus perforation is minimized. A. Gabapentin. furcation is more accessible from the buccal. buccal roots are shorter than palatal root. Patients who have undergone kidney D. C. B. bacterial infections. Note: Some of the items in the Released Test Item Bank may have been discontinued due to outdated science or errors. A. Celecoxib. C. Oligodontia. Lateral pterygoid. ©The National Dental Examining Board of Canada 2017 . When using forceps to extract a maxillary first B. In addition. Osteomyelitis. D. exercise intolerance. Carbamazepine. Medial pterygoid. epulis granulomatosum. transplantation are at an increased risk of developing A. Gabapentin. Codeine. prolonged bleeding. Diazepam. Carbamazepine. Osteosarcoma. Granulation tissue which extends coronally from the pulp of a carious tooth is known as a/an Which of the following muscles is most likely A. brown tumours. Which of the following is associated with C. following? C. D. plasma cell gingivitis. to be affected by a depressed fracture of the B. principally in the buccal direction because the D. buccal bone is thinner than the palatal bone. zygomatic arch? C. Masseter. exercise intolerance. Patients with anemia have an increased risk of The Controlled Drugs and Substances Act A. A. Osteomas. bacterial infections. B. B. D. B. C. the same. C. B. A large. A. D. A. the distance between maximum Bonding composite to enamel is most intercuspation and centric relation. salivary contamination of clinic jackets. ©The National Dental Examining Board of Canada 2017 . ectodermal dysplasia. percutaneous injury. The host defense mechanisms of the gingival C. lichen planus. C. appropriately performed by isolation with A. D. B. B. improved. reduced. production of agglutinins and antibodies. In addition. C. nutritional deficiency. C. C. addition reactions. found in patients with A. low tissue turnover rate. rubber dam. dimension and hinge axis registration. A. the difference between occlusal vertical dimension and rest vertical dimension. local antibody production. ring-opening. geographic tongue. the difference between occlusal vertical B. blood spatter on intact skin. manufacturing defects in gloves. Proliferative verrucous leukoplakia. B. The NDEB periodically reviews the bank to improve its quality. burning mouth syndrome. a matrix system. Squamous cell carcinoma. Note: Some of the items in the Released Test Item Bank may have been discontinued due to outdated science or errors. content. C. the format of some items is not currently used. the same. Down Syndrome. D. condensation reactions. alcohol by-products are called D. B. sulcus do NOT include D. reduced. flushing action of crevicular fluid. and translation. improved. the effectiveness of a quartz Generalized redness and atrophy of the tongue tungsten halogen curing light will be are most often associated with A. the effectiveness of a Light Emitting Diode curing light will be The freeway space is A. deeply furrowed tongue is commonly D. cheek retractors. C.The greatest risk to dental health care Which of the following is NOT a malignant personnel for acquiring a blood-borne pathogen lesion of the gingiva? is through A. cotton rolls. B. D. B. Polymerization reactions which create water or C. Pierre Robin Syndrome. endocrinopathy. cross-linking. Over time. B. Over time. D. Neurofibroma. Sarcoma. A. usually l0mm in the premolar region. A. influences the strength of metals more than dental implant to osseointegrate can potentially ceramics. opacity.Which of the following posts are the most Increasing the crystalline content of a dental radiopaque? ceramic will generally decrease its A. C. The NDEB periodically reviews the bank to improve its quality. C. significantly higher than that of the A material undergoing plastic deformation underlying metal structure. Note: Some of the items in the Released Test Item Bank may have been discontinued due to outdated science or errors. A. A. B. D. the format of some items is not currently used. leads to the development of stress concentrations. A. Plastic. low modulus and high thermal diffusivity. Carbon fibre. is experiencing a breaking of bonds within B. D. is experiencing stress below its A cement base under an amalgam restoration proportional limit. fracture toughness. content. slightly higher than that of the underlying metal stucture. All of the following are ways of characterizing For a ceramometal crown. C. The presence of flaws or cracks in a material The failure of a commercially pure titanium A. D. C. should have C. B. habitual smoking. low modulus and low thermal diffusivity. will typically fail in a sudden. the structure. In addition. equal to that of the underlying metal D. Titanium. titanium oxide formation on the implant tensile strength. high modulus and low thermal diffusivity. D. ultimate stress. D. ©The National Dental Examining Board of Canada 2017 . will recover its original dimensions upon unloading. proportional limit. proportional limit. structure. A. D. type III/IV bone at the implant site. uncontrolled diabetes. impacts compressive strength more than B. high modulus and high thermal diffusivity. decreases its elastic modulus. be attributed to each of the following EXCEPT B. C. yield stress. C. metal structure. A. B. elastic limit. catastrophic manner. C. surface. slightly lower than that of the underlying B. the thermal the resistance of a material to permanent expansion coefficient of the ceramic deformation EXCEPT (porcelain) should be A. D. B. Zirconia. and translation. translucency. D. silver amalgam. crystalline phase dispersion. juice. length equal to the restored crown height. Encourage the patient to identify an alternate date to stop smoking. D. content. impression and then cement a new crown.All of the following display visceolastic A new patient is a heavy smoker and has severe properties EXCEPT periodontitis. When asked if interested in quitting smoking in the next month. Ask the patient what would be the easiest A. Frequency of fermentable carbohydrate A. increase its potential for devitrification upon heating. D. replies. the patient A. decrease its chemical reactivity. damage to an adjacent tooth not involving the B. Explain the benefits of quitting smoking as All of the following are strategies for soon as possible. B. C. An impression of a full metal crown a patient should drink preparation is made. Personalize the risk of continuing smoking ceramics EXCEPT to the patient’s oral health. decrease its fusion temperature. D. Note: Some of the items in the Released Test Item Bank may have been discontinued due to outdated science or errors. What is the most appropriate strategy C. to encourage behaviour change? D. intake. D. dentin. C. and translation. dental porcelain. then polish the configuration. following a call to medical emergency services. In addition. cement the finished crown. then apply topical fluoride to the involved enamel To achieve adequate retention when placing a surface. increase its thermal expansion . Total amount of sugar in the diet. the finished crown. The NDEB periodically reviews the bank to improve its quality. polish the enamel surface and remake the C. cement the finished crown. B. increasing the fracture toughness of dental C. resin luting cement. “I don’t smoke that much so it’s not a B. A. self-glazing. proximal contact. In cases of suspected fluoride overdose. the C. big deal”. length of 7 to 8mm. water. the post requires a involved enamel using a polishing strip. the format of some items is not currently used. slow cooling after sintering. Type of desserts eaten with meals. B. parallel sided design. first step to take toward quitting. Assessment of the impression reveals that there is superficial A. At the next appointment. post in a molar tooth with a normal canal B. ©The National Dental Examining Board of Canada 2017 . milk. most appropriate management is to A. polish the enamel surface and then cement A. Quality and range of nutrients in meals C. tetragonal zirconia addition. Which of the following has the largest effect on caries risk? Decreasing the amount of network modifiers in A. D. irreversible hydrocolloids. a dental porcelain will B. and snacks. B. C. Early lesions involve adjacent enamel. D. B. appropriate. B. only supragingival enamel is etched. The Which of the following is NOT typically crown margin is most appropriately placed associated with failed endodontic treatment? A. bruxism-related attrition. ©The National Dental Examining Board of Canada 2017 . B. D. content. C. B. Actinomyces israelii. D. C. at the amalgam/tooth junction. and translation.Which of the following statements is correct When closing a diastema with direct with respect to root caries lesions? composite. C. A. interproximal gingiva is adequately D. the format of some items is not currently used. care must be taken to ensure that A. B. Prevotella intermedia. smooth surface decalcification. interpoximally. glass-ionomer cement is used C. C. D. C. D. usually of the buccal side of the tooth. C.3 to 0. mercury to alloy in a dental amalgam. of caries activity. A specific microoganism causes root A. Colour of the lesion is a relaible indicator subgingivally. B. usually passing through the tooth. equally likely to be on the buccal or C. entire embrasure is filled. D. In addition. caries. E. A lower molar requiring a crown has an extensive MOD amalgam restoration. A. When a radiograph shows the inferior alveolar canal superimposed on an impacted mandibular third molar. Candida albicans. usually on the lingual side of the tooth. preparation. primer is used to increase bond strength. the canal is C-factor (configuration factor) is defined as the ratio of A. When closing a diastema with direct Fluoridated toothpaste will be most effective in composite. enamel is reduced 0. filler to monomer in a composite resin. Streptococcus mutans. width to height in the upper anterior teeth. width to height relationships are B. care must be taken to ensure that remineralizing the A. on the existing amalgam. Note: Some of the items in the Released Test Item Bank may have been discontinued due to outdated science or errors. 1mm apical to the amalgam margin.5mm prior to etching. The NDEB periodically reviews the bank to improve its quality. Enterococcus faecalis. The majority of lesions begin retracted. bonded to unbonded surfaces in a lingual side of the tooth. A. interproximal caries. pit and fissure caries. B. restoration does not extend subgingivally. The tooth with the largest root surface area is In periodontal ligaments, enzymatic the hydroxylation of proline and lysine residues on nascent procollagen requires A. maxillary canine. B. maxillary first molar. A. vitamin C. C. mandibular canine. B. lysyl oxidase. D. mandibular first molar. C. calcium ions. D. decorin. E. matrix metalloproteinase. The Bass tooth brushing technique is appropriate for oral hygiene instruction because Sympathetic reflexes accompanying A. plaque beneath the gingival margin is hemorrhage removed. B. debris is forced from the embrasures. A. allow blood loss up to 60% of the total C. the horizontal, vertical and circular volume. motions. B. have no effect on the amount of blood that D. it increases circulation to the gingiva. can be lost. C. are part of the long-term compensatory response mechanism. D. have a greater effect on arterial pressure The stratified squamous epithelial layer of oral than on cardiac output. mucosa does NOT contain A. melanocytes. B. Langerhans cells. What is the minimum number of nucleotides C. Merkel cells. that can be deleted from a gene that will result D. Schwann cells. in the smallest change in the amino acid sequence of the encoded protein? A. 1. A patient with leukoplakia had an incisional B. 2. biopsy. Which of the following diagnosis does C. 3. NOT require complete excision? D. 4. E. 5. A. Carcinoma in situ. B. Severe epithelial dysplasia. C. Hyperkeratosis. D. Invasive carcinoma. What are the dimensions of pain? A. Intensive and sensory. B. Sensory and peripheral. Repeat panoramic radiographs are indicated to C. Affective and peripheral. D. Sensory and affective. A. identify early signs of disease. B. follow-up a specific clinical concern or condition. C. monitor high caries risk patients. D. monitor patients with fixed dental prostheses. Note: Some of the items in the Released Test Item Bank may have been discontinued due to outdated science or errors. In addition, the format of some items is not currently used. The NDEB periodically reviews the bank to improve its quality, content, and translation. ©The National Dental Examining Board of Canada 2017 Which of the following is NOT a contributor to Which of the following bacterial components is the pathogenesis of stomatitis? considered to be of key importance in initiating and sustaining inflammatory responses in A. TNF-α. gingiva and other periodontal tissues? B. Bradykinin. C. Substance P. A. Fimbriae. D. Glucocorticoids. B. Capsule. C. Lipopolysaccharides. D. Flagella. Botulinum neurotoxins cause which type of paralysis? During rapid head movement, that occurs on A. Spastic. activities such as running, the mandible is held B. Flaccid. in place due to activation of C. Periodic. D. Brachiofacial. A. temporomandibular proprioceptors. B. fast adapting periodontal pressoreceptors. C. slow adapting periodontal pressoreceptors. D. primary muscle spindle fibres. When a painful muscle is palpated, the provoked pain is A. localized. Hypernatremia may be induced by B. referred to another area. C. described as a dull ache. A. elevated synthesis of atrial natriuretic D. described as a burning sensation. peptide. B. excessive vasopressin secretion. C. impaired aldosterone secretion. D. diabetes insipidus. Which of the following statements is INCORRECT? A. Chronic pain has somatosensory and Which of the following would NOT be psychosocial impacts. expected to cause hypokalemia? B. Pain persists only as long as the injury exists. A. Metabolic acidosis. C. Perception of acute and chronic pain B. Lactate accumulation. involves the same regions of the CNS. C. Increased plasma insulin. D. Pain is a protective mechanism. D. Excess aldosterone secretion. E. Beta-adrenergic stimulation of cells. Dietary consumption of which of the following inhibits the cytochrome P450 system? A. Meat. B. Broccoli. C. Decaffeinated tea. D. Grapefruit juice. Note: Some of the items in the Released Test Item Bank may have been discontinued due to outdated science or errors. In addition, the format of some items is not currently used. The NDEB periodically reviews the bank to improve its quality, content, and translation. ©The National Dental Examining Board of Canada 2017 The principal cellular sites of the cytochrome Which of the following statements about P450 system are chlorhexidine is INCORRECT? A. cardiomyocytes. A. It disrupts bacterial cell membranes. B. pneumocytes. B. It binds tenaciously to oral surfaces. C. splenocytes. C. It interferes with adherence of plaque- D. hepatocytes. forming microorganisms. D. Mutans streptococci are preferentially destroyed. E. It is commonly used for long-term therapy. A hereditary condition affecting odontoblasts can result in A. regional odontodysplasia. Which of the following statements about B. fusion or gemination. sodium is correct? C. dilaceration. D. hypercementosis. A. Salt-sensitive people should avoid foods E. radicular dentin dysplasia. like oranges and bananas. B. A high salt intake aggravates but does not cause hypertension. C. Renal sodium excretion varies indirectly Biopsy of a recent solitary painless ulcer on the with total sodium intake. lip in a 20 year old patient shows the presence D. The sodium recommendation is increased of Treponema pallidum. The ulcer is called (a) during pregnancy. A. lupus vulgaris. B. facies leprosa. C. chancre. A patient wants to reduce her caries risk and D. noma. asks her dentist to recommend an alternate sweetener that can be used in baking. Which of the following should NOT be recommended? The sensation of sour is mediated by A. Equal® (aspartame). B. Splenda® (sucralose). A. transducin. C. Sugar Twin® (sodium cyclamate). B. protons blocking K+ channels. D. Xyla® (xylitol). C. gustducin. D. activation of T1R3 receptors. A 35 year old female patient has multiple petechial hemorrhages of the palatal mucosa Which of the following is NOT used to inhibit and ecchymoses of the right and left buccal calcification of plaque? mucosae. Which of the following is the most likely laboratory finding? A. Fluoride. B. Magnesium. A. Anemia. C. Pyrophosphate. B. Prolonged PTT. D. Zinc. C. Elevated INR. D. Thrombocytopenia. Note: Some of the items in the Released Test Item Bank may have been discontinued due to outdated science or errors. In addition, the format of some items is not currently used. The NDEB periodically reviews the bank to improve its quality, content, and translation. ©The National Dental Examining Board of Canada 2017 A rotary endodontic file separates in the canal. Which of the following statements about The most appropriate management is to fats/lipids is correct? A. inform the patient of the separated file at A. Naturally occurring trans fats have been the time of the incident. confirmed to increase disease risk. B. inform the patient of the separated file if B. LDL-cholesterol is found in foods of only the tooth becomes symptomatic. animal origin. C. make a notation in the patient’s chart C. Most of the cholesterol circulating in the without informing the patient. human body is synthesized by the body. D. recommend an apicoectocy. D. The essential fatty acids are only the omega-3 fats. A patient has been receiving dental treatment over a period of 1 year. A root canal treatment In the Vita® Classical Shade Guide, the hue of has recently been started but not completed the D shade series is when she declares that her husband has lost his job and she is not able to pay the balance of the A. red. fee assessed for her root canal treatment. She is B. grey. presently free of pain. The dentist can terminate C. yellow. treatment D. brown. A. only after completion of the root canal treatment. B. for failure to honor financial commitments. In the Vita® Classical Shade Guide, the hue of C. until the patient is able to pay for it. the A shade series is D. only after completion of the root canal treatment and a full crown restoration. A. red. E. by referring the patient to an endodontist. B. grey. C. yellow. D. brown. Five years ago, a 55 year old patient was treated for squamous cell carcinoma of the oral cavity with ablative surgery, chemotherapy and The percent of elderly persons known to intensity-modulated radiotherapy. The colonize asymptomatic C. difficile in the gut is posterior mandible received a dose of 70 Gy. typically in a range of up to The extraction of tooth 3.7 with local anesthesia is planned because the tooth cannot A. 3%. be restored. The most appropriate management B. 10%. is extraction with C. 30%. D. 50%. A. transfusion of packed red blood cells prior to the extraction. B. hyperbaric oxygen treatment prior to the extraction. C. hyperbaric oxygen treatment prior to and following the extraction. D. local anesthetic containing no vasoconstrictor. Note: Some of the items in the Released Test Item Bank may have been discontinued due to outdated science or errors. In addition, the format of some items is not currently used. The NDEB periodically reviews the bank to improve its quality, content, and translation. ©The National Dental Examining Board of Canada 2017 Which of the following statements regarding Infections transmitted in dental offices have physiologic changes in an elderly patient is been documented NOT correct? A. at a rate comparable to health care A. Reduced gastric emptying can lead to a associated infections in hospitals. decrease in drug absorption. B. occasionally, primarily with hepatitis B. B. Renal blood flow is decreased. C. with equal frequency for viruses and C. Lipid soluble drugs remain in the body for bacteria. a longer period of time. D. primarily in offices, which do not control D. Relative tissue perfusion is unaltered. bacterial levels in water from dental units. Which of the following analgesics is The metabolic clearance of which of the CONTRAINDICATED for a 76 year old following drugs is NOT reduced in an elderly patient with angina? patient? A. Acetaminophen. A. Carbamazepine. B. Ibuprofen. B. Warfarin. C. Codeine. C. Morphine. D. Tramadol. D. Clindamycin. An 89 year old patient has rampant caries Which of the following statements regarding secondary to vestibular food pocketing. The physiologic changes in an elderly patient is LEAST likely predisposing disease is correct? A. Parkinson’s. A. Creatinine production is reduced. B. Lewy body dementia. B. Plasma albumin is increased. C. Alzheimer’s. C. Relative tissue perfusion is unaltered. D. Addison’s. What is the most common intraoral location of After latex gloves have been put on, washing squamous cell carcinoma? the gloved hands with an antimicrobial soap A. Lateral tongue. A. adds a further layer of safety to barrier B. Buccal mucosa. protection. C. Tonsilar bed. B. is unnecessary but acceptable practice. D. Attached gingiva. C. can compromise the integrity of the glove. D. should be done if a surgical procedure is planned. Note: Some of the items in the Released Test Item Bank may have been discontinued due to outdated science or errors. In addition, the format of some items is not currently used. The NDEB periodically reviews the bank to improve its quality, content, and translation. ©The National Dental Examining Board of Canada 2017 A 20 year old has a solitary radiolucent lesion Which of the following is LEAST likely to in the left maxillary tuberosity. The histologic share histological features with lichen planus? diagnosis is an odontogenic keratocyst/keratogenic odontogenic tumour. A. White sponge nevus. What is the recurrence rate of this tumour? B. Oral mucosal cinnamon reaction. C. Oral graft-versus-host disease. A. It does not recur. D. Lupus erythematosus. B. 10-30%. C. 40-60%. D. 70-90%. The palatine tonsils are found A. surfacing the posterior third of tongue A 74 year old patient being treated for dorsum. hypertension requires deep scaling using local B. between the palatoglossal and anesthesia. It is appropriate to defer treatment palatopharyngeal arches. if the patient’s blood pressure is equal to or C. on the posterior wall of the nasopharynx. above D. on the bilateral lateral ventral aspects of the tongue. A. 120/80. B. 140/90. C. 160/110. D. 180/110. A 10 year old girl has poorly formed, distorted permanent right maxillary canine, lateral and central incisors. The remaining teeth and supporting structures are unremarkable. The A dentist who has been diagnosed as a chronic patient most likely has Hepatitis B carrier can perform exposure prone procedures A. ectodermal dysplasia. B. segmental odontomaxillary dysplasia. A. using Standard precautions. C. radicular dentin dysplasia. B. in the absence of HBeAg. D. regional odontodysplasia. C. following a review by an Expert Panel. E. coronal dentin dysplasia. Which of the following is LEAST likely to be A strong association exists between the associated with fissured tongue? presence of fissured tongue and the presence of A. Oral lichen planus. A. geographic tongue. B. Melkersson-Rosenthal syndrome. B. hairy tongue. C. Geographic tongue. C. lingual varicosities. D. median rhomboid glossitis. A feature of primary syphilis is A. atrophic glossitis. B. a gumma. C. a chancre. D. a mucous patch. Note: Some of the items in the Released Test Item Bank may have been discontinued due to outdated science or errors. In addition, the format of some items is not currently used. The NDEB periodically reviews the bank to improve its quality, content, and translation. ©The National Dental Examining Board of Canada 2017 Which of the following is NOT a malignancy? The principal action of caffeine is on the A. Leukemia. A. cerebral cortex. B. Lymphoma. B. corpus callosum. C. Leiomyoma. C. hypothalamus. D. Melanoma. D. spinal cord. A tissue-level implant should be used when A 3 year old complains of a sore left arm for 10 days. A physician has diagnosed a fracture and A. the edentulous site is in the esthetic zone. a radiograph reveals an old fracture. The most B. platform switching is desired. appropriate diagnosis is C. a 2-stage surgical approach is planned. D. ease for oral hygiene is desired to preserve A. infantile osteoporosis. crestal bone. B. osteogenesis imperfecta. C. battered child syndrome. D. Münchhausen syndrome. Crestal bone loss around implants prior to occlusal loading is more significantly associated with Dentists may choose NOT to provide treatment for a patient when the A. smooth collars. B. roughened collars. 1. treatment cannot be completed in a timely C. internal connections. fashion. D. external connections. 2. patient has a serious communicable disease. 3. dentist does not have sufficient training for procedure. In which of the following conditions is there a 4. treatment is for an urgent situation. risk of malignant change after radiotherapy? A. (1) and (3) A. Fibrous dysplasia. B. (2) and (4) B. Mucocele. C. (1) (2) (3) C. Lymphangioma. D. (4) only D. Torus palatinus. E. All of the above. Patients who are positive for the interleukin-1 Which of the following CANNOT be viewed (IL-1) on a lateral cephalometric radiograph? A. are at increased risk for severe periodontal A. Upper airway clearance. disease. B. Mandibular dimensions. B. have a decreased inflammatory response in C. Maxillary constriction. the presence of bacteria. D. Lower face height. C. are more likely to respond favourably to periodontal therapy. D. have decreased bacterial pathogens associated with active periodontal disease. Note: Some of the items in the Released Test Item Bank may have been discontinued due to outdated science or errors. In addition, the format of some items is not currently used. The NDEB periodically reviews the bank to improve its quality, content, and translation. ©The National Dental Examining Board of Canada 2017 A 70 year old patient was diagnosed with Which of the following is NOT a clinical squamous cell carcinoma of the lateral border finding of hypothyroidism? of the tongue. The tumour measures 3.5 x 3.0 cm. A CT of the neck confirms the presence of A. Anxiety. a 2.5 cm ipsilateral submandibular lymph node B. Mild tremor. suggestive of locoregional disease. Chest X- C. Puffy eyelids. rays and liver function tests were within normal D. Exophthalmos. limits. The TNM classification of this patient’s disease is A. T1N1M0. What is the most likely diagnosis of a 9 year B. T1N2M0. old with speech problems, macroglossia, C. T2N1M0. unilateral premature eruption of teeth, and D. T2N2M1. moderate scoliosis? A. Down syndrome. B. Hemi hypertrophy. A 65 year old, underweight patient confirms a C. Gigantism. longstanding problem with alcohol abuse. The D. Cleidocranial dysplasia. medical history reveals complaints of muscle weakness, low appetite, lack of energy and forgetfulness, which the patient attributes to getting old. The patient’s diet is most likely Upon returning to the operatory, the dentist deficient in notices the patient is looking at the dental chart. Which of the following statements is correct? A. ascorbic acid. B. folic acid. A. The patient has a legal right to access the C. thiamin. chart at any time. D. retinol. B. The dentist can charge an administrative fee to the patient to view the chart. C. A written request is required before access to the chart can be granted. A patient with bulimia who purges several times a day should be advised to increase intake of Which of the following has both ectodermal A. lean chicken. and mesodermal tissues? B. bananas. C. whole wheat bread. A. Ameloblastoma. B. Cementoblastoma. C. Odontoma. D. Odontogenic myxoma. A 55 year old patient wants to maintain bone mass following implant surgery and plans to take extra calcium. Supplemental calcium is most beneficial when consumed in A. divided doses with meals. B. divided doses between meals. C. a single dose with meals. D. a single dose between meals. Note: Some of the items in the Released Test Item Bank may have been discontinued due to outdated science or errors. In addition, the format of some items is not currently used. The NDEB periodically reviews the bank to improve its quality, content, and translation. ©The National Dental Examining Board of Canada 2017 A 75-year old female patient is being treated What is the most appropriate design for a study for oral lichen planus with a topical that is started after the exposure to a risk factor corticosteroid. She also has low vitamin D. but before the disease started? The most likely cause of a reduced alveolar bone mass in this patient is A. Case series. B. Randomized trial. A. postmenopausal osteoporosis. C. Case-control. B. senile osteoporosis. D. Cohort. C. drug-induced osteoporosis. D. osteomalacia. Patterns of exposure to a risk factor in a population is important because it helps Pleomorphic adenomas are most likely to exhibit A. understand where preventive interventions can be targeted. A. rapid growth with death of patient. B. define the prevalence of a disease. B. early metastasis. C. define the population attributable fraction C. slow growth but tendency to local of a disease. recurrence. D. give an insight into what statistical tests to D. early ulceration and hemorrhage. use. The findings from a meta-analysis of A study has been conducted comparing rates of 24 clinical trials conducted over 20 years, dental caries in 25 cities with differing levels of involving vitamin D supplementation to fluoride in the water supply. This study is a/an prevent dental caries in children aged 5-14 years, reveal there is wide variation in the A. cross-sectional study. results between the studies. Which of the B. community trial. following differences is LEAST likely C. ecological study. responsible for this variation? D. case series. A. Age of the children. B. Stage of dentition of children. C. Years when the studies were conducted. To assess factors affecting the utilization of D. Baseline caries levels of children. dental services, a cross-sectional study surveyed adults on a number of variables. Considering the study design, which of the following findings CANNOT be derived from An article reports that subjects receiving the results? chlorhexidine varnish had significantly lower (p<0.05) Streptococcus mutans levels than did A. Satisfaction with the services. those receiving fluoride varnish. What type of B. Gender differences. reporting error is of the most concern when C. Caries incidence. interpreting these results? D. Utilization of dental services. A. Type II (ß) error. B. Type I (ß) error. C. Type II (α) error. D. Type I (α) error. Note: Some of the items in the Released Test Item Bank may have been discontinued due to outdated science or errors. In addition, the format of some items is not currently used. The NDEB periodically reviews the bank to improve its quality, content, and translation. ©The National Dental Examining Board of Canada 2017 account for 95% of the dental visits. The odds ratio for severe caries based upon whether the child drank milk was A.84). The NDEB periodically reviews the bank to improve its quality. it excludes subgingival lesions. C. low.586 Canadians found that 63% of older adults had Examination of a 32 year old pregnant woman visited a dentist in the last year. C. no additional precautions beyond standard of these results? Drinking milk procedures are recommended for dental treatment. A randomized clinical trial of pregnant women with periodontal disease. C. root caries is difficult to diagnose. the format of some items is not currently used. the disease is transmissible through air or 0. normal pattern of development. birth weight baby?” Which type of study design B. C. it treats filled and decayed surfaces as the same.23 to 0. the disease is usually fatal within one year. The 95% reveals that she has generalized periodontal confidence interval was 56% to 70%. She is otherwise healthy. older adults had dental visits. This probing depths of 4-6mm. The Root Caries Index may underestimate the Two millimeters of maxillary incisor spacing in amount of disease because a 3 year old is indicative of a A. most of which bleed means that on probing. and translation. B. D. D. had no effect on caries. B. no more than 70% of dental visits occurred A. D. was protective against caries. A. women. there is 95% certainty that 56% to 70% of disease and some of whom do not. months. Territories. content. A systematic review comparing the risk of delivering pre-term low birth weight babies among women with various risk factors. following is the most appropriate interpretation B. was not protective against caries. the disease has an incubation time of 3-6 C. some of whom receive periodontal therapy and some of whom do not. A. Comparing the rates of pre-term low birth weight babies. there is 95% certainty that 63% of older would provide the best answer to her question? adults had dental visits. the number of surfaces at risk cannot be A. A survey of a random sample of 5. D. tooth size-arch size discrepancy. determined. C. including the presence of periodontal disease. some of whom have periodontal D. between 56% and 70% of older adults am I more likely to deliver a pre-term. A prospective cohort involving pregnant in 95% of older adults. persistent tongue thrusting. between women with no risk factors to those with periodontal disease.44 (95% CI = 0. B. Which of the by casual contact.A study examined caries in 349 children For patients with prion disease such as 2 to 6 years of age living in the Northwest Creutzfeldt-Jacob disease. maxillary Bolton deficiency. B. but asks “If I don’t get the proper periodontal treatment. ©The National Dental Examining Board of Canada 2017 . Note: Some of the items in the Released Test Item Bank may have been discontinued due to outdated science or errors. In addition. and translation. B. content. Beneficence. Selection bias. They plan on A. define who is at risk for a disease. What ethical concern(s) would the D. Cross-sectional. researchers have to consider? A. randomized controlled clinical trial is to ensure C. A. Sampling. Justice. Recall. What ethical principle B. Meta-analyses are important to the practice of dentistry because they A. ©The National Dental Examining Board of Canada 2017 . Justice. B. In addition. Privacy and confidentiality. B. of an avulsed permanent tooth. provide a quantitative summary measure outcomes. Non-maleficence. similarity of experimental and control groups. the format of some items is not currently used. from many studies. The NDEB periodically reviews the bank to improve its quality. Beneficence. C. administrative health care data to conduct this C. D. D. C. comparator agents on the second and third days after third molar extraction included 600 officers from dental clinics at 10 military bases. D. Note: Some of the items in the Released Test Item Bank may have been discontinued due to outdated science or errors. B. A double blind controlled clinical trial that C. Splinting with composite resin only. consistent measurement of clinical D. and/or A. Ecological. Retrospective cohort. between community water fluoridation and hip They have found a 40 year old database that fracture? contains information on the oral health of children along with their names. Pulpectomy.A group of researchers would like to study the Which of the following studies provides the relationship between oral health status in best evidence for the possible association childhood and adult systemic health conditions. Case-series. assessed the analgesic effects of etoricoxib and D. Splinting with flexible wire. Autonomy. Splinting with rigid wire. A. does this refer to? C. Selection. research. What form of bias is present in this In addition to the advancement of knowledge. The purpose of calibrating examiners in a B. for society as a whole. Publication. C. for other individuals. Which of the following clinical protocols is the D. measurement of all relevant clinical most appropriate next step in the management outcomes. minimize publication bias. which has just been replanted? A. linking this information with current B. high ethical standards in conducting the trial. guide statistical testing for clinical procedures. study? health research is intended to produce value for research subjects. Replant the tooth followed by endodontic D. Condylar. 400 cells/mm3.Which of the following types of permanent What is the minimum CD4 count to perform teeth is more likely to present with a C-shape elective dental treatment? canal anatomy? A. B. B. A. Note: Some of the items in the Released Test Item Bank may have been discontinued due to outdated science or errors. pull stroke. affected dentin. A 3 year old presents 30 minutes after facial The teeth of a Miller bone file are designed to trauma. B. Down's syndrome. lateral incisors is most likely seen in a patient E. palatal exostoses occur at the C. molars. Tooth 5. A. C. D. treatments in 2 weeks. premolars. How many bones form the orbit? D.1 is avulsed. The highest incidence of congenitally missing D. 200 cells/mm3. B. infected dentin. B. Caries disclosing solution used during the C. the format of some items is not currently used. 100 cells/mm3. In a sagittal split osteotomy of the mandible. A. B. 6. B. D. which segment of the mandible? A. B. D. C. Maxillary second molars. 5. 1000 cells/mm3. content. sawing motion. The NDEB periodically reviews the bank to improve its quality. hyperthyroidism. Most commonly. C. canines. Replant the tooth and monitor for possible A 30 year old male suffers an orbital injury. In addition. Mandibular first premolars. 8. D. reversibly denatured dentinal matrix. circular motion. ©The National Dental Examining Board of Canada 2017 . C. The father has smooth bone with a recovered the tooth and has kept it in a wet napkin. determine the extent of A. excavation of a deep carious lesion will D. and translation. endodontic treatment. Which of the following is the most A. Maxillary first premolars. congenital heart disease. A. the neurovascular bundle should remain in D. Perform a pulpectomy. then replant the tooth. C. unilateral cleft lip and palate. 4. Distal. Mandibular second molars. appropriate management? B. push stroke. Lateral. inorganic dentinal matrix level of deminieralization. 300 cells/mm3. Do not replant the tooth. with A. C. 7. Proximal. incisors. E. C. sulcular epithelium. A. cured in increments limited to a maximum of B. diabetes. 2. and translation. D. Leukocyte adhesion deficiency syndrome. C. B. vascular supply. Crouzon syndrome. 0. atherosclerosis. epithelial attachment. factitious injury. Maxilla. D. osteoporosis. D. B. The NDEB periodically reviews the bank to improve its quality. Which gingival recession in a healthy 17 year old condition should NOT be included in the individual is differential diagnosis? A. C. Papillon-Lefèvre syndrome. occlusal trauma. In addition. 1/3. minimal attached gingiva. 3/4. 1/4. Attachment loss is one of the main D. Supraeruption occurs 60% of the time. A. Mandible. E. A. Chédiak-Higashi syndrome. C. supraeruption of 3. D. D. Unopposed molars have a mean B. content. Note: Some of the items in the Released Test Item Bank may have been discontinued due to outdated science or errors. B. Cyclic neutropenia. Which of the following is true about the supraeruption of unopposed molars? A3 shades of composite resins should be light- A. C. E. the format of some items is not currently used. B. 1.50mm. type of collagen. 2/3. Cranial vault.00mm.0mm. ©The National Dental Examining Board of Canada 2017 . mobile The most likely cause of localized facial teeth and generalized calculus. A 4 year old has generalized bone loss.A significant anatomic difference between the Which of the following sites is formed peri-implant marginal soft tissue and the exclusively by endochondral bone ossification? gingival marginal tissue of a tooth exists in the A. C. D. B. B. C. A. how far below the superior classification includes border of the lower 1/3 of the face are the commissures of the mouth located? A.50mm. At the commissures. C. D. Cranial base.00mm. leukemia. predictors. Periodontitis as a manifestation of systemic disease as described in the Armitage 1999 Proportionally. Supraeruption is more prevalent in the mandibular arch. C. 1. traumatic toothbrushing. Hepatitis C.000 that may be safely administered to a 17kg child is approximately A. The NDEB periodically reviews the bank to improve its quality. B.5. Which of the following restorations is the most appropriate for the replacement of a maxillary permanent lateral incisor where there is 4. B.8ml cartridges of 3% mepivacaine that may be safely administered to a 65kg adult A. E. Hepatitis B. D. HPV. D. 9.5. opaque. is B.8ml The most appropriate gingival contour of a cartridges of 2% lidocaine with fixed partial denture connector is epinephrine 1:100. ©The National Dental Examining Board of Canada 2017 . convex. D. 2. 8. Cantilever pontic FPD using canine abutment. Arrested caries. B. C. Rampant caries. content. Removable partial denture. concave. A. Which of the exposure to a vaccinated dental healthcare following is the most appropriate diagnosis? worker? A. flat. B. E. and translation. A young adult has generalized areas of chalky. When cavitated carious lesions are present there is Ideally. demineralization by matrix metalloproteinases. Secondary caries.5mm of mesial-distal space and an intact central The manufacturer’s maximum recommended incisor? number of 1. 1. 6. HIV. 2.5. 0. C. fixed dental prosthesis. Interproximal caries. C. D. 4. 6. D. 5. exposure to blood borne pathogens? B. 7. B. D. C. within how many hours should one receive medical attention for percutaneous A. Smooth surface caries. E. In addition. A. C. 2. Three-unit metal-ceramic full-coverage C. B. A.The maximum recommended number of 1. C. D. 1. A. Implant supported restoration. exposure of the dentin protein matrix. the format of some items is not currently used. Note: Some of the items in the Released Test Item Bank may have been discontinued due to outdated science or errors. a denatured inorganic phase. C. 8. bacterial protease inhibition. cavitated lesions on the vestibular Which of the following viruses has the greatest surface of multiple teeth and interproximal chance of transmission in an occupational gingiva that is red and swollen. Which of the D. Potassium alginate. C. Cervical. Tanaka-Johnston. 4mm on the lingual interproximal caries. A. 1 . A. Note: Some of the items in the Released Test Item Bank may have been discontinued due to outdated science or errors. Combination. C. adjacent teeth are heavily restored. D. Hyrax appliance. and translation. incompetent lips. A large Class III preparation on tooth 1. Moyers. an Angle Class II A. abutment crown to root ratios are 1:1. incisors and contralateral molars are missing. Zinc oxide. prepared at 90 degrees? A. The thickness of the labial bony wall of a Which of the following statements is correct maxillary anterior tooth is typically about Quantitative Light or Laser Induced Fluorescence used to aid in the early detection A. Which of the following analyses is most C. and onto the root surface. the format of some items is not currently used. Labial. High sensitivity when used to detect extends 2mm on the labial. D. Unable to detect caries around existing restorations.2mm. Bolton. D. A. High-pull. C. An alginate impression is made of the A 9 year old patient has a right posterior maxillary arch and there is abundant unset crossbite and a mandibular midline shift to the alginate remaining on the teeth. B. appropriate for use in an adult patient? D. malocclusion and vertical maxillary excess? B. mesially. B. Low specificity when used to detect smooth surface caries. > 2mm. A. B. Fixed brackets. ©The National Dental Examining Board of Canada 2017 .1 C. second and third molars are tipped C. Which of the following is the most which of the following components may appropriate appliance for this patient? contribute to the slow setting? A. Gingival. Headgear. < 1mm. C. B. Functional appliance. Assesses light reflection as opposed to following cavosurface margins should be light transmission. of caries? B. B. Protraction.A fixed partial denture is an appropriate Which type of headgear is most appropriate for treatment option for replacing a missing the management of a 10 year old patient with mandibular first permanent molar when the an increased lower third of the face. In addition. The NDEB periodically reviews the bank to improve its quality. B. Incisal. D. content. Excess of right. C. Sodium phosphate. Lingual. Calcium sulfate. In addition. D. then angling at least a/an 45° to reach the col area. B. C. B. Adding Sn. a long junctional epithelial attachment. E. probeable vertical depth of 1 to 3mm. D. Proximal guide plane. D. smoking. C. the format of some items is not currently used. 4. Suprabulge area. (1) (2) (3) When designing a removable partial denture. intertubular dentin. then angling slightly A. C. bacterial plaque. (4) only A. B. a Grade I furcation is A. E. D. C. (2) and (4) D. 3. All of the above. Type I reaction. D. resin plugs in the dentin surface. decalcified peritubular dentin. Type II reaction. Impressions for partial dentures. bonding agent at the interface. then angling B. (1) and (3) undercuts are found in which area of teeth? C. line angle is reached. D. B. B. B. probeable horizontal depth of 1 to 3mm. porcelain and the metal. contact area is reached. Mandibular block anesthetic injection. ©The National Dental Examining Board of Canada 2017 . The primary etiological factor associated with periodontal disease is A delayed allergic reaction is called a A. D. contact area is reached. The NDEB periodically reviews the bank to improve its quality. sac with a definite horizontal component. A. calculus. when the porcelain to metal bond in metal-ceramic osteoclasts reach the root surface first. Type IV reaction. content.Which procedure(s) require(s) antibiotic The hybrid layer is composed of resin tags and prophylaxis in a patient susceptible to bacterial endocarditis? A. regenerated periodontium. In and Fe to the metal. Height of contour. D. Which of the following does NOT contribute to During periodontal wound healing. Periodontal surgery. tooth mobility. B. this will crown restorations? result in A. Type III reaction. C. Creating a thick oxide layer on the metal. An accurate interproximal probing depth is obtained by walking the probe parallel to the long axis of the tooth until the According to the Glickman classification for furcation involvement. line angle is reached. root resorption and/or ankylosis. Removing the chemical debris from the A. connective tissue adhesion. A. least 45° to reach the col area. and translation. Forming a mixed oxide layer between the C. involvement. furcation lesion that is essentially a cul-de- slightly to reach the col area. Infrabulge area. Suture removal. incipient or early stage furcation to reach the col area. diabetes mellitus. 2. then angling at C. Note: Some of the items in the Released Test Item Bank may have been discontinued due to outdated science or errors. 1. B. metal surface. C. In addition. Proximal plates. A. on the site where avulsion happened. the exposure of teeth via apical migration there is a bone defect is implant placement with of the gingiva. D. Cast I-bars. functional antagonists. ©The National Dental Examining Board of Canada 2017 . sinus floor elevation with window technique. phase. Double embrasure clasps. simultaneous membrane application. diameter of the probe. Iodophors. the anterior abutments? C. immediately after avulsion. C. the movement of the teeth in the direction of the occlusal plane. and translation. The NDEB periodically reviews the bank to improve its quality. bone splitting the alveolar crest. on the site where avulsion happened. Cingulum rests. Elimination of the source of inflammation. D. content. force used on the probe. intervention? B. Presence of a periradicular radiolucency. E. Development of a periradicular The concept of continuous tooth eruption is radiolucency. D. consisting of both an active and a passive A. A. the format of some items is not currently used. B. Phenols. replantation of an avulsed permanent tooth is B. soaking it in saline for 5 minutes. B. C. D. Note: Some of the items in the Released Test Item Bank may have been discontinued due to outdated science or errors. Chlorine compounds. Formaldahyde. level of inflammation. after canal. D. which of the following is NOT an indication for additional endodontic A. Which of the following is NOT a clinical predictor of successful vital pulp therapy? The most appropriate management for the A. after soaking it in milk for 5 minutes. D. upon arrival at a dental clinic. D. being complete when teeth reach their C. Preoperative health of the pulp. Exclusion of future microbial A. minutes of avulsion. within 60 microleakage. the osteotome technique. D.The depth of penetration of a periodontal probe After a tooth has had nonsurgical endodontic in a sulcus is LEAST affected by the treatment. Presence of calcifications within the root B. upon arrival at a dental clinic. described as A. B. following components for direct retention on B. C. amount of clinical attachment loss. Presence of clinical signs and symptoms. The most complex surgical procedure where B. C. Which of the following disinfectant is NOT appropriate for dental impressions? The rotational path of insertion for the Class IV partially edentulous patient uses which of the A. C. Enlargement of the original periradicular radiolucency. C. Glutaraldehydes. Pulpectomy. B. demonstrate similar clinical presentation. A. D. Extraction. A. EXCEPT A. A patient with suspected hyperthyroidism may C. weight loss. B. the format of some items is not currently used. E. Transparent. A. capable of remineralization? D. can be prevented by hyperbaric oxygen therapy. Teeth showing signs of opacity in pits or fissures. Pulpotomy. Infected. display all of the following early symptoms D. Avulsion. Extrusion. enamel junction). B. D. tachycardia. Uncomplicated crown-root fracture. Inverted U-shape (base towards dentino- enamel junction).Which of the following traumatic injuries does Restoration of non-carious cervical lesions may NOT require immediate management? be the appropriate management in all of the following situations EXCEPT when A. In addition. B. occur more frequently in the maxilla. Inverted V-shape (base towards dentino- C. osteonecrosis A. the defect compromises structural integrity Osteoradionecrosis and medication related of the tooth. What is the most appropriate immediate management? Initial caries of enamel has which of the following shape in pits and fissures? A. excessive sweating. Note: Some of the items in the Released Test Item Bank may have been discontinued due to outdated science or errors. Which of the following is an indication for the C. C. B. A. fine and brittle hair. Teeth that have proximal cavitated lesions. ©The National Dental Examining Board of Canada 2017 . A 5 year old has an enamel fracture of tooth 6. U-shape (apex towards dentino-enamel junction). denture design. B. D. V-shape (apex towards dentino-enamel junction). and translation.1. Teeth that have remained caries-free for 4 years and more. B. Turbid. Alveolar fracture. have identical pathophysiology. tendency to bruise easily. Smooth sharp edges. content. the area is to be included in a partial C. C. Patient’s water supply is non-fluoridated. a brown lesion is present in an elderly patient. D. B. there is increasing thermal sensitivity. The NDEB periodically reviews the bank to improve its quality. Which of the following dentinal caries zones is C. use of occlusal sealants? D. C. Angle Class I occlusion with 30% overbite sublingually. C. Actinomyces viscosus. Enterococcus faecalis. reduces the likelihood of an allergic statistical method. correction of a single tooth anterior crossbite are A. D. D. unpredictable. B.3 mg epinephrine C. D. maxillary first molars. ©The National Dental Examining Board of Canada 2017 . factor means that the mass of permanent teeth C. E. The two main conditions complicating the D. tips. Maxillary mesiolingual cusps that face becomes quickly and visibly swollen on interdigitate with the central fossa of the the left side. Epinephrine in local anesthetic solutions A. apply a cold compress. maxillary first or second premolars. a decreased overbite and a large labial frenum. The NDEB periodically reviews the bank to improve its quality. choosing a 75% probability reaction. is A. lack of spacing and decreased overbite. Note: Some of the items in the Released Test Item Bank may have been discontinued due to outdated science or errors. accurately predicted. Veillonella alcalescens. and 2mm overjet with mild maxillary C. refer for immediate medical treatment. apply a hot compress. C. a slightly narrow food table. normal contour reproduced. mandibular incisors. B. underestimated.Bacteria that are strongly associated with the Which of the following is the most acceptable onset of caries are extraction pattern in the management of an Angle Class II malocclusion in a non-growing A. be to B. B. balances the pH of the solution. A. increases the duration of anesthesia. B. the patient’s A. patient? Extraction of B. the margins extended 1mm subgingivally. A. C. D. Maxillary mesiobuccal cusp tips that contact the mandibular mesiobuccal cusp A. a slightly overcontoured tooth form. 2mm overjet and good arch alignment. using the Moyers B. Which of the following should be corrected through orthodontic intervention? Immediately following a left posterior superior alveolar nerve block injection. a full C. D. C. B. In a mixed dentition analysis. Streptococcus mutans. content. spacing. Angle Class II molar relationship with 30% overbite. overestimated. gold crown should have D. In addition. The immediate treatment should mandibular molars. and translation. mandibular first or second premolars. the format of some items is not currently used. In order to prevent gingival recession. an increased overbite and a large labial frenum. Lactobacillus caseii. administer 0. lack of spacing and increased overbite. the liner will leach fluoride and provide D. amalgam restorations when the A. if the liner extends to the gingival cavosurface margin. arterioles of pulpal tissue are engorged for long periods creating temporary episodes of pain. ©The National Dental Examining Board of Canada 2017 . within the root canal by using B. the preparation will have reduced B. and translation. C. The NDEB periodically reviews the bank to improve its quality. D. a transient invasion of bacterial elements Proximal retention grooves are necessary to into the outer lying stroma of the pulpal provide resistance for proximal-occlusal tissue. it is likely that Gutta-percha may be softened or dissolved A. D. C. D. caries resistance. restoration is a pin-retained cusp amalgam preparation diverge toward the replacement. the tooth will have reduced temperature sensitivity. The mesial and distal walls of a Class I B. Note: Some of the items in the Released Test Item Bank may have been discontinued due to outdated science or errors. maxillary second premolar. resist the forces of mastication. liner dissolution will lead to leakage. alcohol. extend the preparation into areas more readily cleansed. The pulpal floor of a Class II cavity is cut perpendicular to the long axis of the tooth EXCEPT in the A. occlusal extension is wide faciolingually relative to the proximal extension. prevent undermining of the marginal ridges. angles. B. a chronic situation whereby minute angles. C. ethyl chloride. mandibular second premolar. mesio-gingival and disto-gingival line C. mandibular first premolar. xylene (xylol®). bonded amalgam technique is not being B. dilated vessels and increased blood flow. C. the format of some items is not currently used. maxillary first premolar. C. provide resistance and retention form. D. A. an acute inflammation with intermittent paroxysms of pain which may become A. occlusal surface in order to C. occlusal extension is narrow faciolingually relative to the proximal extension. A. occluso-axial and gingivo-axial line continuous. an increased volume of blood within B. eugenol. For a Class II amalgam restoration. In addition.The principal internal retention for a Class V Hyperemia of the pulp is amalgam cavity preparation is established at the A. used. mesio-axial and disto-axial line angles. content. resistance to fracture. D. B. Radiographically. B. C. pulp. scleroderma. The lamina dura is Generalized widening of the periodontal ligament space is a radiologic characteristic of A. cementum. Down syndrome. enamel. B. acromegaly. Bell's palsy. spongy bone. None of the above. continued growth of the mandibular The tissue which cannot be seen on dental condyles. an increase in size of the bones and radiographs is soft tissues of the hands. In addition. B. D. Note: Some of the items in the Released Test Item Bank may have been discontinued due to outdated science or errors. Multiple neurofibromatosis and "café au lait" Which of the following is the most appropriate spots on the skin are typical of early management for a patient with primary herpetic gingivostomatitis? Prescribing A. supraorbital ridges and air sinuses suggest A. D. Raynaud's disease. pituitary adenoma. a systemic antiviral. Parkinson's disease. C. E. canals. A. a systemic antibiotic. Ménière's syndrome. B. D. D. lupus erythematosis. E. D. B. Stevens-Johnson syndrome. A. radiolucency around the crown of an C. ©The National Dental Examining Board of Canada 2017 . content. radiolucency containing multiple E. E. D. In an adult. Plummer-Vinson syndrome. hyperthyroidism. corticosteroids locally and systemically. D. Von Recklinghausen's disease. the format of some items is not currently used. B. mixed radiolucency and radiopacity. Osteosarcoma. a cribiform plate perforated by nutrient C. immature bone. cortical bone. Gardner's syndrome. B. B. The NDEB periodically reviews the bank to improve its quality. osteitis deformans. A. dentin. A. rudimentary teeth. Addison’s disease. feet. impacted tooth. a keratocystic odontogenic Extreme widening of the periodontal ligament tumour (odontogenic keratocyst) can appear as may be seen in a A. C. periodontal ligament. a systemic antifungal. gigantism. C. C. and translation. C. mandibular central incisor. insert a habit-breaking appliance. Following premature deciduous tooth loss. content. The most significant factor in determining the A person who has sickle cell anemia may show prognosis of anterior crossbite correction is the certain radiographic changes in the bones of the skull. encourage habit cessation and observe. Addison's disease. space loss occurs most frequently in the area of the A. shape of the tooth involved. Diphenhydramine. Nitrous oxide. D. interstitial growth. C. A. C. D. E. A 6 year old has an open bite caused by active thumbsucking.In an adult. D. anesthetic in a patient allergic to both amide B. E. maxillary first premolar. Bone tissue grows by Which of the following may be used as a local A. differentiation of cartilaginous tissue. C. B. gross irregularities with exostosis formation. ‟punched-out” radiolucent lesions. mandibular second premolar. D. Ethyl aminobenzoate. D. proliferation of endodermal tissue. B. 10 and 12 years. osteoclastic activity. age of patient. C. refer to a psychologist for evaluation. Cushing's disease. D. hyperthyroidism. B. B. a moth-eaten appearance of the bone. a "hair on end" effect. Note: Some of the items in the Released Test Item Bank may have been discontinued due to outdated science or errors. bond limited fixed bracket. and translation. D. Differential white cell count. C. progressive increase in mandibular The greatest period of cranial growth occurs length and interdental spacing is a feature of between A. Serum alkaline phosphatase. 14 and 16 years. space available mesiodistally. In addition. hyperparathyroidism. These changes may be A. Urinalysis. the format of some items is not currently used. B. C. and ester-type local anesthetics? C. C. D. A. The most appropriate management is to Which one of the following tests is used to confirm the presence of an acute infection? A. A. Phenylephrine. ©The National Dental Examining Board of Canada 2017 . Erythrocyte sedimentation rate. B. B. A. depth of the overbite. 6 and 8 years. birth and 5 years. hyperpituitarism. The NDEB periodically reviews the bank to improve its quality. B. D. maxillary lateral incisor. C. Bupivacaine. E. there is radiolucency in the bifurcation. B. the use of a cavity liner. removal of caries will lead to pulp exposure. be overall smaller. Elimination of local etiologic factors. A. C. D. C. cribriform plate. treatment is a/an B. erosion. D. C. D. attrition. E. observation. the format of some items is not currently used. B. B. The most appropriate A. pulpotomy. pulpectomy. C. C. caries involves dentin to a depth of 1mm. pulpotomy. extraction of the tooth. D. a pulp capping with calcium hydroxide. periodontal ligament space. the eating. Periodontal flap surgery. has a large carious lesion on a resultant die will permanent molar. there is continuous pain. indirect pulp capping. syringe and tray material. B. C. In addition. D. content. be overall larger. In taking an impression with polysulfide or silicone materials. A. Final evaluation and maintenance on a A. specific treatment for the exposed pulp. a routine amalgam restoration without any D. decalcification. Note: Some of the items in the Released Test Item Bank may have been discontinued due to outdated science or errors. Scaling. A. D. D. ©The National Dental Examining Board of Canada 2017 . develop bubbles at the interface of the C. extraction. and translation. C. not be affected dimensionally. B. The radiographic term used to describe the In a 5 year old. cancellous bone. if the heavy bodied tray A 7 year old child who complains of pain when material begins to set before seating. a small mechanical exposure in dense bone of the socket and septal crest is a vital primary molar would be treated by A. lamina dura.The most appropriate treatment for an 11 year Which of the following is essential for old who has intermittent swelling and pain successful periodontal treatment? associated with a central incisor which was traumatized 6 months ago is A. The NDEB periodically reviews the bank to improve its quality. B. direct pulp capping. partial pulpotomy. develop a rough surface texture. abrasion. A radiograph reveals no periapical change. B. one-year recall. cortical bone. The physiologic wear of hard dental tissue An indirect pulp capping procedure for primary resulting from mastication is known as molar teeth is indicated when A. equally from the abutment teeth and the C. C. Clasps should be designed so that upon insertion or removal of a partial denture the A distal extension removable partial denture is reciprocal arms contact the abutment teeth one that receives its support when the retentive arms pass over the height of contour in order to A. the format of some items is not currently used. insufficient horizontal overlap (overjet). percentage elongation. provide needed support to abutment teeth tissues. tooth structure. to reduce the copper-oxide content of the D. increased vertical dimension. cause difficulty in swallowing. the function of the fluoride flux angular cheilitis. D. D. cause injury to the abutment teeth. B. during a period of added stress. described by the A. prevent distortion of the clasps. C. parallel without excessive removal of B. survey line. An excessively thick palatal bar of a maxillary C. Note: Some of the items in the Released Test Item Bank may have been discontinued due to outdated science or errors. A. In addition. the retainers can be so prepared as to have equal retentive qualities. C. The line drawn through the occlusal rests of two principal abutments for a removable partial denture is the A. line of greatest torque. A "broken stress" or "non-rigid" connector is C. D. excessive vertical overlap (overbite). mostly from the tissues. alloy. A. distort under occlusal stress. B. B. axis of rotation. to reduce the formation of chromium The stiffness of a material can best be oxide during soldering. elastic limit.In soldering nickel-cobalt-chromium alloys and A patient wearing complete dentures has stainless steel. constructing a mandibular fixed partial denture will prosthesis. irritate the palatal tissues. 2 or 3 teeth are to be replaced. solder. terminal line. modulus of resilience. A. ©The National Dental Examining Board of Canada 2017 . The NDEB periodically reviews the bank to improve its quality. assure complete seating of the framework. B. indicated for a fixed partial denture when D. to lower the melting temperature of the B. B. B. modulus of elasticity. decreased vertical dimension. and translation. D. C. mostly from the abutment teeth. D. content. from the tissues only. C. to stop the flow of the molten solder onto undesired areas. The most likely cause is is A. the abutments cannot be prepared in A. dividing stress by strain below elastic bringing the teeth into occlusion. B. Distortion of a wax pattern is mainly due to A. refrigeration. dividing strain by stress. increasing early strength. In addition. insufficient bulk of material. Molecular attraction between unlike substances is called Varying the mercury content of an amalgam A. retarding setting rate. and translation. syneresis. insufficient paraffin ingredient. cohesion. when a complete denture opposes natural D. B. having the patient leave the dentures out by strain.For a patient with new complete dentures. D. E. C. C. remounting the dentures in the articulator using remount casts and new interocclusal records. directing the patient to close the jaws. A. D. at the time the dentures are removed from the flask. after the dentures have been in the mouth 24 hours. increasing flow. relaxation of stresses introduced during manipulation. insufficient plasticity during manipulation. C. D. A. A decrease in the particle size of the amalgam B. the articulator. water at room temperature. absorption. zinc. The lower the mercury content the greater the strength. of the mouth for 24 hours. adhesion. The higher the mercury content the D. ©The National Dental Examining Board of Canada 2017 . The modulus of elasticity of a material is errors in centric occlusion are best adjusted by determined by A. B. B. D. alloy will affect the amalgam by C. C. copper. when the dentures have been stored in tap B. D. The higher the mercury content the less the flow. A. results in which of the following? B. squaring proportional limit and dividing C. The NDEB periodically reviews the bank to improve its quality. limit. decreasing expansion. The greatest dimensional change in denture bases will occur Gold alloy can be given a white colour by introducing sufficient A. Note: Some of the items in the Released Test Item Bank may have been discontinued due to outdated science or errors. occlusion and making a transfer record to C. greater the strength. The lower the mercury content the greater the flow. having the patient close in centric B. teeth. multiplying proportional limit by strain. D. palladium. the format of some items is not currently used. C. cadmium. content. A. strength. Paget's disease. C. ©The National Dental Examining Board of Canada 2017 . B. A. D. D. Porion and orbitale. A. Porion and sella. zinc phosphate cement. The NDEB periodically reviews the bank to improve its quality. the format of some items is not currently used. orthodontic tooth movement. show decreased corrosion and marginal soft tissues of the mouth. polycarboxylate cement. Note: Some of the items in the Released Test Item Bank may have been discontinued due to outdated science or errors. return to a point beyond its original dimension. zinc oxide eugenol base. return to its original dimension. and translation. D. A. Nasion and sella. In addition. infection. hypersensitive. breakdown when the copper tin phase is C. scleroderma. content. identify systemic problems affecting the C. buccal contour. constructed by joining which of the following B. soft. D. contain zinc to reduce galvanic corrosion. C. marginal ridge contour. C. lingual contour. C. Widening of the periodontal space is NOT seen radiographically in The Frankfort-horizontal is a reference plane A. determine the level of muscle B. remain distorted. reveal retained roots or residual areas of increased. B. is more porous. In placing an amalgam. D. is stronger. is less brittle. show decreased flow when the tin mercury phase is increased. A. B. glass-ionomer lining. Sclerotic dentin is Using less water for mixing plaster of Paris will result in set plaster that A. B. The elastic limit of a material is the greatest load to which a material can be subjected to in order that it will A. injurious to the pulp. Porion and nasion. contracts. calcium hydroxide lining. E. contain nickel to increase the yield attachments. trauma from occlusion. D. gingival contour. fracture. D. B. D. D.Dental amalgam restorations On an edentulous patient. interproximal wedging will be most effective in controlling the Incomplete polymerization of composite resin will occur when the resin comes in contact with A. B. a panoramic radiograph is used to A. landmarks? C. C. B. assess the vertical dimension. resistant to caries. C. In addition. D. C. Which one of the following cements is anticariogenic because of fluoride ion release? A. Hyperplasia of the gingiva. C. a lead apron. change the catalyst/base ratio. high speed film. Glass ionomer. long buccal. which of the following will NOT be found? C. Lingual. Mental. Zinc phosphate. D. A. The NDEB periodically reviews the bank to improve its quality. Cervical. The best way to increase the working time of a polyvinylsiloxane is to The surgical removal of tooth 3. cement dissolution will lead to leakage. The vertical dimension of occlusion will C. 3-4mm apical to the cemento-enamel junction. ©The National Dental Examining Board of Canada 2017 . occlusal contacts are to be restored. C. the preparation will lack retention form. and translation. Giant cell tumours of bone. the preparation will lack resistance form A. B. A. C. the preparation will lack appropriate junction. reduce mixing time. long buccal. In primary or secondary hyperparathyroidism. added filtration. Note: Some of the items in the Released Test Item Bank may have been discontinued due to outdated science or errors. More than one third of the patients A. add oleic acid. to bulk fracture. content. The patient shows signs of a temporomandibular disorder. Lingual. D.6 requires anesthesia of the inferior alveolar nerve as well A. Alteration of the trabecular bone pattern. Polycarboxylate. If the lining cement is left on the gingival cavosurface margin of a Class II amalgam restoration. the format of some items is not currently used. the most effective A centric relation record must be used to method of reducing patient somatic exposure is articulate casts in which of the following cases? to use A. On bite-wing radiographs. cervical plexus. B. B. at the cemento-enamel junction. the normal alveolar A. B. be modified on the articulator. outline form. D. 1-2mm apical to the cemento-enamel D. The patient's centric occlusion and centric relation do not match. B. Generalized loss of lamina dura. C.In dental radiography. as which of the following nerves? B. collimation. long buccal. Resin. crest on a young adult is B. D. B. C. D. refrigerate the material. rounded internal line angles.Which muscle is LEAST likely to dislodge a A 12 year old female patient has a mandibular denture? developmental age of 8 years. Distal of the primary canine to mesial of craniofacial radiograph shows punched-out the permanent first molar. prilocaine. Polyvinylsiloxane. Mylohyoid. A. A. Radiographic examination reveals mandibular dysplasia. In addition. multiple myeloma. hyposecretion of growth hormone. to 10 years. the format of some items is not currently used. mepivacaine. Mesial of the primary first molar to mesial of the permanent first molar. D. fever and fatigue. E. Clinical examination A. Mesial of the primary first molar to distal B. delayed eruption of the permanent canines and absence of the premolars. A B. Irreversible hydrocolloid. A 72 year old male patient complains of bone pain. B. D. Burkitt’s lymphoma. The NDEB periodically reviews the bank to improve its quality. still produce accurate dies? B. crowding of the primary and permanent teeth. C. abnormal tooth eruption pattern. C. of the permanent first molar. B. lidocaine. C. C. Panoramic radiographs reveal an D. Reversible hydrocolloid. D. the permanent first molar. content. B. chondrosarcoma. Note: Some of the items in the Released Test Item Bank may have been discontinued due to outdated science or errors. hypogonadism. Polysulfide. facial height. bupivacaine. D. hypothyroidism. A. a minimum incisal reduction of 1. The most likely diagnosis is C. a sloping shoulder (long bevel) margin. a minimum margin depth of 0. C. hypoparathyroidism. Dental development is equivalent C. D. radiolucencies. Condensation reaction silicone.0mm. delayed growth of the cranial vault and reduced B. Masseter. The most likely diagnosis is Methemoglobinemia is a potential complication of an excessive dose of A. A bitewing radiograph of an early mixed dentition should include the following proximal surfaces. Buccinator. and translation. A. Mentalis. ©The National Dental Examining Board of Canada 2017 . acute lymphocytic leukemia. The tongue has a nodular appearance.5mm lingually. A full coverage all-ceramic anterior crown requires Which impression material can be stored for more than 24 hours before being poured and A. Distal of the primary canine to distal of shows petechiae on the skin and oral mucosa. D. developing stage. strength). have better resistance to marginal set. maxillary molars. B. Doubling the diameter of a round stainless steel orthodontic wire decreases its springiness how The type of amalgam requiring the least many times? volume of mercury for its setting reaction is A. C. over the C. content. fracture. The NDEB periodically reviews the bank to improve its quality. right posterior maxilla. taurodontism. D. high copper. 12. D. D. B. spherical. B. post-developing rinse stage. The most common location of a lateral periodontal cyst is in the area of the A ghost-like opaque image in a panoramic radiograph caused by a metal earring worn in A. the format of some items is not currently used. D. left mandibular ramus. it is recommended to spherical amalgams A. C. dentinogenesis imperfecta. halide is removed from the emulsion during the C. left posterior maxilla. require shorter trituration time. A. lathe cut. E. mix the cement on a waxed paper pad. 16. B. apply continuous occlusal loading while C. A. mandibular molars. right mandibular ramus. 8. the lobe of the left ear will be superimposed B. The inherited defect of teeth that may be associated with osteogenesis imperfecta is A. full crown. amelogenesis imperfecta. 4. A. D. B. D.When using a zinc phosphate cement to lute a When compared with admixed amalgams. During radiographic film processing. tarnish more. In addition. mandibular premolars. admixed. maxillary incisors. low copper. dentin dysplasia. ©The National Dental Examining Board of Canada 2017 . B. C. D. fixing stage. A. silver B. C. post-fixing wash stage. and translation. leave the tooth moist but not wet. are stronger (24 hour compressive the cement sets. C. Note: Some of the items in the Released Test Item Bank may have been discontinued due to outdated science or errors. E. E. regional odontodysplasia. require less condensation pressure. remove any excess before the cement is D. Occlusal. E. D. remount cast. B. Which of the following conditions is most likely to result in new periosteal bone A 5 year old child is diagnosed with leukocyte formation? adherence deficiency and is also affected with generalized severe bone loss adjacent to his A. A. B. D. A. is a barrier to bacterial invasion. content. equally chemical and mechanical. Gingival diseases modified by systemic factors. B. Bite-wing. D. Generalized chronic periodontitis. C. A. lateral pterygoid. Note: Some of the items in the Released Test Item Bank may have been discontinued due to outdated science or errors. In addition. muscle determining the form of the lingual flange in the molar region is A. primary teeth. During the fabrication of a removable complete B. cervical In gingivitis. and translation. C. ©The National Dental Examining Board of Canada 2017 . D. block out and relief is placed on the C. chemical. C. the format of some items is not currently used. Chronic osteomyelitis. Dentin dysplasia type II.Which radiograph best depicts the buccal The bond between porcelain and metal in a cortex of the mandible? ceramometal (porcelain bonded to metal) crown is A. mylohyoid. What is the diagnosis? B. master cast. C. Which disorder presents with all permanent teeth exhibiting bulbous crowns. C. the systemic disease. C. Generalized aggressive periodontitis. genioglossus. B. B. refractory cast. geniohyoid. Dentinogenesis imperfecta. Paget’s disease of bone. medial pterygoid. Fibrous dysplasia. Amelogenesis imperfecta. undergoes both degenerative and proliferative changes. The NDEB periodically reviews the bank to improve its quality. denture. D. D. is permeable to bacterial enzymes and B. D. mechanical. Dentin dysplasia type I. A. Periodontitis as a manifestation of For a mandibular denture impression. D. Periapical. the sulcular epithelium has the constriction and obliterated pulp canals and following characteristics EXCEPT it chambers? A. Hyperparathyroidism. diagnostic cast. toxins. C. may be ulcerated. Panoramic. neither chemical nor mechanical. C. C. E. secreting the calcospherites. the format of some items is not currently used. Tome’s processes sensitivity following the placement of a are responsible for posterior composite resin restoration with ideal occlusion is A. C. decrease of the freeway space. vertical axis and defines the position of by heat treatment increases the mandibular cast. horizontal transverse axis and defines the C. increase of the freeway space. excessive etching of enamel margins. unchanged. The NDEB periodically reviews the bank to improve its quality. a tendency for tensile fracture. and translation. B. downward and backward mandibular unchanged. E. infection in a healthy elderly patient. D. B. unchanged. A. inadequate polymerization of primer resin. ©The National Dental Examining Board of Canada 2017 . malleability. greater strength in tension than of the compression. elastic modulus. B. content. D. high thermal coefficients of expansion. decreased by one half. excessive primer resin. position of the maxillary cast. yield strength. When prescribing antibiotics for an orofacial D. B. duration E. The dose is An anterior bite plane will NOT result in A. A.The most likely cause of postoperative In the formation of the tooth. D. extrusion of the posterior teeth. D. D. laying down the enamel prisms. decreased by one third. adult dose and duration of the prescription should be written using the following guidelines. horizontal transverse axis and defines the position of the mandibular cast. A. increased by one third. In addition. C. vertical axis and defines the position of the maxillary cast. Hardening of Type IV cast gold dental alloys D. unchanged. duration extended by one half. duration unchanged. the usual E. Ceramics used in dentistry exhibit A facebow record provides an approximation A. rotation. A. Note: Some of the items in the Released Test Item Bank may have been discontinued due to outdated science or errors. ductility. reorganizing the collagen fibres. coring. forming the odontoblastic tubules. C. increase of the vertical dimension. inadequate peripheral seal. B. chemical instability. duration B. duration unchanged. generating the incremental lines. B. C. antacids. C. clinical attachment loss. slough. D. metronidazole should avoid B. pulpectomy. The most likely diagnosis for a 5 year old patient with multiple well-defined multilocular radiolucencies of the maxilla and mandible is Which of the following medications can cause A. which of the radiograph reveals no abnormalities. Periodontal pocketing. B. grapefruit juice. extraction. depth of periodontal probing. D. Asymptomatic apical periodontitis (chronic periradicular periodontitis). A. The most following will elicit the most rapid response in appropriate management is a/an a patient? A. C. D. cemento-osseous dysplasia). the progression of periodontitis can be determined best by the increasing A patient that has been prescribed A. Note: Some of the items in the Released Test Item Bank may have been discontinued due to outdated science or errors. A. and translation. B. Too large a dose. abscess). D. D. Bleeding from the gingiva. In addition. Angiotensin converting enzyme inhibitors. pulpotomy. severity of gingival inflammation. hyperthyroidism. gingival enlargement? B. regular recall for observation. D.In which of the following pathological Which of the following is NOT suggestive of a conditions would a lower central incisor tooth diagnosis of necrotizing ulcerative be expected to respond to heat. A periapical With respect to local anesthetic. ameloblastomas. cold and an gingivitis (NUG)? electric pulp test? A. Diuretics. B. number of bleeding sites on probing. C. E. A. C. alcohol. D. cheese. Slow elimination. “Punched-out” papillae with necrotic periradicular abscess). Slow biotransformation. Beta blockers. B. C. cherubism. Calcium channel blockers. Rapid absorption. E. Metallic taste. The NDEB periodically reviews the bank to improve its quality. C. E. A. Bad breath. the format of some items is not currently used. A 4 year old child presents with a history of trauma and an asymptomatic discoloured primary maxillary left incisor. D. content. hypophosphatasia. Intravascular injection. Chronic apical abscess (chronic B. Periapical osseous dysplasia (periapical E. Acute apical abscess (acute periradicular C. ameloblastic fibromas. C. ©The National Dental Examining Board of Canada 2017 . caffeine. B. Clinically. Acetaminophen.An amalgam coronal-radicular core build-up Failure of bone resorption over an erupting for endodontically treated molar teeth requires tooth is due to lack of A. ferrule and amalgam B. Note: Some of the items in the Released Test Item Bank may have been discontinued due to outdated science or errors. Including occlusal grooves makes the restoration more fracture-resistant compared to a slot preparation. In addition. fibroblasts. A. bonding. Which of the following has anticonvulsant properties? Which statement is true with respect to Class II A. B. A. Meckel’s cartilage. A. of nuclear/cytoplasmic ratio and abnormal mitoses is A. an adequate pulp chamber and ferrule. B. D. maxillary molars. the presence of a post. D. The term used to describe epithelial changes C. alteration D. content. B. Nitrous oxide. acanthosis. C. Extension for prevention is a key element. D. B. Adaptation of the resin during placement is enhanced. Stress concentration is reduced. Tooth structure is conserved. and translation. C. composite resin preparations? B. Ankylosis of primary teeth is most frequently Which of the following statements is FALSE observed in with respect to rounded internal line angles in preparations for Class II composite resins? A. B. C. including nuclear hyperchromatism. backward placement of the film. C. Lack of ramus height is caused by faulty D. The NDEB periodically reviews the bank to improve its quality. Diazepam. the temporomandibular joint. the format of some items is not currently used. a pulp chamber. D. hyperparakeratosis. C. osteocytes. C. A. C. D. membranous bone. The preparation has a “standard” shape for Excessively dark radiographs will result from each individual tooth. overexposure. too little milliamperage. neutrophils. maxillary canines. D. B. the use of retentive threaded pins. C. dysplasia. Codeine. Diphenhydramine. B. mandibular molars. acantholysis. Fissures are only included when carious. D. mandibular incisors. E. ©The National Dental Examining Board of Canada 2017 . Resistance form is compromised. endochondral bone. underdevelopment. development of A. osteoclasts. C. Paget’s disease. initiate periodontitis. The most accurate indicator of caries activity in D. condensation technique. Which of the following should be performed to B. vertical dimension of occlusion. apply caries detector dyes. Fainting. terminology in all explanations. D. phonetic tests can be used D. carving technique. D. B. Swelling. to determine A. C. B. E. The residual mercury content of the amalgam C.7. B. Note: Some of the items in the Released Test Item Bank may have been discontinued due to outdated science or errors. evaluate the hardness. ensure a well-adapted and functional stainless C. The NDEB periodically reviews the bank to improve its quality. assess the colour. B. A single ceramometal crown on tooth 1. Which of the following would require a custom B. Cherubism. flange extension. Which of the following is a sign of local anesthetic overdose? A. A. Convulsions. affect the progression of periodontitis. An all ceramic crown on tooth 4. B. content.7. Trauma from occlusion may A. use bite-wing radiographs. Break interproximal contacts using a tapered bur. involves using scientific dental D. and translation.Which disorder is associated with Tell-show-do technique when used for hypercementosis of teeth? behaviour management of pediatric patients A. A fixed partial denture from tooth 1. age. C. initiate marginal gingivitis. restoration is significantly affected by D.5-3. amount of amalgam used. In addition. A.3. ©The National Dental Examining Board of Canada 2017 . will decrease a child’s fear of the unknown and their anticipation of pain. Hyperparathyroidism. Wheezing. Rash. incisal guide table for a patient with mutually C. C. A. complete dentures. E. A fixed partial denture from tooth 3. Prepare a well-defined chamfer margin. root caries lesions is to A. the format of some items is not currently used.4. Cement the crown with rubber dam in place. steel crown? D. Fibrous dysplasia. esthetics. protrusion. At the wax rim stage of jaw relation records for C. Prepare the tooth with sharp line angles. cavity outline. protected occlusion? D. A. works best for children under 3 years of B. centric relation. B. C.1-1. affect the blood supply to gingiva. C. gingivally. gingival recession. hours each night? C. horizontally. D. D. Convex surface of the labial flange. C. A conical pontic replacing a mandibular first molar should be designed so that A. hyperpituitarism. Osteogenesis imperfecta. A. C. 1 to 2mm apical to the CEJ. In addition. content. Osteomalacia. the porcelain to metal junction is on its gingival surface. The NDEB periodically reviews the bank to improve its quality. scleroderma. Decreased bonding potential to enamel. tooth sensitivity. hyperparathyroidism. Decreased enamel surface porosity. occlusally. Thickness of the border in the vestibule. at the CEJ. gingiva. C. of the simulated gingiva. B. opening of the dentinal tubules. horizontal bone loss. D. B. B. Albright's syndrome. elimination of the smear layer. demineralization of the superficial dentin. it has open gingival embrasures. hyperthyroidism. Which of the following has the potential for D. malignant transformation? A. Festooned carvings on the labial surface D. D. Significant incidence of irreversible pulpitis. elimination of the collagen fibres. A. Note: Some of the items in the Released Test Item Bank may have been discontinued due to outdated science or errors. Labial surface of the teeth and simulated A. and translation. D. A. C. C. Increased tooth mobility and the absence of lamina dura are signs of Which of the following will result from a 2 week regimen of tooth whitening using a 10% A. A Class II amalgam preparation on a primary tooth does NOT require a gingival bevel because the enamel rods in the area incline The normal position of the alveolar crest in A.Proper lip support for a maxillary complete A 10-15 second application of 37% phosphoric denture is provided primarily by which of the acid on prepared dentin will result in all of the following? following EXCEPT A. the format of some items is not currently used. 3 to 4mm apical to the CEJ. healthy periodontium is B. vertically. it seals the mucogingival fold. B. D. C. ©The National Dental Examining Board of Canada 2017 . B. Primary occlusal trauma can cause C. carbamide peroxide gel in a custom tray for 8 B. D. B. 1 to 2mm coronal to the CEJ. B. Paget's disease. furcation involvement. the gingival surface is concave buccolingually. Moderate demineralization of enamel. What statement related to self-threaded pins is FALSE? A reciprocal clasp arm on a removable partial denture will provide A. Acrylic. the space between the tray and the teeth is chambers and apical radiolucencies? 1-2mm. What is the name of the process by which carbamide peroxide bleaches the teeth? Which of the following dental materials is the most radiolucent? A. Which drug should be administered as the initial management for a patient with chest pain consistent with a myocardial infarction? A. Atropine. D. Zinc oxide. D. Amelogenesis imperfecta. D. The drill has a smaller diameter than the pin. Hydrogenation. Note: Some of the items in the Released Test Item Bank may have been discontinued due to outdated science or errors. indirect retention. small crescent-shaped pulp A. Diphenhydramine. The ideal depth for the pin hole is 3mm. C. A. resistance of the restoration. Addition. stabilization. the impression is immersed in disinfectant for one hour before pouring. B. Subtraction. Dentin dysplasia type II. C.The accuracy of alginate impression materials Which disorder presents with all permanent will be improved if teeth exhibiting shortened roots. Dentinogenesis imperfecta. Oxidation. D. B. obliterated pulp canals. allows 4-5mm of alginate. B. D. B. The NDEB periodically reviews the bank to improve its quality. E. direct retention. B. restoration. ©The National Dental Examining Board of Canada 2017 . C. D. B. D. the format of some items is not currently used. the impression is removed slowly from the C. content. C. Acetylsalicylic acid. Epinephrine. In addition. the space between the tray and the teeth A. The pin can increase retention of the C. Gutta-percha. undercuts around the teeth. C. B. Dentin dysplasia type I. and translation. Lidocaine. support. The pin does not increase fracture A. Porcelain. C. D. a full complement of teeth. B. gutta-percha pulpectomy. maxilla to mandible. maxillary first molar. crown and applying apical pressure during C. is successful treatment in 90 percent of Angle's classification of occlusion is based on cases. The NDEB periodically reviews the bank to improve its quality. B. ©The National Dental Examining Board of Canada 2017 . permanent molar relationship is indicative of E. following the loss of the deciduous A. at the cementoenamel junction of the tooth apical surgery.1 involves placing the beaks of the forceps following a trauma 24 hours ago. at the cementoenamel junction of the tooth and applying a rotational force during Space closure is LEAST likely to occur luxation. In addition. B. B. will stimulate apical closure. maxillary central incisor. Paget's disease. hyperthyroidism. D. the format of some items is not currently used. E. is the treatment of choice for small B. A. as far apically on the tooth root as possible appropriate management is a and applying apical pressure during luxation. ideal molar occlusion. radiograph is suggestive of D. A. C. D. Angle Class II malocclusion. luxation. In primary teeth. MTA pulpotomy. A. Angle Class III malocclusion. The absence of lamina dura on a dental C. gutta-percha pulpectomy followed by an C. on the lingual and buccal enamel of the B. MTA apexification. a pulpotomy using calcium D. normally developing occlusion.The proper placement of a lower universal A 7 year old child presents with a 3mm coronal extraction forceps for the removal of tooth 4. B. The tooth is sensitive to hot and cold fluids. and gently pulling upward during luxation. Note: Some of the items in the Released Test Item Bank may have been discontinued due to outdated science or errors. and translation. The most A. E. MTA direct pulp capping. as far apically on the tooth root as possible and gently pulling upward during luxation. D. mandibular second molar. vertical relationships in the lower face. In the mixed dentition. vitamin D deficiency. content. antero-posterior skeletal relationship of mechanical exposures. C.6 fracture with pulp exposure of tooth 2. C. hyperparathyroidism. mandibular canine. A. will cause an acute inflammatory reaction. acromegaly. E. will cause internal resorption. hydroxide A. antero-posterior relationship of maxillary and mandibular first permanent molars. an end-to-end first D. Exclusive of third molars. apposition on the free edge of the palatine resorption. B. A 12 year old patient with a complete dentition The body of the mandible increases in length to has enamel hypoplasia of tooth 1. the downward and forward growth of the appointment. an abscessed tooth 5. A. D. An amalgam placed at the same C. border of the ramus and apposition of C. B. the permanent dentition is usually completely erupted by the age of During orthodontic therapy. 15 to 17 years. the format of some items is not currently used. bone on the posterior border of the ramus. maxillary canine. D. decreased. the principle factor to consider is the permanent tooth is usually the A. a febrile illness. apposition of bone at the symphysis and posterior border of the ramus of the Which of the following permanent restorations mandible. mandibular canine. C. morphology of the anterior teeth. sequence of eruption of the permanent C. radiograph demonstrates no internal B. maxillary first molar.4. A. C. D. In addition. systemic tetracycline. A stainless steel crown placed when a A. high fluoride intake. B. D. C. A stainless steel crown placed at the same The palate grows in length by appointment. B. anterior growth. endochondral growth.Prior to the correction of a one tooth anterior The last primary tooth to be replaced by a crossbite. is the most appropriate after a formocresol E. bone. content. This accommodate the permanent second molar by condition was most likely caused by A. increased. pulpotomy has been completed on a primary molar? A. adequacy of mesio-distal space. A. D. An amalgam placed when a radiograph indicates no bone destruction between the roots. B. mandibular second molar. D. and translation. the width of the periodontal ligament radiographically appears A. resorption of bone along the anterior B. apposition of bone in the condyle. B. apposition of bone on the alveolar margin E. developmental age of the patient. and lower border of the body of the mandible. 18 to 21 years. dentition. low calcium intake. The NDEB periodically reviews the bank to improve its quality. 12 to 14 years. ©The National Dental Examining Board of Canada 2017 . 9 to 11 years. nasal septum. C.4. C. Note: Some of the items in the Released Test Item Bank may have been discontinued due to outdated science or errors. unchanged. D. ©The National Dental Examining Board of Canada 2017 . rigid. flexible enough to permit easy insertion. the bond between the materials is A. C. C. mainly chemical. content. point. In metal-ceramic crowns. and translation. tends to reduce both hygroscopic and thermal expansions. B. D. B. a porcelain modifier. porosity due to boiling of the monomer. C. cover as much mucosa as possible. be constructed to have an occlusal surface D.Which of the following materials are LEAST A metal ceramic posterior fixed partial denture suitable for impressions for cast gold pontic should restorations? A. volumetric expansion of the denture causing an open bite. denture porosity is most likely to appear in the C. thinnest portion. mainly mechanical. buccal surface. D. D. B. carefully polished on the interior. desired because of the possibility of B. contributes to the overall expansion of the mold. In addition. Polyethers. Polysulfides. B. C. The binder in casting investments not only strengthens the investment. Impression trays should be E. decreased by oxides on the metal surface. shrinkage of the denture. A. thickest portion. C. a correct temperature control is A. Note: Some of the items in the Released Test Item Bank may have been discontinued due to outdated science or errors. but also A. Irreversible hydrocolloids. B. C. the format of some items is not currently used. increases the thermal expansion of the mold. D. pure methyl alcohol. the correct viscosity is achieved by mixing the porcelain powder with In processing (polymerizing) an acrylic denture in a water bath. crazing of the denture base around the necks of the teeth. replaces. A. creates a reducing atmosphere in the mold. B. wider than the width of the tooth it replaces. palatal area. distilled water. provide adequate embrasure spaces. one quarter the width of the tooth it B. C. warpage. For application of porcelain to a ceramo-metal alloy. a mild detergent. decreased when the metal has a high yield A. In the processing of methyl methacrylate. Polyvinyl siloxanes. D. held in place by the patient. D. be constructed to have an occlusal surface A. The NDEB periodically reviews the bank to improve its quality. removed if no cortical bone is involved. A. C. medial pterygoid. resorption of the supporting edentulous B. sinusitis. muscle determining the form of the lingual C. the format of some items is not currently used. an allergy to the anesthetic agent. the patient's upright position in the chair. of the horizontal condylar guidance plane of B. guiding planes. B. modulus of elasticity. C. centric relation interocclusal record. anterior open bite. mastication. cuspal inclination of the teeth selected. effort is made to secure parallel tooth surfaces B. lowering of the yield strength. area. removed surgically. D. flow. In addition. angina. Immediately after tooth extraction. lateral pterygoid. mouth breathing. C. increased ductility. content. superior pharyngeal constrictor. orientation of the occlusal plane. skeletal malocclusion. A. In treatment planning for a complete denture. A. B. flange in the molar region is D. severe undercuts on the edentulous ridge should be In complete denture fabrication. ©The National Dental Examining Board of Canada 2017 . E. This is most likely precipitated by A. B. bracing areas. proportional limit. geniohyoid. A. D. D. D. Habitual thumbsucking that continues after the age of six commonly results in A. For a mandibular denture impression. B. dislodgement of the saddle area during D. increasing of the modulus of elasticity. to act as C. the epinephrine in the anesthetic agent. indirect retention. anxiety and anticipation of pain. Note: Some of the items in the Released Test Item Bank may have been discontinued due to outdated science or errors. occlusal rest areas. mylohyoid. with a known cardiac problem experiences E. Repeated clasp adjustment can result in fracture due to In designing a removable partial denture. incisal guidance. D. an A. C. strain hardening. temporomandibular joint dysfunction. a patient D. B. C. The NDEB periodically reviews the bank to improve its quality. the articulator is determined by the C. protrusive interocclusal record.A measure of the stiffness of a dental gold alloy A Kennedy Class II denture with no provision is expressed as its for indirect retention causes A. gingivitis. ultimate tensile strength. the angulation A. and translation. C. considered as retentive aids. retained but avoided by the denture base extension. the B. D. increasing vertical dimension. discrepancies. content. per unit of stress of the alloy in hard or soft condition. retraction of the maxillary molars. follolwing eruption of the lateral incisors. C. A. D. repositioning the mandible forward. B. ©The National Dental Examining Board of Canada 2017 . poorly formed enamel. C. A. In addition. lack of systemic fluoride. Which of the following conditions is most bite of the permanent maxillary central incisor appropriately treated upon diagnosis? is A. and translation. A. Posterior cross-bite with midline B. greater plastic deformation per unit of A. C. following eruption of the canines. excessive bottle use. radiolucent.The most appropriate time to correct a cross. D. Improper axial inclination. End-to-end molar relationships. C. nonexpansile. D. stress than the same alloy in a softened B. lack of calcium during pregnancy. B. maintain the alveolar ridge morphology. The NDEB periodically reviews the bank to improve its quality. C. C. B. D. For which of the following malocclusions is The Frankel functional regulator appliance serial extraction most appropriate? performs all of the following EXCEPT A. Angle Class II. follolwing eruption of the central incisors. the format of some items is not currently used. D. expansion of the dental arches. condition. Angle Class III. B. Angle Class I. A maxillary midline diastema. C. B. B. Note: Some of the items in the Released Test Item Bank may have been discontinued due to outdated science or errors. Idiopathic osteosclerosis is Mandibular overdentures are used to A. painful. during eruption of the central incisors. increase the strength of the denture. found mostly in the maxilla. improve periodontal health of abutment teeth. decrease costs. less plastic deformation per unit of stress incisors in a 12-month old child is most likely than the same alloy in a softened caused by condition. A hardened gold alloy will exhibit Cervical caries on the maxillary primary A. no difference in the plastic deformation D. C. Light is the diameter carious exposure on a vital permanent first molar in a 7 year old is A. and translation. Note: Some of the items in the Released Test Item Bank may have been discontinued due to outdated science or errors. that is nonvital. strength. decreased expansion and increased posterior teeth. initiator. B. B. B. limit the distal extension of the A. B. increased expansion and decreased mandibular denture. content. Light-cured dental composites set when The most appropriate management of a 4mm exposed to light. In addition. strength. direct pulp capping. inorganic mercuric compounds. C. C. terminator. phenolic cement. increased expansion and increased denture. polyalkenoic acid cement. partial pulpotomy. B. Zinc oxide eugenol cement is a/an The use of an intra-coronal attachment is CONTRAINDICATED for a tooth A. myocyte. activator. C. The NDEB periodically reviews the bank to improve its quality. E. the format of some items is not currently used. organo mercurial compounds. epithelial basal cell. C.In an edentulous patient. pulpectomy. limit the distal extensions of the maxillary D. extraction. ©The National Dental Examining Board of Canada 2017 . strength. D. C. requiring a core procedure build up. with short crown length. C. determine the location of the posterior C. D. phosphate cement. D. supporting a partial denture. decreased expansion and decreased palatal seal. A. neuron. affect the position and arrangement of the B. D. E. elemental mercury. inorganic mercurous compounds. The most toxic form of mercury is The most radiosensitive cell type is a/an A. B. the coronoid process Using more water when mixing dental stone may will result in a cast that exhibits A. D. catalyst. D. strength. A. adhesive resin cement. reactor. B. A. chondrocyte. resin modified glass ionomer cement. D. C. B. modulus of elasticity. Position of the mucogingival junction. Crown. Diabetes. D. Diameter of the probe tip. Heart disease. Ceramic inlay. midline deviation. D. Note: Some of the items in the Released Test Item Bank may have been discontinued due to outdated science or errors. C. superficial enamel demineralization. and translation. cytotoxicity. Rheumatoid arthritis. In addition. control the salivary flow. When gypsum is mixed with water and sets to form a dental cast. associated with the fully edentulous state? B. Bonded amalgam restoration. D. B. soft tissue reaction. C. Which of the following is the most appropriate Strain hardening a metal will reduce its treatment for an endodontically treated mandibular first molar with a previously placed A. Genetics. calcium phosphate. The NDEB periodically reviews the bank to improve its quality. sodium phosphate. calcium sulfate. A. sodium sulfate. C. The chemical that is used to retard the setting reaction in alginate impression materials is Which of the following conditions is NOT A. Angulation of the probe. C. ductility. ensure adequate impression material in B. MOD amalgam restoration? B. C. D. D. D. tooth sensitivity. calcium sulfate. D. C. the powder particles are replaced by crystals of Which of the following is NOT considered a risk factor for periodontal disease? A. yield strength. content. proportional limit. Altered taste perception. potassium phosphate. Probing force. C. A. Residual ridge reduction. C. Subgingival calculus. B. C. pain on closure. B. A. Tobacco use. trisodium phosphate. C. severe crowding. E. eliminate the intrasulcular debris.Which of the following does NOT affect The primary function of gingival retraction probing depth measurement? cord is to A. ©The National Dental Examining Board of Canada 2017 . Decrease in masticatory function. the format of some items is not currently used. displace the epithelial attachment. B. B. potassium sulfate. A. A. the sulcus. D. arch asymmetry. D. The most common clinical characteristic of a functional crossbite is The most common risk associated with vital bleaching using 10% carbamide peroxide in a A. Composite resin restoration. custom tray is B. D. mesial to distal. C. bacteria. C. A. medicate the canal to eliminate remaining A. A. less bone loss. distal to mesial. stimulate healing in the apical region. 6 months in utero. B. B. the format of some items is not currently used. maxillary skeletal expansion. growth modification. tooth pain upon percussion. In addition. prevent discolouration of the tooth. B. Compared to nonsmokers. Note: Some of the items in the Released Test Item Bank may have been discontinued due to outdated science or errors. more attachment loss. Root uprighting. increased restoration strength. anchorage augmentation. The NDEB periodically reviews the bank to improve its quality. nasal bone. buccal to lingual. and translation. birth. B. vertical tooth mobility. failure to remove the denture at night. should slope apically from B. 6 months. mandible. B. B. D. D. C. C. C. D. D.The primary function of root canal sealer is to The pulpal floor of an occlusal amalgam preparation on a mandibular first premolar A. Crown tipping. an allergy to the denture material. molar distalization. a candida infection. C. lingual to buccal. D. fill space between gutta-percha and pulp C. 12 months. A. B. superior translucency. more bleeding on probing. C. B. D. canal wall. mobility during occlusion. Orthodontic headgear can be used for all of the Fremitus is following EXCEPT A. D. cigarette smokers Which tooth movement is NOT possible with a are more likely to have removable appliance? A. D. B. A. superior esthetics. more conservative vestibular preparation. C. D. tooth mobility of grade II. maxilla. The last bone in the craniofacial complex to Papillary hyperplasia under a denture is usually stop growing is the due to A. ©The National Dental Examining Board of Canada 2017 . less gingival recession. content. frontal bone. C. An advantage of a metal-ceramic crown over Calcification of the permanent first molars an all-ceramic crown is normally begins at A. Crown rotation. an ill fitting denture. The technique of amalgam bonding can be Hypoglycemia in the conscious patient is most A. E. The occlusal cavosurface margin for a Class I amalgam restoration should be Which of the following tissues is LEAST sensitive to ionizing radiation? A. ischemic myocardial necrosis. when contamination with moisture is D. The NDEB periodically reviews the bank to improve its quality. D. D. Oral mucosa. chamfered. B. and translation. the format of some items is not currently used. Bone. Note: Some of the items in the Released Test Item Bank may have been discontinued due to outdated science or errors. atherosclerosis. C. Which of the following statements is correct What is the most effective local anesthetic with respect to zinc containing amalgams? technique for a patient with trismus who requires a pulpectomy on a mandibular molar? A. B. C. Mental nerve block. used in conjunction with copal varnish. Buccal nerve block. Amalgams containing zinc should be used C. females under the age of 80 years. insulin. Inferior alveolar nerve block. survival rates. occlusive coronary artery disease. E. glucagon. High copper amalgam restorations containing zinc demonstrate better overall A. C. E. seal. Gingival overhang. beveled. Inadequate marginal ridge morphology. C. effective in providing an improved initial C. 90°. D. B. Amalgams containing zinc produce a significantly better seal than zinc-free amalgams. content. D. B. Enamel. Gow-Gates block. relied upon to reinforce severely weakened A. oxygen. Inadequate proximal contact. D. Vazirani-Akinosi block. cusps. epinephrine. B. ©The National Dental Examining Board of Canada 2017 . C. B. In addition. A. A. acute. Zinc is added during the manufacturing operation to increase the solubility of tin in silver. Salivary gland. crushing substernal pain. D. C.What is the most likely cause of food impaction A major difference between stable angina at the site of a recently placed Class II pectoris and myocardial infarction is that stable composite resin restoration? angina pectoris does NOT involve A. carbohydrates. B. D. unavoidable during condensation. Poor oral hygiene. used as the primary means of retaining an appropriately managed with amalgam restoration. B. over trituration. Replace the restoration with a bonded amalgam restoration. the risk of When using alginate impression material. Adhesive porcelain fracture.0%. Remove the impression slowly from the occlusal area before removal. E. the format of some items is not currently used. posterior composite resin restoration with C. Ibuprofen. Replace the restoration with a reinforced zinc oxide eugenol restoration. D. The NDEB periodically reviews the bank to improve its quality. Acetylsalicylic acid.1 – 1. Note: Some of the items in the Released Test Item Bank may have been discontinued due to outdated science or errors. ©The National Dental Examining Board of Canada 2017 . D. Acetaminophen. Control the setting time by changing the D. acceptable occlusion? D. Store the impression in water at 37°C marginal ridge before removal. marginal ridge fracture of an amalgam which one of the following statements is restoration is reduced by correct? A. Sore throat. 10 – 15%. sensitivity 4 weeks after placement of a B. under trituration. 0%. Reversible tooth sensitivity. content. Recurrent caries. lack of zinc content. A dry and crumbly mix of amalgam can be the result of What is the most likely cause of long term clinical failure for a metal-ceramic crown with A. C. high copper content.Which of the following does NOT block The volumetric polymerization shrinkage of a cyclooxygenase-2? hybrid composite resin is in the order of A. Celecoxib. Which of the following is NOT a consequence of vital bleaching with 10% carbamide peroxide? Which is the most appropriate treatment for a patient who reports persistent thermal A. contouring and wedging the band. A. C. C. B. D. Soft tissue sloughing. C. and translation. leaving an excess of amalgam in the B. undercuts. Diflunisal. During matrix band removal. completing most of the shaping of the A. D. porcelain occlusal coverage on a molar? B. C. Replace the restoration with a bonded composite resin restoration. Cohesive porcelain fracture. A. B. C. B. B. D. Loss of retention. Pour the cast immediately following disinfection. D. A. prior to pouring the cast. and bands. B. Adjust the restoration slightly out of occlusion. In addition. 2 – 8%. 0. Weakening of the enamel. C. using universal circumferential retainers water/powder ratio. Reposition the teeth and do not splint. Anesthetize the long buccal nerve with the teeth? remaining lidocaine. In addition. Anesthetize the lingual nerve with the increase the retention of a Class III composite remaining lidocaine. Active zone.5ml of lidocaine 2% with interlocks with dentinal collagen? 1:100. resin restoration in primary incisor and canine C. Inhibition of prostaglandin synthesis. Smear layer. content. Monitor the teeth for re-eruption. Tinnitus.000 epinephrine and shows signs that the block was effective. placement of a rubber dam clamp on the B. amoxicillin. clindamycin. tetracycline. Using labial or lingual dovetails. the child complains that C. following is the most appropriate management? A. central incisors which are not in proximity to B. Antipyresis. Hybrid layer. Adhesive zone. A. the permanent successors. Which of the following procedures will B.What is the name of the area in which the resin A child has received an inferior alveolar nerve of the adhesive system micromechanically block using 1. Constipation. the “tooth ring” is hurting. The NDEB periodically reviews the bank to improve its quality. A patient has a prosthetic heart valve and is allergic to penicillin. D. However. appropriate management? D. The most appropriate antibiotic to be used prophylactically is A 2½ year old child has intruded maxillary A. permanent first molar. Extract the teeth. D. ©The National Dental Examining Board of Canada 2017 . C. Which of the following is NOT associated with the administration of acetylsalicylic acid? A. B. possible. E. the format of some items is not currently used. What is the most C. Reposition the teeth and splint. C. during A. Keeping the preparation as small as dam. Analgesia. B. Etching the preparation for an additional 30 seconds. Proceed with treatment without rubber A. and translation. erythromycin. D. metronidazole. C. Note: Some of the items in the Released Test Item Bank may have been discontinued due to outdated science or errors. Wait 15 minutes until more profound anesthesia is achieved. E. B. Which of the D. Benzodiazepines. B. permanent teeth? E. A. B. D. A prescription for antibiotics. Vigorous curettage of the socket. A. Opioids. Internal root resorption. ©The National Dental Examining Board of Canada 2017 . No treatment. for oral sedation in the adult patient? C.Which of the following is NOT a function of What is the most probable complication of a the wedge in the restoration of a Class II cavity local anesthetic injection into the parotid with amalgam? gland? A. Trismus. Be treated with fissure sealants. B. Placement of a dressing in the socket. 8½ years. E. Which of the following is LEAST likely to D. and translation. Hydrogen peroxide irrigation of the A 10 year old child with no previous caries socket. allowing the restoration to be placed in a dry field. content. Too high a water:powder ratio of the stone. In addition. 4½ years. Vertical root fracture. It separates the teeth to allow restoration of A. experience has proximal carious lesions in the B. At what age is a child expected to have C. band to the proximal portion of the D. It absorbs moisture from the cavity preparation. A. enamel only of several primary molars. Periradicular abscess. Poor flow of the impression material. It provides stability to the matrix band and cause swelling of the gingiva adjacent to an retainer assembly. C. Poor mix of the stone. Poor mix of the impression material. 6½ years. C. Barbiturates. Note: Some of the items in the Released Test Item Bank may have been discontinued due to outdated science or errors. Diplopia. E. D. Be treated with topical fluoride. Be restored with composite resin. How C. incorporating air into the material. B. the contact. Formation of saliva droplets on the palate Which class of drugs should first be considered while the impression is setting. B. Which of the following is the most appropriate management for a dry socket? A. The NDEB periodically reviews the bank to improve its quality. Phenothiazines. It assists in the adaptation of the matrix C. Be smoothed with abrasive strips. B. Infection. should the lesions be managed? D. Periodontal abscess. B. endodontically treated tooth? A. proper home care and observation. D. What is the most likely cause of voids in a C. Facial paralysis. maxillary diagnostic cast? D. C. A. Antihistamines. preparation. the format of some items is not currently used. 11½ years. 12 erupted primary teeth and 12 erupted D. cracked tooth syndrome. retention of the definitive restoration.In a restoration following endodontic therapy. B. Orange. B. A new bone grafting material has been E. panoramic radiograph and bitewings. resistance of the tooth to fracture. is unhappy with the natural colour of their B. This potential can best be described as an example of A patient complains of intermittent spontaneous pain in a tooth that was previously A. smooth surface carious lesion is D. M. For a complete denture patient. White. is A. D. P. has existing all-ceramic crowns that are D. osteoconduction. C. The most appropriate management of a C. noncavitated. B. S. applying topical fluoride. The most likely diagnosis D. sealing of the root canal. has teeth discoloured by tetracycline. distribution of forces along the long axis. full mouth periapicals. Dentist supervised at-home bleaching the primary function of the post is to improve technique is NOT indicated for a patient who A. B. In addition. ©The National Dental Examining Board of Canada 2017 . Both hot and cold C. full mouth periapicals and bitewings. the format of some items is not currently used. osteogenesis. Which porcelain stain colour is added to give E. osteoinduction. J. panoramic radiograph. produce severe pain. E. A. for a new patient with an extensively restored B. prescribing a chlorhexidine rinse. has a single dark coloured tooth. (a) D. Yellow. content. D. reversible pulpitis. E. B. The NDEB periodically reviews the bank to improve its quality. C. D. The tooth is not B. osteopenia. which letter or sound is a guide for the position of the incisal edge of the maxillary incisors? The most appropriate radiographic examination A. Grey. and translation. lighter than the natural teeth. C. F. Blue. placing an amalgam restoration. A. demonstrated to generate bone tissue through the recruitment of progenitor cells. symptomatic irreversible pulpitis. has extrinsic stains. dentition and generalized periodontal disease is C. C. symptomatic apical periodontitis (acute periradicular periodontitis). placing a composite restoration. observation. the appearance of translucency? A. E. A. teeth. Note: Some of the items in the Released Test Item Bank may have been discontinued due to outdated science or errors. C. cone beam computed tomography. sensitive to percussion. treated with a direct pulp cap. D. An occlusal rest should be prepared so that Opioids would be CONTRAINDICATED for the management of A. composite resins. B. gingival recession. content. D. The completed. crown retention. compatible with the expansion of B. frequent decementation of the temporary C. Antidepressants. C. A. after all restorative procedures are seconds. ©The National Dental Examining Board of Canada 2017 . diarrhea. C. crown. the tooth is prevented from rotating. glass ionomer cements can be used as a base because they are Which of the following drug groups is LEAST A. B. C. pain. administer vitamin K after the extraction. B. discontinue anticoagulation drugs one week before extraction. increasing the post preparation diameter The problem most likely to result from a will most likely increase temporary crown with inadequate proximal contacts is A. C. Antibiotics. long axis of the tooth. and translation. C. D. the format of some items is not currently used. B. D. biocompatible. D. cervical resistance. D. In composite resin restorations. most appropriate management is to B. Note: Some of the items in the Released Test Item Bank may have been discontinued due to outdated science or errors. The normal value is 15 seconds. after each restorative procedure. A. D. difficulty in seating the permanent crown. The NDEB periodically reviews the bank to improve its quality. C. sedative to a hyperemic pulp. occlusal forces are directed perpendicular to the long axis of the tooth. In addition. occlusal forces are concentrated near the D. Diuretics. interferences should be corrected A patient on anticoagulant drugs who requires an extraction has a prothrombin time of 20 A. marginal ridge. If a complete occlusal adjustment is necessary. A. extract the tooth and use local measures to control bleeding. C. occlusal forces are directed parallel to the B. In the restoration of an endodontically treated tooth. before starting any restorative treatment. an esthetically compromised restoration. depression. during treatment. stress to the root. Anticholenergics. likely to cause xerostomia? B. A. severe cough. D. neutral in colour. administer vitamin K before the extraction. post stability. E. The tooth is mobile with a large mesio- B. the patient's face products will increase the compressive strength becomes quickly and visibly swollen. Decreasing the mixing time. the treatment of B. The of the set material? immediate treatment should be to A. mandibular canine. C. E. The NDEB periodically reviews the bank to improve its quality. B. When preparing a posterior tooth for an extensive amalgam restoration. administer diphenhydramine hydrochloride (Benadryl) 50mg intravenously. D. Adding a small amount of salt to the water A. Using warmer water. D. intravenously. erythema multiforme. C. A light weight dam was used. What is the most likely cause of the interdental papilla protruding from beneath the rubber dam after placement? The most typical location for the development of early childhood caries is the A. The most likely C. use pressure followed by cold packs over before mixing. D. B. content. necrotizing ulcerative gingivitis. gingival area of the mandibular incisors. a hyperemic pulp. C. an exfoliating tooth. administer 100mg hydrocortisone D. incisal edges of the maxillary incisors. traumatic occlusion. maxillary molar. In addition.Which of the following modifications to the Immediately following a posterior superior standard procedure for mixing gypsum alveolar block injection. A. an acute pulpitis. ptyalism. B. herpes simplex. A. the swelling. E. Decreasing the water/powder ratio by a B. mandibular molar. eating. D. parallel to the contour of the final restoration. use hot packs over the swelling. C. angled 30° away from the pulp chamber. occlusal amalgam restoration. the format of some items is not currently used. Note: Some of the items in the Released Test Item Bank may have been discontinued due to outdated science or errors. D. B. C. incisal edges of the mandibular incisors. ©The National Dental Examining Board of Canada 2017 . and translation. A. C. gingival area of the maxillary incisors. maxillary canine. B. C. The teeth were not individually ligated. submaxillary cellulitis. small amount. refer the patient to a hospital. The holes were placed too close together. perpendicular to the pulpal floor. a retentive pin hole preparation should be placed Systemic or topical cortisone therapy is used in A. diagnosis is D. The most common primary tooth to become ankylosed is a A 10 year old patient complains of discomfort in a maxillary primary second molar when A. D. parallel to the external root contour. a hyperplastic pulp. The holes were placed too far apart. widening of vascular canals. the capillaries of the lesion will cause prolonged bleeding. quadrupled. Erythromycin. pulp capping. E. B. ©The National Dental Examining Board of Canada 2017 . the preferred treatment for asymptomatic teeth with very deep dentinal caries is Space closure is LEAST likely to occur A. Note: Some of the items in the Released Test Item Bank may have been discontinued due to outdated science or errors. if target-skin distance is doubled. hydroxide. loss of intercanine space. direct pulp capping using calcium C. pus in the lesion will infect the blood clot. D. Clindamycin. the lesion may undergo cystic degeneration.The extraction of a maxillary primary central A circumscribed radiolucent lesion associated incisor at the age of 6 years will cause with a retained root tip should be curetted because A. B. tripled. D. Azithromycin. content. mandibular canine. In addition. no change in intercanine space. B. B. To maintain pulp health and vitality. Which antibiotic is CONTRAINDICATED for a patient with a history of a Type 1 In radiology. acidic primers. D. to remove all affected dentin. An early radiographic sign of chronic operative sensitivity following the placement of periodontitis is posterior composite resin restorations is A. prevent new bone formation. decreased overjet. "notching" of the crestal lamina dura. the fibrous tissue of the lesion will D. D. enlargement of the interdental medullary C. The NDEB periodically reviews the bank to improve its quality. B. increased 10 times. doubled. D. C. maxillary first molar. the format of some items is not currently used. microleakage. Cephalexin. B. maxillary second molar. B. residual caries. increased intercanine space. increased overbite. C. B. The most common cause of persistent post. D. E. A. C. A. decreased radiolucency of the interdental trabeculae. direct pulp capping using a dentin bonding following the premature loss of the primary agent. A. and translation. C. C. hyperocclusion. D. mandibular second molar. C. spaces. to avoid pulp exposure and use indirect A. anaphylactic reaction to penicillin? the exposure time must be A. maxillary central incisor. The patient’s head size is large with prominent A 3 year old. E. A. Eliminate the collagen fibrils.5mm from the cavosurface margin. Glandular. After a few minutes the child becomes lethargic. infraorbital foramen. osteopetrosis. Reduce the dentinal fluids in the tubules. dentino-enamel junction. Note: Some of the items in the Released Test Item Bank may have been discontinued due to outdated science or errors.5mm from the C.Decreased size or obliteration of pulp chambers What is the most appropriate location for the and canals is diagnostic of placement of a retentive pin? A. impacted permanent teeth. cleidocranial dysplasia. be taken by panoramic radiography only. be taken. pterygopalatine fossa.000 epinephrine for features. C. pterygoid plexus. A 17 year old male patient exhibits delayed eruption of permanent teeth. amelogenesis imperfecta. A. The most probable cause of this C. ©The National Dental Examining Board of Canada 2017 . osteogenesis imperfecta. Gardner's syndrome. The NDEB periodically reviews the bank to improve its quality. B. D. C. D. disorientated and eventually begins to B. Radiographs indicate multiple. Hand-Schuller-Christian disease. What is the goal of dentin conditioning? C. 16kg child is given 2 cartridges of frontal eminences and slightly constricted facial 4% prilocaine with 1:200. B. B. be taken only if difficulty is encountered during surgery. At the dentino-enamel junction. extraction. A. cleidocranial dysplasia. overdose of local anesthetic. To obtain block anesthesia of the second B. In dentin a minimum 0. allergic reaction to the local anesthetic. C. D. content. B. At least 2. D. A patient who is 4 months pregnant requires an D. A. acromegaly. D. and translation. A radiograph may E. Which of the following tissues is the LEAST sensitive to the effects of x-radiation? A.5mm from the dentino-enamel junction. epilepsy precipitated by epinephrine. B. A. foramen spinosum. reaction is D. In addition. Bone. not be taken. epinephrine. Expose the hydroxyapatite. osteitis deformans. C. Gonadal. the format of some items is not currently used. In enamel a minimum 0. the solution C. Remove the smear layer. Hematopoietic. B. A. must be deposited in proximity to the D. convulse. The most likely diagnosis is extraction of primary teeth. foramen ovale. division of the trigeminal nerve. the curve of Spee. B. involved by the carious process. type 1 allergic reaction. The extraction of a primary maxillary central E. content. increased intercanine space. hematoma. lateral spread of caries along the dentino. A. C. the curve of Monson. a patient suddenly develops a mounted on an articulator. apply firm pressure for ten minutes. When removing bone or sectioning roots of D. the format of some items is not currently used. tapered root and a long C. C. The most appropriate initial management is to A. B.6. Tooth rotated and tipped out of line. and translation. administer local anesthetic with On a semi-adjustable articulator. air and water on. need to terminate the margins on a cusp ridge or marginal ridge crest. neither air nor water. D. establish the occlusal plane. loss of intercanine space. extent to which the enamel has been A. horizontal guidance. The most probable diagnosis is a/an A. extension that must be made along the clinical crown. C. A. Four hours later. Tooth with minimal coronal structure. determine the vertical dimension of occlusion. The outline form for a Class I amalgam Which of the following teeth is the LEAST preparation is determined by all of the desirable to use as an abutment tooth for a following EXCEPT the fixed partial denture? A. Tooth with short. horizontal and vertical overlap. B. obtain an INR and complete blood count. Tooth with pulpal involvement. C. Following a simple extraction of tooth 4. B. enamel junction. apply dry socket dressing. D. a centric relation swelling of the left face and neck with record is used to crepitation. In addition. teeth with a high-speed handpiece. extent of undermining of the enamel by incisor at the age of 6 years will cause the carious process. air only.7. guidance is the mechanical analogue of B. there is renewed bleeding from the extraction D. water only. the incisal epinephrine and suture. the air/water combination should be set with A. hemostasis was achieved. The NDEB periodically reviews the bank to improve its quality. C. record the inclination of the condylar D.During preparation of a subgingival Class V on Assuming a maxillary cast is accurately tooth 3. B. C. guidance. C. D. cervicofacial emphysema. no change in intercanine space. B. D. B. A. ©The National Dental Examining Board of Canada 2017 . D. fissures in order to achieve sound and smooth margins. site. Note: Some of the items in the Released Test Item Bank may have been discontinued due to outdated science or errors. decreased overjet. mount the mandibular cast. anaphylactoid reaction. Maxillary incisors. taken by a patient who has been prescribed B. Which of the following teeth are most often lost as a result of periodontitis? Anginal pain is A. Mandibular incisors. D. treat the patient in the office as a normal A. C. C. Mesiolingual rotation of permanent first maxillary molars A healthy 66 year old patient who had a myocardial infarct eight years ago requires an A. Mandibular molars. The NDEB periodically reviews the bank to improve its quality. D. Oxycodone. D. ©The National Dental Examining Board of Canada 2017 . aggravated by deep inspiration. position of the anterior teeth. For complete dentures. Acetaminophen. D. Ibuprofen. C. patient. Tetracycline. B. Flurbiprofen. palatal thickness. A. B. lithium? C. D. with local anesthesia. extraction. C. The most appropriate management B. admit the patient to hospital for extraction molars. Root tip elevator. reproduction of the palatal rugae. Cow horn forceps. increases arch length. placing an initial increment of flowable resin. D. relieved by digoxin. causes impaction of the second permanent A. C. Maxillary universal forceps. Metronidazole. results is an Angle Class II molar is to relationship. admit the patient to hospital for extraction with general anesthesia. D. placing a glass ionomer liner. Rongeur. position of the molar teeth. A. B. C. beveling the gingival margin. In addition. C. B. A. extract the tooth in the office using Which of the following drugs is most likely to preoperative sedation and local anesthetic induce photosensitivity? without a vasoconstrictor. B. D. the most important factor affecting speech is the Which of the following medications can be A. the risk of marginal leakage is reduced by A. the format of some items is not currently used. Maxillary molars. Acetaminophen.Which of the following instruments is When the gingival margin of a Class II CONTRAINDICATED when extracting composite resin preparation is located less than primary teeth? 1mm occlusal to the cemento-enamel junction. B. C. Note: Some of the items in the Released Test Item Bank may have been discontinued due to outdated science or errors. provoked by exercise or cold weather. incremental curing. not relieved by rest. and translation. B. content. Naproxen. be 2mm into the dentin. agent. Increased number of open dentinal C. intercuspal distance. D. Note: Some of the items in the Released Test Item Bank may have been discontinued due to outdated science or errors. holes punched too far apart. B. The pulpal floor of an occlusal amalgam A preparation for a Class II amalgam preparation on a mandibular first premolar restoration in primary molars should include should A. A. bonded silver amalgam. cast gold inlay. ©The National Dental Examining Board of Canada 2017 . The genial tubercles are best visualized on which type of radiograph? A. be perpendicular to the long axis of the C. content. an axial wall that parallels the dentino- C. Increased radiographic density is caused by The most radiosensitive salivary gland is the A. cast restoration with cuspal coverage. D. The most appropriate treatment for an An immediate toxic reaction to a local endodontically treated molar with an existing anesthetic administration is caused by a/an MOD restoration is a A. D. an isthmus that occupies two thirds of the tooth. B. D. slope apically from mesial to distal. B. the format of some items is not currently used. C. Bacterial contamination. tubules. Panoramic. C. walls. parotid. too much tension on the rubber dam C. D. A. B. B. D. too broad a rubber dam arch form. B. be parallel to the buccolingual cusp plane. In addition. decreased target-object distance. sublingual. hypersensitivity to the anesthetic agent. B. undercut enamel and dentin. B. decreased kVp. C. enamel junction. Loss of dentin bulk. deterioration of the anesthetic agent.A rubber dam in which there is leakage What is the most likely cause of sensitivity interproximally most likely has related to a noncarious cervical lesion? A. Hyperocclusion. submandibular. Occlusal. C. holder. decreased mA. hypersensitivity to the vasoconstrictor. bonded composite resin. and translation. excessive blood level of the anesthetic D. The NDEB periodically reviews the bank to improve its quality. B. occlusally divergent buccal and lingual A. increased object-film distance. D. C. Periapical. A. Bitewing. holes punched too close together. Amalgam. C. moderately mobile. Note: Some of the items in the Released Test Item Bank may have been discontinued due to outdated science or errors. a coated tongue. C. and translation. A. Which of the following conditions should NOT commonly be treated during the mixed dentition stage? During cardiopulmonary resuscitation (CPR). A. ankylosed. Gold castings. D.20cm.5cm. patient’s anxiety is to administer E. Gingival recession. 10 . D. B.10cm. uremia and liver disease is An apprehensive 77 year old patient. excessive force? C. Areas of isolated gingival recession are most frequently seen on teeth that are Which of the following outcomes may be the A. approximately 60kg. external chest compression of an adult patient A. C. a smooth tongue. A clinical finding common to alcoholism. C. should be B. Bleeding on probing. B. the format of some items is not currently used. Posterior crossbite. intravenously. The most appropriate management is to A. several mandibular teeth under local anesthesia. content. B. Underestimation of attachment loss. ©The National Dental Examining Board of Canada 2017 .Radiographic examination of a 9 year old The level of streptococcus mutans has been patient reveals the pre-eruptive position of a shown to be significantly higher in the bacterial mandibular second premolar is tipped 20 plaque adjacent to which type of posterior degrees from vertical. Angular bony defect. A. 1 . D. recall the patient for reassessment in 6 months. B. A. 100mg of secobarbital. Glass-ionomer. The most appropriate method to manage this D. C. labial fissures. Class II molar relationship. requires the removal of B. B. C. surgically upright the premolar. The NDEB periodically reviews the bank to improve its quality. increased blood pressure. labially prominent in the arch. nitrous oxide and oxygen. Maxillary incisor rotation. 7 . halitosis. result of probing a healthy periodontium with B. D. nonvital. In addition. remove the primary second molar and D. orally. 20mg of diazapam. The primary second restoration? molar exhibits no root resorption. 4 . poorly-controlled diabetes mellitus.2cm. D. place a space maintainer. 20mg of diazapam. C. weighing A. Anterior crossbite. orally. Composite resin. C. with B. Risk of osteomyelitis. Acetaminophen 300mg with A. What is the most likely B. permit use of the long cone technique. reduce exposure time. Langerhans cell histiocytosis. Coxibs are more appropriate than standard NSAIDs for patients with a history of Solitary eosinophilic granuloma is associated A. exposure. Codeine 60mg. E. B. fatigue. the format of some items is not currently used. C. C. permit the use of lower kilovoltage during codeine 30mg. Vitamin B deficiency. radiotherapy. B. Risk of trismus. D. atrophic glossitis. D. D. renal dysfunction. root canal therapy. An oral exam shows angular cheilitis and A. hemi-mandibulectomy. C. A. hypoparathyroidism. Risk of spread of infection. filter out useless short rays. content. D. reduce the diameter of the primary beam. D. Chronic lymphocytic leukemia. Which of the following is the most potent analgesic? The main purpose of collimation of an x-ray A. Difficulty achieving anesthesia.The most appropriate treatment for a unicystic A 27 year old woman complains of burning ameloblastoma is mouth. curettage. E. B. diagnosis? C. D. C. Ibuprofen 600mg. hyperparathyroidism. cardiac problems. block resection. The NDEB periodically reviews the bank to improve its quality. beam is to B. D. C. In addition. ©The National Dental Examining Board of Canada 2017 . palpitations and lack of energy. A. Note: Some of the items in the Released Test Item Bank may have been discontinued due to outdated science or errors. What is the most appropriate reason to delay the extraction of a mandibular third molar with acute pericoronitis? A. Crohn’s disease. Iron deficiency anemia. Acetylsalicylic acid 650mg. asthma. multiple myeloma. B. and translation. gastrointestinal bleeding. B. benign mucous membrane pemphigoid. D. Overadjustment of a wrought wire denture C. lichen planus. sweat glands. In addition. implanted epithelium. hyperkeratosis. diagnosis of A. decrease in ductility due to strain hardening. C. cystic formations. aphthous ulceration. following is most appropriate to confirm a diagnosis? A. restoration to seal the access. Crown/root ratio. sebaceous glands. and translation. Placing a bonded composite resin incisive canal cyst. Direct visual fluorescent examination. B. C. Which of the following is the most important C. Exfoliative cytology can be of value in the D. All cusps A. The between temporary restoration was lost two months ago. Removal of 4-5mm of coronal gutta. A. content. erythema multiforme. Endodontic retreatment followed by a core build-up. abscess. ©The National Dental Examining Board of Canada 2017 . periapical osseous dysplasia (periapical cemento-osseous dysplasia) and an A. to placing a full gold crown? B. Exfoliative cytology. B.A nonsurgical endodontic treatment was Pulp vitality tests are used to differentiate completed on tooth 1. E. periapical osseous dysplasia (periapical are present but are weak and unsupported. Which of the seal. The NDEB periodically reviews the bank to improve its quality.6 six months ago. Culture. followed by a core build-up. Post placement in the palatal canal cyst. percha from all canals followed by a core build-up. B. D. A. periapical granuloma and a periapical B. increase in fracture toughness. Note: Some of the items in the Released Test Item Bank may have been discontinued due to outdated science or errors. decrease in the yield strength. determinant for the maximum length of a post D. increase in modulus of elasticity. herpes simplex. Fordyce spots or granules is/are A. C. clasp can lead to fracture because of a/an D. the format of some items is not currently used. adherent. Biopsy. Distance between the crestal bone and root apex. The tooth has no caries or fractures. white C. in an endodontically treated tooth? E. cemento-osseous dysplasia) and a What is the most appropriate management prior periapical granuloma. D. A patient has an asymptomatic. B. periapical granuloma and a periapical C. Maintenance of an apical gutta-percha patch on the oral mucosa. content. Hutchinson’s teeth. the format of some items is not currently used. E. incisional biopsy. mucous patches. Analgesic and hydration therapy. D. A. C. B. Gingival enlargement observed in acute leukemia is due to Most cases of erosive oral lichen planus are effectively treated with A. hypertelorism. B. Antibiotic therapy. C. antimalarials. E. herpetic gingivostomatitis? B. C. D. tissue infiltration by neoplastic cells. In addition. The most likely diagnosis is experiencing an acute asthmatic attack? A. Note: Some of the items in the Released Test Item Bank may have been discontinued due to outdated science or errors. Topical antifungal therapy. perform a cytological smear. recall the patient in six months. C. A. B. curettage. Salbutamol. contact the pathologist to discuss the case. malignancy. lichen planus. D. white sponge nevus. Down syndrome is associated with Which of the following is the most appropriate management for a child with acute primary A. antifungals. observation. reactive fibrosis. The most appropriate management is to A. C. B. papular rash. capillary fragility. D. B. A. cytologic smear. B. Topical corticosteroid therapy. macroglossia. D. reassure the patient that there is no C. C. ©The National Dental Examining Board of Canada 2017 . edema. A. Isoproterenol. A 14 year old boy presents with bilateral white thickening of the buccal mucosa which has Which of the following drugs is most been present since birth. leukoplakia. Hydrocortisone. Epinephrine. and translation. The NDEB periodically reviews the bank to improve its quality. excisional biopsy. B. C.An incisional biopsy of a clinically suspicious The most appropriate management for a malignant lesion fails to support the clinical gingival cyst of the newborn is diagnosis. D. D. D. antibacterials. His brother has similar appropriate first line management for a patient lesions. corticosteroids. mandibular prognatism. E. The most likely diagnosis of a 1cm mobile Which of the following is caused by a mass in the parotid is microorganism? A. Replacement of amalgam restorations. Pyogenic granuloma. D. B. Note: Some of the items in the Released Test Item Bank may have been discontinued due to outdated science or errors. Carbon dioxide laser ablation. ectodermal dysplasia. C. Dentigerous cyst. B. A. Observation. Geographic tongue. Simple bone cyst/traumatic bone cyst. Change in diet. Gorlin-Goltz’s syndrome. acinic cell carcinoma. A. C. mucocele. D. mandible. In addition. hyperplasia of the duct. Biopsy. The NDEB periodically reviews the bank to improve its quality. ©The National Dental Examining Board of Canada 2017 . Which of the following is NOT a true cyst? A mucocele results from A. 20 years. E. Periapical osseous dysplasia (periapical E. C. pleomorphic adenoma. characterized by dysplastic changes. content. D. Calcifying epithelial odontogenic tumor. C. D. Lichen planus. D. and translation. maxilla. damage to the duct. Central cementifying fibroma. mucoepidermoid carcinoma. B. 40 years. D. aplasia of the duct. C. cemento-osseous dysplasia). Warthin’s tumour. C. D. A. 20 years. B. Median rhomboid glossitis.Nicotinic stomatitis is The most common site and patient age for a solid (conventional) ameloblastoma are the A. C. B. irreversible. the format of some items is not currently used. D. C. mandible. B. Gardner’s syndrome. B. pleomorphic adenoma. A. cleidocranial dysplasia. characterized by small red dots on an B. maxilla. A. Ameloblastoma. associated with squamous cell carcinoma posterior of the palate. Multiple congenitally missing teeth may be The most common benign tumour of the characteristic of salivary glands is a/an A. hypersecretion. elevated pale mucosa. Warthin’s tumour. D. C. adenoid cystic carcinoma. caused by smokeless tobacco. B. Radicular cyst. C. Nasopalatine cyst. D. 40 years. Which of the following lesions is most What is the most appropriate management for commonly found in the anterior region of the erythroleukoplakia? mandible? A. B. canalicular adenoma. Verrucous carcinoma. D. Which of the following lesions is most Which of the following is the most common commonly found in the anterior region of the site in the oral cavity for a squamous cell mandible? carcinoma? A. C. resembling the enamel organ. Which of the following lesions has the LEAST C. B. A. the format of some items is not currently used. B. Which of the following is NOT associated with D. The NDEB periodically reviews the bank to improve its quality. Pain. ©The National Dental Examining Board of Canada 2017 . gingiva. smoking. In addition. Ameloblastoma. Inflammation. Pseudomembrane. neurofibroma. D. Vesicle. posterior maxilla. tetracycline therapy. an aphthous ulcer? A. probable diagnosis is a/an C. a post-extraction infection. trauma. anterior mandible. pulpal necrosis. Note: Some of the items in the Released Test Item Bank may have been discontinued due to outdated science or errors. cemento-osseous dysplasia). The most B. ameloblastoma. and translation. Palate. A. A. Basal cell carcinoma. Mottled enamel is associated with E. C. D. B. B. dentigerous cyst. B. Buccal mucosa. C. D. content. Central cementifying fibroma. B. periodontal inflammation. C. regional odontodysplasia. Gingiva. C. periapical osseous dysplasia (periapical D. resolves within a few weeks after quitting E. most commonly affects the anterior C. hard palate. fluorosis. A. C. D. D. posterior mandible. B. central ossifying fibroma. lateral periodontal cyst. A. A dentoalveolar abscess most frequently Tissue from a multilocular radiolucent area of originates from the posterior mandible histologically shows follicular areas lined with columnar cells A. B. Floor of the mouth. Melanoma. favourable prognosis? D. Calcifying epithelial odontogenic tumour. anterior maxilla. is painful.The most common site for breast carcinoma to Smoker's melanosis metastasize to the maxillofacial regions is the A. amelogenesis imperfecta. may be pre-cancerous. Ameloblastoma. malignant tumour. The most common malignant tumour of the B. D. D. does not normally metastasize. C. C. B. and translation. Melanoma. Healing of a recurrent herpes simplex lesion Difficulty in mouth opening. dysphagia. Adenocarcinoma. B. A. the format of some items is not currently used. B. Note: Some of the items in the Released Test Item Bank may have been discontinued due to outdated science or errors. granular cell myoblastoma. radiolucency involving the apices of the C. D. normally metastasizes to the submaxillary tongue is a/an lymph nodes. Vesicles. squamous cell carcinoma. tongue occurs within stiffness and generalized induration of the skin are characteristic of A. lymph nodes. 7-14 days without scar formation. radiolucency most frequently seen B. Ameloblastoma. normally metastasizes to the pre-auricular C. E. mandibular incisors.A cementoblastoma is typically a solitary What type of lesions are seen in primary circumscribed herpetic gingivostomatitis. posteriorly to the maxillary molars. lymph nodes. molar. normally metastasizes to the cervical A. 7-14 days with scar formation. The NDEB periodically reviews the bank to improve its quality. Which of the following tumours has the best prognosis? A simple bone cyst/traumatic bone cyst is a A. C. adenocarcinoma. In addition. Pustules. in the mandibular ramus. erythema multiforme. fibrosarcoma. radiopacity with a radiolucent rim involving the apex of a mandibular incisor. in the maxillary premolar area. herpes simplex. radiopacity with a radiolucent rim involving the apices of a mandibular A. Macules. D. B. from the symphysis to the ramus of the mandible. Basal cell carcinoma on the nose A. Osteosarcoma. E. A. D. herpes zoster and varicella? A. lupus erythematosus. 2-4 weeks without scar formation. C. C. content. scleroderma (systemic sclerosis). D. D. normally metastasizes to the submental lymph nodes. ©The National Dental Examining Board of Canada 2017 . B. Papules. lichen planus. 2-4 weeks with scar formation. C. D. B. radiolucency in an edentulous area. Naproxen. non- restorable primary first molar is to What is the most appropriate medication to manage postoperative pain for an asthmatic A.000. with epinephrine 1:200. Patients with primary herpetic D. A. caused by a different organism than is requiring the surgical extraction of tooth 3. For a 4 year old child. A lingual plate is indicated as a major connector for a removable partial denture when A.000. Which of the following drugs is LEAST likely B. prednisone. C. D. with epinephrine 1:100. Ibuprofen. with epinephrine 1:200.5%. prescribe antibiotics. B. A. Acetylsalicylic acid/codeine combination.Recurrent herpes labialis is Which local anesthetic should be used to achieve the longest pain relief for a patient A. Lidocaine 2%. benzocaine. acyclovir. For a 1 year old child with primary herpetic E. C. A. B. swab the lesions with chlorhexidine. Note: Some of the items in the Released Test Item Bank may have been discontinued due to outdated science or errors. A. maintain adequate fluid intake. observe it until it becomes symptomatic. extract it and place a space maintainer. Bupivacaine 0. Prilocaine 4%. E. The NDEB periodically reviews the bank to improve its quality. acetaminophen. a palatal torus is present. a contagious lesion. B. D. B. Phenytoin. gingivostomatitis should NOT receive E. D. a form of disease which heals by scarring. C. observe it until it exfoliates. immunize against chicken pox. a retromylohyoid undercut is present. with epinephrine C. Cyclosporine. D. Nifedipine. In addition. patient taking beclomethasone and salbutamol? B. Acetaminophen/codeine combination. chlorhexidine. occurs more frequently in children. to cause gingival hyperplasia? C. the most appropriate management is to A. Mepivacaine 3% plain. Fluoxetine. content. gingivostomatitis. 1:100. B. the most appropriate management for a chronically infected.000. D. D. the format of some items is not currently used. Acetylsalicylic acid. there is a low attachment of the lingual frenum. E. D.8? primary herpetic gingivostomatitis. and translation. extract it only. insufficient room exists for a lingual bar. ©The National Dental Examining Board of Canada 2017 . C. B. debride the lesions. C. C. Articaine 4%.000. Which of the following is the most B. B. Mesognathic. unilateral. Inflammatory. Have him eat a chocolate bar. D. skeletal. mandibular canines. and translation. appointment. C. usual. A 50 year old obese patient was diagnosed with type 2 diabetes last year and has recently The term "subdivision" in malocclusion started taking an oral hypoglycemic.m. increase with the eruption of the permanent A. D. dental and neuromuscular. dental. B. D. An extreme overjet. Have him drink 175ml of diet cola. before the maxillary first premolar. Replacement.The etiology of an Angle Class II malocclusion The permanent maxillary canine most in a 12 year old with an SNA = 82 and an commonly erupts SNB = 75 is most likely A. Surface. E. B. when the enamel organ is complete. content. C. B. a recessive chin and a deep Between the ages of 6 and 12. D. Note: Some of the items in the Released Test Item Bank may have been discontinued due to outdated science or errors. premolars. increase with eruption of the mandibular C. molar. following eruption. During his 8:30 a. D. Ankylosis. appropriate management? C. He classification refers to molar relationships that frequently skips meals in order to reduce his are weight. at eruption. A. present radiographically with shortening or blunting of the apical tip of a root. severely abnormal. before the maxillary second permanent C. In addition. prior to eruption but after root formation C. Give him 15g of glucose as tablets or in a solution. Dismiss the patient and advise him to eat. before the mandibular permanent canine. D. decrease with the eruption of the A tooth with a nonvital pulp may occasionally mandibular premolars. B. mandibular arch labial mento-labial sulcus are common findings length will normally in which facial type? A. C. Retrognathic. decrease with the eruption of the permanent mandibular incisors. A. The loss of apical cementum and dentin would be classified as what type of resorption? Enamel maturation is completed A. his speech becomes slurred and he is less alert than A. molar. after the maxillary second permanent E. ©The National Dental Examining Board of Canada 2017 . has begun. A. The NDEB periodically reviews the bank to improve its quality. D. C. skeletal and dental. B. bilateral. B. D. remain the same. Prognathic. neuromuscular. Crossbite. mildly abnormal. the format of some items is not currently used. maxillary second premolar. most likely canine. A. severity of the Angle Class II C. amount of overbite. If the B. Angle Class III. maxillary first premolar. Ankylosis of the primary maxillary central incisors is commonly related to The area most susceptible to excessive space loss following premature loss of a primary A. D. inclination of maxillary incisors. division 2 malocclusion. an Angle Class II. over time. C. Which of the following is a possible cause of an anterior crossbite? A 4 year old with a thumbsucking habit A. B. tooth is in the region of the B. tooth size . and translation. presents with a 1. a bimaxillary protrusion.An Angle Class II. A. the anterior C. D. decrease. trauma. amount of crowding present. B. B. B. D. ©The National Dental Examining Board of Canada 2017 . The NDEB periodically reviews the bank to improve its quality.jaw size discrepancy. A skeletal Class II malocclusion. D. decrease and the midline will correct. B. An abnormal labial frenum. incisor. remain the same. C. thumbsucking. content. trauma. with division 2 malocclusion based upon the A. The early loss of a primary maxillary open bite will. malocclusion. C. a deep overbite. Angle Class I. Note: Some of the items in the Released Test Item Bank may have been discontinued due to outdated science or errors. the format of some items is not currently used. results in which of the following permanent B. sleeping habits.5cm anterior open bite. Angle Class II. increase. molar relationship. first molar relationships? C. An over-retained primary maxillary A. D. D. C. chronic gingivitis. delayed eruption of the permanent teeth. habit ceases within six months. mandibular second premolar. C. A. E. an anterior open-bite. D. growth discrepancy. mandibular first premolar. division 1 malocclusion can A tongue thrust is most often found in a child be differentiated from an Angle Class II. In addition. A Class III malocclusion is normally associated with A mesial step relationship most commonly A. Midline diastema. result in inadequate arch length. Note: Some of the items in the Released Test Item Bank may have been discontinued due to outdated science or errors. just after the end of the prepubertal C. D. removable space maintainer. Angle Class II division 1. C. content. A. the distance between the maxillary and B. the optimum time for permanent teeth. ©The National Dental Examining Board of Canada 2017 . treatment of an anterior crossbite of dental C. mesiobuccal cusp rotating lingually. bonded resin bridge (Maryland bridge). and molars and the permanent cuspids and bicuspids. Which of the following malocclusions is most appropriately corrected immediately upon diagnosis? When the primary maxillary second molar is A. A. D. following the completion of growth. conventional fixed partial denture. prepubertal growth spurt. contact to maximum intercuspation. will most likely A. lost prematurally. be present in the permanent dentition. In addition. C. B. Cross-bite with a lateral functional shift. crown tipping buccally. B. A. B. there is a functional shift from initial mandible is in rest position. following completion of jaw growth. mandibular occlusal surfaces when the C.The most appropriate time for surgical The most appropriate management of an treatment of a patient with mandibular avulsed maxillary central incisor in a 10 year prognathism is old is the placement of a A. crown tipping lingually In a child. growth spurt. the difference in size between the D. D. D. and translation. D. C. just before the beginning of the A. B. D. just after eruption of the second permanent molars. A posterior crossbite in the deciduous dentition E. usually moves with the C. single tooth implant. after the maxillary premolars erupt. self-correct with the eruption of the In an 8-year old. B. Angle Class II division 2. the space between primary anterior teeth. D. self-correct with the cessation of an origin is associated habit. a quadhelix is used for treatment. B. after the maxillary canines are in position. the format of some items is not currently used. the space distal to the mandibular primary canine and mesial to the maxillary primary canine. The NDEB periodically reviews the bank to improve its quality. just before eruption of the second permanent molars. the maxillary posterior teeth are centred on the alveolar process. as soon as possible. correction of a bilateral posterior constriction of the maxillary arch has the WORST long term prognosis for stability if The leeway space is A. the permanent first molar B. mesiobuccal cusp rotating buccally. there is a history of prolonged thumb combined widths of the primary cuspids sucking. C. is easiest in the preadolescent patient. Note: Some of the items in the Released Test Item Bank may have been discontinued due to outdated science or errors. maxillary central incisors. direction of growth of the mandible. incisors. D. angulation of the teeth in the permanent produced from a single turn of a palatal dentition. manipulation is A. requires surgery in skeletally immature patients. during intra-oral emergence of the Maxillary skeletal arch expansion maxillary canines. D. 1. C. large forces. continuous and light. translation of the maxillary incisors. In addition. C. extrusion of the maxillary posterior teeth. after skeletal growth is complete. 0. Ideal orthodontic tooth movement involves C. D.5mm. intermittent and light. the format of some items is not currently used. frontal resorption. decreases arch space. velocity. after complete eruption of the maxillary A. 0. The most appropriate opportunity for C. subcrestal incisions. during the late primary dentition. prior to eruption of the maxillary lateral D. ©The National Dental Examining Board of Canada 2017 . undermining resorption. A. B. B.Orthodontic closure of a 2mm maxillary The use of a maxillary anterior bite plane may midline diastema is most appropriately result in accomplished A. B. C. Interceptive orthodontic treatment has NO B. The NDEB periodically reviews the bank to improve its quality. and translation. increases overbite. In orthodontics. 0. D. effect on the C.25mm. B. C. content. continuous and heavy. D. Appropriate forces for orthodontic tooth movement are A. orthodontic treatment involving growth D. how much activation is D. during the period of greatest growth A. prior to complete eruption of the B. after complete eruption of the maxillary lateral incisors. C. after the eruption of third molars. A. space available for the eruption of the permanent dentition. E. canines.75mm. no occlusal changes. B. extrusion of the mandibular incisors. intermittent and heavy. B. expansion screw? A. final basal bone width of the mandible.0mm. 85kVp . 65kVp . the format of some items is not currently used. D. the size of the lead diaphragm. Maxillary anterior periapical. tube voltage. B.40mA. D. The maxillary left central incisor has not yet A. the size of the filter.10mA. remove the mesiodens. contrast of the image. density of the image. grey levels. contrast. image definition. D. too high a temperature of the C. In addition. The most appropriate management B. and decreasing object-to-film distance. D. the longest side of the film. 10kVp . allow the mesiodens and the right central B. decreasing focal spot-to-object distance examinations. A. C. density. B. D. A. A. secondary radiation. determine their relative positions. C. B. decreasing focal spot-to-object distance greater than and increasing object-to-film distance.5mA. E. radiographic contrast. 7cm. Mandibular anterior periapical. at the patient's skin surface should NOT be B. Mean x-ray beam energy is a function of Which of the following exposure factors will result in the most penetrating x-rays? A. C. tube current.65mA. allow the mesiodens to erupt before attempting extraction. is to C. and translation. D.A periapical radiograph reveals a mesiodens in Proper collimation of the useful beam for film a 7 year old. In radiography. Intraosseous nutrient canals are most frequently Increasing the kVp results in decreased seen on which of the following radiographs? A. increasing focal spot-to-object distance C. immediately band developer will cause increased the unerupted central incisor and initiate orthodontic therapy. incisor to erupt into the oral cavity to C. resolution. 90kVp . A. and increasing object-to-film distance. C. appeared. increasing focal spot-to-object distance A. Radiographic image distortion can be minimized by Regardless of the focal spot-to-film distance employed for intraoral radiographic A. B. collimation. intensity of the central beam. ©The National Dental Examining Board of Canada 2017 . The NDEB periodically reviews the bank to improve its quality. exposure time.15mA. C. the diameter of the primary beam and decreasing object-to-film distance. The maxillary right central size and focal spot-film distance reduces incisor has erupted only partially. D. content. Maxillary posterior periapical. remove the mesiodens and observe progress carefully. D. energy of the x-ray beam. B. B. Note: Some of the items in the Released Test Item Bank may have been discontinued due to outdated science or errors. 75kVp . Mandibular posterior periapical. the format of some items is not currently used. D. D. A.5-2mm. A. 1. content. Chronic gingivitis. D. A. periapical radiograph using the bisecting B. C. Calcifying odontogenic cyst. absorb scatter radiation. B. calcifying epithelial odontogenic tumour. B. limit the area of surface exposure. periapical radiograph using the paralleling D. D. C. B. Fibrous dysplasia of bone. 3-5mm. Gardner’s. Gorlin. D. ©The National Dental Examining Board of Canada 2017 . Sjögren’s. generalized demineralization. panoramic radiograph. C. C. punched-out radiolucent lesions. C. and translation. multiple radiopaque lesions. bite-wing radiograph. B. B. D. constant pain in his jaws upon awakening? A. Peutz-Jeghers. C. compound odontoma. C. complex odontoma. diffuse “ground glass” appearance. C. Gold inlay. Cortical bone. 0. angle technique. absorb long wave length radiation in the E. Acute pulpitis. Composite restoration. C. primary beam. The NDEB periodically reviews the bank to improve its quality. technique.Crown-root ratio and residual bone support can A radiopaque area within the alveolar process best be seen radiographically in a containing several rudimentary teeth suggests a/an A. Sinusitis. make it possible to use higher kilovoltage for improving image quality. Which syndrome has multiple cysts of the jaws? Which of the following lesions is most A. Enamel. B. What is the most likely diagnosis for a patient who complains of dull. In clinically normal periodontium. radiopaque? B. Which of the following is most radiopaque? A filter is inserted in the path of the x-ray beam to A. B. the distance between the bottom of the gingival sulcus and the alveolar crest is The radiographic change most suggestive of multiple myeloma is A. Complex composite odontoma. adenomatoid odontogenic tumour. A. Note: Some of the items in the Released Test Item Bank may have been discontinued due to outdated science or errors.5-1mm. D. Ameloblastoma. Lamina dura. Bruxism. In addition. assess the periodontal status of a patient. B. B. clinically identical to a pyogenic B. (immunoglobulin A). B. not associated with local irritants. number of bony walls. D. measurement of the clinical attachment probing depths after surgical periodontal loss. increased tooth mobility. decreased collagenase in periodontal tissue. B. B. a benign neoplasm. A. C. C. tongue thrust. periodontitis. in random cycles. content. B. altered microbial flora. increased salivary antibodies B. A. impinging on the biologic width. and translation. causing furcation involvement. System (PSR®) is designed to C. C. When smokers are compared to nonsmokers. D. C. treatment are C. creating an environment conducive to an A. change in consistency of the attached D. Radiographs of a periodontally related osseous defect can be used to confirm the A. The Periodontal Screening and Recording B. A. ©The National Dental Examining Board of Canada 2017 . Loss of attachment in periodontal disease occurs A. C. C. measure loss of attachment for large Pathologic migration of teeth is associated with patient populations. periodontal pathogens. change in color of the attached gingiva. greater.Compared to nonsmokers. presence of bone loss. The NDEB periodically reviews the bank to improve its quality. lip incompetence. In addition. A dental restoration with a marginal discrepancy (void) located 1mm subgingivally can affect gingival health by The earliest clinical sign of gingivitis is A. into the gingival crevice. D. document the progression of periodontitis. A. at a consistent rate. evaluate oral hygiene. causing leaching of restorative materials gingiva. bruxism. location of the epithelial attachment. patients who smoke A "'pregnancy tumor"' is have A. smaller. D. the format of some items is not currently used. bleeding on probing. decreased serum T lymphocytes. the same. D. increased serum IgG antibody response to granuloma. in cycles lasting for about 3 months. Note: Some of the items in the Released Test Item Bank may have been discontinued due to outdated science or errors. C. hyperplasia. D. A. Hypertrophy of the gingiva. A. mandibular premolar region. E. A. dysplasia. The NDEB periodically reviews the bank to improve its quality. D. recurrent periodontitis. Familial aggregation. C. leukoplakia. Presence of Aggregatobacter C. Pocket formation without suppuration. due to chronic infection. C. Pemphigus vulgaris.Which of the following is NOT a feature of A commom feature of the oral mucosal lesions aggressive periodontitis? of lichen planus. and translation. D. C. E. B. In addition. D. B. B. Erosive lichen planus. the format of some items is not currently used. Periapical osseous dysplasia (periapical attachment loss. If mucous glands are seen in the epithelial lining of a dentigerous cyst. B. chronic periodontitis. C. C. D. hyperkeratotic. Alveolar bone resorption. maxillary posterior region. D. Necrotic tissue. C. D. B. actinomycetemcomitans. Cementoblastoma. gingivitis. Mucous membrane pemphigoid (cicatricial pemphigoid). C. neoplasia. Destruction of the periodontal ligament. this is most Which of the following is NOT characteristic appropriately called of periodontitis? A. B. Note: Some of the items in the Released Test Item Bank may have been discontinued due to outdated science or errors. There is no clinical or radiographic evidence of further A. B. An aphthous ulcer. maxillary anterior region. hyperkeratosis and discoid lupus erythematosus is that they are A. Localized and generalized forms. granulomatous proliferations. ©The National Dental Examining Board of Canada 2017 . A. What is the most likely diagnosis of an The most common location for periapical ulcerated gingival lesion whose biopsy report osseus dysplasia (periapical cemento-osseus confirms epithelial basal layer separation from dysplasia) is the the lamina propria? A. metaplasia. D. aggressive periodontitis. Peripheral odontogenic fibroma. A patient successfully treated 8 years ago for moderate chronic periodontitis now presents Which of the following can be associated with with generalized erythematous and edematous an impacted tooth? gingiva with bleeding on probing. anaplasia. associated with drug use. Ameloblastoma. mandibular anterior region. B. The most likely diagnosis is cemento-osseous dysplasia). premalignant. content. Hypertension. Lungs. administer diphenhydramine 50mg. for the administration of local anesthetic for an B. Inferior to the pterygomandibular raphe. radiation therapy. alteration in circulating blood volume. repeat biopsy. the format of some items is not currently used. observation. Superior to the mandibular foramen. a nervous 22 year old male with well controlled A. infraorbital block.0mg. Acute apical abscess of a mandibular What is the correct position of the needle tip central incisor. Beads of sweat have C. Hypovolemia. ©The National Dental Examining Board of Canada 2017 .8ml of associated with multiple spider angiomas? 2% lidocaine with 1:100.5cm white patch after C. E. Chronic apical abscess of a mandibular third molar. administer glucagon 1. Infected dentigerous cyst. and translation. must be extracted. patient is to A.3. In addition. incisional biopsy is severe epithelial dysplasia. E. The initial management of this E.000 epinephrine. C. Sepsis. injection. He D. D. middle superior alveolar block. Which of the following is common to all forms B. B. Anterior to the buccinator muscle. E. Liver. Kidneys. inferior alveolar nerve block? C. Mid facial cellulitis. C. Note: Some of the items in the Released Test Item Bank may have been discontinued due to outdated science or errors. Pancreas. D. call 911 and begin CPR. B. A. A patient has an acute apical abscess (acute B. is quite pale. D. Medial to the medial pterygoid muscle. Which of the following clinical conditions is the most serious? A. Lateral to the ramus of the mandible. content.5mg. The diagnosis of a 1. the most appropriate local anesthetic technique is a/an A. dizzy and weak. Hemorrhagic shock is a circulatory disturbance characterized by A. The tooth C. Gallbladder. elevation of temperature. The NDEB periodically reviews the bank to improve its quality. B. A.Which of the following organs is typically Following the injection of 1. decrease in amount of interstitial fluid. accumulated on his forehead and upper lip. In addition to a palatine D. D. D. elevate the patient's legs and administer The most appropriate management is 100% oxygen. increase in blood pressure. C. periradicular abscess) on tooth 1. Vasoconstriction. D. A. buccal infiltration. B. Impaired tissue perfusion. of hemorrhagic shock? C. complete excision. insulin dependent diabetes states that he feels B. administer epinephrine 0. intraligamentary injection. Following a simple extraction of tooth 4. C. acute apical abscess (acute periradicular angles. The NDEB periodically reviews the bank to improve its quality. B. C. A. A. at the gingival crest.5 is periradicular abscess). abscess). about lmm below the gingival crest. B. Forty-eight hours later. B. surfaces. axiopulpal. C. give local anesthetic. D. Marginal leakage of a composite resin After completion of root canal therapy on a restoration will maxillary first premolar with moderate mesial and distal lesions and intact buccal and lingual A. decrease with longevity. B. A line angle NOT present on a Class I cavity D. asymptomatic apical periodontitis (chronic periradicular periodontitis).7. Note: Some of the items in the Released Test Item Bank may have been discontinued due to outdated science or errors. Diphenhydramine. A. buccopulpal. linguopulpal. C. chronic apical abcess (chronic preparation on tooth 1. D.Which drug is indicated as part of the Nickel-chromium alloys designed for porcelain management for a codeine overdose? bonded to metal crowns should be used with caution because A. obtain an international normalized ratio (INR) and a complete blood count. MOD bonded amalgam. A. D. above the gingival crest. Naloxone. MOD onlay. C. C. MOD composite resin. B. apply firm pressure and ice for 10 minutes. not be detectable. ©The National Dental Examining Board of Canada 2017 . The principal internal retention for a Class V amalgam cavity preparation is established at the The diagnosis for a patient having a sinus tract A. C. MOD pin retained amalgam. these alloys cannot be soldered. the format of some items is not currently used. there is renewed bleeding from the extraction site. B. Flumazenil. nickel is an allergen. mesiopulpal. C. The most appropriate management margins of crowns are placed is to A. E. mesio-gingival and disto-gingival line A. C. with no symptoms is B. mesio-axial and disto-axial line angles. Epinephrine. B. D. periodontal abscess. Firm pressure fails to achieve The gingival tissues remain healthier when hemostasis. content. give local anesthetic and electrocauterize the socket. pack and suture. the modulus of elasticity is low. be minimized by use of a bonding agent. In addition. and translation. Atropine. occluso-axial and gingivo-axial line associated with a nonvital pulp who presents angles. hemostasis was achieved. the restoration of choice is a/an B. an odontogenic infection. B. D. C. D. root perforation. anterior maxilla. aan C. C. B. anterior mandible. reduced enamel epithelium. Bell's palsy. Composite resin. Pulpectomy. C. B. Sjögren’s syndrome. sialolithiasis. iron deficiency. The NDEB periodically reviews the bank to improve its quality. folic acid deficiency. apical abscess (periradicular). D. Retromolar pad. apical abscess. pulp testing? B. B. B. the format of some items is not currently used. vitamin B12 deficiency. Chelating agents are useful to treat a tooth with B. E. C. A. C. D. Unilateral numbness of the chin is associated What is the most appropriate initial with management of a tooth which is sensitive to percussion but responds normally to electric A. E. C. A. ©The National Dental Examining Board of Canada 2017 . ranula. malignancy. D. posterior maxilla. Note: Some of the items in the Released Test Item Bank may have been discontinued due to outdated science or errors. D. Type IV alloy. The smear layer present on the root canal wall Acute or subacute suppurative osteomyelitis after cleaning and shaping is best removed by occurs most frequently in the using A. B. In addition. A. EDTA. Occlusal assessment. aminopyrine therapy. C. content. D. C. D. sarcoidosis. A. Mylohyoid muscle. epithelial rests of Malassez. posterior mandible. Enamel. D. sclerotic canal. and translation. Pulpotomy. Restoration replacement. A. Buccinator muscle. Hypochromic anemia is associated with A. trigeminal neuralgia. odontoblastic layer. The epithelium covering the lesions of hyperplastic pulps is believed to be derived Recurrent unilateral submandibular swelling from the and pain just prior to meals is indicative of A. C. B. remnants of the dental lamina. epithelium of the gingiva. hydrogen peroxide. chlorhexidine. Amalgam. D. curved canal. isopropyl alcohol.Which of the following is the hardest? What structure is most likely to dislodge a complete mandibular denture? A. Pterygomandibular raphe. B. usually occurs after the age of 20. A. malaise or is lymphadenopathy is present. acetylsalicylic acid is B. In addition. C. central ossifying fibroma. C. D. Clinical examination of a 42 year old heavy smoker reveals a white patch in the A 29 year old patient developed multiple retromolar/tonsillar pillar region. has a slow onset. coagulation necrosis. A. B. aphthous stomatitis. chickenpox. aphthous ulcers. B. D. The patch ulcerated lesions on the buccal attached gingiva cannot be wiped off. The most likely diagnosis one day ago. lichen planus. B. Retropharyngeal lymph nodes. B. Mediastinal lymph nodes. B. The most lymph nodes? likely diagnosis is a/an A. hyperparakeratosis. primary herpetic gingivostomatitis. scarlet fever. the format of some items is not currently used. Bilateral neck lymph nodes. recurrent herpes simplex infection. content. D. periradicular cyst. No fever. mumps. C. necrotizing ulcerative gingivitis. lasts for 1 . Heavy cigarette smoking significantly increases Koplik's spots are seen in the oral cavity of the incidence of patients with A. C. D. Unilateral neck lymph nodes. D. ©The National Dental Examining Board of Canada 2017 . leukoplakia. Note: Some of the items in the Released Test Item Bank may have been discontinued due to outdated science or errors. atrophic glossitis. C. acanthosis.Primary herpes simplex virus infection The white appearance of the oral mucosa following extended local application of A. smallpox. mucosal pigmentation. B. C. A. pseudomembranous candidiasis. and translation. B. The most likely diagnosis is A. affects only the gingiva.2 days. D. C. E. white sponge nevus. The NDEB periodically reviews the bank to improve its quality. A squamous cell carcinoma located near the Histological sections of a lesion removed from midline of the anterior floor of the mouth the apex of a carious tooth show fibrous tissue normally spreads to which of the following and chronic inflammatory cells. E. E. E. C. edema. D. lichen planus. periradicular granuloma. D. odontogenic fibroma. A. may be subclinical. geographic tongue. measles. frictional hyperkeratosis. mandible. less hard and more colour stable. tooth 1. D. Mesial lingual cusp tip. topical fluoride application. Note: Some of the items in the Released Test Item Bank may have been discontinued due to outdated science or errors. D. so that it is not in supraocclusion. a well-defined line that maintains a distinct C. Existing periodontal disease. pit and fissure sealants. shape and location. Which of the D. C. tooth brushing and oral hygiene A. less hard and less colour stable. preventive resin restorations. volumetric changes in the casting. D. Central fossa. superior surface on the cast. The use of chemical retraction agents. C. harder and less colour stable. B. palate. content. superior accuracy. D. A. C. ©The National Dental Examining Board of Canada 2017 . B. and translation. vertical axis of the maxilla to the D. D. hinge axis to the maxillary arch. Inadequate use of water with the high. harder and more colour stable.6. B. The NDEB periodically reviews the bank to improve its quality. D.Which of the following is the most likely cause Compared to heat-cured acrylic resins. B. The principal advantage of polyether impression materials over polysulfide polymer impression materials is A facebow relates the A. B. the junction between the movable and immovable soft palate. mandibular arch to the glenoid fossa. C. A metal-ceramic crown is being placed on B. The patient has ideal occlusal C. A. speed handpiece. casting porosity from inclusion of foreign relationships and the crown has been adjusted bodies. following occlusal contact points should be eliminated? A. B. maxilla to the mandible at the vertical dimension of occlusion. C. less dimensional change with time. longer shelf life. casting porosity from inclusion of gases. The most appropriate management of an 8 year old patient with cavitated occlusal enamel The vibrating line in a completely edentulous lesion of the permanent first molars is maxilla is A. The use of a reservoir on the sprue of a wax pattern decreases A. In addition. the format of some items is not currently used. B. the junction between the hard and soft instruction. an imaginary line that crosses the soft palate posterior to the fovea palatini. casting porosity during solidification. cold- of periodontal pocket formation following cure acrylic resins are crown preparation? A. C. Inclined lingual plane of the mesiobuccal cusp. Mesial marginal ridge. Invasion of the biological width. in a cold environment. under vacuum. immersed in a fixing solution before A. The working time of zinc-phosphate cement The primary purpose of surgical therapy for the treatment of periodontitis is to A. D. control plaque. apically position the flap. foramen. The most likely reason for porcelain fracturing off a long and narrow metal-ceramic fixed The rate of "set" of polysulfide impression partial denture is that the framework alloy had materials is accelerated by an insufficient A. B. C. content. coated with a thin film of separating medium. C. cementum deposition into the apical B. is shortened if the mixing slab is cooled. A. immersed in water 10 minutes before C. increasing the mixing temperature. The NDEB periodically reviews the bank to improve its quality. it is Following successful root canal therapy. D. a gypsum-bonded investment material will undergo an additional expansion A. elastic modulus. poured immediately. B. start mixing. Before performing periodontal surgery. E. D. In addition. proportional limit. D. C. in a dry environment. B. D. decreasing the mixing temperature. the important to most desirable apical tissue response would be A. under water. is lengthened if the powder is mixed with B. prescribe systemic antibiotics. and translation. is shortened if moisture condenses on the mixing slab during the mixing process. eliminate periodontal pockets. periodontal pocket. a chronic low grade inflammatory response surrounded by normal bone. remove the ulcerated epithelium of the C. pouring. adding oleic acid to the mix. pouring. if it occurs B. poured within an hour. B. prescribe a mouthwash. proliferation of epithelium from the apical periodontal ligament. is shortened by adding a small quantity of D. A. fracture toughness. ©The National Dental Examining Board of Canada 2017 . B. C. C. D. A. the liquid as quickly as possible. Note: Some of the items in the Released Test Item Bank may have been discontinued due to outdated science or errors. the format of some items is not currently used. tensile strength. increasing the base/catalyst ratio. improve access for removal of local powder to the liquid a minute prior to etiologic factors. formation of a connective tissue capsule over the foramen. C.Polysulfide rubber base impressions should be During setting. convex only in the mesiodistal direction. less irritating to the soft tissues. a palatal strap is used instead of a narrow bar because it is The primary stress bearing area of the A. C. natural recess that does not need to be B. the cingulum of the canine provides a kept to a minimum. Proximal plate. B. D. after the diagnosis and treatment plan has D. maxillary complete denture is the B. In a removable partial denture. visibility and access are better. D. C. the force transmitted per unit area will be D. The NDEB periodically reviews the bank to improve its quality. finalize the location of the occlusal rest. more rigid with less thickness. phonetics is improved. be placed. Circumferential clasp. C. concave faciolingually and convex B. less leverage is exerted against the tooth The base of a distal extension partial denture by the rest. more hygienic. and translation. zygoma. D. minimize the preparation needed for the A removable partial denture rest should be occlusal rests. alveolar ridge. B. upon delivery of the denture. B. hard palate. A. C. compression. strength of the base is increased. the format of some items is not currently used. E. after the teeth have been set on the trial A.Guiding planes on abutment teeth for a Which component of a partial denture removable partial denture should be prepared framework provides the best indirect retention? before the occlusal rests are prepared in order to A. mesiodistally. If adjustment of the occlusal plane of natural teeth opposed by a complete or partial denture The gingival aspect of a pontic which touches is required. more stable. A. C. median palatal raphe. D. fabricated to produce slight tissue been established. C. Rest. A. should cover the maximum support area B. it should be completed the alveolar ridge should be A. In addition. C. avoid post-insertion adjustment. D. maximum number of artificial teeth can prepared. the enamel is thicker on the lingual because surface. immediately after making the final casts. denture. small and convex in all directions. improve the retention of the direct retainer assembly. ©The National Dental Examining Board of Canada 2017 . facilitate surveying of the cast. easier to polish. placed on the lingual surface of a canine rather than on the incisal surface because A. C. content. Note: Some of the items in the Released Test Item Bank may have been discontinued due to outdated science or errors. B. E. Lingual strap. D. D. occlusion. eliminate the need for a tray adhesive. facilitate removal of the impression. an excessive vertical dimension of B. When a partial denture framework fits the B. mouth. use of indirect retention. C. The extension of the lingual anterior border of a mandibular denture is limited by the A. fibres of the digastric muscle. casting of the framework. B. A. polysulfide rubber base impression materials to A. In addition. A. the format of some items is not currently used. preserves more alveolar bone. Clindamycin. For a patient allergic to penicillin. genioglossus muscle. Ampicillin. cheek affected by the biting may result from A. custom trays D. B. Erythromycin. B. C. length of the retentive arm. causes less soft tissue inflammation. B. Metronidazole. insufficient horizontal overlap of the posterior teeth. D. The NDEB periodically reviews the bank to improve its quality. Cephalexin. location of the occlusal rest. what is the B. content. the error is likely to be in the D. insufficient coverage of the retromolar pad areas. Note: Some of the items in the Released Test Item Bank may have been discontinued due to outdated science or errors. impression making. is less expensive. incorporating all undercut areas available. geniohyoid muscle. prophylaxis? D. preparation of the teeth. design of the framework. most appropriate drug for antibiotic C. and translation. When compared to a conventional complete C. an overdenture material. establishing harmonious occlusion. A. D. allow for a more uniform setting of the denture. C. position on the abutment tooth. D. master cast but does not fit properly in the C. C. is more fracture resistant. B. use of cast clasps. D.The flexibility of a retentive clasp arm is In a patient with complete dentures. Long term stability in partial dentures is best ensured by A. mylohyoid muscle. A. location of the reciprocal arm. ©The National Dental Examining Board of Canada 2017 . In complete denture construction. C. are recommended for polyvinyl siloxane and E. obtain a uniform thickness of material. the use of steep-cusped posterior teeth. 0. Improper wax-up of the partial denture. D. In addition. C.02 – 0. 0. C. A. B.3 – 0.0 – 1. B. the format of some items is not currently used. 8-10mm.8ml. B. D. function of the occlusal rest to counteract for the cast. rigidly connect the denture components. physiological response to additional forces D. Not enough water in the mix of the stone A.5ml. occlusal force transmission. C. is 2-4mm. resistance to flexion of the retentive clasp D. Note: Some of the items in the Released Test Item Bank may have been discontinued due to outdated science or errors.04ml. B. content. Duplication impression slightly oversized. C. ©The National Dental Examining Board of Canada 2017 . function of the reciprocal clasp arm to When epinephrine 1:1000 is administered counteract the retentive clasp arm. C.5ml. arm. C. Resorption of bone. the most appropriate volume for an initial dose is Rests on terminal abutment teeth for a A. D.The major connector of a removable partial When making maxillomandibular records for a denture should be designed to complete denture patient. distribute forces to the soft tissues. it is equal to the rest vertical dimension. The NDEB periodically reviews the bank to improve its quality. 1. D. dissipate vertical forces. D. occlusal forces. placed on abutment teeth? E. the difference between the occlusal vertical dimension and the rest vertical dimension A. interocclusal distance (freeway space) is C. 1. act as a stress-breaker. Increase in trabeculation. A. the vertical dimension of occlusion is acceptable when A. and translation. indirect retention. B. B. the maxillary rim shows just below the upper lip and the mandibular rim is even Which of the following is NOT a direct with the corners of the mouth. intramuscularly for the management of anaphylaxis in an adult. Too much water in the mix of the stone dentures design refers to the for the cast. Decrease in width of periodontal ligament. primary retention. removable partial denture provide B. lateral force transmission. occlusal rims contact evenly and bilaterally at the same time as the lips touch. Which of the following factors could cause a partial denture framework to fit tighter in the mouth than on the cast? Reciprocation as applied to removable partial A. return to a passive state of the flexed clasp. Increase in width of cementum.6 – 1. hide the porcelain-metal junction on their The efficacy of pit and fissure sealants is gingival aspect. increasing the temperature of the mixing water. remove the tooth and replant it in a more can be increased by favorable position. half of the crown is missing. B. The incisal D. Pulpotomy. have minimal soft tissue coverage. In which of the following will the effects of polymerization shrinkage be greatest? Radiographic examination of an intruded primary maxillary central incisor reveals no A. A 7 year old patient presents with a crown C. D. root fracture and no displacement in the B. opacity of the sealant. an overextended border on the partial.At the first post-insertion appointment. Pulp capping. a What is the most appropriate endodontic patient with a new removable partial denture management for a 9 year old patient for complains of a tender abutment tooth. Preventive resin restoration. Pulpectomy. B. The most appropriate C. C. Class I occlusal restoration. D. Direct veneer restoration. extract the tooth. D. adding sodium phosphate to the mixing fracture of a permanent maxillary central water. management would be to D. C. B. content. improper path of insertion. increasing the water/powder ratio. systemic fluoride treatment. incisor that occurred 3 days ago. A. Note: Some of the items in the Released Test Item Bank may have been discontinued due to outdated science or errors. type of polymerization reaction. D. occlusal relationship. Apexification. affected by A. A. A. B. and translation. resulting in a 2mm exposure of vital pulp. have a concave surface touching the mucosa. C. completely replace the missing C. Class IV restoration. stage of tooth eruption. D. B. Pulpectomy. ©The National Dental Examining Board of Canada 2017 . The most tooth 1. the format of some items is not currently used. inadequate polishing of the framework. B. with an open apex and a necrotic likely cause is pulp? A. the occlusion. supragingival tooth structure. E. B. Revascularization. C. decreasing the amount of mixing. Apexification. E. Pulpotomy. Apical surgery. observe periodically. permanent tooth germ. The NDEB periodically reviews the bank to improve its quality. In addition. A. The rate of set of alginate impression materials D. What is the most appropriate initial management for this tooth? Fixed partial denture pontics should A.1. bring the tooth into position and ligate it. C. B. E. C. E. ©The National Dental Examining Board of Canada 2017 . indirect pulp capping. A labial frenum causes a diastema between the B. Remove the soft tissue overlying the occlusal surface of the permanent molar. D. observe the case until the eruption of The most appropriate immediate management permanent maxillary lateral incisors and is to canines. Extract the primary molar. all primary teeth erupt. carefully remove both incisors. agent. child have been traumatically intruded 4mm. elastic and (or) separation wire. Use of a bonding agent prior to sealant application. close the space with a removable A. Use of separate etching and bonding agents rather than a self-etching bonding A. D. D. D. E. Which In an 8 year old patient the most appropriate of the following is the most appropriate treatment of a vital first permanent molar with management? closed apices and a large carious exposure is A. the first tooth erupts. disturbing the teeth.Which of the following procedures will NOT The most appropriate restoration for a primary improve the retention of a fissure sealant? first molar with extensive carious destruction of the crown is a A. Maintaining a dry field until the sealant is set. content. posterior composite resin. pulpotomy with MTA. orthodontic appliance. is to A. close the space with a fixed orthodontic The maxillary central incisors of a 2 year old appliance. Wait until more tuberosity growth occurs. B. C. lateral incisors and canines have not erupted. B. A. the format of some items is not currently used. Unlock the permanent first molar with hydroxide. D. resin-modified glass ionomer. close the space after the eruption of the C. pin retained amalgam. B. direct pulp capping with calcium C. D. reposition and splint the intruded teeth. order an occlusal radiograph. bur. In addition. Reduce the distal surface of the primary B. Oral hygiene for infants’ teeth should begin when A. pulpectomy. perform a frenectomy. molar. D. The NDEB periodically reviews the bank to improve its quality. the infant is weaned from the nursing The most appropriate immediate management bottle or breast. Roughening of the enamel with a 2 round C. B. Note: Some of the items in the Released Test Item Bank may have been discontinued due to outdated science or errors. make the patient comfortable without permanent lateral incisors. The eruption of a maxillary permanent first molar is prevented by a slight interference with the crown of the primary second molar. reposition the intruded teeth. C. The C. and translation. permanent maxillary central incisors. stainless steel crown. the first primary molars erupt. the cusp with the largest pulp horn is the The principal reason for a needle aspiration of an intraosseous radiolucent lesion of the A. and translation. pulpotomy. tendon of the temporalis muscle. immediate maxillary denture is used to C. reposition and perform a pulpectomy. indicate areas that require additional hard or soft tissue reduction. drain purulent material. B. A. D. The most appropriate management of an ceramic restoration prior to bonding is intruded 5. leave in place and perform a pulpectomy. mesiobuccal. Note: Some of the items in the Released Test Item Bank may have been discontinued due to outdated science or errors. retrograde filling. roughening the surface with a diamond C. B. acid etching with hydrofluoric acid. In primary molars. E. improved retention of the denture. A.1 with the apex displaced toward the achieved by labial bone plate is to A. obtain a specimen for histopathological D. B. mandibular denture is determined primarily by the A. content. A. distolingual. ©The National Dental Examining Board of Canada 2017 . the format of some items is not currently used. C. C. C. greater incidence of denture fracture. A. a maxillary denture opposing a full and a wide open apex is complement of natural teeth is more often associated with A. distobuccal. control hemorrhage while the new denture is being fabricated.The most appropriate management for a Compared to a set of opposing complete permanent central incisor with a necrotic pulp dentures. B. root canal therapy using gutta-percha. E. C. protect the extraction sites while fitting the denture. less denture tooth wear. C. In addition. determine the presence of a vascular lesion. decompress a fluid-filled lesion. mandible is to B. The NDEB periodically reviews the bank to improve its quality. E. bur. masseter muscle. The shape of the distobuccal border of a D. root canal therapy followed by a D. apexification. assist in remounting the denture prior to refining the occlusion. buccinator muscle. improved stability of the denture. The preconditioning of a high glass content all. improved appearance of the denture. sandblasting. B. mesiolingual. allow spontaneous repositioning. B. degreasing with acetone. A surgical template (stent/guide) for an B. revascularization. D. D. extract. D. acid etching with phosphoric acid. C. analysis. The most appropriate management is diagnosis is to A. Place amalgam restorations as quickly as B. remove the mesiodens and orthodontically C. the format of some items is not currently used. D. The most probable erupted. Lingual holding arch. the presence of hyperplastic vascular pulp possible. allow the mesiodens to erupt. wait for the mesiodens and the tooth 2. Place the patient on a preventive regime the normal and affected areas. In addition. the strength of the material. pulpal necrosis.The characteristic colour seen in the crowns of Which is the most appropriate initial treatment teeth with internal resorption is due to for a 16 year old patient presenting with multiple extensive carious lesions on 20 teeth? A. A. Distal shoe appliances.1 is A 4 year old has a primary central incisor that partially erupted and tooth 2. content. E. X-rays at the edge of the x-ray beam. C. remove the mesiodens and re-assess. deposition of pigment in the cells of the odontoblast layer. Long wavelength x-rays. High frequency x-rays. Gamma radiation. and delay any treatment. internal resorption. and translation. Note: Some of the items in the Released Test Item Bank may have been discontinued due to outdated science or errors. Bilateral band and loops. B.1 has not yet is yellow but asymptomatic. D. B. D. Excavate and place provisional C. restorations. Restore all teeth with gold inlays to utilize tissue. A. a change in the consistency of the dentin. B. A periapical radiograph of a 7 year old child reveals an inverted mesiodens. C. tissue. D. erupt the central incisor. The aluminum filter in an x-ray machine Which of the following space maintainers is/are prevents which of the following from reaching most appropriate for a 4 year old child whose the patient? mandibular first primary molars have been extracted? A. C.1 to erupt. pulpal calcification. A. D. The NDEB periodically reviews the bank to improve its quality. Tooth 1. B. A removable appliance. the degeneration and necrosis of the pulp D. external resorption. the difference in the refractive indices of C. B. ©The National Dental Examining Board of Canada 2017 . B. the format of some items is not currently used. A. B. There is a slight hemorrhage B. A. Volume fraction of filler particles. B. extract the primary mandibular canines Which of the following has the greatest effect only. values are normal. a base and restoration. C. fusion. pulp capping. Reduction of mandibular fracture risk. Bilateral distal shoes. sound. Prevention of mandibular incisor Which of the following is the most appropriate crowding. resistance to root fracture. and translation. A. Weight fraction of the filler particles. Horizontal impaction. perform pulpotomies and interproximal discing on the mandibular primary canines. The NDEB periodically reviews the bank to improve its quality. ©The National Dental Examining Board of Canada 2017 . A. trauma to a crowding of the mandibular incisors. gemination.An 8 year old patient presents with 4mm After the crown completion stage. Which of the following space maintainers is/are D. management for this patient is C. Recurrent pericoronitis. monitor and recall in 6 months. mandibular third molars? A. reinforcement of the remaining root Which of the following conditions is an structure. extract the primary mandibular canines and resin? place a lower lingual holding arch. there is a small mechanical exposure of one of A. Cone beam CT. Magnetic resonance imaging. most appropriate for a patient with the bilateral loss of mandibular first primary molars prior to the eruption of the permanent molars and permanent incisors? When preparing a cavity in a primary molar. temporomandibular joint disc? D. enamel hypoplasia. B. Modulus of elasticity of the filler particles. In addition. Size of the filler particles. Hardness of filler particles. All cephalometric A. C. The developing tooth may be responsible for patient has a Class I malocclusion and the arches are well aligned. The most appropriate treatment is D. treated tooth with minimal coronal tooth D. D. The placement of a post in an endodontically C. extraction and space maintenance. for determining the morphology of the C. a ferrule effect. Arthrography. A. and the dentin surrounding the exposure is C. C. the pulp horns. C. B. Lingual holding arch. pulpectomy and restoration. The most appropriate B. B. Bilateral band and loop. structure provides E. Corrected tomography. content. base and restoration. dilaceration. on the mechanical properties of composite D. D. retention for the core. Note: Some of the items in the Released Test Item Bank may have been discontinued due to outdated science or errors. indication for the removal of impacted D. Fixed palatal arch with an anterior button (a Nance appliance). 6. bone on the lingual surface is fenestrated prednisone once a day. Streptococci species. Resin-modified glass ionomer. Note: Some of the items in the Released Test Item Bank may have been discontinued due to outdated science or errors. The NDEB periodically reviews the bank to improve its quality. there is no response to an electric pulp test. It is indicated for the early management of A. ©The National Dental Examining Board of Canada 2017 . During endodontic treatment. B. Bonded amalgam. C. C. which of the following microorganisms is most likely to cause endocarditis in a patient with valvular A root fragment of an impacted third molar heart disease? could be displaced into the submandibular space during its surgical removal when the A. D. 8. E. is a postmenopausal woman with a full dentition and generalized osteoporosis. D. B. Stainless steel crown. D. True vitality can be determined. root of the mandibular third molar lies close to the buccal cortex. inferior alveolar canal. has an impacted third molar close to the A. C. It does not confirm health of the pulp. Vascularity can be determined. A patient with amyloidosis is taking 20 mg D. down. D. atrophy of the mandible. the periradicular region is affected if A. C. It is indicated for the definitive testing? management of angina pectoris. Its administration will increase blood D. there is pain on percussion. What is the minimum below the mylohyoid muscle. In addition. B. 4. B. and translation. apical of the roots. D. Prevotella species. buccal cortical bone in the mandibular third molar area is extremely thin. A. Staphylococcus species. response. Composite resin. content. B. what is occur during mastication if the patient the most appropriate restoration? A. B. attachment of the mylohyoid muscle is C. the format of some items is not currently used. Thicker enamel will lead to a quicker pressure. is a child with unerupted premolars. Candida albicans.Following a pulpotomy in a second primary Spontaneous fracture of the mandible may molar with extensive occlusal caries. 2. is edentulous and there is advanced C. It should be administered sublingually. number of weeks taking the drug that triggers the need for corticosteroid prophylaxis before proceeding with major dentoalveolar surgery under general anesthesia? In a tooth with pulpal necrosis. a myocardial infarction. C. there is pain to thermal stimuli. Which of the following is NOT correct regarding nitroglycerin used for medical emergencies? Which statement is true regarding electric pulp A. the tooth throbs when the patient is lying D. C. B. Exposure to air or light shortens the shelf- life of the tablets. B. A. ©The National Dental Examining Board of Canada 2017 . A post is used in an endodontically treated What is an early clinical manifestation of local tooth to anesthetic toxicity (overdose)? A.Which of the following is NOT a characteristic Which of the following is NOT a component of of an acute apical abscess (acute periradicular a dental cartridge containing 2% lidocaine with abscess)? 1:100. C. ask the patient to lightly tap on the D. Exfoliative cytology. C. Water.000 epinephrine. Radiographic examination alone. C. B. Articaine 4% with 1:100. D. obturate the canal. C. Pain on percussion. A. A. Note: Some of the items in the Released Test Item Bank may have been discontinued due to outdated science or errors. nonsurgical root canal therapy before periodontal therapy. nonsurgical root canal therapy only. Pain on palpation. periodontal surgical therapy only. Prilocaine 4% with 1:200. B. C. Decreased blood pressure. B.000 epinephrine. and translation. Tooth feels elongated. Aspiration cytology. C. the best way to Which of the following is necessary to make a ensure accurate seating is to diagnosis of a keratocystic odontogenic tumour (odontogenic keratocyst)? A. time for insertion. The most appropriate treatment of a true E. Inability to focus. B. A. C. Lightheadedness. D. D. B. B. provide the longest duration of anesthesia? D. B. D. relieve the internal angles of the inlay combined endodontic-periodontal lesion is before insertion. content. D. C. the format of some items is not currently used. D.000 epinephrine.5% with 1:200. apply a firm pressure on the inlay until the cement is set. retain the restoration.000 epinephrine? A. Lidocaine 2% with 1:100. reinforce the remaining crown. Methylparaben. strengthen the root. Histopathologic examination.000 epinephrine. force the restoration in place with an orange wood stick and mallet. When cementing a gold inlay. Sodium chloride. Bupivacaine 0. Pain to a cold stimulus. In addition. periodontal surgical therapy before non- Which of the following local anesthetics surgical endodontic treatment. A. Vomiting. Metabisulphite. A. restoration until the occlusion is comfortable. mix the cement rapidly to allow ample B. The NDEB periodically reviews the bank to improve its quality. 2 days. The NDEB periodically reviews the bank to improve its quality. Perform a pulp extirpation. D. D. under the distobuccal cusp. and translation. using calcium hydroxide as a treatment root canal filling. The most appropriate management is to C. E. gradual dissolution of the liner into the that the master cone is 2. Which is the B. paper point medicated with formocresol. replantation. Nothing. Corticosteroids. C. proceed with the filling as the cone is D. A. the format of some items is not currently used. The disappearance of a calcium hydroxide liner under a restoration after a length of time is most likely a result of The canal of a maxillary canine has been instrumented to within 1mm of the apex and is A. C. C. A. gutta percha. within acceptable limits. 2 hours. most appropriate initial management? C. evidence of internal resorption. residual bacterial action on the hydroxyl ions. immersing the tooth in fluoride solution D. zinc-oxide eugenol. apex. D. length of time between avulsion and replantation. canal. C. completion of endodontic therapy before B. B. between the distobuccal and the mesiobuccal orifices. C. lingual to the orifice of the mesiobuccal E. B. microleakage. 2 weeks. before replantation. C. silver cone. maxillary first molar is most often found B.The success of replantation of an avulsed tooth The minimum time to wait before placing is dependent upon composite restorations after the completion of a bleaching (whitening) treatment is A. Observe and re-evaluate in 3-6 months. A. 2 months. incorporation into the adhesive agent. In addition. Perform an apicoectomy. ©The National Dental Examining Board of Canada 2017 . Perform a pulpotomy. between the palatal and the distobuccal orifices. cut the cone 1mm and insert. discard the cone and fit a smaller one. A radiograph indicates B. B. content. Antibiotic powder. ready to be obturated. Note: Some of the items in the Released Test Item Bank may have been discontinued due to outdated science or errors. D. D. A. Oxidized cellulose. fit a larger cone within 2mm of the apex. The material of choice for obturating the root canal system of a primary tooth is The orifice to the fourth canal in a permanent A. Bone wax. which of the following should be placed in the bony defect prior to suturing the flap? Radiographic examination reveals early A. zinc oxyphosphate. After performing an apicoectomy.5mm short of the restorative material. D. B. Weight loss. B. pulp necrosis. viral disease. D. thrombocytopenic purpura. invade and metastasize.After a thermal stimulus has been removed A definitive diagnosis of osteosarcoma is from a tooth persistent pain suggests established by A. An antipyretic drug A characteristic of malignant tumors is the A. myeloid leukemia. D. C. E. healing is expected to be uncomplicated. E. provides analgesia. biopsy. C. fungal disease. counters the tendency for epileptic capsule. A. fragments are not displaced by muscle B. non-specific bacterial infection. Exophthalmos. causes loss of consciousness. Note: Some of the items in the Released Test Item Bank may have been discontinued due to outdated science or errors. D. radiographic examination. ©The National Dental Examining Board of Canada 2017 . grow to large size and remain within their E. Which of the following epithelial changes is A decrease in the neutrophil count is present in most likely to be precancerous? A. irreversible pulpitis. E. and translation. A. the format of some items is not currently used. D. Acanthosis. The NDEB periodically reviews the bank to improve its quality. A fracture is considered to be favourable when Which is a characteristic of a patient with myxedema? A. Testosterone. content. leukocytosis. In addition. iron deficiency anemia. C. D. not exposed to the oral environment. C. Prednisolone. aspiration cytology. B. D. Heat intolerance. reduces fever. Dysplasia. Which of the following steroids can produce Cushing's syndrome? Histoplasmosis is a A. B. C. operating time will be short. D. Tachycardia. ability to B. Estradiol. A. B. a normal pulp. A. pull. C. C. Progesterone. C. A. creates heat sensitivity. exposed cervical dentin. B. reversible pulpitis. B. clinical examination. D. grow slowly. C. remain localized. Hyperkeratosis. B. Parakeratosis. B. C. granulocytopenia (agranulocytosis). seizures. D. protean disease. Diethylstilbestrol. Lethargic appearance. D. E. D. concentration of fluoride in public water C. A. content. myocardial infarction. at the cemento-enamel junction. allows safe temporary separation of the teeth is B. ©The National Dental Examining Board of Canada 2017 . the most The Fluorosis Index is used to measure the important first aid measure is to A. Salivary secretion A. The NDEB periodically reviews the bank to improve its quality. C. D. 5 to 7 days. In addition. 24 to 48 hours. E. Patients with a history of rheumatic fever and known heart valve damage should be given prophylactic antibiotic coverage before dental extractions because of the risk of In a fully erupted tooth with a healthy periodontium. Note: Some of the items in the Released Test Item Bank may have been discontinued due to outdated science or errors. be parallel to each other. B. The facial and lingual walls of the occlusal portion of a Class II cavity preparation for an amalgam in deciduous teeth should A characteristic of the periodontium which A. converge toward the occlusal surface. and translation. degree or severity of mottled enamel. shows no difference with age. C. shows a decrease in mineral content in the B. epithelium is located B. C. D. diverge toward the occlusal surface. C. subacute bacterial endocarditis. control infection. not follow the direction of the enamel A. B. B. elderly. less than 24 hours. the C. 1. rods. C. passive eruption. total amount of fluoride ingested. in the cervical third of the crown. 2 to 4 days. opposition to fluoridation by citizens' groups. Deposition of plaque on teeth occurs in B. establish and maintain an airway. nature of acellular cementum. the format of some items is not currently used. decreases with age. increases with age.5mm below the cemento-enamel junction. D. the apical end of the junctional A. A. degree of protection offered against A. modified continuous eruption of the teeth. D. supplies. All of the above. D. prevent shock. in the cervical third of the root. elasticity of bone. D.In facial injury management. B. cardiac arrest. C. dental caries by fluoride supplements. control the bleeding. D. E. leaving the site undisturbed for a period C. B. horizontal relationship of the mandible to the maxilla. prevent hemorrhage. occlusion. B. flap operation. a salt water rinse. In addition. D. reduce occlusal trauma. root planing. microorganisms and calculus. the same as the vertical relation of C. gram-positive cocci. the format of some items is not currently used. B. The tooth surfaces LEAST susceptible to caries are The vertical relation of rest is A. D. vertical relationship of the mandible to D. protect the wound. B. splinting. A. root planing and curettage. ©The National Dental Examining Board of Canada 2017 . lingual of the mandibular arch. eliminated by B. gram-negative cocci. D. B.Following periodontal surgery. B. the predominant microflora of gingival plaque are Centric relation is a A. D. gram-negative facultative and anaerobic skull. D. A. occlusal of the mandibular arch. subgingival curettage. regular use of a water-irrigating device. D. C. gingival surgery. C. The NDEB periodically reviews the bank to improve its quality. gingivoplasty. the most The primary objective of initial periodontal important factor to promote healing is therapy is to A. remove the colonized masses of of 3 months. greater than the vertical relation of E. thorough plaque control. lingual of the maxillary arch. A. the same as the interocclusal distance. content. Chronic gingival inflammation is best eliminated by A. adjustments. and translation. gingival massage. eliminate crowded and tilted teeth. the maxilla. mesial of the maxillary arch. D. protect the sutures. less than the vertical relation of occlusion. B. C. make adequate dietary and nutritional C. rest position of the mandible. B. The primary reason for placing a surgical dressing after a gingivectomy is to Residual soft tissue interdental craters NOT associated with underlying bony changes are A. C. stabilize the teeth. occlusal correction. C. distal of the maxillary arch. rods. spirochetes. D. In normal gingiva. A. Note: Some of the items in the Released Test Item Bank may have been discontinued due to outdated science or errors. relation of the maxilla to the rest of the C. occlusion. B. A. Rosacea. C. In addition. Which of the following procedures is NOT Which of the following bacterial types is indicated for the management of infrabony implicated in the initiation of gingivitis? defects? A. Streptococcus salivarius. smokes 2 packs of cigarettes daily. Measles. C. A. D. the pit or groove is the full enamel thickness. Mylohyoid. C. the format of some items is not currently used. destruction of the adrenal cortex. content. B. caries is detected radiographically. B. balancing occlusion. B. centric relation. Temporalis. C. either centric relation or balancing D. B. hypersecretion of the posterior pituitary. Note: Some of the items in the Released Test Item Bank may have been discontinued due to outdated science or errors. the angle ANB is frequently Which disease could cause both renal and used. horizontal protrusive relation. greater than the normal. occlusion. Leptothrix buccalis. Flap surgery. B. B.To evaluate an existing occlusion. the pit or groove is less than full enamel D. bundle bone. The NDEB periodically reviews the bank to improve its quality. associated hypothalamic centres. destruction of the posterior pituitary or thickness. less than the normal. ©The National Dental Examining Board of Canada 2017 . Digastric. membranous bone. D. D. Which of the following patients has the POOREST prognosis when placing dental Which of the following is classified as a muscle implants? A patient who of mastication? A. Gingival graft. B. A. caries has reached the DEJ. is on thyroid replacement therapy. lamellar bone. A. Buccinator. D. A. is over age 80 years old. D. C. endochondral bone. C. Scarlet fever. and translation. Gingivectomy. C. this angle is A. diagnostic The maxilla is formed from casts should be mounted on an articulator in A. B. C. Diabetes mellitus is the result of Enameloplasty can be used when A. Regenerative surgery. Actinomyces viscosus. D. D. normal. atrophy of the islands of Langerhans. C. C. Streptococcus mutans. Rubella. In patients with severe Class II cardiac complications? malocclusion. has generalized osteoporosis. In cephalometric analysis of children with malocclusion. B. and translation. protect the wound from mechanical B. C. Fish. Promote wound healing. Note: Some of the items in the Released Test Item Bank may have been discontinued due to outdated science or errors. inner surface of the enamel. C. weight loss and general malaise. pemphigus. D. container with a lid. Temporal. buccal mucosa. D. Double lip. Use of air spray when condensing. Use of high volume evacuation when A. B. most likely B. Glossopharyngeal. as well as some irregularly shaped ulcerations on the A. polishing or removing amalgam. Storage of amalgam scrap in a dry E. outer surface of enamel and dentin. C. D. prevent microbial colonization of the wound. outer surface of the dentin. varicella. E. She has which cranial nerve? a rash on the malar area and nose. recurrent herpes. C. B. A. outer surface of the enamel. A quarterly mercury assessment for office personnel. Commissural pits. inner surface of the dentin. In addition. content. Wheat germ. Which of the following procedures must be done to ensure acceptable mercury hygiene in a Smooth surface caries begins at localized areas dental office? on the A. D. A. B. an allergic response. D. B. erythema multiforme. The NDEB periodically reviews the bank to improve its quality. E. D. C. lupus erythematosus. Lettuce. The parasympathetic post ganglionic fibers A 32 year old female patient complains of leaving the otic ganglion will travel along fever. working with amalgam. Facial.Which of the following is the most frequent Which of the following would maximize major congenital malformation of the head and vitamin E intake following osseous surgery? neck? A. C. Eggs. the format of some items is not currently used. Cystic hygroma colli. A. C. B. injury. C. Encephalotrigeminal angiomatosis. achieve hemostasis. Branch of the mandibular. D. D. herpes zoster. bullous pemphigoid. lichen planus. A periodontal dressing is placed following a gingivectomy to: Oral lesions that do NOT cross the midline are A. Cleft palate. ©The National Dental Examining Board of Canada 2017 . The most likely diagnosis is B. By definition. exposure to the external environment. estrogen. The most likely diagnosis is B. AIDS. storing radiographs at 30°C. a tooth in the line of fracture. hands. 4.5. -3mm -4.5mm in the mandible. ©The National Dental Examining Board of Canada 2017 . frequent diarrhea and weight loss. add bicarbonate ions to saliva. B. A current mixed dentition analysis yields the following data: Decreased alveolar bone density is associated with decreased levels of R L -5mm -3mm A. AIDS. E. Note: Some of the items in the Released Test Item Bank may have been discontinued due to outdated science or errors. D. incomplete fixing and/or washing of A. crusting of the lips and red circular lesions on the palms of the A. Diabetes mellitus. erythema multiforme. C. acute herpetic gingivostomatitis.5mm B. B. content. ulcers with hyperplastic margins in the buccal C. processing at an excessive temperature. infectious mononucleosis. parathyroid hormone. E. D. The NDEB periodically reviews the bank to improve its quality. 7.A 22 year old patient has been experiencing Yellow or brown stains appearing on general malaise. B. C. using expired film. Space maintainers were not placed. D. B. help form a hypertonic saliva ( relative to oral clinical examination shows linear mucosal serum ). vestibule.5mm in the mandible. sore throat and radiographs months after processing result coughing for one week. There are multiple from ulcerations of the oral mucosa. What is the most likely diagnosis? D. remove K+ ions from saliva. C. radiographs. Crohn’s disease. C. the format of some items is not currently used. 8mm in the maxilla. C. gonorrhea. In addition. and translation. 2mm in the maxilla.5 and tooth 7. a compound fracture of the D. fever. A. add salivary amylase to saliva. One function of the striated ducts in the parotid and submandibular glands is to A 24 year old patient complains of abdominal A. The B. mandible must have A. A. Leukemia. D. add Na+ ions to saliva. multiple bone fragments. A 7 year old patient is missing tooth 5. The actual space loss is D. displacement of the fractured segments. thyroxin. hydrocortisone. pain. C. adding powder to the water in the mixing A. D. general anesthetic agent. D. D. D. Periodontal atrophy. Absence of occlusal contacts on a provisional Which has the WORST prognosis? restoration may result in the definitive restoration exhibiting A. A. Fremitus. an apically positioned flap. Note: Some of the items in the Released Test Item Bank may have been discontinued due to outdated science or errors. hyperthyroidism. C. using room temperature water. B. mixing beyond the recommended time. aid in healing. D. B. B. content. B. A. The primary use of nitrous oxide and oxygen in Guided tissue regeneration surgery selectively dentistry today is as a(n) promotes the growth of all of the following EXCEPT A. E. the initial incision Which of the following is NOT a sign of is made to occlusal trauma? A. Gingival recession. In addition. agent for the management of chronic C. epithelial cells C. D. D. expose the sulcular lining of the pocket. the format of some items is not currently used. cementoblasts. open proximal contacts. C. malignant tumours. High telomerase activity is associated with A. continued initial therapy. ©The National Dental Examining Board of Canada 2017 . bowl. obstructive pulmonary disease. heavy occlusal contact. Widened periodontal ligament. C. tight proximal contacts. and translation. agent for conscious sedation. cystic fibrosis. gingivectomy. B. C. The NDEB periodically reviews the bank to improve its quality. Gingivitis. endothelial cells. Aggressive periodontitis. sever the attachment of the oblique fibres C. osteoblasts. substitute agent for local anesthesia. excise the keratinized gingiva. A. In periodontal flap surgery. C. Occlusal traumatism. D. B. C. no occlusal contact. Chronic periodontitis. diabetes mellitus. D. of the periodontal ligament. B. Tooth migration. B. gingival curettage. B.The physical properties of alginate impression A patient fails to demonstrate effective plaque materials will be adversely affected by control during initial periodontal therapy for moderate periodontitis. "tumbling" the alginate container prior to management is filling the dispensing scoop. The most appropriate A. Reduce bleeding during the surgery. appropriate initial management is (a) C. C. and translation. Which of the following is NOT correlated to early implant failure? The periodontal ligament A. muscles and bones. buccal to lingual. Increase the width of keratinized tissue. lingual to buccal. Occipital. C. seen as perikymata on the surface of B. increases in width with age. C. bones and teeth. responsible for the incremental lines in enamel. content. the format of some items is not currently used. Upper medial. structural faults that span the entire lymphadenopathy is present. systemic antiviral therapy. topical steroid. has osteoblasts as its principle cells. D. skin and teeth. B. aberrant dentinal tubules that cross the ulcerated lesions on the buccal attached gingiva dentinoenamel junction. Lower medial. Frontal. Age of the patient. The most thickness of the enamel. B. C. achieves its final structural form after complete eruption. Upper lateral. for one day. In addition. newly erupted teeth. B. Regular smoking habit. only has oblique fibres during the eruptive C. Enamel spindles are A 29 year old patient has had multiple painful A. C. Temporal. B. stage. Reduce infrabony pockets. D. Sphenoid. D. D. Note: Some of the items in the Released Test Item Bank may have been discontinued due to outdated science or errors. D. A. A. The NDEB periodically reviews the bank to improve its quality.A drug that affects alkaline phosphatase The pulpal floor of an occlusal amalgam activity would target preparation on a mandibular first premolar should slope apically from A. B. D. Lower lateral. A gluteal intramuscular injection may be safely administered in which quadrant? Which of the following is a reason to perform initial periodontal debridement before A. palliative mouth rinse. D. malaise or B. mesial to distal. Poorly controlled diabetes. distal to mesial. ©The National Dental Examining Board of Canada 2017 . The pterygomaxillary fissure is formed by the maxilla and which other bone? A. A. muscles and skin. Increase the attachment levels. C. systemic antibiotic. C. A. periodontal surgery? B. D. No fever. Type III/IV bone around the implant. B. D. 900. Note: Some of the items in the Released Test Item Bank may have been discontinued due to outdated science or errors. A. B.000/mm3 C. mutans in White blood cell count: 5. anti-HBsAg. fever and bilateral cervical on the entire dorsal surface of the tongue. lesion is more than halfway through the D. The NDEB periodically reviews the bank to improve its quality. B. C. Actinomyces naeslundii. enamel. He reveals angular cheilitis and a smooth redness has malaise. hepatitis A. D. B. White blood cell count: 870. and translation. E. 100. Red blood cell count: 3. enamel.000. B. HBeAg. lesion is less than halfway through the A. Platelets: 82. A. content. necrotizing ulcerative gingivitis. squamous cell carcinoma. hepatitis B. C. the format of some items is not currently used.000/ml. A 47 year old female patient complains of a A 20 year old male presents with a three-day “burning sensation” of the tongue.A smooth surface coronal white spot carious The form of hepatitis that poses the greatest lesion that is visible when the tooth is both wet risk of transmission from patient to dental and dry indicates that the health care worker is A. B.000/mm3 mixed saliva at which a patient is deemed “high risk” for caries? The most likely diagnosis is A. D. infectious mononucleosis. 10. acute myelogenous leukemia.000-10. B. Lactobacillus casei. Streptococcus salivarius. D. C. C. ©The National Dental Examining Board of Canada 2017 . enamel is stained and not demineralized.000-400. Normal Values: Hb: 14-18g/100ml Platelets: 150.000. hepatitis C. Actinomyces viscosus.9g/100ml A.000/ml.000/mm3 D. anti-HBcAg. thrombocytopenic purpura. In addition. Examination history of an acute generalized gingivitis.000/ml. D. C. 100/ml. HBsAg. The lymphadenopathy. hepatitis D. A blood examination reveals most likely diagnosis is Hb: 8. primary herpes.000/mm3 B. anemia. 1. C.000/mm3 Red blood cell count: 4-5million/mm3 What is the threshold count of S. median rhomboid glossitis. The risk of a health care worker contracting hepatitis B through a needlestick injury from a Which microorganism does NOT contribute chronic hepatitis B carrier is increased when significantly to the progression of dentinal the patient’s serology report indicates the caries? presence of A. caries involves the inner half of the dentin. D. epithelial dysplasia. facial to the contact area. D. vascular dementia. Note: Some of the items in the Released Test Item Bank may have been discontinued due to outdated science or errors. The Glycemic Index of foods should be used when planning the diet. elderly is motor neurons in the swallowing center are activated to A. taste. Which of the following conditions is managed The tooth preparation for a porcelain veneer by the administration of bisphosphonates? must have a/an A. inhibit respiration. content. Pyridoxine. A. D. Vitamin B12. Sucrose up to 10% of total daily energy intake (e. D. B. olfaction. Food Guide to Healthy Eating must be C. ©The National Dental Examining Board of Canada 2017 .5mm. A. multi-infarct dementia.g. B. Ascorbic acid. the format of some items is not currently used. and translation. The NDEB periodically reviews the bank to improve its quality. open the lower esophageal sphincter. D. C. butt joint gingival margin. open the palatopharyngeal folds. C. A diet planned according to Canada’s B. modified for a person with diabetes. A. hearing. The most common senile dementia in the During the pharyngeal phase of swallowing. D. which of the following nutrients is of special concern Which of the following statements about the during pregnancy? nutritional management of diabetes is correct? A. In addition to iron. C. D. C. margin at least 1mm supragingivally. D. B. C. incisal reduction of 0. space for the veneer material. A. Osteomyelitis.An incipient lesion on an interproximal surface The most common sensory change in the is usually located healthy elderly is a decrease in A. 50% of 2000 kcal/day) is acceptable. lingual to the contact area. gingival to the contact area. B. initiate the secondary peristaltic wave. Vitamin D. D. at the contact area. B. Alzheimer’s disease. Osteoarthritis. B. calcium and folate. Osteoporosis. occlusal to the contact area. touch perception. In addition. Osteopetrosis. a result of cerebral arteriosclerosis. C. E. B. The fat content of the diet should be 30- 35% of energy intake. C. C. Vertical root fracture. B. The NDEB periodically reviews the bank to improve its quality. a lacerated lip and a by black eye. including the source of the injuries. Respect the patient’s request regarding B. Zygomatic process of the maxilla and maxillary sinus border. Forward all information. She is with her 6 year old son. What is the dentist’s obligation? A. content. A. Hypercarotenemia. junction. and translation. shows that the sclera of the eyes are yellow. confidentiality. questioning. The clinical examination B. can be depicted as a cone with Which anatomical structures form the inverted the apex of the cone at the dentino-enamel Y (Y line) in maxillary periapical radiographs? junction. apples. initially begins as an enamel defect in the contact area. mother “Why was Daddy hurting you?” Upon C. Nasopalatine/incisive canal and floor of the base of the cone at the dentino-enamel the nasal fossa. to the specialist. B. Thrombocytopenic purpura. Viral hepatitis. abdominal pain A. the patient confides that her D. Inadequate clinical crown length. cheese. B. D. C. husband was the source of her injuries and indicates that she would not be pursuing any action. of the oral mucosa. the format of some items is not currently used. A. Anterior nasal spine and D. interdental col. B. Note: Some of the items in the Released Test Item Bank may have been discontinued due to outdated science or errors. C. Report the situation to a relevant child A smooth-surface proximal carious lesion protection agency. Iron deficiency anemia. interdental space. initially begins as a subgingival enamel nasopalatine/incisive canal. can be depicted as a cone with A. embrasures. ©The National Dental Examining Board of Canada 2017 . The dentist’s receptionist A. She is referred to a specialist due to the complexity of her dental injuries. in enamel.An adult female patient presents to the dental The greatest cariogenic potential is exhibited office with fractured teeth. Class III furcation defect. D. defect covered with plaque. In addition. C. Which of the following is the most appropriate indication for resective osseous periodontal surgery? A patient complains of fatigue. chewing gum. and lack of appetite. who is crying and upset. Report her injuries to an adult protection D. There is also a yellowish diffuse discolouration D. Floor of the nasal fossa and maxillary sinus border. What is the most likely diagnosis? A. C. discretely reports that the child was asking his B. in enamel. raisins. D. agency. line angles. C. Advanced attachment loss. but she The proximal surfaces of two adjacent teeth in requests that the source of her injuries not be contact form the borders of the disclosed. A. B. the antiviral Which tooth has the best prognosis in a patient drugs should ideally be administered within with generalized periodontal disease? A. D. the gloving is regenerative cells originate primarily from the A. Which of the following statements is true thickness (split-thickness) flap will regarding local anesthetic syringes and needles for dental anesthesia? A. In addition. patient-specific. and translation. B. D. Needles should be recapped after use. B. reduce healing time. Note: Some of the items in the Released Test Item Bank may have been discontinued due to outdated science or errors. provide improved surgical access. If post-exposure prophylaxis is recommended following a significant percutaneous injury from an HIV-positive patient. lamina propria. reduce infraosseous defects. C. procedure-specific. Gain of clinical attachment. for injections when the needle is inserted D. needles may be left uncapped away from the working area after use. the format of some items is not currently used. Delayed hypersensitivity. 1. 4 weeks of the injury.4. Regeneration of the periodontal ligament. To avoid percutaneous injury. for a procedure on a patient with AIDS. device. Restoration of lost alveolar bone. periodontal ligament. increase the amount of attached gingiva. A new anesthetic needle should be used After the elimination of occlusal trauma. 2 weeks of the injury. Reduction in tooth mobility. B. content. B. C. following is most likely to result? D. C. of tactile sensitivity. a partial. cellular cementum. increase the loss of marginal bone. 1.1. C. E. 1. even when the elapsed time between multiple in the presence of inflammation. for a procedure that requires a high degree D. Immediate type I allergic reaction. A. Irritant contact dermatitis. Type IV immunologic reaction. The most appropriate indication for double During guided tissue regeneration therapy. Which is the LEAST common type of adverse reaction associated with the use of latex gloves? A. Bending a needle is an acceptable practice C. collagen membrane. 1. 1-2 hours of the injury. B. The NDEB periodically reviews the bank to improve its quality. 1-2 days of the injury. which of the injections is more than 30 minutes. C. D. ©The National Dental Examining Board of Canada 2017 . C.6. D. D. B. B. A.3. more than 5mm into soft tissue.Compared to a full thickness flap. C. using a scoop method or mechanical A. D. D. During the intravenous administration of Which is the most appropriate method to diazepam. Fibromatosis. Mandibular canal. C. C. B. and translation. The root canals may join and have a C. E. D. The NDEB periodically reviews the bank to improve its quality. precisely at the apex of the tooth. External oblique ridge. C. medial to the pterygomandibular raphe. Storage of amalgam capsules in sealed C. are thixotropic. always necessary. apex of the tooth. the format of some items is not currently used. The dentino-cemental junction is E. Which of the following is the most Which of the following statements regarding characteristic gingival feature of Informed Consent is FALSE? It is agranulocytosis? A. B. D. the inferior projected closest to the occlusal plane when alveolar nerve passes taking a mandibular posterior periapical radiograph? A. C. procedure specific. D. Ulnar. B. E. A cross section of the canal in the apical region is relatively round. which would be In the pterygomandibular space. ©The National Dental Examining Board of Canada 2017 . Radial recurrent. B. Ulceration. A. are exothermic. treatment. B. A. Hematoma. able to be withdrawn. B. Deep brachial. Erythema. The major foramen is precisely at the A. D.Of the following structures. B. D. given only by the person receiving the B. exhibit a high elastic modulus. A. C. Root canals bifurcate and have dual dentate patients because they foramina. content. Mental foramen. Disposal of scrap amalgam in a landfill site. are hydrophilic. D. Use of ISO approved amalgam separators. exhibit viscoelasticity. Radial. C. Submandibular salivary gland fossa. Note: Some of the items in the Released Test Item Bank may have been discontinued due to outdated science or errors. In addition. medial to the medial pterygoid muscle. Brachial. lateral to the sphenomandibular ligament. anterior to the deep tendon of the temporal muscle. containers. Which of the following statements concerning root canals and their foramina is NOT true? Polyether impression materials should be used with caution for full arch impressions of A. which of the following arteries may minimize loss of dental amalgam and mercury accidentally be entered in the antecubital fossa? from dental offices into sewage systems? A. single foramen. Use of mercury vapour scavengers. herpangina. C. parasitic infestations. C. a radiolucent area. The primary muscle of the cheek is the Pernicious anemia may cause A. D. B. osteopetrosis. sclerotic dentin. zygomaticus major. D. viral diseases. E. D. D. A. syphilis. woven bone. C. A. hypercementosis. C. fibrous dysplasia. bacterial diseases. lingual ulceration(s). C. buccinator. ©The National Dental Examining Board of Canada 2017 . juvenile hyperthyroidism. and translation. Adult hypothyroidism. E. aphthous stomatitis. glossitis. D. In addition. the format of some items is not currently used. B. Which of the following can result in post- developmental jaw growth? Blue sclera is characteristic of A. C. space. The NDEB periodically reviews the bank to improve its quality. alteration of the lamina dura. Hypoparathyroidism. thrush. gastric hyperacidity. B. Note: Some of the items in the Released Test Item Bank may have been discontinued due to outdated science or errors. osteogenesis imperfecta. ankylosis. lichen planus. giant cell reparative granuloma. C. B. rickettsia. E. numbness of the jaw and occlusion is unexplained tooth mobility is A. fibrous dysplasia. C. D. cortical bone around surgical site. D. A. B. parotid swelling.The initial histological appearance of a Premature eruption of primary teeth is successful apicectomy would show on a associated with radiograph as A. congenital hypothyroidism. hyperparathyroidism. D. B. Osteitis deformans (Paget's). Coxsackie A virus is the etiologic agent in Inclusion bodies in the nucleus or cytoplasm of cells are diagnostic of A. cleidocranial dysplasia. B. content. B. orbicularis oris. osteopetrosis. B. E. C. osteitis deformans. congenital gingival fibromatosis. malignant neoplasm. swelling. D. Hyperthyroidism. Hyperparathyroidism. masseter. A. B. The most likely diagnosis of a patient with The earliest radiographic sign of traumatic pain. C. widening of the periodontal ligament D. root resorption. A. the only acid-forming bacteria in the mouth. C. cheilitis may be associated with a deficiency of C. B. both acidogenic and aciduric. A magenta-coloured tongue and angular B. Cyanosis. lactobacilli are B. Deafness. content. cocci. B. D. Clubbing of the fingers and toes. In the initiation of dental caries. E. thiamine. D. abundant in calculus. penetrating microorganisms are B. A. Which of the following drugs potentiates the action of sedative drugs? In necrotizing ulcerative gingivitis the deepest A. filamentous rods. Aphasia. serial extraction. monitor until eruption of all permanent D. immediate orthodontic correction. removal of supragingival calculus. D. Motor paralysis. anticholinergic effects. Propranolol. C. In addition. B. Methyldopa. A. capable of surviving without nutrients. C. Heart murmur. immediate orthodontic correction only with sufficient overbite. diplococci. A. D. annoying but controlled with salicylates. E. sharp and prolonged. C. Which of the following conditions may develop as a result of juvenile diabetes mellitus? Ultrasonic scalers are most effective in A. regularly recurrent. D. stabbing and excruciating. E. B. photosensitivity. the format of some items is not currently used. infrequent. dull and persistent. removal of toxins from cementum. Blindness. C. riboflavin. Chlorpromazine produces all of the following EXCEPT A. and translation. old is B. B. antiadrenergic effects. dull and prolonged. excessive salivation. teeth. spirochetes. The NDEB periodically reviews the bank to improve its quality. Which of the following is NOT a sign of congenital heart disease? A. Right ventricular hypertrophy. planing root surfaces. C. Telangiectasia. D. C.The characteristic pain of trigeminal neuralgia is The most appropriate management of a maxillary central incisor in crossbite in a 7 year A. removal of subgingival calculus. Digitalis. A. ascorbic acid. B. Note: Some of the items in the Released Test Item Bank may have been discontinued due to outdated science or errors. A. Ataxia. Spironolactone. C. D. Phenothiazine. ©The National Dental Examining Board of Canada 2017 . C. sharp. D. niacin. E. D. hyaluronidase. small. A. improved healing after surgery. sites. A 30 year old HIV positive patient presents for B. D. streptokinase. D. In a healthy patient whose chief complaint is Initial scaling and oral hygiene instruction in bleeding gums after tooth brushing. C. C. ©The National Dental Examining Board of Canada 2017 . B. correction of pathological migration of teeth. cusp surfaces of teeth. content. division 2 malocclusion is the A. impaction of the maxillary canines. electro-chemical action. D. D. double glove before starting any surgical B. B. microleakage. D. E. yellowish spots on the oral mucosa. and translation. treat the patient in the same way as all other patients. Elimination of local plaque retention A. B. B. B. the format of some items is not currently used. crowding of the mandibular incisors. what is the the treatment of periodontitis results in all of most appropriate initial management? the following EXCEPT A. decreased hemorrhage during surgery. malposition of the maxillary lateral incisors. myasthenia gravis. the removal of an abscessed second molar. A. E. D. Occlusal examination and recording. most appropriate management is to D. multiple sclerosis. pits and fissures of teeth. C. Periodontal examination and recording.Streptococcus mutans colonizes The most damaging characteristic of an Angle Class II. a rhomboid-shaped red patch on the dorsum of the tongue. E. D. C. amylase. papillary lesions on the palate. D. The most likely cause is A bacterial enzyme capable of altering the ground substance of the periodontal ligament is A. muscular dystrophy. Dietary analysis. relationship of the molars. The NDEB periodically reviews the bank to improve its quality. The C. Note: Some of the items in the Released Test Item Bank may have been discontinued due to outdated science or errors. B.5. deep overbite. overhanging margins. Bell’s palsy. pocket shrinkage. schedule appointments at the end of the It is advisable to polish any restorative material day. A. as smoothly as possible in order to prevent C. on the tongue. dextranase. A. procedures. refer to another dentist because universal infection control procedures are insufficient. In addition. at pH of 2. A patient must push up on his mandible to close his mouth. necrotizing fasciitis. C. evaluation of the patient's motivation. C. melanin pigmentation of the lips. B. accumulation of plaque. Characteristics of Peutz-Jeghers syndrome include A. C. D. Prognathic maxilla. attached gingiva and multiple areas of gingival D. increased over bite. B. and translation. D. Open bite. open bite. A panoramic radiograph shows diffuse widening of the periodontal ligament. advanced adult periodontitis. Temporalis. C. maxilla. D. A 52 year old patient presents with a limitation B. magnesium. Retrognathic mandible. Increased lower anterior face height. eosinophils. prognathic mandible. D. hyperparathyroidism. E. third molar. B. scleroderma. retrognathic A. B. monocytes.A patient complains of lip and tongue Which of the following Class II Division 1 hypersensitivity (allodynea) following intake of malocclusion(s) is/are most likely to be hot. retrognathic maxilla. erythema multiforme. C. An advantage of glass ionomer cement is A. basophils. A. Masseter. prognathic maxilla. central incisor. first molar. D. Lateral pterygoid. A-delta and C. C. ©The National Dental Examining Board of Canada 2017 . adhesion to hard tooth tissues. Medial pterygoid. low incidence of sensitivity. prognathic The most likely cause of a cavernous sinus mandible. spicy food. A-beta. B. second premolar. height. of mouth opening. A-gamma. B. C. decreased lower face C. D. In addition. potassium. C. content. cicatricial pemphigoid. B. Note: Some of the items in the Released Test Item Bank may have been discontinued due to outdated science or errors. ammonium. The patient has loss of C. The inorganic ion which is the chief offender in hypertension is A bacterial infection causes the most significant increase in A. recession. C. neutrophils. low solubility. The most likely diagnosis is Which of the following is the most powerful A. wear resistance. jaw-closing muscle? B. thrombosis is a periradicular abscess of a maxillary A. The sensory nerve fibers that corrected with a cervical headgear? are associated with this form of pain are A. the format of some items is not currently used. A-alpha. D. sodium. E. The NDEB periodically reviews the bank to improve its quality. A. lymphocytes. Sjögren’s syndrome. Paget’s disease. expansion. CONTRAINDICATED for a patient with D. catalyst. Note: Some of the items in the Released Test Item Bank may have been discontinued due to outdated science or errors. Addison’s disease. What B. serum alkaline phosphatase and elevated urinary hydroxyproline levels? A. ©The National Dental Examining Board of Canada 2017 . basophils. “ground glass” appearance of the bone. is the most likely diagnosis? C. the mucogingival junction. Hyperparathyroidism. loss in compressive strength. Warm saline solution rinses. D. contraction. A. Radiographic examination shows generalized loss of the lamina dura and a A. to eliminate the suprabony pockets when the pocket wall is fibrous and firm. lymphocytes. Topical steroid therapy. B. Phenytoin. C. A. E. necrotizing ulcerative gingivitis? E. In addition. when the bottom of the pocket is apical to D. C. content. Cyclosporine. fibrous dysplasia. disinfectant. During the setting phase. monocytes. Which of the following medications does NOT B. D. The NDEB periodically reviews the bank to improve its quality. B. cause gingival hyperplasia? C. Nifedipine. the format of some items is not currently used. B. Gardner’s symdrome. reactor. Hyperthyroidism. B.In alginate impression materials. neutrophils. retarder. Antibiotic therapy. C. Gingivectomy is recommended B. C. D. E. to treat moderately deep pockets with The predominant cells in the inflammatory mild intrabony defects. Hypoparathyroidism. cross linking agent. D. and translation. Which condition is associated with elevated D. A. Carbamazepine. Hypothyroidism. Which of the following treatments is C. exudate of an acute periodontal abscess are A. C. sodium A 65 year old patient has a tendency for urinary phosphate (Na3PO4) is the tract stones. Local debridement. eosinophils. A. a dental stone mixture will exhibit A. B. gain in moisture content. Hands should be washed thoroughly and D. Advanced adult periodontitis. A. Erythema multiforme. B. allergic reaction. D. B. Lamellar. Laterally positioned flap. Temporalis. D. IgA. Lateral pterygoid. retrovirus. coronavirus. Note: Some of the items in the Released Test Item Bank may have been discontinued due to outdated science or errors. IgE. The NDEB periodically reviews the bank to improve its quality. A 52 year old patient presents with restricted or D. A. C. B. Buccinator. B. B. D. B. B. Hyperparathyroidism. IgG. ©The National Dental Examining Board of Canada 2017 . E. Free autogenous gingival graft. E. Woven. soap. Bundle. C. bacterial infection. decrease in antibody production. and translation. D. Progressive systemic sclerosis. adenovirus.Which of the following types of bone contain Type I hypersensitivity results from cross- the insertions of the periodontal ligament linking of which immunoglobulin on mast fibres? cells? A. seconds. C. content. A. picovirus. An alcohol hand rub is an acceptable limited mouth opening. C. blood clotting defect. In addition. C. Which statement best describes hand washing Which muscle is primarily responsible for for the prevention of disease transmission? moving the mandible to a lateral position? A. Masseter. IgM. Hands should be washed for at least 15 A. Subepithelial connective tissue graft for C. vigorously prior to placement and upon E. Which mucogingival surgical procedure does NOT increase the zone of attached gingiva? Acquired immunodeficiency syndrome (AIDS) is caused by a/an A. Medial pterygoid. Cortical. removal of gloves. IgD. Cicatricial pemphigoid. Transient flora is more difficult to remove during routine hand washing and needs vigorous scrubbing action. The patient has loss of alternative to hand washing. C. root coverage. D. Coronally positioned flap. the format of some items is not currently used. D. always with an antimicrobial B. attached gingiva and multiple areas of gingival recession and tight skin. What is the most likely A high neutrophil count is indicative of a/an diagnosis? A. C. A panoramic radiograph shows diffuse widening of the periodontal ligament. In addition. It tests C. keratinized tissue. from saliva is D. in pocket depth is primarily due to D. Scaling and root B. B. mandibular central incisor. connective tissue regeneration. distal to the main canal. Gingival curettage. content. IgD. vital and is not mobile. planing have not resolved the condition. 6mm three-wall infrabony mesial defect. In gingivitis. A. plasma cells. The NDEB periodically reviews the bank to improve its quality. Which C. Gingival curettage. is the most appropriate treatment? A. it is most frequently found papillae of 2 months duration are interfering with orthodontic treatment. Following periodontal debridement. Modified Widman flap. Osseous resective surgery. A. reattachment of gingival fibers. Note: Some of the items in the Released Test Item Bank may have been discontinued due to outdated science or errors. suprabony pockets. gingival overgrowths. neutrophils. D. There is no A. C.3 has a 9mm probing depth with a B. mesial to the main canal. B. B. The predominant immunoglobulin isolated C. Guided tissue regeneration. IgG. of the following is the most appropriate D. labial to the main canal. Osseous resective surgery. greatest variation in size and shape is the D. Gingivoplasty. B. gingival pockets. and translation. A. the initial cellular immune D. infrabony pockets. mandibular lateral incisor. Tooth 3. response predominantly involves A. T lymphocytes. maxillary lateral incisor. The permanent anterior tooth that exhibits the C. lingual to the main canal. maxillary central incisor. evidence of attachment loss. B. reduction C. epithelial "adhesion" to the tooth. ©The National Dental Examining Board of Canada 2017 . In a patient with an adequate band of D.When a second canal is located in mandibular Generalized enlarged fibrotic interdental incisors. C. Guided tissue regeneration. gingivectomy is indicated for all EXCEPT A. B. IgA. Which of the following D. the format of some items is not currently used. B lymphocytes. intervention? A. C. IgM. decreased inflammation. exophthalmos and heat B. Osteoporosis. Gram-positive cocci. Focal fibrous hyperplasia. bleeding index. B. Streptococcus salivarius. Epithelial dysplasia. Which of the following lesions of the oral or maxillofacial region is premalignant? Changes in direction of groups of enamel A. Clinical management of gingival enlargement A functional parathyroid adenoma would most caused by Phenytoin (Dilantin®) therapy likely result in includes A. generalized loss of lamina dura. The NDEB periodically reviews the bank to improve its quality. C. intolerance. result from incremental growth. mobility. C. The earliest colonizers of dental plaque are A. Note: Some of the items in the Released Test Item Bank may have been discontinued due to outdated science or errors. extend to the enamel surface. D. D. a mouth guard to control mouth breathing. Occipital lobe. A. C. multiple recurrent periodontal abscesses. In addition. nervousness. plaque index. Actinomyces naeslundii. D. D. C. A. B. E. an increased caries rate. the use of analgesics to control pain. the format of some items is not currently used. make enamel resistant to fracture. Basal ganglia. prisms B. Gram-negative cocci. D. extraction of the teeth. C. C. C. Cerebellum. A. Gram-positive rods. Smoker’s melanosis. B. Anticoagulant therapy. withdrawal of the medication. Age greater than 80 years. oral hygiene. Gram-negative rods. and translation. A. B. C. implants? B. Uncontrolled diabetes mellitus. E. ©The National Dental Examining Board of Canada 2017 . content. Frontal lobe.The re-evaluation of periodontal debridement Which microorganism does NOT contribute effectiveness after 4 to 6 weeks is best assessed significantly to the progression of dentinal by examining clinical attachment levels and caries? A. D. Fibrous dysplasia. B. C. B. Actinomyces viscosus. gingivectomy and maintenance of good D. have no functional importance. radiographic bone density. Which part of the brain is NOT primarily involved in motor control? Which of the following conditions CONTRAINDICATES treatment with dental A. D. Ectodermal dysplasia. D. Lactobacillus casei. a past exposure to hepatitis B with A. In addition. movement? C. easier placement into the cavity carcinomas. C. forces. congenitally missing first premolars. Composite resin. A. mandibular second molar in a working side B. C. Mesiolingual cusp of the maxillary first immunity. Distolingual cusp of the maxillary second carious surfaces in a high caries-risk child? molar. Lichen planus. What is the most likely diagnosis of a white. Mesiolingual cusp of the maxillary first E. B. A. B. recovery from hepatitis B with liver B. hamartomatous polyps of the small B. Linea alba. neoplasms of endocrine organs. Bonded amalgam. molar. multiple palmar or facial basal cell D. D. Resin modified glass ionomer cement. intestine. greater thermal conductivity. C. wrinkled appearing lesion of the buccal mucosa which diminishes in prominence or In an Angle Class I occlusion. D. C. Mesiolingual cusp of the maxillary second Which of the following restorations is the most molar. molar. an inconclusive immune status. Candidiasis. Leukoedema. a highly infectious individual. A. a cast gold neuromas should be further evaluated for the inlay is superior to an amalgam because the possibility of inlay has A. Distolingual cusp of the maxillary first damage. content. appropriate for a primary molar with 3 or more D. B. Note: Some of the items in the Released Test Item Bank may have been discontinued due to outdated science or errors. In an Angle’s Class I occlusion. Mesiolingual cusp of the maxillary second D. diffuse.A patient diagnosed with multiple mucosal For a patient who exhibits bruxism. Distolingual cusp of the maxillary first molar. White sponge nevus. better adaptation to the cavity walls. Stainless steel crown. D. B. Distolingual cusp of the maxillary second molar. D. which cusp of disappears upon stretching? which permanent tooth moves between the mesiolingual and distolingual cusps of the A. ©The National Dental Examining Board of Canada 2017 . preparation. molar. and translation. C. molar. The NDEB periodically reviews the bank to improve its quality. premalignant adenomatous polyposis coli. better ability to withstand masticatory C. the format of some items is not currently used. E. the cusp of The presence of more than 104 copies/mL of which permanent tooth is in contact with the hepatitis B DNA in blood is indicative of central fossa of the mandibular first molar? A. D. asthmatic attacks can be triggered by B. and translation. The NDEB periodically reviews the bank to improve its quality. C. vitamin K. A chronic alcohol abuser has just been diagnosed with Wernicke-Korsakoff’s There is a difference between girls and boys syndrome. immediate generalized (Type 1) bronchospasm and anaphylaxis. mounting of the casts on the articulator. hypothyroidism. D. hypoparathyroidism. D. B. immediate generalized (Type 1) anxiety. A. promote osteoclast proliferation and D. processing of acrylic. A. B. are necessary for calcium absorption from A. Girls are two years ahead of boys. C. C. immediate localized (Type 1) contact C. the format of some items is not currently used. placed supragingivally. He is suffering from a severe deficiency of A. inhibit osteoblast activity. asthma has a decreasing prevalence. setting of condylar guidance. B. glossitis. proximal walls diverge occlusally. postmenopausal osteoporosis. margins must just clear contact with the adjacent tooth. asthma is an acute inflammatory disorder.In determining the ideal proximal outline form Bone loss at menopause accelerates because for a Class II amalgam cavity preparation in a estrogens molar the A. ascorbic acid. A. urticaria and erythema. gingival cavosurface margin must be B. Low serum levels of parathyroid hormone and vitamin D combined with low bone mass in the A laboratory remount of processed dentures is skeleton are consistent with the diagnosis of done in order to correct occlusal disharmony produced by errors primarily in the A. facial and lingual proximal cavosurface differentiation. C. Boys are six months ahead of girls. He complains of pain and burning with respect to the age at which the growth mouth and presents with angular cheilitis and velocity reaches its peak. B.5mm deep. D. C. the small intestine. conjunctivitis and rhinitis. Girls are six months ahead of boys. B. D. D. dietary calcium deficiency. C. axial wall should be 1. asthma leads to increased caries. delayed (Type IV) contact dermatitis. content. thiamin. ©The National Dental Examining Board of Canada 2017 . B. Note: Some of the items in the Released Test Item Bank may have been discontinued due to outdated science or errors. In pediatric patients The most common form of latex allergy is a/an A. registration of jaw relation records. vitamin A. C. Girls are one year ahead of boys. In addition. D. inhibit bone resorbing cytokine synthesis. D. Buccally positioned first premolars. Hematoma. The lesions can be wiped off to reveal an erythematous base. An adult with liver cirrhosis. risk of patient contamination. apex pointing to the enamel surface. Immediate allergic reaction. base facing toward the pulp. old patient. The most appropriate procedure when a glove has been punctured during a restorative appointment is to On a bite-wing radiograph. The NDEB periodically reviews the bank to improve its quality. B. Epstein-Barr virus. C. a smooth surface A. D. Lack of canine labial bulges in a 10 year the surface of the hands. Candida albicans. A chronic alcoholic. dryness of the skin prior to donning gloves. complete the procedure as expeditiously as C. change the glove as soon as possible. D. quantity of transient microorganisms on D. 4 year old child. triangle with the B.Which patient would NOT be predisposed to What type of insurance must a dentist carry in liver toxicity following a dose of 1000mg of order to practice dentistry in Canada? acetaminophen? A. D. D. C. the patient complains of a tenseness in the left A. B. C. General liability. Herniation of buccal fat pad. The lesions Shortly after the administration of a local are most likely caused by anesthetic for the removal of tooth 2. B. Long term disability. D. C. possible. B. Office overhead. the format of some items is not currently used. Early exfoliation of primary canines. What is the most likely diagnosis? C. put on an overglove and complete the A. Surgical emphysema. An HIV-positive patient has white lesions on the buccal mucosa. layers. A diabetic. Midline central diastema. In addition. cheek and left cheek swelling is observed. Malpractice. Which clinical sign indicates palatal impaction The purpose of hand hygiene in infection of the permanent maxillary canines? control is primarily to reduce the A. E. content. ©The National Dental Examining Board of Canada 2017 . wash the gloved hands using antimicrobial proximal carious lesion in enamel appears as a soap and complete the procedure. C. A. apex pointing to the dentino-enamel junction.8. Straphylococcus aureus. Employment. A. B. A 15kg. resident microflora in the deep tissue B. procedure. A. and translation. base at the dentino-enamel junction. C. Streptococcus viridans. B. D. Note: Some of the items in the Released Test Item Bank may have been discontinued due to outdated science or errors. C. B. B. Amputate the apex. B. Note: Some of the items in the Released Test Item Bank may have been discontinued due to outdated science or errors. treatment is rendered. Extirpate the pulp. A. composed chiefly of macrophages. granulation tissue. should not be administered lidocaine for C. Ivy bleeding time. and translation. C. which laboratory two adjacent implants is test provides the most accurate information on coagulation time? A. C. What is the most appropriate space management for a 4 year old patient who has lost all their primary maxillary incisors due to Which viral hepatitis does NOT have a chronic trauma? carrier state? A. A. should receive dental treatment only on the A.An inflammatory cellular infiltrate found in the The statute of limitations states that the period connective tissue of healthy gingiva is during which an action in negligence against a dentist can be initiated starts when the A. C. lack of interdental papilla. A 9 year old presents immediately after the avulsion of a permanent maxillary central incisor. D. Hepatitis D. Hepatitis B. C. Activated partial thromboplastin time. is likely to have an increased red blood cell count. content. The NDEB periodically reviews the bank to improve its quality. D. INR. A fixed Nance button appliance. D. patient first meets the dentist. Hepatitis A. Rinse the tooth with saline. a tissue response to food decomposition. C. D. treatment is paid for. an indication of systemic disease. Curette the root surface. A removable Hawley appliance. D. B. A. Thrombin time. B. B. E. D. B. No treatment. is at greater risk for infection. patient seeks a second opinion about the problem. A patient on hemodialysis implantation? A. A fixed lingual holding arch. D. Which of the following is the most appropriate management prior to re. local anesthesia. Platelet count. The most common problem associated with For a patient taking warfarin. Hepatitis C. peri-implantitis. day of hemodialysis. In addition. a routine microscopic finding. E. the format of some items is not currently used. C. patient first becomes aware of the problem. B. ©The National Dental Examining Board of Canada 2017 . C. Note: Some of the items in the Released Test Item Bank may have been discontinued due to outdated science or errors. Prevotella intermedia. D. B. C. In addition. actinomycetemcomitans. following eruption of the maxillary first A. An 8 year old patient presents 4 hours post. habit. contains insoluble glucans. excursion. normal bilateral excursions. becomes less anaerobic as it matures. C. several months prior to the pubertal C. excursion. The anticariogenic effect of systemic fluoride is related principally to the Long term stability of the orthodontic A. following cessation of a thumb sucking cariogenic bacteria. B. when the correction is accomplished by enamel. Aggregatibacter (Actinobacillus) A. bacteriostatic action on oral flora. organisms. D. contains predominantly Gram-negative A. growth spurt. B. D. C. when serial extractions are performed. buffering effect on acids produced by A. undetected disease. C. root canal treatment. D. Treponema denticola. following eruption of the maxillary B.A patient with a unilateral left disc The most appropriate time to begin orthodontic displacement without reduction is most likely correction of an Angle Class II malocclusion is to present with A. in patients with decreased lower anterior face heights. normal left excursion and limited right permanent molars. Dental plaque A diagnostic test for dental caries which has low sensitivity and high specificity will result A. Porphyromonas gingivalis. content. in B. apexogenesis. limited left excursion and normal right permanent central and lateral incisors. intrusion of the posterior teeth. correction of an anterior open bite is better B. limited bilateral excursions. D. and translation. contains food particles. alteration in the composition of the B. C. bactericidal action on oral flora. The NDEB periodically reviews the bank to improve its quality. apexification. extraction. B. The most appropriate pulpal treatment is A. patients being alarmed unnecessarily. a test whose performance does not justify its cost. D. the format of some items is not currently used. Which of the following is associated with trauma with an oblique crown fracture of 2. D. D. ©The National Dental Examining Board of Canada 2017 .1 aggressive periodontitis in adolescents? exposing 2mm of vital pulp. unnecessary treatment being provided. C. at the start of the pubertal growth spurt. Gram-positive and aerobic. Note: Some of the items in the Released Test Item Bank may have been discontinued due to outdated science or errors. the alveolus. D. Gram-positive and anaerobic. stimulated by sympathetic activity. D. Albumin. is twice as sweet as table sugar. breaks down at high temperatures and cannot be used in baking. cartilage. B. Chronologic age. Gram-negative and anaerobic. B. B. Dental age. inhibited by cyclic AMP. D. presence of gingival edema. B. B. stimulating auto immune responses. the frontomaxillary suture. Sucralose D.6 has a disto-occlusal amalgam Which of the following plasma proteins has the restoration with a gingival overhang. Pressure and tension have little effect on C. anterior teeth. D. C. B. microorganisms are most likely associated with C. the subgingival environment in this site? D. E.What is the most reliable indicator with respect "Dens in dente" is most commonly associated to the timing of treatment for growth with modification? A. B. Gamma globulin. Fibrinogen. B-lipoprotein. releasing toxins. In addition. Saliva production is increased when blood flow C. amelogenesis imperfecta. stimulated by parasympathetic activity. content. The primary factor for selecting periodontal flap surgery rather than gingivectomy is A. The NDEB periodically reviews the bank to improve its quality. Tooth 3. inhibited by acetylcholine. need for access to the bony defect. D. dentinogenesis imperfecta. C. A. E. the mandible. Calculus causes gingival inflammation by A. causes constipation when ingested in excessive amounts. and translation. A. mechanical irritation. osteogenesis imperfecta. A. growth of D. pocket depth. supernumerary teeth. A. is a derivative of sucrose. Gram-negative and aerobic. A. to the salivary glands is D. C. There is greatest ability to bind drugs? radiographic evidence of bone loss and deep probing depths with bleeding upon probing. Hemoglobin. C. the format of some items is not currently used. B. A. Tanner weight chart. presence of subgingival calculus. retaining micro-organisms. Which of the following types of B. C. ©The National Dental Examining Board of Canada 2017 . Skeletal age. C. C. C. the format of some items is not currently used. D. reflex. A. C. periodontal decreased function. 7-10mm. In a peripheral nerve such as the trigeminal. When tumour cells revert to a more primitive. B. C. likely etiologic factor? B. Pubertal hormones. B. and cementum is composed primarily of A. Note: Some of the items in the Released Test Item Bank may have been discontinued due to outdated science or errors. dental papilla. this is called ligament. Tannerella forsythia. Actinomyces viscosus. 11-13mm. In addition. Mouth breathing habit. muscle spindle information. hypoplasia. Prevotella intermedia. Predominant organisms in necrotizing When compared with that of its permanent ulcerative gingivitis are successor. and translation. embryonic or undifferentiated form with an The most common type of collagen found in increased capacity for reproduction and a the gingival connective tissue. unmyelinated C type fibres convey Tooth development begins when the basal layer of cells proliferates to form a ridge called the A. D. 3-6mm. A. bleeding and Which periodontal pathogen can use the swollen gums. The NDEB periodically reviews the bank to improve its quality. B. content. C. A protrusive relation record should be made by instructing the patient to protrude the mandible A. C. B. hyperplasia. D. C. Aggregatibacter (Actinobacillus) actinomycetemcomitans. equal buccolingually and mesiodistally. B. nociceptive pain information. A. reflexes such as the jaw jerk reflex. D. Insulin-dependent diabetes mellitus.A 13 year old complains of red. Porphyromonas gingivalis. ©The National Dental Examining Board of Canada 2017 . dental lamina. D. anaplasia. B. type II. Prevotella intermedia. C. Porphyromonas gingivalis. equal mesiodistally. narrower mesiodistally. D. larger mesiodistally. What is the most A. type III. type I. odontoblastic matrix. D. spirochetes and fusiforms. Blood dyscrasia. B. type IV. the occlusal surface of the primary mandibular second molar is A. B. reflexes such as the jaw opening C. A. metaplasia. Clinical examination reveals hormone estrogen as a growth factor? this is present only on the labial gingiva of the maxillary anterior teeth. a dentist can A. All of the following are possible sequelae to the permanent successor EXCEPT A. and translation. D. an apically repositioned full thickness and may fracture. “severe pain in my right ear” which began C. the recording material will be too weak B. right anterior disc displacement with reduction. occlusal adjustment. three hours ago. B. with deflection to the right. the format of some items is not currently used. A. B. further plaque control instruction. should be done with caution as it is metabolized more slowly. delayed eruption. flowable composite resin. C. reduction. an apically repositioned split thickness mandible. for the patient’s benefit. C. glass ionomer cement. increases the incidence of insomnia. gingivectomy. requires higher doses for clinical effects. left anterior disc displacement without B. After root planing and plaque control perforation of the recording material must be instruction. a 22 year old patient still shows avoided because swollen and edematous gingiva with 3mm pockets and a 4 to 6mm band of attached A. collect the third party payment without A patient presents with a chief complaint of requesting the patient's portion. silver amalgam. reduction. Note: Some of the items in the Released Test Item Bank may have been discontinued due to outdated science or errors. C. right anterior disc displacement without D. flap. D. hybrid composite resin. B. right lateral excursion of 9mm and left lateral excursion of 2mm. C. the material will undergo dimensional gingiva. E. to restore these lesions is B. C. compensate a referring dentist. charge different fees when warranted by region. reveals tenderness over the right preauricular D. The most appropriate management is changes. bill an insurance company a higher than when eating. D. enamel hypoplasia. A. D. flap. content. The NDEB periodically reviews the bank to improve its quality. is a safe and reliable practice. root dilaceration. B. ectopic eruption. left anterior disc displacement with with root caries. contact of teeth could deflect the A. The most appropriate material reduction. maximum interincisal opening of 21mm clinical conditions. Ethically. C. The most likely diagnosis is A 67 year old patient with xerostomia presents A. In addition. The use of benzodiazepine in the elderly to A 6 year old patient has an intrusive injury to manage anxiety tooth 5.In recording centric relation registration.2. ©The National Dental Examining Board of Canada 2017 . An examination usual fee. it is important to determine the position of the base of the pocket One week following extraction of teeth 1. D. Which of the following tests is/are specific in The blood vessel which may be injured when the diagnosis of anemia? harvesting a connective tissue graft from the palatal mucosa is the A. salivary gland atrophy. Hematocrit determination. mucogingival junction. D. B. Note: Some of the items in the Released Test Item Bank may have been discontinued due to outdated science or errors. mandibular premolars. thickness of the alveolar process. D. ibuprofen intake. hyperparathyroidism. D. A complete blood count. B. The most frequent cause of hyposalivation in elderly patients is In healthy gingiva. B. B. cirrhosis of the liver. maxillary lateral incisors. hemophilia A. E. Subsequent blood D. an 18 year old male returns to the dental office complaining of persistent bleeding from A. adult rickets. amount of underlying trabecular bone.8. posterior superior alveolar. position of the cemento-enamel junction. D. C. determination. Vitamin C deficiency. D. C. a dry socket. hemoglobin. B. C. greater palatine. vitamin K deficiency. likely related to B. and translation. A. A. C. C. the level of the interproximal alveolar crest is related to the A. C. The NDEB periodically reviews the bank to improve its quality. excessive oropharyngeal muscular tonus D. content. The medical history is B. B. The most likely cause of the bleeding is The physiopathology of sleep apnea is most A. B. cemento-enamel junction. hemoglobin concentration and red cell count. the extraction sites. D. except for episodes of bruising C. furcation. maxillary premolars. Sjögren’s syndrome. multiple medications. In periodontal surgery. Measuring the concentration of A. White cell count and hematocrit C. the format of some items is not currently used. obstruction in lower airways. central nervous system respiratory depression. and joint swelling as a child. In addition. C. A. during sleep. tip of the inter-dental papilla. location of the gingival margin. protein deficiency. unremarkable. sphenopalatine. Alzheimer’s disease. nasopalatine.The labial/buccal attached gingiva on Osteomalacia is seen in permanent teeth is normally widest at the A. ©The National Dental Examining Board of Canada 2017 . a severe Angle Class III malocclusion.8 and to the 4. tests show normal bleeding time and a factor VIII level of 14%. mandibular canines. A 78 year old patient presents with several carious lesions on the root surfaces of the maxillary posterior teeth. none of these pathways. Class I occlusion. and translation. acute asthmatic attack. A. B. encourage mesial movement of the deciduous molars. The opening pathway from C. ©The National Dental Examining Board of Canada 2017 . herpetic ulcer. E. A. pathway H. Resin-modified glass ionomer. predict human cell growth or death. B. the format of some items is not currently used. reproduce the in vivo environment. hybrid composite resin. of a mandibular incisor point in an Angle’s B. C. Administration of succinylcholine to a patient 3mm. provide space for alignment of the likely result in permanent incisors when crowding is 3- 5mm. microfilled composite resin. A. D. A. C. gingivitis would most likely be the result of a/an A. gumma. C. lower primary cuspids is indicated to C. Biocompatibility tests conducted in vitro D. chancre. The NDEB periodically reviews the bank to improve its quality. maximum intercuspation follows D. aphthous ulcer. In addition. D. pathway A and B. Interproximal reduction of the mesial of the B. amalgam. undercontoured crown. D. overcontoured crown. B. subgingival finish line. C. chronic studies. E. hypertension. convulsions. provide space for alignment of the permanent incisors when crowding is 1. supragingival finish line. B. B. prolonged apnea. pathway C. Note: Some of the items in the Released Test Item Bank may have been discontinued due to outdated science or errors. follow screening with an animal model. pathway A. deficient in serum cholinesterase would most C. mucous patch. D. decrease inter-cuspid arch width. are more easily standardized than clinical After the cementation of a crown.A primary infection of syphilis occurring on the tongue is referred to as a/an A. content. D. The most appropriate restorative material for these lesions is The above diagram demonstrates a lateral view of a tracing illustrating the border movements A. This is most likely the result of C. acromegaly. B. E. A. C. A. A 28 year old patient who has severe anterior B. radiographic appearance. results of vitality testing. A. caninus. B. a basal cell carcinoma. the architecture at the CEJ. increased bleeding time. Which of the following is the most D. an Angle Class II malocclusion. ©The National Dental Examining Board of Canada 2017 . depth of periodontal pockets. epithelial cells and tissue phagocytes. chronic periodontal disease. a sebaceous cyst. has been increasing in size. and C. In addition. verruca vulgaris. diagnosis is C. Note: Some of the items in the Released Test Item Bank may have been discontinued due to outdated science or errors. treatment. Fluoride rinse prescription. risorius. D. crowding. appropriate initial step in management of this E. diabetes. orbicularis oris. A patient who has until recently been on prolonged corticosteroid therapy may have A. Osteoporosis may result from C. Tetracycline. mobility of the teeth. D. D. The most likely B. changes in dietary pattern. labii inferioris. Penicillin. It has a rolled border and A. high level of plasmatic cortisol. lupus erythematosus. hyposensitivity to pain. C. E. D.Which antibiotic is primarily bactericidal? A patient presents with a non-healing lesion on the side of the nose. an epulis. D. B. case? A. fibroblasts and eosinophils. C. an increased metabolic rate. The most important diagnostic element in assessing the periodontal status of a patient is the Granulation tissue is composed of A. decreased tolerance to physiological a high DMFS score wants orthodontic stress. Chloramphenicol. B. Caries risk assessment. B. the format of some items is not currently used. content. Cephalometric analysis. plasma cells and giant cells. age related changes in cementum E. D. inflammatory cells. composition. B. Clindamycin. capillaries and chronic D. The muscle attached to the labial surface of the maxilla above the region of the central incisors is A healthy 78 year old patient presents with A. three new carious lesions on root surfaces. C. Diagnostic wax-up. labii superioris. prolonged steroid therapy. and translation. hypothyroidism. The NDEB periodically reviews the bank to improve its quality. C. B. D. A. fibroblasts. Erythromycin. decrease marginal percolation. 135/86 mmHg. ©The National Dental Examining Board of Canada 2017 . D. the format of some items is not currently used. C. which of the following a gold inlay preparation is to would be considered normal blood pressure? A. Williams probe. Tooth migration. (PSR) probe. increase resistance and retention forms. Michigan “O” probe. D. 175/95 mmHg. fibroblast. delayed healing. remove undermined enamel. histiocyte. improve marginal adaptation. mismanagement of the airway. infection. D. B. Gingival recession. B. The NDEB periodically reviews the bank to improve its quality. Bradykinin. 185/94 mmHg. B. C. B. Maxillary furcation involvements are best Which of the following conditions is most assessed clinically by probing with a likely to lead to thrombosis? A. Note: Some of the items in the Released Test Item Bank may have been discontinued due to outdated science or errors. A. C. The most frequent cause of death occurring Which of the following compounds released by under general anesthesia is inflammatory cells induces bone resorption? A. B. C. alveolar osteitis. B. Presence of bacteria in blood. A. Stasis of blood. and translation. Nabers probe. C. Lowered oxygenation of hemoglobin. odontoblast. hemorrhage. B. Periodontal Screening and Recording D. Increased concentration of plasma. C. Alkaline phosphatase. C. D. D. E. content. D. cardiac arrest. Nitric oxide. traction on the viscera. overdosage of premedication. mesenchymal cell. Widened periodontal ligament. E. The most likely complication following surgery for a patient with thrombocytopenic purpura would be A. 130/100 mmHg. B.The principal reason for a cavosurface bevel on In a healthy 75 year old. Fremitus. C. A. overdosage of anesthetic agent. The cell of the dental pulp most capable of Which of the following is NOT a sign of transforming into other cells is the occlusal trauma? A. Deficiency of circulating platelets. B. angina. In addition. D. C. E. Interleukin-1. A. D. hyperglycemia. birth defects. of this patient is to E. Manual brushing. has never been infected with hepatitis B. method of cleaning endodontic instruments C. C. prior to sterilization? D. D. hypoglycemia. place a provisional crown on the tooth indefinitely. with bilateral temporomandibular joint clicking without pain is A. The most appropriate management is to Folate deficiency is associated with an A. B. restore the tooth with composite until the D. The vital signs are observed. Note: Some of the items in the Released Test Item Bank may have been discontinued due to outdated science or errors. C. cholesterol. Washing in an ultrasonic bath for 5 crown to a patient who requires restoration of a minutes. and translation. use local anesthetic with no epinephrine. microcytic anemia. A. B. High pressure water jet. Although a dentist recommends an all-ceramic D. content. an anterior repositioning splint. the patient complains endocarditis requires the extraction of a of being lightheaded and weak. C. hyperventilation. The presence of hepatitis B surface antigen (HBsAg) and hepatitis B e antigen (HBeAg) in blood indicates the individual A. cortisone injections. The most appropriate immediate management D. A. is infectious for hepatitis B. requested. Respiratory Rate: 16/min B. the patient requests a full gold crown be placed. A. insist on placing a more esthetic crown. C. the format of some items is not currently used. Sweating is mandibular molar. cerebrovascular accident. syncope. D. has acquired immunity to hepatitis B.A 45 year old with insulin-dependent diabetes A 65 year old patient who has recovered from a mellitus has a morning dental appointment. Which of the following is the most effective B. temporomandibular joint surgery. no treatment. stroke 6 months previously and has a history of During the examination. increased risk of B. In addition. delay treatment for 3 months. The most appropriate management for a patient D. C. elevated high-density lipoprotein patient reconsiders. C. Washing with antimicrobial soap. ©The National Dental Examining Board of Canada 2017 . low serum homocysteine. fractured maxillary lateral incisor. is not infectious for hepatitis B and has not acquired immunity to hepatitis B. The NDEB periodically reviews the bank to improve its quality. proceed with the treatment. The patient is most likely experiencing Blood Pressure: 135/85 mmHg Pulse: 76/min A. place a full gold crown as the patient B. use prophylactic antibiotics. B. calcifying odontogenic cyst. Pentazocine. Which of the following muscles of mastication is associated with the condylar head and articular disc? A. B. C. Codeine. hemangioma. C. C. ameloblastoma. Oxycodone. In addition. C. C. C. D. A. Lidocaine. Granular cell tumour. Nitrous oxide. E. B. eruption cyst. Temporalis. Cleft palate. C.Which of the following entities has the greatest A smooth-surfaced. Oral melanotic macule. ©The National Dental Examining Board of Canada 2017 . B. E. content. Which of the following is the most frequent A large pericoronal radiolucency associated major congenital malformation of the head and with an impacted third molar tooth is most neck? likely a/an A. dentigerous cyst. Which of the following increases the risk for bronchospasm in asthmatic patients? Ocular lesions may be associated with A. D. D. B. predominance in elderly patients. D. Meperidine. C. D. atrophic filiform papillae. A. traumatic bone cyst. lichen planus. D. and translation. D. Junctional nevus. E. the format of some items is not currently used. Prednisone. D. Ibuprofen. B. E. mucocele. Naloxone. intramucosal nevus. C. Double lip. E. Lateral pterygoid. association with hairy tongue. The NDEB periodically reviews the bank to improve its quality. B. Cystic hygroma. Masseter. Encephalotrigeminal angiomatosis. D. B. papilloma. mucous membrane pemphigoid. B. Medial pterygoid. Which of the following drugs is used in treating opioid-dependent individuals? Geographic tongue is characterized by A. A. exophytic oral malignant potential? mucosal lesion with no colour change is most likely a/an A. A. necrotizing ulcerative gingivitis. Note: Some of the items in the Released Test Item Bank may have been discontinued due to outdated science or errors. solid. congenital soft tissue deformity. Methadone. White sponge nevus. B. fibroma. herpangina. leukoplakia. ultrasonic scaling. The coronoid process of the mandible can be seen in all of the following types of radiographs EXCEPT Typically. B. An incipient carious lesion on an interproximal B. B. B. Supernumerary tooth in the midline. osteoradionecrosis. In addition. form dentinal bridges after appropriate pulp capping procedures. 10-19. surgery and scaling. content. D. osteomalacia. The NDEB periodically reviews the bank to improve its quality. Ameloblastoma. Odontogenic myxoma. Stafne's bone defect/static bone cavity. Premature loss of a primary maxillary second Total removal of subgingival calculus on a molar usually produces a malocclusion in the tooth with pockets more than 5mm deep is best permanent dentition that is characterized by achieved by A. A. cavity and decreased bone density are radiographic features of A.The normal differential white cell count for Which of the following does NOT have a neutrophils is multilocular radiolucent appearance? A. D. B. D. C. D. Cherubism. vestibular to the contact area. A. an Angle Class II molar relationship on the affected side. gingival to the contact area. occlusal to the contact area. central incisors. E. E. A. enlargement of the medullary D. osteopetrosis. anterior crowding. Note: Some of the items in the Released Test Item Bank may have been discontinued due to outdated science or errors. panoramic. B. 40-65. Abnormal labial frenum. Failure of fusion of the premaxillae. Central giant cell granuloma. 20-29. lingual to the contact area. D. maxillary occlusal. show an increase in myelinated nerves between the erupting permanent maxillary when compared to pulps of younger adults. ©The National Dental Examining Board of Canada 2017 . Normal eruption pattern. collagenous fibres. subgingival irrigation. D. C. C. the format of some items is not currently used. What is the most likely cause? A. C. A. 30-39. labially displaced maxillary canines. show a decrease in the number of C. surface is usually located C. periapical. an Angle Class III molar relationship on the affected side. have decreased reparative capacity compared to younger adults. lateral cephalometric. B. trabeculation. B. molar. B. delayed eruption of the permanent first C. decreased cancellous C. An 8 year old patient has a 3mm diastema D. E. vital pulps of elderly patients A. osteoporosis. Thinned cortical bone. C. D. root planing. 66-90. and translation. A. B. Excessive overjet and overbite. fifth week. D. Lateral pterygoid. E. Bite-wing radiographs show a C. maximal opening of 55mm. the format of some items is not currently used. CONTRAINDICATION for an all-ceramic E. Systematic review articles from dental research journals. measure the space between the C. observation. A low caries index. condyle.Which of the following is suggestive of a Which of the following types of publications is malignant tumour of the parotid gland? the most reliable source for making evidence- based clinical decisions? A. A. B. Excessive salivation. appropriate treatment is E. An endodontically treated tooth with a should be completed by the cast post and core. and translation. B. patient have pronounced. Professional association journals. predict the probability of canine impaction. conservative amalgam restorations. maxillary anterior crown? A. glass-ionomer restorations. Seventh cranial nerve paralysis. The NDEB periodically reviews the bank to improve its quality. Temporalis. Sudden swelling. Which of the following is a clinical D. tenth week. calculate the amount of crowding in the maxilla. topical fluoride application. content. limitation in right laterotrusion. normal dentino-enamel junction. deep occlusal fissures B. The most D. Embryologically. Dental manufacturer’s product C. age is indicated to B. D. D. Which of the following muscles causes displacement of the condyle in a subcondylar fracture of the mandible? The permanent first molars of a 7 year old A. C. assess the severity of midline deviation. C. B. fifteenth week. that are stained. A. temporomandibular joint disc and the D. Masseter. Fluctuation. D. In addition. Case reports in dental journals. Normal overjet and excessive overbite. B. twentieth week. Medial pterygoid. Note: Some of the items in the Released Test Item Bank may have been discontinued due to outdated science or errors. application of pit and fissure sealants. fusion of the palatal shelves C. ©The National Dental Examining Board of Canada 2017 . C. C. B. A. information. deflection of the mandible to the right during protrusion. deflection of the mandible to the left at A panoramic radiograph taken at 10 years of maximal mouth opening. reciprocal click in the right TMJ. A clinical sign of an acute disc dislocation without reduction in the right TMJ is A. Superior pharyngeal constrictor. D. What A loss of sensation in the lower lip may be is the most appropriate management? produced by A. In addition. there is sufficient tooth bulk in the abutment teeth for retention. D. pigmentation associated with geniohyoid muscles. malignancy in the body of the mandible. What is the most B. Reversible pulpitis. dorsum of the tongue. B. Acute apical abscess (acute periradicular E. The most likely diagnosis is A. D. The mandibular lateral incisor has a occur most often on the shallow restoration. Reposition the teeth and splint for 7 to 10 C. physiologically tolerated. buccal mucosa. medial pterygoid. Eosinophilic granulocyte. fracture in the mandibular canine region. A. days. muscles. a favorable crown-root ratio is established. A. palate. test. is tender to percussion and gives a positive response to an electric pulp A. the interocclusal distance will be B. and translation. lateral pterygoid and masseter muscles. tetracycline pigmentation. B. D.A 5 year old child has yellow pigmentation of The muscles used when closing the jaws to the deciduous teeth which under ultraviolet maximum intercuspation include light gives a bright yellow fluorescence. A patient experiences pain and some gingival swelling in the anterior segment of the Dysplastic lesions of squamous epithelium mandible. D. will be improved. abscess). A 3 year old presents with subluxated maxillary central incisors. C. the most important consideration is that Which of the following is the most active cell in synthesizing and secreting antibodies? A. content. The occlusion is normal. E. B. temporalis and geniohyoid muscles. Place the child on a soft diet and monitor the teeth. D. chromogenic bacteria. B. temporalis and masseter C. D. enamel hypoplasia. C. Mast cell. when the vertical dimension has to be increased. ©The National Dental Examining Board of Canada 2017 . the format of some items is not currently used. lateral pterygoid. C. the aesthetic appearance of the patient D. Periodontal abscess. masseter. T-cell lymphocyte. Macrophage. Extract the traumatized teeth. floor of the mouth. There is some mobility. A. a soft diet. The NDEB periodically reviews the bank to improve its quality. likely diagnosis? C. medial pterygoid. amelogenesis imperfecta. In fixed bridge construction. C. gingiva. masseter and B. D. temporalis. Plasma cell. Reposition the teeth and place the child on A. B. medial pterygoid. C. Bell’s palsy. trigeminal neuralgia. Note: Some of the items in the Released Test Item Bank may have been discontinued due to outdated science or errors. Symptomatic irreversible pulpitis. E. both appositional and sutural growth. myofascial pain. D. presence of a pulp exposure. Which of the following muscles is NOT Periapical odontogenic cysts are primarily affected? associated with A. content. trismus of the masticatory muscles. D. A. internal derangement of the TMJ without D. Note: Some of the items in the Released Test Item Bank may have been discontinued due to outdated science or errors. D. In addition. E. The NDEB periodically reviews the bank to improve its quality. exact location of the apical foramen.Which of the following is a proliferative Desquamation of the gingiva usually occurs as response of the soft tissue to an irritant? a result of A. ©The National Dental Examining Board of Canada 2017 . a developmental abnormality. D. C. Levator palpebrae superioris. vitality of the pulp. A. B. A. Preoperative endodontic radiographs will show The postnatal increase in width of the maxilla the results from A. an inflammatory infiltrate of plasma cells reduction. nonvital teeth. B. presence of active infection. normal cell turnover. and translation. Cellulitis. hyalinization of the principal fibres of the periodontal ligament. polymorphonuclear cells predominate. C. C. C. The histopathologic changes in chronic gingivitis are characterized by The most likely diagnosis for a patient with an interincisal opening of 30mm before feeling A. B. sutural growth. right facial paralysis is noted. size of the pulp chamber and root canal(s). Mentalis. B. E. benign neoplasia. D. B. internal derangement of the TMJ with C. loss of rete pegs and destruction of the pain and a maximum opening of 44mm with basement membrane. Following the administration of a right inferior alveolar nerve block. Aphthous ulcer. Abscess. Pyogenic granuloma. inflammation. the format of some items is not currently used. pain is B. A. appositional growth. impacted wisdom teeth. and lymphocytes. congenitally missing teeth. C. Buccinator. B. Nasalis. C. B. an inflammatory infiltrate in which reduction. subluxation of the TMJ. Orbicularis oris. C. hypercapnia. B. chewing occurs in centric relation. ignores recurrent dental caries. position. D. approximately the D. Primary teeth start to calcify in the fetus at C. mucosa. Note: Some of the items in the Released Test Item Bank may have been discontinued due to outdated science or errors. one year before eruption of the mandibular permanent first molar. A. third month. D. Pain on percussion. gastric ulceration. attached gingiva. missing. Guided tissue regeneration is a surgical B. first month. immediately. Codeine administration is associated with all the following side effects EXCEPT A. C. B. constipation. C. Which of the following is consistent with a diagnosis of reversible pulpitis? A. repair cemental defects. during eruption of the mandibular C. and translation. content. centric relation is a good reference permanent first molar. C. The NDEB periodically reviews the bank to improve its quality. it makes it easier to set up the teeth. alveolar bone. regenerate the periodontium. Maximum intercuspation on a complete denture The mandibular primary second molar is should be established to correspond with extracted in a 5 year old patient. measures dental caries experience. hypoxia. The most centric relation because appropriate time to construct a space maintainer is A. B. C. baroreceptors.The respiration of a patient with chronic The DMF (decayed. pulp. D. regenerate long junctional epithelial attachment. alkalosis. D. A. Lingering pain to heat. In addition. it prevents cheek biting. filled) index for obstructive pulmonary disease (COPD) is dental caries in adults primarily controlled by A. No spontaneous pain. B. does not apply to all populations. C. A. response in the A. includes teeth missing for any reason. B. B. Lingering pain to cold. The percussion test is used to test nerve D. B. nausea. B. D. seventh month. sedation. periodontal ligament. E. ©The National Dental Examining Board of Canada 2017 . fifth month. permanent first molar. one year after eruption of the mandibular D. D. the format of some items is not currently used. C. repair connective tissue. A. procedure used to C. D. penicillin. high incidence of facial clefts. keratocystic odontogenic tumours. Mandibular central and lateral incisors. multiple supernumerary teeth. Maxillary first and second molars. clindamycin. pulpotomy. In addition. the format of some items is not currently used. Streptococcus salivarius. Which of the following is most indicative of a sialolith in the submandibular duct? Which of the following does NOT improve the retention of a Class II inlay? A. horizontal condylar inclination. C. D. Vertical releasing incisions for a full-thickness The predominant microorganisms in deep flap should cross the free gingival margin at the dentinal caries are A. mucogingival problems. Increasing the parallelism of walls. observation. B. B. D. ©The National Dental Examining Board of Canada 2017 . The NDEB periodically reviews the bank to improve its quality. C. C. suprabony pockets.Which of the following teeth are most The gingivectomy is a useful technique to commonly lost due to periodontal disease? eliminate A. Adding an occlusal dovetail. A. metronidazole. Mandibular first and second premolars. infrabony pockets. Collagenase activity is inhibited by A facebow is used to record the A. D. tetracycline. Cleidocranial dysplasia can be associated with The most appropriate management for a concussed tooth is A. B. Pain and swelling associated with eating. pulpectomy. Note: Some of the items in the Released Test Item Bank may have been discontinued due to outdated science or errors. Drainage of pus from Stensen's duct. Lactobacillus species. vertical dimension of occlusion. C. D. and translation. Actinomyces species. D. content. Placing a gingival bevel. Maxillary first and second premolars. A. D. B. D. D. incisal guidance. tip of the interdental papillae. B. C. splinting. D. C. B. relationship of the maxilla to the horizontal hinge axis. the mouth. A. most prominent aspect of the root. B. C. Fluctuant bluish swelling in the floor of A. B. fragile bones. Periodic swelling of the cheek. line angles of the tooth. B. Streptococcus mutans. C. B. C. thick osseous ledges. midfacial aspect of the tooth. C. A. Lengthening the axial walls. B. D. A patient is currently on warfarin. A. from the tissues. ©The National Dental Examining Board of Canada 2017 . stellate reticulum cells. osmotic property. the format of some items is not currently used. The proximal surfaces of the crown. Before a planned extraction of tooth 3. C. Note: Some of the items in the Released Test Item Bank may have been discontinued due to outdated science or errors. INR (International Normalized Ratio). D. small contact angle. original die has a uniform margin opening of C. The internal surfaces of the crown. neuronal projections. Bleeding time. increase stability of the remaining Which of the following occurs if there is a abutment teeth. B. Repaired congenital heart disease. capillary extensions. prevent the distal extension moving away D. B. Which of the D. Existing mitral valve prolapse. Hypomineralized dentin. Partial thromboplastin time. B.4. A crown with an acceptable marginal fit on the B. Presence of a coronary artery stent. B. B. content.Which of the following conditions requires The hydrophilicity of an impression material is further information prior to making a decision defined by its regarding antibiotic prophylaxis for an invasive procedure? A. A. water content. following should be modified to address this problem? A. D. C. C. D. the majority of The primary purpose of an indirect retainer in a the inner dentinal tubules contain distal extension removable partial denture is to A. C. odontoblast projections. A. 1mm when placed intraorally. The occlusal surface of the preparation. Upon completion of primary dentin formation in a healthy permanent tooth. increase the retention of the removable B. and translation. increase stress distribution among the abutment teeth. 8mm. the patient’s A tooth with 3mm of gingival recession and a coagulation mechanisms should be evaluated probing depth of 5mm at the same site has an using which test? attachment loss of A. high water absorption. The NDEB periodically reviews the bank to improve its quality. No cementum. No eruption. D. In addition. 2mm. defective development of the epithelial root sheath? A. The occlusal surface of the crown. partial denture. C. Previous coronary bypass. 3mm. Brittle dentin. C. 5mm. C. D. hepatitis A. junction between the mucoperiosteum and gingiva. separation between the marginal gingiva D. Oral hairy leukoplakia primarily occurs with A. D. In addition. Prosthetic reconstruction. at the gingival margin. Recurrent pericoronitis. initiate periodontitis. B. Papillon-Lefèvre syndrome. Prevention of crowding of mandibular D. B. HIV. hypophosphatasia. A drain should not be used if drainage B. parafunctional habits and stress reduction fluctuant swelling. content. and translation. of sufficient ideally should be placed A. calcium channel occurs during the procedure. C. A. blockers and benzodiazepines. D. Physiotherapy and removal of occlusal interferences. separation between the attached gingiva and the marginal gingiva. C. D.The gingival margin of the preparation for a Achieving root coverage with a connective full crown on a molar.0mm subgingivally. buccal and lingual soft and hard tissue. supragingivally. A. control of C. Muscle relaxants. Generalized aggressive periodontitis. A. C. that satisfies the tissue graft is most dependent on the presence requirements for retention and resistance. C. Profound. A. maxillary incisor retrusion. interdental soft and hard tissue.5mm subgingivally. 0. A. The mucogingival line denotes the B. B. D. diabetes. The procedure is indicated for a localized. B. Note: Some of the items in the Released Test Item Bank may have been discontinued due to outdated science or errors. C. B. unattached gingiva adjacent to the site. 1. techniques. mandibular incisor protrusion. E. D. B. anterior open bite. exacerbate gingival hyperplasia. Which of the following is true regarding the incision and drainage of an acute periradicular abscess? What is the most appropriate management for A. C. Occlusal trauma can Which of the following is the best indication for the removal of mandibular third molars? A. The NDEB periodically reviews the bank to improve its quality. C. cause gingival recession. increase tooth mobility. incisors. and the alveolar mucosa. junction between the attached gingiva and the alveolar mucosa. Impaction. deep anesthesia is needed to nocturnal bruxism in the presence of perform the procedure. attached gingiva adjacent to the site. the format of some items is not currently used. Relief of the pressure and pain will take 24 hours to occur. ©The National Dental Examining Board of Canada 2017 . malocclusion? B. The effects of constant mouth breathing include D. C. Occlusal appliance. expansion of the maxilla. obtain adequate access. posterior mandibular buccal gingiva. oral candidiasis. dental procedures should be performed on All of the following are oral complications of non-dialysis days. D. pressure. ©The National Dental Examining Board of Canada 2017 . irrigate copiously. D. molar. C. mesiobuccal cusp of the mandibular first B. The most appropriate method to prevent root The buccal (long buccal) nerve is a branch of canal obstruction during the instrumentation phase of endodontic treatment is to A. C. Which of the following is the maximum daily In an Angle Class I occlusion. B. D. C. Erythema. dialysis can be used to measure blood C. C. Note: Some of the items in the Released Test Item Bank may have been discontinued due to outdated science or errors. irreversible pulpitis. B. hairy leukoplakia. A. and translation. 8. B. increased blood flow. uncontrolled diabetes mellitis EXCEPT for B. 4. B. Increased pH. A. content.000 mg. pulp necrosis. What localized changes occur in soft tissue when it becomes infected? A. In addition. periodontal bone loss. V3 and provides motor innervation to the buccinator muscle. the present with a diagnosis of alveolar osteitis? most likely diagnosis is A. normal bleeding can be expected if the platelet count is normal. 6. C. Decreased pH. use a chelating agent. reversible pulpitis.If there is persistent pain after a thermal Which of the following symptoms is always stimulus has been removed from a tooth. B. C. V2 and provides motor innervation to the buccinator muscle. A. use reamers instead of files. D. C.000 mg. the mesiobuccal dose of acetaminophen for a healthy adult? cusp of the maxillary first molar occludes with A.000 mg.000 mg. Decreased pH. Fever. decreased blood flow. decreased blood flow. molar. mesiobuccal groove of the mandibular first D. 2. Increased pH. V3 and provides sensory innervation to the D. the arm used for vascular access during B. posterior mandibular buccal gingiva. A. The NDEB periodically reviews the bank to improve its quality. A. Pain. increased blood flow. mesial marginal ridge of the mandibular first molar. delayed healing. V2 and provides sensory innervation to the B. D. For a patient with chronic renal failure. Swelling. C. the format of some items is not currently used. central fossa of the mandibular first molar. appropriate treatment is D. endodontically treated tooth is to C. strengthen the root. A. and translation. B. D. debridement of the roots. Cigarette smoking. Hypothyroidism. B. Which of the following is a major risk factor A flush terminal plane will convert to an for the progression of chronic periodontitis? Angle Class I occlusion by A. denudation of the periosteum. C.When a pit and fissure sealant is placed over The degree of curing of a composite resin is early but noncavitated caries dependent on all of the following EXCEPT the A. E. wavelength of the emitted light. trauma with an oblique crown fracture of tooth B. D. the likelihood of caries progression is surface. content. decortication of bony defects. maxillary growth. D. sealant. intensity of the light source. the sealant retention rate is greatly A. The most C. the caries will be arrested only if it is a surface. bacterial loads generally increase under the C. Human papillomavirus infection. ©The National Dental Examining Board of Canada 2017 . amount of light reflected by the composite D. primary tooth. avoid leakage into the root canal. distal movement of the maxillary first permanent molars. retain the core. the next step is A 7 year old patient presents 4 hours post.1 exposing 2mm of vital pulp. Osteoporosis. 2. B. The NDEB periodically reviews the bank to improve its quality. C. A. closure of mandibular primate space. extraction and prosthesis. distance from the light tip to the composite C. maximize esthetics. Note: Some of the items in the Released Test Item Bank may have been discontinued due to outdated science or errors. vital pulpotomy and placement of calcium hydroxide. reduced. removal of inflammatory tissue. reduced. pulpectomy and obturation with calcium The primary function of a post used in an hydroxide. mesial movement of the mandibular first permanent molars into the leeway space. D. In addition. A. B. B. mandibular forward growth exceeding C. B. pulpectomy and obturation with gutta. the format of some items is not currently used. D. A. duration of light exposure. percha. After the initial incision and raising of a flap in the modified Widman technique. B. remaining. cemento-enamel junction (CEJ). C. and translation. retrusive maxillary incisors. B. pulpal hyperemia. E. equal to the pocket depth. C. B. B. moisture contamination during placement. remains stationary. the failure to use primer prior to its placement. C. Note: Some of the items in the Released Test Item Bank may have been discontinued due to outdated science or errors. in one direction and the root apex in the developing a deficiency of which one of the opposite direction. Nifedipine.A 39 year old patient has lingering pain to heat A CONTRAINDICATION for a pulpectomy in the area of tooth 3. Vitamin B12. B.6 and has referred pain to on a primary molar is the left ear. B. Sodium. In addition. There is a large MOD amalgam on tooth 3. the format of some items is not currently used. D. following nutrients? B. D. in the same direction as the root apex. normal pulp. D. less than 1/3 of the root structure C. A. expansion of the palatal suture. buccinator muscular pull. C. the loss of B. bone apposition laterally. The NDEB periodically reviews the bank to improve its quality. use of a chemical-cured sealant. A. spontaneous nocturnal pain. symptomatic irreversible pulpitis. The most likely reason for extracting four first premolars in orthodontics is because of Which of the following drugs is most likely to cause gingival overgrowth? A. asymptomatic irreversible pulpitis. pulpal necrosis. posterior crossbite. reversible pulpitis. Potassium. along the line of force while the root apex A. B.6. Tetracycline. D. A lateral force applied to the crown of a tooth will result in the crown moving A strict vegetarian is at greatest risk for A. C. D. endochondral ossification. greater than the pocket depth. D. attachment is C. placement of the sealant over enamel caries. less than the pocket depth. Growth in maxillary width occurs mainly via When the gingival margin is at the level of the A. Insulin. The most likely diagnosis is A. A. pulp tissue necrosis. a tooth size/arch size discrepancy. C. The most likely cause of loss of a pit and fissure sealant is the A. D. content. C. Hydrochlorothiazide. ©The National Dental Examining Board of Canada 2017 . an excessive overbite. Ascorbic acid. migraine. B. pain that lasts less than one minute. herpetic gingivostomatitis. A 58 year old patient is being managed with the development of a greyish discolouration in topical corticosteroids for ulcerative oral the crown usually indicates mucosal lesions. D. incise into the buccal sulcus. The most likely diagnosis is A. a bone scan for metastatic tumours. A 55 year old Caucasian with a long-term tobacco habit has diffuse pigmentation involving only the anterior mandibular labial gingiva. a more potent topical corticosteroid. affecting the lower D. A. racial pigmentation. smoker’s melanosis. D. soft white A. pulp necrosis. lingual surface of the anterior mandible. of a hematoma following a posterior superior alveolar nerve block is to A. and translation. drug-induced pigmentation. excisional biopsy of the nodules. bony hard nodules on the A. hemochromatosis. The pain is C. risk for spread of infection by bacteria B. Addison’s disease. E. vitality test of teeth adjacent to the nodules. most appropriate management is C. spontaneously sloughs and can be removed E. extreme and lancinating. trigeminal neuralgia. The B. would be C. C. A patient has gingival epithelium that D. C. external root resorption. mucous membrane pemphigoid. a systemic corticosteroid. aspirate with a wide bore needle. place hot towels over the cheek. D. incisional biopsy. C. The most likely diagnosis is A patient with an orofacial infection is most at A. C. plaques. D. D. with minor manipulation. antifungal medication. myofascial pain. D. content. cluster headache. Note: Some of the items in the Released Test Item Bank may have been discontinued due to outdated science or errors. antibiotic therapy. travelling through C. A recall examination reveals widespread. the maxillary sinus. The most appropriate management B. B. internal root resorption. B. The most appropriate immediate management E.Many months after trauma to a primary incisor. pulp canal calcification. airways. aphthous stomatitis. ©The National Dental Examining Board of Canada 2017 . The most likely diagnosis is A 30 year old patient has bilateral asymptomatic. A. observation at routine recalls. arteries. the format of some items is not currently used. right face and jaw and can be brought on by light touch of the skin over the lower right lip and chin. hyperplastic candidiasis. A patient has a one year history of recurrent B. B. peripharyngeal spaces. loosely adherent. A. referral for potential premalignant colonic polyps. In addition. apply firm direct pressure. The NDEB periodically reviews the bank to improve its quality. C. Oral hairy leukoplakia. radicular cyst. Median rhomboid glossitis. C. Anoxia. D. and translation. aureus infection with of penicillin is often complicated by the A. inability of penicillin to penetrate the B. simple/traumatic bone cyst. C. Carbohydrates. C. swell and expand. Acetaminophen. Note: Some of the items in the Released Test Item Bank may have been discontinued due to outdated science or errors. D. Which condition has NOT been associated with Candida albicans infection? Which analgesic interferes with platelet A. D. the format of some items is not currently used. C. Lipids. hyperthyroidism. Constipation. vitamin D overdose. A. dentinal tubules become occluded A. C. membrane of S. Paresthesia. Oxycodone. vitamin B12 deficiency. C. because odontoblast cellular projections B. Lethargy. occult malignancy. sialorrhea. E. residual cyst. C. lichenoid reaction to amalgam. B. Angular cheilitis. Collagen. produce organic molecules. In addition. allergic reaction caused by staphylococcal protein. secretion of penicillin acetylase by S. Generalized loss of lamina dura is an early sign Treatment of a S. has had prior exposure to Mycobacterium species. aggregation? B. A positive tuberculin test means that the patient B. DNA. The NDEB periodically reviews the bank to improve its quality. Aphthous ulcerations. Ibuprofen. D. Cushing’s disease. A. C. aureus. Addison’s disease. Which of the following is NOT a symptom of hypercalcemia? With age. deposit more mineral. absorb dentinal fluid. B. D.Glossodynia can be associated with Which of the following is NOT a significant component of dental plaque? A. D. ©The National Dental Examining Board of Canada 2017 . should be vaccinated against tuberculosis. B. aureus. Radiologic interradicular scalloping of bone is a characteristic of a A. A. dentigerous cyst. is susceptible to tuberculosis. Denture stomatitis. D. A. hyperparathyroidism. A. content. production of penicillinase by S. B. D. aureus. C. B. E. B. 1. B. decreased with the use of a high strength material.5 had nonsurgical root canal treatment A. The tooth responds within normal limits to all tests performed. saliva. Nystatin. C. Some areas of enamel are more susceptible D. “white spot” lesion. Incision and drainage. to demineralization. a small perforation was produced in C. Frequency of sugar intake is less important than total consumption. C. span. content. Sulfamethoxazole. The NDEB periodically reviews the bank to improve its quality. Immediate surgical repair. Follow-up in 6 months. A. management? A. and translation. Delayed seal of the perforation. D. B. A. E. Metronidazole. High salivary or plaque counts of A healthy 38 year old has a well-defined S. proportional to the cube of the width of its B. ©The National Dental Examining Board of Canada 2017 . What D. B. the pontic is overcontoured gingivally. Open the tooth for drainage. the gingival margins are overcontoured. is the most appropriate management? E. C. Omeprazole. B.Which of the following is the most appropriate Flexing of a fixed partial denture under in the management of chronic periodontitis? occlusal loads is A. C. E. Prescribe an antibiotic.6. Caries development requires a tooth. Isoniazid. Extraction. During an endodontic access preparation on B. Prescribe an analgesic. The earliest clinical evidence of caries is a B. tooth 3. What is the most appropriate D. Which of the following drugs is indicated for D. What is the most appropriate management? Tooth 2. tooth 4. may be radiolucency 2mm in diameter at the apex of related to high caries risk. the radiograph shows apical rarefaction. Note: Some of the items in the Released Test Item Bank may have been discontinued due to outdated science or errors. casting. D. Acyclovir. C. the gingival margins are open. plaque bacteria and dietary A. Doxycycline. the furcation area. performed 10 years ago. The tooth became B. C. a void exists on the internal surface of the carbohydrate. linearly proportional to the length of the D. the format of some items is not currently used. sensitive to percussion in the last 6 months and C. the management of a Gram-negative anaerobic infection? A. Follow-up in 6 months. Immediate seal of the perforation. Clotrimazole. In addition. Endodontic retreatment. Which of the following statements about dental caries is INCORRECT? Seating of a fixed partial denture may be hindered if A. which is reversible. Extraction. Apicoectomy. pontics. mutans and Lactobacillus spp. Chlorhexidine. D. parallel to the floor. polylactic acid. anemia. palpable mass of normal colour located on the buccal mucosa. ©The National Dental Examining Board of Canada 2017 . D. D. prescribe a muscle relaxant. dentoalveolar malocclusions. aphasia is A. surgical treatments primarily address circumscribed. A. Note: Some of the items in the Released Test Item Bank may have been discontinued due to outdated science or errors. Which of the following is a principle of mucoperiosteal flap design? For a patient with dementia. C. There is normal post-operative suture material that should be used is swelling with no evidence of fluctuance or discharge. Combined orthodontic and orthognathic A 55 year old has a 12mm diameter well. C. appropriate margin of normal tissue to include C. content. C. B. prepubescent child. In addition. D. chromic catgut. D. the width of the base. A patient returns three days following the the maxillary sinus lining is accidently extraction of a carious mandibular first molar. perforated. incise and drain. D. 20º to the floor. B. The NDEB periodically reviews the bank to improve its quality. 90º to the floor. to B. B. occlusal discrepancies due to multiple around the perimeter of the lesion? tooth loss. 4-5mm. tension. C. catgut. What is the discrepancies. The flap margins should be approximated A. A. Nonintegration of dental implants in a 76 year For the extraction of maxillary teeth. After 10 days of observation. 2-3mm. The worsening pain. C. moderate to severe dentoskeletal biopsy is going to be performed. The base of the flap should not be under C. the most old patient taking anti-rheumatic drugs and appropriate position of the maxillary occlusal glucocorticoids is most likely due to plane is A. prescribe an antibiotic. 60º to the floor. 8-9mm. The most appropriate management is A. B. appreciate the consequences of behaviour. vertical growth anomalies in a A. D. execute personal oral hygiene tasks. make decisions. express ideas or thoughts. The height of the flap should be more than E. hyperglycemia. over the bony defect. recognize people. and translation. A. the format of some items is not currently used. place an intra-socket dressing. immunosuppression. The base of the flap should be narrower characterized by a diminished ability to than the free margin. bad taste and suture that will resorb in 10-14 days. polyethylene. D. C. The wound is to be closed with a complaining of a foul smell.While performing a bony tuberosity reduction. B. elevated. B. 6-7mm. an excisional B. osteodystrophy. septic shock. patient with minimal overjet.6. ago. epilepsy. Which of the following NSAIDs is most D. She requires the extraction of tooth 4. acute asthma.A patient returns to his dentist four days Intensifying screens are used when exposing following the extraction of a grossly carious extraoral analogue radiographic films to 4. Ketolorac. The NDEB periodically reviews the bank to improve its quality. C. antibiotic to prescribe? D. remain unchanged. Celecoxib. with Ludwig’s angina is a/an A. The last time the patient A. (FosamaxTM) for 4 years. was prescribed penicillin. Metronidazole. The most serious complication for a patient D. increase penetration. What is the most appropriate C. generalized rash. relationship is most likely to develop into an Angle Class II relationship? A. Order a bone density test. B. B. Straight. Mesial step. B. optic nerve involvement. In addition. Which of the following is the most appropriate management? A. There is a fluctuant swelling involving the submandibular space. Use hyperbaric oxygen before and after the extraction. content. angina pectoris. B. he developed a B. A. Naproxen. Salbutamol is the most appropriate drug to B. cavernous sinus thrombosis. airway obstruction. tip labially. Discontinue the alendronate preoperatively. ©The National Dental Examining Board of Canada 2017 . Proceed with the extraction without adjusting the dose of alendronate. decrease scatter radiation. Distal step. incisors will most likely A. decrease exposure time. manage C. Clindamycin. D. C. A. B. A. A patient had a myocardial infarction 6 months C. and translation. the format of some items is not currently used. An elderly female patient with a history of C. tip lingually. mediastinal sepsis. improve detail.5. myocardial infarction. osteoporosis has been taking oral alendronate D. tip laterally. C. Tetracycline. Note: Some of the items in the Released Test Item Bank may have been discontinued due to outdated science or errors. the mandibular E. Ibuprofen. Which primary molar terminal plane D. vasodepressor syncope. B. Amoxicillin. C. appropriate for the patient? E. With excessive forward mandibular growth in a D. B. B. use Gates-Glidden drills as end-cutting instruments to open the coronal portion of canal. B. Residual ridge. start at the same time as comprehensive orthodontic treatment. D. integrity of the apical one third should be maintained. the format of some items is not currently used. content. be delayed until the completion of growth. use the next larger file as soon as a tight fit of the preceding file is achieved. C. start in the primary dentition. What is the most stable area for support of a mandibular complete denture? A. Buccal shelf.During initial endodontic preparation of the apical portion of the canal. A. E. In addition. The NDEB periodically reviews the bank to improve its quality. Mylohyoid ridge. C. ©The National Dental Examining Board of Canada 2017 . D. sodium hypochlorite should be used sparingly. C. Retromylohyoid area. occur during the early to middle mixed dentition. Note: Some of the items in the Released Test Item Bank may have been discontinued due to outdated science or errors. D. Genial tubercle. enlarge with a wider file. E. if there is a canal restriction. and translation. Interceptive orthodontic treatment using appliances to treat the effects of oral habits should A.
Copyright © 2024 DOKUMEN.SITE Inc.